Está en la página 1de 338

1.Cuál de las siguientes es una causa de cianosis central?.

Señale la respuesta correcta:

a-Menor gasto cardiaco sin fallo retrógrado.

b-Hipoventilación alveolar con bradipnea.

c-Exposición al frío con vasoconstricción en dermis.

d- Fenómeno de Raynaud secundario

2) ¿Cuál de los siguientes medicamentos es el de mayor efectividad en vómitos y náuseas


inducidas por quimioterapia, siendo su mecanismo de acción el antagonismo de los receptores 5 –HT3?.

Señale la respuesta correcta:

a- Dimenhidrinato.
b- Escopolamina.
c- Ondasentron
d- Domperidona.

3) ¿Cuál de las siguientes es causa de hipopigmentación?. Señale la


respuesta correcta:

a- Pelagra.
b- Tiña versicolor
c- Deficiencia de vitamina B12.
d- Queratosis seborreica,

4) En un paciente de 30 años con diagnóstico de neumonía extrahospitalaria, que se encontraba


sano previamente y no había recibido antibióticoterapia en los últimos 90 días. ¿Qué antibiótico indica
inicialmente ?. Señale la respuesta correcta:

a- Trimetroprima sulfametoxazol EV
b- Ceftriaxona endovenoso (EV)
c-Claritromicina vía oral.
d-Piperacilina tazobactan (EV)

5)¿Cuál de los siguientes es un criterio diagnóstico del colon irritable?. Señale la resp uesta correcta:

a- El dolor abdominal mejora con la defecación


b- La presencia de dolor abdominal sin relación con la frecuencia de evacuaciones
c- Presencia de dolor abdominal sin relación con la frecuencia de las evacuaciones y con los
cambios en la forma de las heces
d- Presencia de dolor abdominal, dos veces por día en los últimos seis meses.

6) Un paciente que presenta HBsAg +, Anti HBs negativo, Anti HBc IgM, HBeAg + y Anti HBe
negativo. ¿Cuál es la interpretación?.

Señale la respuesta correcta:

a- crónica de alta infectividad


b- aguda de alta infectividad
c- crónica de baja infectividad
d- aguda de baja infectividad
7) ¿Cuál de los siguientes son factores predisponentes para los cálculos de colesterol?:

a- Fibrosis quística
b- Ayuno
c- Hemólisis crónica
d- Cirrosis alcohólica

8) ¿Cuál de las siguientes es causa de anemia con un índice de producción de reticulocitos mayor
o igual a 2,5?

a- Talasemia
b- Nefropatía
c-Efectos tóxicos de fármacos
d-Hemólisis intravascular

9) ¿Cuál de los siguientes tumores se asocia a eritrocitosis como síndrome paraneoplásico?

a- Cáncer de pulmón
b- Hepatocarcinoma
c- Cáncer de próstata
d- Cáncer de mama

10) ¿Cuál de los siguientes es un criterio mayor en la endocarditis infecciosa?

a- Fiebre mayor a 38°


b- Nueva insuficiencia valvular
c- Embolia arterial recidivante
d- Cardiopatía predisponente

11) ¿Cuál es el tratamiento para una gangrena gaseosa?. Señale la respuesta correcta:

a- Nafcilina u oxacilina 2gr ev cada 4 a 6 hs


b- Clindamicina 660 a 900mg ev cada 6 a 8/hs más penicilina G 4 millones de U ev cada 4-6hs
c- Aciclovir 800 mg PO 5 veces al día durante siete a diez días más eritromicina 500 mg PO cada 6
hs.
d- Vancomicina 1 gr ev cada 12hs

12) ¿Cuál de las siguientes es una característica del vértigo laberíntico?. Señale la respuesta
correcta:

a- Nistagmo horizontal
b- Acúfenos o hipoacusia
c- Romberg negativo
d- Nistagmo vertical

13) ¿Cuál de las siguientes patologías se asocia con enfermedad intersticial difusa en la radiografía
de tórax?. Señale la respuesta correcta:

a- Absceso bacteriano
b- Infarto pulmonar
c- Fibrosis pulmonar idiopática
d- Malformación vascular
14) En un paciente con riesgo alto de síndrome de lisis tumoral, ¿Cuándo tiene indicación de
comenzar la hemodiálisis?. Señale la respuesta correcta:

a- Potasio sérico mayor a 4,8 meq/l


b- Ácido úrico sérico mayor a 10 mg/100ml
c- Creatinina sérica de 5 mg/dl
d- Fosfato sérico mayor a 6 mg/100ml

15) Un paciente de 16 años de edad con antecedente reciente de infección gastrointestinal por
E.coli, consulta por fiebre, cefalea intensa, y en el laboratorio se constatan: anemia, trombocitopenia y
aumento de LDH. En el frotis de sangre periférica esquistocitos. Los estudios de orina revelan hematuria
y leve proteinuria. ¿Cuál es el diagnóstico presuntivo más probable?

a- Necrosis tubular aguda isquémica secundaria a las enterotoxinas.

b- Síndrome urémico hemolítico/trombótico trombocitopénico


c- Pielonefritis bilateral aguda con bicitopenia autoinmune.
d- Nefritis intersticial alérgica y enfermedad tubular con bicitopenia .

16) ¿Cuál de las siguientes es causa de enfermedad tubulointersticial aguda?. Señale la resp uesta
correcta:

a- Nefropatía por células de mieloma.


b- Nefritis por radiación
c- Nefritis por citomegalovirus
d- Reflujo vesicoureteral

17) ¿Cuál de los siguientes síndromes vasculíticos es producido por depósito de complejos
inmunitarios patógenos?. Señale la respuesta correcta:

a- Poliangeítis microscópica
b- Arteritis de células gigantes
c- Crioglobulinemia mixta, secundaria a hepatitis C
d- Síndrome de Churg Strauss

18) ¿Cuál de las siguientes es una característica de la hiponatremia hipervolémica?. Señale la


respuesta correcta:

a- Deficiencia de ACTH
b- Edema generalizado y ascitis
c- Sodio urinario alto
d- Ácido úrico bajo

19) ¿Cuál de las siguientes es causa de hipertiroidismo secundario?. Señale la respuesta correcta:

a- Enfermedad de Graves
b- Adenoma tóxico
c- Adenoma hipofisario secretor de TSH
d- Ingesta excesiva de hormona tiroidea

20) ¿En cuál de las siguientes patologías pulmonares se observan granulomas de células
epitelioides semejantes a sarcoides no caseificantes?
a- La silicosis
b- La asbestosis
c- La neumoconiosis de los trabajadores de las minas de carbón
d- La beriliosis crónica

21) Dentro de las manifestaciones musculoesqueléticas de la artritis inducida por depósito de


cristales, ¿Cuál de las siguientes afecciones considera la correcta?.

a-Síndrome del túnel carpiano


b-Epicondilitis
c- Epitrocleítis
d- Fascitis

22) En una paciente con probable prolactinoma y elevación de la prolactina, ¿Cuál de los siguientes
estudios solicita?. Señale la respuesta correcta:

a-Mamografía
b-Resonancia magnética nuclear de silla turca
c- Tomografía computada con contraste de ambas carótidas
d- Angio TC de circulación vertebral

23) Paciente de 25 años de edad, que 15 días antes presentó una infección respiratoria alta,
continuando en la actualidad con tos seca. Es llevada a la guardia con cuadro de desorientación y fiebre.
Se interna para optimizar diagnóstico y terapéutica. Se constata que en 24 hs tiene una diuresis de 7 l/d.
acompañada de polidipsia. Osmolaridad urinaria de 200 mosmol/l. Restringiendo la vía oral, la paciente
continúa poliúrica.

¿Cuál de los siguientes diagnósticos es el más probable?:

a- Polidipsia primaria
b- Diabetes insípida central
c- Diabetes insípida central secundaria a encefalitis viral
d- Diabetes insípida nefrogénica secundaria a necrosis tubular.

24) ¿Cuál es la deformidad valvular observada más frecuentemente en la cardiopatía reumática


crónica?

a- La estenosis mitral
b- La estenosis tricúspidea aislada
c- La insuficiencia mitral aislada
d- La asociación de estenosis mitral con estenosis aórtica

25) ¿Cuál de las siguientes es causa de acortamiento del intervalo QT en el electrocardiograma?

a- Hipocalcemia
b- Hipopotasemia
c- Hipercalcemia
d- Hiponatremia

26) ¿Cuál de las siguientes es causa de insuficiencia cardiaca con fracción de eyección mayor a
50%?. Señale la respuesta correcta:
a- Obstruccion moderada de la descendente posterior
b- Miocardiopatía hipertrófica
c- Miocardiopatía dilatada no isquémica
d- Bradiarritmias crónicas
27) La causa más frecuente de vértigo es:

a- Síndrome de Menière.
b- Síndrome de Wallenberg.
c- Neuronitis vestibular.
d- Vértigo posicional benigno.

28) ¿Cuál de los siguientes, es considerado Criterio de Exclusión de infusión de fibrinolíticos en


pacientes con enfermedad cerebrovascular isquémica?:

a- Glucemia de 130 mg%.


b- Tensión arterial diastólica > 110 mmHg.
c- Plaquetas < 130.000/mm3.
d- Cirugía Mayor en el último año.

29) ¿Cuál de las siguientes es indicación de tratamiento con insulina en un paciente diabético tipo
2?:

a- Aquellos pacientes con normopeso


b- Pacientes con glucemia en ayunas de 200mg%
c- Diabético tipo 2 que fracasa al tratamiento con dosis máximas de drogas orales y se mantiene
hiperglucémico.
d- Paciente con infección urinaria.

30) Ante la sospecha de estenosis hipertrófica de píloro, ¿cómo iniciaría los exámenes
complementarios?

a- Transito digestivo
b- Radiología simple
c- Ecografía.
d- pHmetría gástrica.

31) ¿Cuál de las siguientes cefalosporinas no requiere ajuste de dosis con Filtrado Glomerular entre
25-10 ml/min?

a- Ceftazidima
b- Cefepime
c-Ceftriaxona
d-Cefuroxima

32) ¿Cuál de los siguientes fármacos antirretrovirales produce depresión de la Medula Ósea
(medulotóxico)?

a- Efavirenz
b- Raltegravir
c -Atazanavir
d- Zidovudina
33) ¿Cuál de los siguientes no se incluye como un grupo de riesgo para recibir la vacuna
antineumocócica?

a- EPOC
b- Fisura etmoidal con fístula de LCR y rinorraquia
c- Enfermedad de Whipple
d- Asplenia funcional o anatómica

34) Paciente de 53 años con antecedente de reemplazo de válvula aórtica hace 3 años. Consulta
por síndrome febril de dos semanas de evolución asociado a astenia. Refiere un episodio de hematuria
hace 5 días que cedió espontáneamente. Al examen físico presenta buen estado general, febrícula (T°
axilar: 37.4°), hemorragias en astilla en pulgar derecho. Señale la conducta más adecuada:

a- Descarta diagnóstico de endocarditis por ser un paciente de bajo riesgo.


b- Realiza hemocultivos, de ser negativos descarta el diagnóstico de endocarditis.
c- Realiza hemocultivos y ecocardiograma transtorácico. De ser negativo realiza ecocardiograma
transesofágico.
d- Realiza hemocultivos y ecocardiograma transtorácico. De ser negativo descarta el diagnóstico de
endocarditis

35) ¿Cuál es el signo o síntoma más frecuente en un carcinoma papilar de mama?

a- Nódulo palpable subareolar


b- Derrame sanguinolento por el pezón
c- Adenopatía axilar y nódulo mamario reconocido semiológicamente
d- Retracción del pezón

36) ¿En qué casos está indicada una biopsia pleural con aguja?

a- Paciente con caverna tuberculosa que desarrolla un derrame pleural


b- Paciente con un nódulo pulmonar hiliar y atelectasia segmentaria del lóbulo superior
c- Paciente que desarrolla un derrame pleural de origen desconocido
d- Recidiva de un derrame pleural en un paciente tratado por cáncer de pulmón.

37) ¿Cuál es la localización más frecuente, dentro de las enunciadas, de una angiodisplasia?

a- Colon descendente
b- Colon ascendente
c- Colon sigmoide
d- Recto

38) En una hemorragia digestiva baja, intermitente, con pérdida de sangre a razón de 0.1 ml/ min,
¿cuál de los siguientes métodos diagnósticos tiene mayor sensibilidad?

a- Arteriografía selectiva
b- Colonoscopía
c-Glóbulos rojos marcados con 99mTc
d-Centellografía con sulfuro de tecnesio coloidal, marcado con 99mTC

39) En un cáncer de recto localizado a 9 cm del margen anal y que abarca 1/3 de su circunferencia,
¿cuál es la intervención quirúrgica de elección?
a- Proctocolectomía izquierda
b- Amputación abdominoperineal u operación de Miles
c- Resección local endoanal
d- Resección anterior u operación de Dixon
40) En un paciente de 70 años de edad, ¿cuál de las siguientes causas es la más frecuente de
isquemia colónica aguda?
a- Embolia arterial
b- Trombosis arterial
c- Vólvulo sigmoide
d- Episodio de bajo volumen minuto cardíaco

41) ¿Cuál es el tumor más frecuente del mediastino anterior?


a- Timoma.
b- Quiste broncogénico. c- Quiste esofágico.
d- Tumor neurogénico.

42) Paciente con trauma torácico penetrante en cara lateral del hemitórax derecho. Signos clínicos
de derrame pleural masivo y radiografía de tórax con velamiento completo y descompensación
hemodinámica. ¿Qué conducta adoptaría?
a- Arteriografía de vasos centrales
b- Tomografía computada
c-Toracocentesis
d-Toracotomía exploradora inmediata

43) En un herido de bala cuyo trayecto perforó el mediastino de lado a lado, y la Rx. presenta
neumotórax y neumomediastino leve: ¿Qué estudio diagnóstico indica como imprescindible?

a- Broncofibroscopia para confirmar perforación de tráquea y/o grandes bronquios.


b- TAC de tórax.
c- Punción pleural. d- Esofagograma.

44) ¿Las hernias inguinales de tipo directas, se presentan por?

a- Dilatación del orificio inguinal profundo.


b- Persistencia del conducto peritoneo vaginal.
c- Debilidad de la pared posterior.
d- Dilatación del orificio del orificio inguinal superficial.

45) Un paciente con cefaleas, náuseas, diplopía y edema de papila se encuentra en:

a- La etapa descompensada del síndrome de hipertensión intracraneano


b- La etapa final del síndrome de hipertensión intracraneano
c- La etapa compensada del síndrome de hipertensión intracraneano
d- La etapa de las hernias encefálicas

46) Los tumores intracraneales intraxiales:

a- Son extracerebrales
b- Se separan fácilmente del parénquima cerebral
c- Suelen ser gliomas
d- Suelen ser meningiomas

47) ¿En qué tipo de hidrocefalia está contraindicada la punción lumbar?

a- En el pseudotumor cerebral
b- En la hidrocefalia comunicante
c- En la hidrocefalia no comunicante
d- En la hidrocefalia normotensiva

48) Durante el ayuno, el acetil-CoA producido en la oxidación de ácidos grasos es utilizado para la
síntesis de:

a- Glucosa y glucógeno en hígado


b- Acetoacetato e hidroxibutirato en tejido hepático
c- Piruvato y glucosa en hígado
d- Hidroximetilglutaril CoA y colesterol en tejidos extrahepáticos

49) En un paciente con maniobras de Barlow y Ortolani positivas Ud. sospecha:

a- Epifisiolisis de cadera.
b- Necrosis ósea avascular.
c- Coxa vara.
d- Displasia congénita de cadera.

50) En una paciente de 45 años de edad que presenta sinovitis en ambas muñecas, e videnciándose
en las radiografías disminución del espacio articular y osteopenia subcondral, ¿Qué patología sospecha?

a- Artritis Reumatoidea.
b- Artritis Séptica bilateral.
c- Esclerodermia.
d- Lupus Eritematoso Sistémico.

51) En una paciente de 80 años que se presenta en la guardia con dolor, rotación externa y
acortamiento del miembro inferior, Ud. sospecha una:

a- Fractura de cadera.

b- Luxación de cadera.
c- Fractura de fémur.
d- Luxación de rodilla.

52) Un joven de 22 años es llevado a la guardia. Sus padres informan que desde hace varios meses
se ha ido aislando de su entorno social, y hace unas semanas tuvo un enfrentamiento en su trabajo. En
la entrevista se muestra poco colaborador pero orientado en tiempo y espacio. Durante el
interrogatorio confiesa que sus compañeros están conspirando con su jefe para expulsarlo de su puesto
laboral. No se infiere riesgo inminente para sí o para terceros. Antecedentes de importancia: los padres
refieren que de chico era solitario, y su rendimiento escolar fue bajo. Tuvo algunas novias con las que se
peleaba por celos.

¿Cuál es el diagnóstico más probable y la conducta más adecuada?

a- Trastorno Bipolar. Interconsulta con psiquiatra de guardia para iniciar tratamiento

b- Trastorno de personalidad paranoide. Derivación a psiquiatría para seguimiento por


consultorios externos.

c- Esquizofrenia paranoide. Interconsulta con psiquiatra de guardia para iniciar tratamiento

d- Trastorno delirante de tipo persecutorio. Derivación a psiquiatría para seguimiento por


consultorios externos
53) Una mujer de 70 años, ama de casa, acude a la consulta acompañada por su esposo que
informa que hace unos meses se ha vuelto retraída, apática e irritable. Presenta dificultad para
acordarse de cosas que habían sucedido el día anterior y problemas para calcular el dinero al ir de
compras. A la evaluación está desorientada en tiempo y espacio, desprolija en su vestimenta, lúcida
pero notoriamente ansiosa y con dificultad para encontrar las palabras para expresarse. No se
detectaron alteraciones en el examen físico. Todos los análisis de laboratorio dieron negativos, pero una
tomografía computada mostró una marcada atrofia cortical y un Mini Mental Test de Folstein arrojó un
puntaje de 19. ¿Cuál es el diagnóstico más probable y la conducta más adecuada?

a- Deterioro cognitivo normal para la edad, seguimiento y reevaluación periódica para


monitorear la aparición de algún síntoma de enfermedad

b- Síndrome confusional agudo, tratamiento de su causa. Generar un ambiente familiar


para el paciente.

c- Síndrome demencial (compatible con


enfermedad de Alzheimer), inicio de tratamiento de los síntomas cognitivos y no cognitivos.
d- Cuadro depresivo, inicio de tratamiento con fármacos antidepresivos.

54) ¿Cuál de las siguientes opciones es una complicación posible ante una infección por virus de la
hepatitis A?

a- Evolución a la cronicidad.
b- Evolución a la cirrosis.
c-Falla hepática fulminante.
d-Hepatocarcinoma.

55) ¿Qué diagnóstico hace en un niño de tres años de edad que comienza con dolor en sus tobillos
al cual se agrega dolor abdominal y un exantema purpúrico en cintura y miembros inferiores?. En los
exámenes complementarios se observa proteinuria, hematuria, y e n la biopsia renal se muestra
proliferación mesangial y depósitos de Ig A por inmunofluorescencia.

a-Púrpura de Schonlein Henoch.


b-Granulomatosis de Wegener.
c- Displasia renal.
d- Sindrome de Alport.

56) En un niño de tres años con síndrome febril de e tiología desconocida, linfadenopatía cervical,
rash cutáneo eritematoso e inyección conjuntival, cuya biopsia ganglionar evidenció infartos en parches
asociados a trombos de fibrina en pequeños vasos

¿Qué diagnóstico sospecha?

a- Lupus eritematoso diseminado.


b- Enfermedad de Kawasaki.
c- Mononucleosis infecciosa.
d- Toxoplasmosis.

57) En un lactante de 6 meses con una tumoración intrarrenal macroscópicamente sólida y


amarillenta, constituida histológicamente por células fusiformes, acompañadas por islote s de cartílago y
focos de hemopoyesis. ¿Cuál es el diagnóstico?

a-Tumor rabdoide.
b-Sarcoma de células claras.
c- Tumor de Wilms.
d- Nefroma mesoblástico

58) ¿Qué virus se encuentra involucrado en etiología de la papilomatosis laríngea juvenil?

a- HPV 11 y 6.

b-Virus de Epstein Barr.


c- Herpes Virus 11.
d- Virus Sincicial Respiratorio.

59) En un niño de diez años que presenta a nivel de miembro superior derecho pápulas de hasta 3
mm planas de color carne agrupadas e histológicamente constituidas por infiltrado compuesto por
histiocitos y células gigantes aisladas, circunscripto a una papila dérmica. ¿Cuál es el diagnóstico más
probable?

a- Liquen nítido.
b- Liquen plano.
c- Xantogranuloma juvenil.
d- Pitiriasis liquenoide aguda

60) En una biopsia de piel que muestra una lesión vesiculoampollar intracórnea o subcórnea con
escasas células inflamatorias, ¿qué diagnóstico realizaría?

a- Impétigo.
b- Eritema tóxico neonatal.
c- Sindrome de la piel escaldada estafilocóccica.
d- Enfermedad de Hailey.

61) ¿Cuál de las siguientes dermatosis nutricionales se caracteriza por la malabsorción de Zinc?

a- Acrodermatitis enteropática.
b- Pelagra.
c- Enfermedad de Hartnup.
d- Frinodermia.

62) ¿Cuál es la complicación digestiva más frecuente en un Recién Nacido pretérmino de bajo peso
al nacer, que presenta letargia y manifestaciones de acidosis metabólica?

a- Atrofia vellositaria.
b- Vólvulo intestinal.
c-Enterocolitis necrotizante.
d-Reflujo gastroesofágico.

63) Un niño de 2 años, quien es portador de anemia falciforme presenta tumefacción simétrica de
las manos. El diagnóstico más probable es:

a- Osteomielitis
b- Artritis reumatoide pauciarticular
c- Síndrome mano pie (Dactilitis)
d- Artritis juvenil
64) Usted asiste a un niño de 6 años por presentar desde hace 24 horas un cuadro caracterizado
por dolor periumbilical. Posteriormente se agregan náuseas y anorexia y el dolor, que se ha
reagudizado, se localiza ahora en el cuadrante inferior derecho. Al examen físico el abdomen se halla
blando, depresible y doloroso en dicho cuadrante, con dolor a la descompresión y ruidos hidroaéreos
presentes. El diagnóstico más probable es:

a- Apendicitis aguda
b- Estreñimiento
c- Gastroenteritis vírica
d- Infección urinaria

65) Usted recibe un neonato con ictericia, dificultad para alimentarse, hipoglucemia y cataratas. El
diagnóstico más probable es:

a- Deficit de galactosinasa
b- Galactosemia
c- Deficit de piruvato dehidrogenasa
d- Enfermedad de Hurler

66) Un joven de 16 años llega a la consulta porque presenta vómitos, anorexia, astenia,
constipación y poliuria de instalación progresiva en las últimas semanas. En los exámenes de laboratorio
solicitados se destacan: calcio sérico 13 mg/dl; concentraciones elevadas de 25 -D < a 150ng/ml) e
hiperfosfatemia. Una ecografía renal muestra signos de hipercalcinosis. Surge del interrogatorio que ha
estado consumiendo suplementos vitamínicos diariamente durante los últimos siete meses, como parte
de un programa de entrenamiento físico intensivo. ¿Al exceso de qué vitaminas se pueden atribuir estos
hallazgos?

a- A
b- C
c- D
d- B1

67) El Screening para Streptococo beta agalactiae

a- Se realiza en forma universal entre las 35-36 semanas mediante un hisopado vaginal y perianal.

b- Se realiza en forma universal entre las 35-36 semanas mediante un hisopado de la cavidad oral
c- Se utiliza para prevenir una infección en la madre.

d- Se efectúa mediante un hisopado de la cavidad oral, para iniciar antibiótico terapia antes del trabajo
de parto.

68) La maduración pulmonar se realiza:

a- Entre la semana 24 y 34
b- Después de las 34 semanas
c- Antes de las 24 semanas
d- Entre las 22 y 24 semanas

69) El diagnostico de enfermedad de Chagas

a- Se realiza mediante la dupla serológica a saber HAI/IFI O HAI/EL ISA


b- Se realiza mediante la serología por el ELISA
c- Se realiza por la presencia del signo de romania
d- Se realiza por la presencia se manifestaciones cardiacas en pacientes infectadas.

70) En la Enfermedad hemolítica perinatal, la inmuno profilaxis se realiza en puérperas:

a- RH negativa que han tenido un recién nacido RH positivo


b- RH negativa que han tenido un recién nacido RH negativo
c- RH negativa que han tenido un recién nacido RH positivo con prueba de Coombs directa
positiva.
d- RH negativa con recién nacido RH positivo y prueba de Coombs directa negativa.

71) Señale la opción correcta en el manejo del hipertiroidismo durante el embarazo:

a- Seguimiento de concentración de T4 cada 2 a 4 semanas.

b- Realización de ecografía durante el primer trimestre del embarazo para evaluar


respuesta del tratamiento.

c- Se considera rango normal de T4 tres veces por encima del valor de T4 de la no


embarazada.

d- Dosaje de TSH cada 2 semanas.

72) Si se confirma sífilis en una embarazada por las pruebas serológicas pero nunca se evidenció la
clínica de la enfermedad y es de evolución desconocida

¿Cuántas dosis Ud. Le indicaría de Penicilina Benzatinica?

a- Una dosis
b- Dos dosis
c- Tres dosis
d- Ninguna pues debe ser un falso positivo.

73) ¿Cuál es el tratamiento de elección para la prevención de recurrencia de convulsiones para una
paciente que presenta Eclampsia?

a- Fenitoina
b- Coctel lítico
c- Sulfato de Magnesio
d- Diazepam

74) El sangrado masivo en Obstetricia se define como:

a-Descenso del hematocrito más de 10 puntos.

b-Descenso de la hemoglobina mayor a 8 gramos/dl. c- Pérdida brusca de 500 ml.


d- Pérdida de más del 30% de la volemia

75) ¿Cual de los siguientes gestágenos, usados en anticonceptivos orales, tiene menor efecto sobre
el metabolismo lipídico?
a) Medroxiprogesterona
b) Desogestrel
c) Clomifeno
d) Noretisterona

76) Como nuevas tecnologías se encuentra el anillo vaginal en anticoncepción. Considere la


correcta: Este anillo se coloca:

a-Durante 21 días y se retira una semana.


b-Antes de la relación sexual y luego se lo retira c- Durante 12 días, debiéndose reinsertar luego un
nuevo anillo.
d- Durante 7 días, debiéndose reinsertar luego un nuevo anillo.

77) Indique la opción correcta respecto de la mamografía:

a-Se debe realizar en el control del segundo trimestre del embarazo.


b-Sirve para diferenciar imágenes sólidas de quísticas.
c- Es de gran utilidad en las pacientes más jóvenes.
d- Se debe realizar entre los 35 y 40 años de edad una mamografía de base.

78)Las fístulas rectovaginales, son las más frecuentes de este tipo de fístulas e intervienen varios
factores.

¿Cuál es la correcta?

a- Las de origen actínico en general son tardías por necrosis del tabique rectovaginal.
b- Las de causa obstétrica se deben a una lesión quirúrgica.
c- Son sumamente raras y son casi siempre de naturaleza traumática.
d- Su localización en general es alta en la vagina o en la cúpula vaginal.

79) Se considera que el tratamiento inicial de una paciente con diagnóstico de cáncer de mama es
quirúrgico cuando:

a-No tiene metástasis a distancia


b-Tiene compromiso de piel
c- Tiene úlcera y eritema
d- Posee infiltración cutánea

80) La Neoplasia intraepitelial vaginal (VAIN) se caracteriza por cambios progresivos


intraepiteleales.

¿Cuál considera correcta?.

a- Presenta una incidencia de 0.2 al 0.3 por 100000 mujeres


b- Representa el 1% de las neoplasias intraepiteliales del tracto genital inferior
c- El 30% se encuentra asociado con otras neoplasias intraepiteliales
d- El 40% se relaciona con el papiloma virus humano (HPV)

81) Señale la opción correcta con respecto a la sífilis:

a- El periodo de incubación es de 9 a 120 días (promedio 31 días)


b- El periodo de incubación es de 9 a 60 días (promedio 21 días)
c- La vía de contagio es exclusivamente sexual
d- La sífilis latente aquella que se manifiesta por reacciones serológicas y por la prueba de campo
oscuro

82) El CA 19.9 es un marcador tumoral que se encuentra generalmente elevado en:

a- Carcinoma seroso del ovario.


b- Carcinoma de endometrio.
c-En cáncer digestivo y en menor medida en mama
d-Tumores de las células germinales.

83) El cáncer de vagina es un tumor poco frecuente, con una incidencia del 2% de los tumores
ginecológicos. Señale la correcta:

a- Epidemiológicamente son tumores que afectan a una edad entre 25 y 55 años.


b- Estadio I, carcinoma limitado a la pared vaginal.
c- Estadio III, carcinoma que compromete el tejido subvaginal y no se extiende a la pared pelviana.
d- El más frecuente es el adeno carcinoma de células claras.

84) Los tumores trofoblásticos gestacionales son raros en el mundo occidental, pero altamente
curables. Elija la opción correcta.

El Corion carcinoma es la forma histológica:

a- Maligna más frecuente, es muy quimiosensible.

b- Maligna menos frecuente y quimio-resistente.


c- Maligna más frecuente y es quimio-resistente.
d- Más rara.

85) La falla Ovárica Oculta afecta a un grupo de pacientes con insuficiente dotación folicular
ovárica, en presencia de ciclos menstruales regulares. ¿Cuál es la correcta?:

a- Se caracteriza por presentar FSH basales elevadas (mayor a 20 UI/ML) o estradiol superior a 75
pg/ml.

b- Se caracteriza por presentar FSH basales elevadas (mayor a 10 UI/ML) o estradiol de 30 pg/ml.

c- Se caracteriza por presentar FSH basales elevadas (mayor a 15 UI/ML) o estradiol superior a 75
pg/ml.

d- Hay una producción inadecuada de progesterona por parte del cuerpo amarillo, posiblemente
como consecuencia directa de una génesis folicular anormal

86) Los estrógenos son hormonas capaces de producir y mantener las características femeninas en
la mujer. Su administración produce:

a- Aumento de HDL y LDL colesterol


b- Aumento de HDL y disminución del LDL colesterol
c- Disminución del HDL y aumento del LDL colesterol
d- Disminución de los triglicéridos y del HDL colesterol
87) Las vías genitales se forman en la mujer a partir de los conductos de Muller. Considere la más
correcta:

a- Entre la 6º y 9º semana transcurre la etapa de la formación tubárica exclusivamente.


b- Entre la 6º y 9º semana transcurre la etapa de la formación tubárica, uterina y cérvico vaginal.
c- En la 8º semana se unen los conductos de Muller
d- En la 5º semana transcurre la etapa de la formación tubárica.

88) Se define a la endometriosis (EDT) como la presencia de glándulas endometriales y estroma


funcionante. Considere la correcta:

a- Las células T citotóxicas y las natural killer (NK, asesinas) actúan como segunda línea de defensa
del sistema inmune.
b- Las mujeres con EDT tienen una alteración en la actividad de los linfocitos B y anticuerpos
anormales.
c- Las células T citotóxicas no están involucradas en la histólisis directa de las células foráneas.
d- La reabsorción del tejido endometrial deportado por el flujo retrógrado no esta a cargo del
sistema inmune.

89) ¿Cuál es el tipo de defecto atribuible a glaucoma que se encuentra en el campo visual?

a- Cuadrantanopsia.
b- Escotoma arcuato.
c- Escotoma central.
d- Cuadrantopsia.

90) ¿Cuál es la causa de muerte en un cuadro de picadura por escorpión?

a- Deshidratación severa
b- Insuficiencia hepática
c- Insuficiencia renal
d- Shock cardiogénico

91) Si en el electrocardiograma de un paciente se observa un QRS isodifasico en D1, el eje cardiaco


en el plano frontal se encuentra en:

a- 0°
b- 30°
c- 60°
d- 90°

92) Si un enfermo presenta una deficiencia de los factores de la coagulación dependientes de la


vitamina K presentará:

a- Tiempo de sangría prolongado


b- Tiempo de sangría y tiempo de trombina prolongados
c- Tiempo de tromboplastina parcial activada (APTT o KPTT) y tiempo de protrombina (de Quick)
prolongados
d- Tiempo de sangría, KPTT y tiempo de Quick prolongados

93) Las lesiones talámicas se caracterizan por:


a- Afectación de la artería estriada lateral.
b- Producen un impacto importante en el hemicuerpo contralateral principalmente por afectación
de vías sensitivas
c- No afectar los sistemas motores ni circuitos cognitivos.
d- No comprometer las vías nociceptivas.

94) ¿De qué segmentos medulares se originan las raíces nerviosas motoras del nervio femoral?

a- L1, L2 Y L3.
b- L2, L3 Y L4.
c- L3, L4 Y S1.
d- L4, S1 Y S2.

95) Indique cuál es el origen de los tejidos embrionarios que forman parte del corazón:

a-Del mesodermo lateral de la placa cardiogénica y endodermo.


b-Unicamente del mesodermo lateral somático.
c- Del mesodermo lateral somático y septum transverso.
d- Del mesodermo lateral, crestas neurales y mesénquima extracardiaco del septum transverso.

96) ¿Cómo será la descendencia de una mujer afectada de enfermedad fibroquística del páncreas
(autosómica recesiva) con un varón sano no portador?:

a- El 100% de sus hijas mujeres serán portadoras


b- El 100% de sus nietas serán portadoras
c- El 25% de su descendencia estará afectada
d- El 25% de su descendencia será homocigota dominante

97) El principal componente de la barrera hemato- testicular está constituido por:

a- Endotelio capilar peritubular


b- Células de Sertoli
c- Membrana basal del tubo seminífero
d- Células mioides

98) El nucleosoma está compuesto por:

a- Una parte fibrilar densa y una parte granular periférica.


b- Ribonucleoproteínas ricas en uridina.
c- Por un octámero de histonas (H2A, H2B, H3 y H4) más ADN.
d- Un octámero de proteínas ácidas más ADN

99) La muestra más apropiada para el diagnóstico de la brucelosis es:

a- Orina para cultivo.


b- Sangre para hemocultivos y pruebas serológicas.
c- Materia fecal para cultivo y detección de toxinas.
d- Orina para detección de antígenos por aglutinación.

100) ¿Cuál de los siguientes patógenos puede atravesar la placenta y causar infección en el feto?
a- Listeria monocytogenes.
b- Streptococcus pneumoniae.
c- Bacillus cereus.
d- Bordetella parapertussis.
SEGUNDO EXAMEN ENAM
1.Paciente que acude por intento de suicidio tras dejar a su novia. Tenía problemas con el consumo de alcohol. ¿Qué tipo de
personalidad tiene?
a) Doble
b) Neurasténica
c) Adictivo
d) Histérica
e) Paranoica

2.¿Cuál es la probable causa de presbiacusia o sordera gradual que acompaña al envejecimiento?

a) Inflamación crónica del oído externo


b) Acumulación de líquido en el oído medio
c) Perforación de la membrana timpánica
d) Pérdida acumulada gradual de células ciliares
e) Otoesclerosis

3.Una paciente de 34 años de edad presenta un cuadro de cirrosis descompensada con ascitis y encefalopatía. Se detecta virus de la
hepatitis C y un hepatocarcinoma de 3 cm en el lóbulo hepático derecho. ¿Cuál es el tratamiento de elección?

a) Resección del lóbulo hepático erecho


b) Resección limitada del tumor
c) Interferón
d) Trasplante hepático
e) Derivación porto-sistémica

4.Una mujer de 23 años acude a consulta por irregularidades menstruales e infertilidad. Su evaluación clínica es completament e normal.
Sus exámenes bioquímicos muestran una reducción en los niveles de estradiol y hormona folículo estimulante (FSH). Se le diagn ostica
un hipogonadismo central. ¿Cuál de las siguientes es la función más importante de la FSH?

a) Provocar ovulación
b) Estimular la secreción de progesterona c) Provocar la fase secretoria urinaria
d) Inhibir la secreción de estrógenos e) Estimular la maduración del folículo

5.Un varón de 54 años acude a urgencias por dolor severo en el dedo gordo del pie derecho. En el pasado había tenido episodios
similares de menor gravedad, que siempre trataba con analgésicos. El dedo esta enrojecido, inflamado y muy sensible a cualquier
movimiento. Una aspiración mediante aguja confirma la presencia de cristales de ácido úrico y se le trata con indometacina (AINE) oral.
Un mes más tarde continua libre de síntomas y se le recomienda alopurinol como tratamiento
preventivo ¿Cuál de los siguientes mecanismos de acción corresponde con el del alopurinol?

a) Inhibición de la xantina oxidasa


b) Solubilización del ácido úrico
c) Reactividad con hipoxantina
d) Efecto antiinflamatorio sobre el tejido articular
e) Secreción tubular aumentada de ácido úrico

6.El cuadro más frecuente en el adulto mayor es:

a) Psicosis aranoide
b) Neurosis de ansiedad
c) Depresión
d) Alzheimer
e) Ninguna de las alternativas es correcta.

7.Según el proceso de Reforma en Salud del MINSA. ¿Qué se prioriza en la organización de los servicios de salud?

a) Asistencia y recuperación
b) Detección y tratamiento precoz
c) Promoción y prevención
d) Rehabilitación
e) Recuperación y rehabilitación
8.Escolar de 6 años presenta hace dos días fiebre de 38.5°C, hiporexia, malestar general, dolor en hipocondrio derecho y náus eas. Se
ha reportado 4 casos similares en su aula. No tiene antecedentes patológicos de importancia. Al examen: activo, hemodinárnicamente
estable, hígado a 4cm DRCD y doloroso a la palpación. Resto del examen sin alteración. ¿Cuál es el diagnóstico más probable?

a) Hepatitis C
b) Colecistitis Aguda
c) Hepatitis B
d) Pancreatitís
e) Hepatitis A

9.En relación a la infección por VIH, marque la respuesta CORRECTA:

a) En el periodo asintomático no hay producción viral.


b) La carga viral indica el pronóstico del paciente y el recuento de CD4 es el marcador inmunológico.
c) La quimioprofil axis con cotrimoxazol protege contra Cryptosporidium y Pneumocystis carinii Isoniazida se indica como prof ilaxis
siempre y cuando el paciente tenga CD4 menor de 200 x mm3
d) Las infecciones oportunistas se presentan
e)solamente con CD4 menor de 200 x mm3

10.Lactante de 11 meses, inicia con vómitos, fiebre de 38,2°C e irritabilidad. Se agregan deposiciones líquidas sin moco ni s angre. Al
examen: mucosas secas, ojos hundidos y llenado capilar de 2 segundos ¿Cuál es el estado de hidratación?

a) Hidratado
b) Deshidratación leve
c) Deshidratación severa sin shock
d) Deshidratación moderada.
e) Deshidratación severa con shock

11.¿Qué vacuna está contraindicada en un niño alérgico a la ovoalbúmina?

a) Vacuna pentavalente
b) BCG
c) Vacuna triple SPR d) Vacuna HVB
e) Vacuna contra el neumococo

12.Niño de 9 meses de edad que tiene un peso de 10kg ,cabello despigmentado, edema en miembros, queilitis y eccema ¿Cuál es el
diagnóstico más probable?

a)Marasmo
b) Eutrófico
c) Desnutrición calórica-proteica
d) Kwashiorkor
e) Con bajo peso

13.Adolescente de 12 años, acude a consulta por desplazamiento del pezón y la areola y ausencia del pectoral mayor y menor. ¿ Cuál
es el diagnóstico más probable?

a)Distrofia muscular b)Miopatia tiroidea c)Escápula alada d)Síndrome de Poland e)Síndrome de Turner

15¿Cómo se denomina el enanismo en niños originado por el maltrato y el descuido crónico?

a)Psico-social b)Síndrome de Laron c)Hipofisiario d)Acondroplásíco e)Cretinismo

15.Si al evaluar a un neonato durante la atención inmediata encuentra: respiraciones espontáneas, frecuencia cardiaca de 80 p or minuto
y acrocianosis. Luego de los pasos iniciales, usted ¿qué indicaría?:

a)Bolseo (VVP), masaje cardiaco e intubación


b)Adrenalina, expansores de volumen y bicarbonato. C)Bolseo (VVP)
d)Intubación y masaje cardiaco e)Masaje cardiac

16.¿Cuál es el tratamiento más adecuado en un paciente con hipokalemia severa?:

a)Administrar K por vía oral


b)Reposición de K alrededor de 20-40 mEq/por hora infusión por vía central Utilizar c)cloruro de potasio al 2% en forma rápida
d)Tratar la causa que originó la hipokalemia e)Administrar bicarbonato de sodio al 8.4%

17.El signo de Grey Turner que aparece en pancreatitis aguda hemorrágica consiste en equimosis en:

a)Fosas iliacas
b)Ángulos costovertebrales c)Zona periumbilical d)Cuello
e)Apéndice xifoides

18.Lo primero que debe realizarse en un paciente con sospecha de cáncer del tercio medio recto es:

a)Tacto rectal
b)Proctoscopia
c)Proctosigmoidoscopia
d)Colonoscopia
e)Rx contrastada de colon

19.El tumor más frecuente del epiplón es:

a)Carcinoma metastásico b)Fibroma


c)Lipoma d)Leiomiosarcoma e)Neurofibroma

20.La operación de elección para la estenosis pilórica congénita es:

a)Gastroyeyunostomía b)Gastroduodenostomía c)Piloroplastía d)Pilorotomía e)Antrectomía

21.Adolescente de 17 años, acude a consulta 3 meses después de sufrir abuso sexual y robo de sus pertenencias, por recuerdos
recurrentes del episodio referido, le produce angustia y depresión. ¿Cuál es tipo de trastorno que presenta?

a)Estrés post traumático agudo b)Pánico


c)Mixto ansioso depresivo d)Obsesivo compulsivo
e)Estrés Post traumatico crónico

22.Un centro de salud recibe un ecógrafo remitido por la Red de Salud. ¿En qué documento de gestión debe incluirse la actividad para
poner en funcionamiento el ecógrafo?

a)Plan de mantenimiento b)Plan operativo institucional c)Plan anual de adquisiciones


d)Reglamento de organización y funciones e)Manual de organización y funciones

23.Varón de 30 años sufre accidente de tránsito. Al examen: herida


contuso cortante en región parietal derecha, escoriaciones en tórax y abdomen, equimosis en flanco derecho, fr actura de clavícula
derecha y fémur izquierdo. Flexión patológica del cuello, apertura ocular al provocarle dolor y no emite sonidos, murmullo ve sicular no
pasa en HTD, matidez a la percusión; abdomen: distendido, RHA presentes. ¿Cuál es la conducta a seguir?

a)Dos vías endovenosas gruesas y cortas b)Colocar tubo de drenaje torácico c)Colocación de tubo endotraqueal d)Lavado peritoneai
diagnóstico e)Tomografía cerebral

24.Varón de 58 años, acude por presentar ortópnea, edema de miembros inferiores y disn ea. Se hace el diagnostico de falla cardíaca
crónica descompensada. ¿Cuál de los siguientes fármacos utilizados en el tratamiento, tiene como mecanismo de acción ser anta gonista
aldosterona?

a)Espironolactona b)Valsartan c)Bisoprolol d)Enalapril e)Furosemida

25.Fractura de la tibia con dolor y disminución de pulso pedio ¿Cuál es el diagnóstico más probable?

a)Síndrome compartimental b)Hematoma de pierna c)Lesión de nervio tibial


d)Embolia grasa
e)Trombosis venosa profunda

26.¿Qué vacuna está contraindicada en gestantes?

a)Hepatitis B b)Influenza c)Paperas d)Meningocócica e)Rabia

27.¿Cuál es la reacción adversa más frecuente en el tratamiento con heparina?


a)Leucopenia b)Agranulocitosis c)Trombocitopenia d)Anemia aplásica e)Reacción leucemoide

28.Paciente con dolor abdominal crónico tipo cólico e ictericia . Exámenes de laboratorio: BT: 10mg/ dl ¿Cuál es el diagnósti co más
probable?

a)Pancreatitis aguda b)Colelitiasis


c)Cáncer de páncreas d)Coledocolitiasis e)Tumor de Klatskin

29.En hiperkalemia con cambios electrocardiográficos ¿Cuál es el tratamiento inicial?

a)Bicarbonato IV b)Hemodiálisis c)Gluconato de calcio d)Insulina


e)B-2 agonistas

30.Señale el factor de riesgo predisponente para IMA según el Estudio Framingham:

a)Obesidad
b)Sexo masculino
c)Historia familiar de IMA d)HTA sistémica e)Dislipidemia

31.La evaluación de la permeabilidad de la trompa de Eustaquio, se realiza mediante:

a)Maniobra de Valsalva. B)Maniobra de Toynbee c)Catecismo con sonda de YTAR


d)Todas las alternativas son correctas e)Ninguna de las alternativas

32.¿En cuál de los siguientes casos está indicada la profilaxis antibiótica?

a)Apendicitis aguda con peritonitis localizada b)Apendicitis aguda con peritonitis generalizada c)Plastrón apendicular
d)Apendicitis aguda no complicada e)Absceso apendicular

33.Ictericia neonatal a predominio directo progresiva, BT: 31 mg/ dl ¿Cuál es el diagnóstico más probable?

a)Atresia biliar Anemia hemolítica


b)Hipoplasia de vías biliares c)Quiste del colédoco
d)Estenosis de los conductos biliares

34.Para lograr que se consolide una fractura, debemos lograr la:


a)Analgesia b)Estabilidad c)Inestabilidad
d)Todas las alternativas. E)Ninguna de las alternativas.

35.Varón de 60 años, sufre atropello hace 15 minutos. Al examen: FC: 140 lat/min PA: 60/40 mm Hg, frialdad distal y dolor en
hemiabdomen inferior. Ecografía: liquido libre en regular cantidad en fondo de saco rectovesical, ¿Cuál es la conducta a seguir?

a)Laparotomía exploratoria b)TAC de abdomen


c)Ecografía doppler
d)Lavado peritoneal e)Paracentesis

36.Lactante que no quiere su papilla ¿cuál debe ser la conducta?

a)Utilizar endulzantes artificiales b)Agregarle sal a la papilla


c)Darle solo puré de fruta d)Fraccionar las tomas de papillas. E)Agregar condimentos naturales

37.Pelvis con conjugado diagonal de 10.5 o sea el obstétrico queda en 9cm.


¿De qué tipo de pelvis se trata?

a)Pelvis normal
b)Estrechez pélvica
c)Pelvis estrecha no viable d)Pelvis ginecoide
e)Pelvis platipeloide
38.¿Cuál de las siguientes medidas, produce el mayor impacto en la disminución de las infecciones intrahospitalarias?

a)Uso de alcohol gel antes de examinar al paciente b)Profilaxis antibiótica a los pacientes
c)Lavado de manos del personal de salud d)Uso de máscara N - 95
e)Uso de mascarilla quirúrgica

39.A qué edad recomienda la American Academy of Pediatrics que se detecte la hipoacusia?

a)Al nacer
b)3 meses
c)2 años
d)4-5 años
e)12 años

40.Niño de 3 años, afebril, con lesiones papuloequimoticas en miembros superiores e inferiores, simétricas, edema, miembros inferiores
y dolor abdominal, no visceromegalia, ¿cuál sería su diagnóstico clínico?

a)Purpura trombocitopénica b)Púrpura de Henoch-Schönlein c)Leucemia infantil


d)Linfoma
e)Ninguna de las anteriores

41.De las siguientes afirmaciones, una no es cierta con respecto a infección urinaria en niños:
a)El cuadro clínico depende en parte de la edad, localización e intensidad de la infección
b) La E. coli es el agente etiológico más frecuente
c)Las recurrencias son raras
d)A menor edad mayor tendencia a formar cicatrices renales
e) Ninguna alternativa es correcta.

42.¿Cuál es la anomalía del aparato genital femenino causada por el progreso caudal insuficiente de los conductos paramesonéf rlicos?

a) Hipertrofia de clítoris
b) Tabique vaginal
c) Agenesia de la vagina d) Himen perforado e) Aplasia uterina

43.La bronquitis en los primeros meses de vida puede dejar secuelas. Marque usted lo correcto:

a)Bronquiectasia b)Sobreinfección bacteriana


c)Desarrollo de asma en algunos niños d)Todas de las alternativas son correctas. E)Ninguna de las alternativas es correcta.

44.La meningoencefalitis bacteriana en recién nacido es más frecuentemente producida por:


a)Haemophilus influenzae b)Bacterias gram negativas
c)Streptococcus pneumoniae
d)Todas de las alternativas son correctas. e)Ninguna de las alternativas es correcta.

45.Pre-escolar de 3 años, con calendario de vacunación incompleto. Presenta rinorrea blanquecina. Al examen: T»: 37.8°C, erupción
macular generalizada con adenopatías retroauriculares y occipitales. ¿Cuál es el diagnóstico?
a)Rubeola b)Urticaria c)Poséola d)Varicela e)Sarampión

46.En salud comunitaria, ¿cuál es la estrategia para el trabajo en zonas urbanas y rurales?
a)Reunión vecinal
b)Asamblea de padres de familia c)Visita domiciliaría
d)Reunión de junta directiva e)Feria comunal

47.¿Cuál de los siguientes signos es un hallazgo tardío en el cáncer de mama?


a)Tumor palpable
b)Retracción reciente del pezón c)Oscurecimiento de la areola d)Asimetría del tamaño de las mamas e)Secreción sanguinolenta por el
pezón

48.Gestante de 32 semanas, preeclámptica con manejo expectante, presenta convulsiones. Luego de tratamiento con sulfato de
magnesio presenta paro cardiorespiratorio. ¿Cuál es el antídoto?
a)Bicarbonato de sodio b)Permanganato de potasio c)Nitroprusiato de sodio d)Gluconato de calcio e)Carbonato sódico

49.¿Qué vía tiene menor biodisponibilidad? :


a)Oral b)Intramuscular
c)Subcutánea d)Intradérmica e)Endovenosa

50.El embarazo molar ocurre por la unión de : a)Espermatozoide anucleado y óvulo normal b)Espermatozoide bicéfalo y óvu lo normal
c)Espermatozoide normal y ovulo normal d)Espermatozoide sin cola y óvulo anucleado e)Espermatozoide ndmial y óvulo anucleado

51.El prolapso uterino de II grado según POPQ es cuando el punto más declive se encuentra:
a)A dos centímetros distales de la carúncula hymeneal
b) Más de un centímetro por debajo del nivel del himen
c)A dos centímetros proximales de la carúncula himenea!
d)Entre el centímetro p revio al nivel himeneal y el centimetro distal a este
e)Sobrepasando a 3 centímetros del introito

52.Mujer con antecedente de hemorragia obstétrica que no pudo dar de lactar, actualmente con astenia e hipotensión ¿Cuál es el
diagnóstico más probable?
a)Hipotiroidismo
b)Hipogonadismo
c)Hipopituitarismo
d)Insuficiencia suprarrenal
e)Hipoparatiroidismo

53.Paciente con pulso arrítmico y deficitario. ¿A qué enfermedad corresponde?


a)Taquicardia supraventricular
b)Fibrilación auricular
c)Taquicardia ventricular
d)Fibrilación ventricular
e)Bloqueo AV

54.En relación a las hemorragias del puerperio inmediato, marque la combinación de respuestas verdaderas (V) o falsas (F) que
corresponda:
1. La hemorragia del puerperio inmediato es la principal causa de muerte materna en el Perú.
2. El pujo antes de la dilatación completa puede condicionar desgarros cervicales
3. La edad muy joven es la principal causa de Placenta Previa
3. El parto instrumentado puede condicionar laceraciones perineo vaginales
4. La versión interna y extracción podálica pueden condicionar ruptura uterina

a) F,V,F,V,V
b) V,F,F,V,V
c) F,V,V,F,F
d) V,V,V,V,V
e) V,V,F,V,V

55.Una medida que puede prevenir la hemorragia y formación de hematomas al suturar un desgarro perineo vaginal, cervical o
episiotomía es la siguiente.
a)Poner suturas interrumpidas e intervalos de 0.3 cms
b)Usar sutura crómico 1
c)Usar aguja redonda en vez de cortante
d)Hacer sutura continua
e)Poner el primer punto a 0.5 a 1 cm, por encima del borde superior del desgarro o episiotomía

56.Pre-escolar de 3 años es llevado a consulta con dermatitis, enteritis, alopecia y retraso del crecimiento. ¿Cuál es la probable
deficiencia de vitamina que presenta?
a)Ácido antoténico
b)Tiamina
c)Cianocobalamina
d)Ácido fálico
e)Niacina

57.Mujer de 32 años, que hace 2 días presenta malestar general y sensación febril. Se agrega cefalea intensa, vómitos, compro miso de
sensorio y 3 episodios de convulsiones. Al examen: Glasgow 10, rigidez de nuca, T°: 38,9 °C ¿Cuál es el diagnóstico más probable?
a)Meningoencefalitis aguda bacteriana b)Meningitis vira!
c)Meningoencefalitis tuberculosa d)Meningoencefalitis herpética e)Sindrome convulsivo refractario

58.Mujer de 42 años, desde hace 2 meses presenta tos, episodios de disnea y pérdida de 5 Kg. Al examen: 38°C, adelgazada,
crepitantes en vértice derecho y algunos roncantes en ambos campos pulmonares, ¿Cuál es el diagnóstico más probable?
a)Neumonía crónica b)Bronquitis crónica c)Tuberculosis pulmonar d)Asma bronquial
e)Enfermedad pulmonar intersticial

59.Recién nacido de 33 semanas, hospitalizado en la UCI neonata] por 1 semana, las últimas 24 horas presenta hipotermia, dist ensión
abdominal, hipoactividad marcada y disminución de la presión arterial. Laboratorio: leucocitos 23,000 x campo y PCR: 1.5 mg/L, ¿Cuál
es la conducta a seguir?
a)Repetir el hemograma antes de iniciar antibióticoterapia b)Tomar hemocultivo y estudio de líquido cefalorraquídeo c)Tomar
hemocultivo e iniciar antibióticoterapia
d)Repetir el hemograma y PCR e iniciar antibióticoterapia.
e)Estudio del líquido cefalorraquídeo e inicio de antibióticoterapia

60.Se denomina afaquia a:


a)Presencia de pus en cámara anterior
b)Estado normal de refracción
c)Presencia de dos cristalinos
d)Ausencia de cristalino
e)Sinequia posterior del iris

61.Preescolar de 2 años previamente sano, presenta puntitos morados en cara, miembros inferiores tronco, niega otras molestias,
Al examen: petequias diseminadas.
¿Cuál es el diagnóstico más probable?
a)Anemia aplásica
b)Púrpura de Henoch-Schónlein
c)Púrpura trombocitopenica idiopática
d)Síndrome de Kasabach-Merritt.
e)Leucemia linfoblástlea.

62.Varón de 60 años, fuma 15 cigarrillos al día desde los 25 arios de edad. Hace un mes presenta expectoración hernoptoica escasa.
Radiografía de tórax:
nódulo paratraqueal derecho de 3cm de diámetro. Biopsia por broncoscopia: Ca broncogénico, Estadiaje: T1, NO; MO. ¿Cuál es el
tratamiento?
a)Radioterapia
b)Quimioterapia
c)Radioterapia y Quimioterapia
d)Cirugía y radioterapia
e)Cirugía

63.Mujer de 25 años sexualmente activa que acude por disuria, polaquiuria y tenesmo vesical. Al examen: T°: 38.5°C dolor en e l flanco
derecho y PPL derecho positivo. ¿Cuál es el germen más probable de infección?
a)Proteus mirabilis
b)Klebsielia sp
c)Enterococo sp
d)Estreptococo
e)Escherichia coli

64.A los niños de un colegio pre-escolar se les realiza un test de Graharn para detectar oxiurasis. Si el resultado es negativo, al día
siguiente se les repite la prueba. ¿Qué ocurre en esta secuencia?
a)Disminuye la sensibilidad y aumenta la especificidad b)Aumenta la sensibilidad y aumenta la especificidad c)Disminuye la sensibilidad
y disminuye la especificidad d)Aumenta la sensibilidad y disminuye la especificidad
e)No hay variación de la sensibilidad ni de la especificidad

65.Gestante multípara que en las últimas cinco semanas se despierta y encuentra que se ha producido una hemorragia vaginal, s u parto
anterior fue normal, no presenta dolor ni contracciones. Con el síndrome expuesto, el diagnostico que usted haría en este caso es:
a)Ruptura del seno marginal
b)Desprendimiento de placenta
c)Placenta previa
d)Trabajo de parto inminente
e)Carcinoma cervical

66.La incontinencia urinaria de urgencia en la mujer post-menopausica se debe:


a)Pérdida del ángulo uretra vesical posterior
b)Aumento del ángulo uretra vesical posterior
c)Trigonitis senil
d)Infecciones crónicas inevitables de vías urinarias que ocurren en mujeres post -menopaúsicas
e)Pérdida de la inervación del cuello vesical

67.Mujer de 33 años presenta lesiones dérmicas en rostro que aumentan por exposición al sol, dolor articular migratorio y convulsiones.
Antecedente de 2 abortos. Laboratorio: anemia, plaquetopenia, ANA (+), Rx. De tórax: derrame pleural leve bilateral, ¿Cuál es el
diagnóstico?
a)Espóndiloartropatía seronegativa
b)Lupus eritematoso sistémico
c)Esclerosis sistémica progresiva
d)Polimiositis
e)Artritis reumatoide

68.Gestante adolescente de 32 semanas por FUR, acude a Emergencia por cefalea frontal persistente desde hace 3 días. Al Examen:
PA: 160/110 mm Hg. Anasarca e hiperreflexia. ¿Cuál es el tratamiento indicado para el control de la hipertensión?:
a)Nifedipino
b)Furosemida
c)Clorotiazida
d)Captopril
e)Verapamilo

69.Paciente gestante de 22 semanas, presento 3 abortos a las 12, 13, 14 semanas, en la gestación actual se procedió a realizar una
operación cervical con el objeto de llevar adelante el embarazo:
a)Amenaza de aborto b)Aborto incompleto c)Aborto frustro d)Aborto inevitable e)Aborto habitual

70.La gestante presenta:


a)Falso trabajo de parto
b)Trabajo de parto en fase activa
c)Trabajo de parto en fase de desaceleración
d)Trabajo de parto en fase latente
e)Trabajo de parto en fase acelerada

71.Una mujer de 69 años presenta una pigmentación oscura aterciopelada en las axilas. Había perdido unos 5 kg de peso en los últimos
tres meses y se quejaba de acidez y saciedad precoz. No tenía otros síntomas. ¿Cuál de las siguientes enfermedades debe investigarse?
a)Carcinoma en algún órgano
b)Linfoma
c)Diabetes mellitus
d)Sarcoidosis
e)Alergia

72.La detección de la proporción de obesos en una fecha determinada (18 -11-2012) es:
a)Prevalencia
b)Incidencia
c)Incidencia acumulada
d)Riesgo atribuible
e)Riesgo relativo

73.¿Cuál es la situación de la famiIia que con relativa frecuencia se da en la etapa del adulto mayor?
a)Formación
b)Expansión
c)Contracción
d)Disolución
e)Comprensión

74.Gestante de 8 semanas, acude a su control prenatal. No refiere síntomas o problemas al orinar. Trae resultado: urocuitivo: coli + de
100,000 UFC/ml, sensible a todos los antibióticos. ¿Cuál de los siguientes indicaría?
a)Doxiciclina b)Rifampicina
c)Trimetropin y Sulfametoxasol d)Cefuroxima
e)Ciprofloxacina
75.Varón de 33 años, presenta cefalea intensa luego de esfuerzo físico, casi inmediatamente pierde el sensorio, cayendo al suelo. Al
examen: PA: 120/80 mm Hg. Glasgow 8, sin compromiso motor, rigidez de nuca (+), signo de Kerning (+), ¿Cuál es el diagnóstico ?
a)Hematoma subdural agudo b)Hemorragia subaracnoidea c)DCV isquemico embólico d)DCV isquémico gigante e)Neurocisticercosis

76.Postmenopáusica con sangrado genital de útero. ¿Cuál es la causa más frecuente ?:


a)Miomatosis uterina b)Cáncer endometrial c)Cáncer de ovario d)Quiste ovárico e)Endometriosis

77.Posta con mal saneamiento y no hay protocolo para atención , en el análisis FODA tenemos:
a)Debilidad más amenaza b)Solo amenaza
c)Solo debilidad d)Oportunidad más amenaza e)Debilidad más oportunidad

78.La amenorrea primaria es característica de todas las siguientes condiciones, EXCEPTO:


a)Resistencia a las Gonadotropinas b)Síndrome de Turner
c)Disgenesia Gonadal Pura d)Síndrome de Ovario Poliquístico e)Síndrome de Rokitansky

79.Gestante que presenta cefalea, PA >160/ 105mmHg y albuminuria +++ ¿Cuál es el diagnóstico más probable ?
a)Preeclampsia severa b)Preeclampsia leve
c)Hipertensión gestacional severa d)Hipertensión gestacional leve e)Preeclampsia sobreagregada

80.Niña que no menstrua, presenta dolor abdominal cíclico. ¿Cuál es el diagnóstico?


a)Hipogonadismo b)Hipotiroidismo c)Síndrome de Turner d)Himen imperforado e)Ovario poliquístico

81.Mujer de 45 años presenta dolor abdominal episódico y refiere, que el dolor aumenta posterior mente a la ingestión de comidas ricas
en grasas. ¿La acción de cuál de las siguientes hormonas es responsable de la intensificación de sus síntomas?:
a)Gastrina b)Secretina
c)Colecistocinina (CCK) d)Pepsina
e)Somatostat ina

82.¿Cuál de los siguientes neurotransmisores produce el ingreso de los iones cloro, en la membrana post sin áptica?:
a)Glutamato b)GABA
c)Aspartato d)Serotonina e)Noradrenalin

83.En la fórmula del perfil vaginal la 3era cifra corresponde a:


a)Uretrocele b)Enterocele c)Rectocele d)Histerocele e)Cistocele

84.En la mujer climatérica la dislipidemia más frecuente es:


a)Aumento colesterol total b)Aumento de HDL c)Aumento de LDL d)Descanso de triglicéridos e)Todas las anteriores

85.En el manejo de la hemorragia post parto se consideran necesario las siguientes medidas generales, excepto:
a)Abrir vía endovenosa con bránula N° 18. b)Administrar cloruro de sodio + oxitocina c)Laparotomía exploratoria inmediat a d)Mantener
vías aéreas permeables e)Cateterizar vejiga

86.El diagnostico de trabajo de parto incluye todo, Excepto:


a)Confirmación del trabajo de parto b)Periodo y fase del trabajo de parto c)Encajamiento y descensos
d)Identificación de la presentación y posición de la cabeza fetal e)Tiempo transcurrido entre el inicio y la hora de la evolu ción

87.Un aborto frustro, la complicación que se asocia con mayor frecue ncia es:
a)Hiperhemesis gravídica b)Sinequias intrauterinas c)Coagulopatía por consumo d)Endometritis
e)Ninguno de los anteriores

88.Lactante de 18 meses de edad, peso 11.8 kg, asintomático, hace 24 días que recibe la visita de su tío, sintomático respiratorio,
evaluado en el hospital con DX TBC pulmonar BK ++. ¿Cuál es la conducta con el lactante?
a)PPD
b)BK con sonda nasogástrica c)INH medio comprimido diario d)Rx tórax
e)Todas las alternativas son correctas.

89.Varón de 29 años acude con múltiples ganglios cervicales, fiebre y dolor de garganta. Por factores de riesgo que mencionó el paciente
se sospecha de infección por el VIH.
¿Qué examen es útil si pensamos que presenta un síndrome retrovirai agudo?
a)Elisa-VIH b)Western Blot c)Recuento de CD4 d)IFI-VIH
e)Carga Viral
90.A cualquier atributo o circunstancia de una persona o de la comunidad asociado al aumento de la probabilidad de que desarrolle un
proceso mórbido, se le denomina:
a)Riesgo atribuible b)Riesgo relativo c)Prevalencia d)Factor de e)riesgo Incidencia

91.Todo fenómeno de salud de poblaciones humanas está biológica y ……………….. determinado:


a)Genéticamente b)Culturalmente c)Hereditariamente d)Socialmente
e)Ninguna de las alternativas.

92.Varón de 19 años acude por dolor abdominal en rnesogastrio que luego se localiza en FiD, náuseas y vómitos en 2 oportunidades e
hiporexia A la palpación dolor en FID, este signo se denomina
a)Dunphy b) Mc Burney C)Rovsing d)Blumbere)Rebote

93.Escolar de 6 años, presenta desde los 11 meses, episodios recurrentes de erupción papulovesicular, eritematosa y pruriginosa en
cuero cabelludo y región retroauricular, Antecedente de rinitis intermitente. Al examen: piel xerótica con zonas de liquenificación. ¿Qué
tratamiento le indicaría?
a)Jabón de glicerina y antihistamínicos b)Corticoide sistémico y antihistamínícos c)Crema humectante y antibió ticos d)CorticoIde tópico
y antihistamínicos e)Tacrolimus y ciclosporina A

94.La oferta eficaz y sistematizada de servicios básicos de salud que sa tisfagan las necesidades de la población se denomina:
a)Cobertura b)Seguridad social c)Demanda
d)Servicios básicos de salud (SBS) e)Necesidad de salud

95.Un niño presenta meningoencefalitis por meningococo, él estuvo en contacto íntimo con sus 2 hermanitos a quienes se les tuvo que
dar medicamentos para evitar la presencia de la enfermedad, esta acción a qué medida de prevención corresponde:
a)Promoción b)Primordial c)Protección
d)Tratamiento oportuno e)Diagnóstico precoz

96.Un estudio observacional en el que se comparan 110 individuos con cáncer de senos paranasales con 220 individuos sin la
enfermedad, corresponde a un estudio: Longitudinal
a)Caso-control b)Cohortes concurrente c)Transversal
d)Cohortes no concurrente

97.La presencia continua de una enfermedad causada por un grande infeccioso en una zona geográfica determinada se denomina:
a)Holoendemia b)Epidemia c)Brote d)Hiperendemia e)Endemia

98.En la prueba de hipótesis, el investigador comete un error tipo II o beta cuando:


a)No establece el nivel de significancia
b)No rechaza la hipótesis nula, siendo falsa c)Rechaza la hipótesis alterna, siendo falsa d)Rechaza la hipótesis alterna, sie ndo verdadera
e)Rechaza la hipótesis nula, siendo verdadera

99.La medida de validez de una prueba diagnóstica que evalúa “la probabilidad de que el paciente tenga la enfermedad con su resultado
positivo de la prueba evaluada”, se denomina:
a)Eficacia diagnostica b)Especificidad c)Sensibilidad
d)Valor predictivo negativo e)Valor predictivo positivo

100.Los principios éticos para la investigación en seres humanos están definidos por la:
a)Declaración de alma – Ata b)Declaración de Helsinki c)Declaración de OMS
d)Declaración de las naciones unidas
e)Declaración universal de los derechos humanos

101.La medición de asociaciones en epidemiologia utiliza:


a)El riesgo atribuible ocupacional
b)El riesgo atribuible
c)El riesgo diferencial
d)El riesgo relativo
e)La fracción etiológica de riesgo

102.¿Cuál de los siguientes signos demuestra ovulación en la mujer?:


a)Aumento de la viscosidad del moco cervical b)Desaparición de células cornificadas c)Aumento de 1°C de temperatura corporal
d)Desaparición del glucógeno
e)Todas muestran ovulación

103.En un paciente deshidratado cuál de las siguientes hormonas esta disminuida:


a)Antidiurética b)Aldosterona c)Angiotensina II
d)Factor natriurético atrial e)Ninguna

104.La articulación radiocarpiana es una artculación del tipo:


a)Enartrosis
b)Artrodia c)Condilea d)Tróclea e)Trocoide

105.La vena que pasa por el canal deltopectoral es:


a)V. Humeral
b)V.Intercostac)l V. Cefálica
d)V. Basílica
e)V. Circunfleja

106.Es el sustrato utilizado para sintetizar serotonina:


a)Acetil CoA b)Serina c)Tirosina d)Fenilalanina e)Triptófano

107.Las glándulas que destruyen parte de su citoplasma para formar la secreción glandular se llaman:
a)Holocrinab)Apocrinac)Merócrinad)Endocrinae)Africrinas

108.Fractura de maxilar y unguis con ojo rotado en medial y aducido. Indique cuál es la lesión:
a)Lesión de músculo recto interno b)Lesión de músculo recto superior c)Lesión de músculo recto externo d)Lesión de músculo recto
inferior e)Lesión de músculo oblícuo menor

109.Niño con fiebre, inyección conjuntival, lesiones descamativas en las manos y adenopatía s ¿Cuál es el diagnóstico más probable?
a)Celulitis
b) Carbunco
c)Pioderma gangrenoso d)Enfermedad de Kawasaki e)Enfermedad de Lyme

110.Mujer con lesiones vesiculares en los labios mayores asociadas a adenopatías inguinales. ¿Cuál es el diagnóstico más probable?
a)Herpes Zoster
b)Chancroide c)Herpes genital d)Gonorrea
e) Sífilis

111.¿Cuál es la hernia que sale por el triángulo de Hasselbach?


a)Hernia inguinal indirecta b)Hernia inguinal directa c)Hernia de Litre
d)Hernia de Spiegel e)Hernia femoral

112.¿Cuál es contraindicación absoluta de anticonceptivos orales?


a)Trombosis venosa profunda b)Migraña
c)Hipertensión arterial d)Diabetes mellitus e)Cirugía electiva

113.Paciente con flujo vaginal espeso y cremoso asociado a prurito ¿cuál es el diagnóstico más probable?
a)Trichomoniasis vaginal b)Vaginitis por levaduras c)Vaginosis bacteriana d)Candidiasis vaginal e)Vaginitis atrófica

114.Paciente mujer de 80 años, presenta disnea, además, dolor de pecho, con antecedentes de bronquitis crónicas, TBC pulmo nar. Al
Examen clínico: 2do. Ruido aumentado a predominio pulmonar y signos de insuficiencia derecha. La Rx muestra derecha, onda P
picuda. El diagnóstico del paciente será:
a)Infarto de Miocardio agudo
b)Shock cardiogénico Cor
c)pulmonale crónica
d)Diabetes mellitas
e)Insuficiencia Cardiaca Izquierda

115.Una distribución difusa del infiltrado pulmonar en una radiografía de pulmones con un patrón en panal de abeja sugiere:
a)Neumonía por Staphilococus aureus b)Neumonía por Streptococcus pneumoniae c)Neumonía por Hemophilus influenzae d)Neumonía
por Neumocystis carinii e)Neumonía por Pseudomona aeruginosa

116.Una paciente post menopáusica acude a consulta para informarse en la prevención de eventos vasculares (IMA, ACV) relacionados
con la arterioesclerosis. Usted aconsejaría:
a)Estatinas
b)Dosaje de lipoproteína A
c)Antioxidantes
d)Mayor consumo de ácidos grasos poliinsaturados
e)Todas las opciones mencionadas
117.Mujer con fiebre, signos meníngeos sin lesiones focales ni sig nos de hipertensión endocraneana. ¿Cuál es el paso inmediato a
seguir?
a)Realizar una RM cerebral b)Realizar una TC cerebral c)Solicitar hemocultivos d)Realizar fondo de ojo e)Realizar punción lumbar

118.Traumatismo encéfalo- craneano con intervalo lúcido y luego caída del Glasgow ¿Cuál es el diagnóstico más probable?
a)Hematoma epidural b)Hematoma subdural agudo c)Hematoma subdural crónico d)Hemorragia subaracnoidea e)Hemorragia
intraparenquimal

119.¿Cuál es el agente etiológico de la colitis pseudomembranosa?


a)Clostridium perfringes
b)Clostridium botulinum
c)Clostridium difficcile
d)Clostridium tetani
e)Clostridium novyi

120.Mujer de 65 años con osteoporosis sin tratamiento, acude por dorsalgia. Al examen: palidez y dolor en región dorsal. Labo ratorio:
Hb: 8.5g/dL, leucocitos: 3900 cel/mm3, plaquetas: 119000 x mm3. Proteínas: 8.5 g/dL, Albumina: 2,5 g/dL, Globulina: 6 g/dL, Calcio: 13
mg/dl, Creatinina: 1.8 mg/dL ¿Cuál es el diagnóstico más probable?
a)Garnmapatia monocional de importancia incierta b)Mieloma múltiple
c)Plasmocitoma solitario
d)Síndrome mielodisplásico
e)Insuficiencia renal crónica

121.Niño con epistaxis nasal por rinofaringitis ¿Cuál es el tratamiento inmediato?


a)Taponamiento anterior b)Presión interdigital c)Taponamiento posterior

d)Observación
e)Administrar vitamina K

122.Paciente con cuadro clínico de obstrucción intestinal por hernia, el examen auxiliar que ayude al diagnóstico es:
a)Tomografía computarizada de abdomen b)Radiografia de abdomen en decúbito c)Radiografia de abdomen en bipedestación
d)Resonancia magnética de abdomen e)Ecografía abdominal

123.Niño con fiebre , convulsión tónico clónica generalizada , de 3minutos de duración y sin secuelas ¿Cuál es el diagnóstico más
probable?
a)Convulsión febril complicada b)Convulsion febril simple c)Crisis mioclónica
d)Ausencias e)Crisis atónica

124.Niño con contacto de TBC , sin sintomatología . El PPD es 10mm ¿que indicaría?:
a)INH 10mg/ kg por 1 mes b)INH 10mg/ kg por 3 meses c)INH 10mg/ kg por 6 meses d)INH 10mg/ kg por 12 meses. E)INH 10mg/ kg
por 9 meses.

125.Paciente con sarcoma con metástasis pulmonar , recibe sólo morfina y oxigeno. ¿Qué término corresponde?
a)Ortot anasia b)Distanasia c)Eutanasia pasiva d)Eutanasia activa

126.Varón de 29 años con quemadura del 55% de su superficie corporal por explosión de motor y que pierde el conocimiento ¿Cuál es
el tipo de shock más frecuente en este caso?
a)Cardiogénico b)Distributivo c)Hipovolémic d)Anafiliáctico e)Séptico

127.Varón de 82 años con neumonía intrahospitalaria. Tiene Pa02/ Fi02 < 250, lacta to en sangre elevado, diuresis en las últimas 24
horas 200 cc con alteración del sensorio y que responde a la administración de líquidos intravenosos. El paciente se encuentra en:
a)Shock séptico
b)Septicemia grave
c)Shock séptico resistente
d)Septicemia
e)Sepsis

128.Niña con cuadro clínico de hepatitis A. ¿Qué exámenes de laboratorio se debe solicitar?
a)Transaminasas y anticuerpos IgM antiHVA b)Transaminasas y anticuerpos IgG a ntiHVA c)Transaminasas
d)Transaminasa y bilirrubina
e)Transaminasas y anticuerpos IgM e IgG antiHVA

129.En una neumonitis intersticial asociada a infección por VIH ¿Cuál es el agente etiológico?
a)Micoplasma pneumoniae
b)Estreptococo pneumoniae
c)Legionella pneumophila
d)Pneumocistis jiroveci e)Coronavirus
130.Lesión de cargas negativas de la membrana basal y ampliación de los poros de los podocitos. ¿Qué se espera encontrar?
a)Proteinuria masiva b)Microalbuminuria c)Hematuria d)Macroalbuminuria e)Piuria

131.Signo más sugerente de hipotiroidismo primario:


a)Piel seca b)Hipoactividad c)Hiperrreflexi
d)Reflejo osteotendinoso lentificado e)Alopecia

132.Hipotensión arterial , ruidos cardiacos disminuídos e ingurgitación yugular. ¿Cuál es el diagnóstico más probable?
a)Pericarditis constrictiva b)Miocardiopatía restrictiva c)Infarto agudo de miocardio d)Taponamiento cardiaco e)Insuficiencia cardiaca
derecha

133.Paciente con cuadro clínico de pancreatitis aguda. ¿Cuál es el examen auxiliar que se debe solicitar inicialmente?
a)Procalcitonina b)Amilasa sérica c)TGO
d)Proteína C reactiva e)Deshidrogenasa láctica

134.Mujer de 32 años, con dolor abdominal bajo desde hace 3 meses y secreción vaginal desde hace 6 días. Al tacto vaginal: cé rvix
doloroso a la movilización, anexos dolorosos a la palpación; se evidencia cérvix con secreción mucopurulenta ¿Cuál es el diagnóstico
más probable?
a)Apendicitis b)Torsión anexial c)Cistitis
d)Enfermedad inflamatoria pélvica e)Endometriosis

135.Hematoma organizado en necropsia por fractura de temporal ¿De dónde procede el sangrado?
a)Arteria cerebral media b)Arteria cerebral posterior c)Arteria cerebral anterior d)Arteria meníngea media e)Arteria cerebelosa

136.Paciente diagnosticada de varicella en el tercer trimestre. Nace un niño eutrófico, actualmente tiene lesiones en costra. ¿Qué
recomendaría?
a)Evitar la lactancia
b)No dar antihistamínicos c)No bañar al niño d)Facilitar la lactancia.
e)Administrar ácido acetil salicílico

137.Mujer de 25 años, con quemadura con agua hirviendo en la totalidad del miembro superior derecho, y cara anterior del miembro
inferior del mismo
lado. Según la regla de los 9 de Pulaski y Tennisson, ¿cuál es el porcentaje del área afectada?
a)9
b)18
c)27
d)36
e)45

138.Sangrado del tercer trimestre, indoloro, rojo rutilante. ¿Cuál es el diagnóstico más probable?
a)Rotura uterina
b)Desprendimiento prematuro de placenta c)Placenta previa
d)Rotura prematura de membranas
e)Miomatosis uterina

139.Adulto mayor con PA 170/70 mmHg . ¿Cuál es el tratamiento?


a)IECA b)ARA II
c)Diurético tiazídico más calcio-antagonista d)Calcio-antagonista
e)Betabloqueador

140.Niña con dolor abdominal en la fosa iliaca derecha y masa dolorosa al tacto ¿Cuál es el d iagnóstico más probable?
a)Apendicitis aguda b)Adenitis mesentérica c)Enfermedad de Chron d)Quiste a pediculo torcido e)Linfoma intestinal

141.¿Cuál es la acción para prevención en el segundo nivel en un paciente con TBC 3+++?
a)Mantener una ventilación adecuada b)Control de contact os
c)Mejorar la alimentación del paciente d)Incinerar el moco y la flema del apaciente e)Formar Comités de vigilancia comunal

142. Paciente en el sexto mes de vida ¿Qué vacuna que debe recibir?:
a)Vacuna contra Rotavirus b)Antihepatítica-HVB
c)BCG
d)APO y pentavalente
e)Vacuna contra el neumococo

143.Mejor terapia para evitar el suicidio :


a)Psicoterapia
b)Tratamientos cognitivos-conductuales
c) Terapia familiar
d)Terapia dialéctico-conductual e)Antidepresivos

144.Principal diagnóstico diferencial de EPOC con disnea variable:


a)Cáncer de pulmón
b)Fibrosis pulmonar c)Bronquiectasias
d)Enfermedad pulmonar intersticial difusa
e)Asma bronquial

145.A los cuantos días se hace el control familiar del RN al dar el alta :
a)24 horas
b)48 horas
c)72 horas
d)96 horas
e)5 días

146.Contracciones uterinas cada 10 minutos sin modificación cervical en paciente con 34 semanas de gestación. ¿Cuál es el dia gnóstico
más probable?
a)Amenaza de aborto b)Aborto inminente
c)Amenaza de labor de parto pretérmino d)Amenaza de labor de parto a término e)Amenaza de labor de parto post-término

147.El divertículo de Meckel, su anatomía se caracteriza por:


a)Se localiza dentro del meso intestinal b)Más cerca de la válvula ileocecal c)Más cerca del ángulo de Treitz
d)Parte media del intenso delgado e)Se localiza en colón ascendente

148.¿Cuál de los siguientes tipos de hernias sigue el trayecto del cordón espermático dentro del músculo cremaster?
a)Hernia femoral
b)Hernia inguinal indirecta c)Hernia inguinal directa d)Hernia Obturatriz e)Hernia de Spiegel.

149.¿Qué cantidad de sodio lleva una solución salina fisiológica normal?


a)124 m Eq/L
b)134 m Eq/L
c)144 m Eq/L
d)154 m Eq/L
e)164 m Eq/L

150.Paciente con FEV 62% que tratamiento debe recibir :


a)Corticoide b)Cromoglicato sódico
c)B2 agonista
d)Bromuro de ratropio e)Teofilina

151.La duración del TIA es :


a)36 horas
b)48 horas
c)72 horas
d)24 horas
e)7 días

152.Niño con pancitopenia y visceromegalia ¿Cuál es el diagnóstico más probable?


a)Leucemia mielomielocítica crónica b)Leucemia mieloide aguda c)Leucemia linfática aguda d)Leucemia mielógena crónica e)Leucemia
linfocítica crónica

153.Lesión típica de la varicela:


a)Vesícula b)Pápula c)Ampolla d)Flictena e)Placas

154.Varón con descarga uretral mucosa y luego purulenta. ¿Cuál es el agente etiológico más probable?
a)Mycoplasma gentalium b)Haemophilus vaginalis c)Chlamydia trachomatis d)Gonococo e)Thricomonas vaginalis
155.Niño que ingirió caustico y la madre le dio leche ¿Cuál es el tratamiento que debe recibir?
a)Observación
b)Endoscopía digestiva alta c)Colocar sonda nasogástrica d)Hacer lavado gástrico e)Hidratación enérgica

156.Factor de riesgo para neumonía aguda:


a)Obesidad b)Estrés c)Alcoholismo
d)Desviación del tabique nasal
e)Talla baja

157.¿En una hernia gigante, que debe hacerse en el preoperatorio que facilite la cirugía?
a)Profilaxis antibiótica b)Hidratación enérgica c)Neumoperitoneo progresivo d)Colocación de sonda rectal e)Administrar coloides IV

158.Traumatismo torácico que posteriormente produce “pulmón blanco” en la radiografía de tórax ¿Cuál es la complicación?:
a)Atelectasia
b)Hemorragia alveolar difusa c)SDRA
d)Hemotórax
e)Edema pulmonar agudo no cardiogénico

159.Centro de Salud con médico general , pediatra , ginecólogo, laboratorio, patología ¿A qué nivel de atención pertenece?
a)I-4
b)I-3
c)I-2
d)II-1
e)II-2

160.Niño de 5 años a quien un antihistaminico que le provoca una torsaide de pointes


¿Cuál es el medicamento?
a)Levocetirizina b)Terfenadina c)Clorfeniramind)Loratadina e)Desloratadina

161.Señora que sufre caída con miembro inferior en flexión, aducción y rotación interna
¿Cuál es el diagnóstico más probable? a)Luxación anterior de la cadera b)Luxación posterior de la cadera c)Luxación congénita de
cadera d)Luxación obturatriz de cadera e)Luxofractura de cadera

162.Buzo que sufre dolor de rodilla y trastorno del sensorio ¿Cuál es el diagnóstico probable?
a)Obstrucción de vasos pequeños.
b)Rabdomiólisis c)Esguince d)Encefalopatía hipóxica
e)Intoxicación por oxígeno

163.En un paciente con shock séptico ¿qué se debe indicar inicialmente?


a)Dopamina b)Fluidos EV c)Coloides d)Adrenalina e)Dobutamina

164.¿Cuál es el parásito que causa anemia megaloblástica?


a)Tenia solium b)Hymenolepis nana c)Diphylobothrium latum d)Necator americano e)Trichuris trichura

165.Señale la lesión benigna más frecuente en hallazgo de hígado:


a)Adenoma b)Hemangioma c)Angiomiolipoma d)Hamartoma
e)Hiperplasia nodular focal

166.¿Cuál es el muestro para atención de calidad?


a)Aleatorio sin restitución b)Aleatorio sistemático c)Muestreo estratificado
d)Muestreo por conglomerados
e)Ninguna de las alternativas es correcta.

167.Señale el criterio de severidad de depresión :


a)Duración menor de una semana b)Delución
c)El episodio es atribuible a abuso de sustancias psicoactivas
d)No presenta alteraciones del sueño de cualquier tipo
e)No presenta cambios del apetito

168.¿Cuál de las siguientes enfermedades presenta soplo pansistólico?


a)CIA
b)Estenosis aórtica c)PCA
d)Insuficiencia aórtica
e)CIV

169.¿En cuál de los siguientes órganos se encuentra células epiteliales estratificadas no queratinizadas?
a)Estómago b)Ileon c)Esófago d)Colon e)Yeyuno

170.En el llamado pulmón blanco u opacificación completa de los campos pulmonares, la ausencia de broncograma aéreo con
borramiento de la silueta cardíaca corresponde a que grado de la enfermedad de membrana hialina:
a)Grado i b)GradoII c)GradoIII d)GradoIV e)Grado V

171.Respecto a sepsis y shock séptico, marque la respuesta falsa:


a)Sepsis se define como síndrome de respuesta sistémica inflamatoria más evidencia clínica de un foco de infección
b)Shock séptico se define como sepsis severa más oliguria
c)Cualquier microorganismos (bacteria, hongo, y virus) pueden iniciar sepsis
d)En sepsis severa hallazgos clínicos más importantes son oliguria y alteraciones agudas del estado mental
e)Se ha propuesto al Factor de Necrosis tumoral alfa como el mediador central de sepsis

172.Respecto a la enfermedad de Hansen, marque la alternativa correcta:


a)M. Leprae en un bacilo ácido alcohol resistente, fácil de cultivar
b)En África son más prevalentes las formas clínicas de menor resistencia inmune c)Baciloscopia se obtiene como mínimo de codos,
orejas y rodillas
d)Test de lepromina en caso de lepra tuberculosis generalmente es negativo
e)En las formas multibacilares se indica tratamientocon dapsona, clofacimina y rifampicina por seis meses

173.Respecto a tuberculosis, marque la alternativa correcta:


a)Profilaxis se indica en inmunosuprimidos y niños, siendo su duración seis meses aunque en personas con infección por VIH se indica
por un año
b)Cavidades son frecuentes de observar en las lesiones pulmonares inmunosuprimidos c)Tuberculosis raramente es sistémica
d)Hemoptisis es más frecuente de observar en inmunosuprimidos que inmunocompetentes
e)El tratamiento es prolongado por ser una bacteria de crecimiento rápido

174.Lactante de 6 meses, inicia con vómitos y fiebre de 38,2 °C. Horas después se agregan deposiciones líquidas abundantes sin moco
ni sangre. La madre decide no darle leche ni papillas. Al continuar con vómitos, tornarse irritable y ojos hundidos, es llevado a emergencia
¿Cuál es la conducta a seguir?
a)Hidratación Parenteral b)Hidratación oral
c)Hospitalizar para hidratación oral d)Administrar antibiótico e)Administrar antiemético

175.Lactante con diarrea , está irritable, con signo del pliegue positivo. ¿Cuál es el grado de deshidratación?
a)No deshidratación b)Deshidratación leve c)Deshidratación severa d)Deshidratación moderada e)Shock

176.¿Qué nervio se lesiona con la fractura diafisiaria de húmero?


a)Cubital b)Mediano c)Radial d)Circunflejo
e)Musculocutáneo

177.Mujer de 34 años, asintomática sin antecedentes de hepatopatía. Al examen: Coloración amarilla de la piel, no hepatornegalia. Perfil
hepático normal. La seudoictericia se debe a la cantidad incrementada de:
a)Caroteno b)Bilirrubina c)Cianocobalamina. D)Melanina e)Fenilalanina

178.La asociación de fiebre con escalofríos. Dolor en hipocondrio derecho e ictericia, conocida como triada de charcot, es muy sugerente
de:
a)Colangitis aguda
b)Colangitis esclerosante primaria c)Colecistitis aguda
d)Pancreatitis aguda e)Colelitiasis

179.Varón 60 años, acude por dolor agudo en ojo derecho. Al examen: fotofobia y aumento del tamaño del globo ocular ¿Cuál es el
diagnóstico más probable?
a)Conjuntivitis
b)Retinitis c)Glaucoma d)Cataratas e)Queratitis

180.Mujer de 45 años. Inicio enfermedad hace 1 semana con dolor intenso en hipocondría derecho, náuseas y vómitos, ictericia, hace
dos días fiebre y malestar general. Impresión diagnóstica:
a)Hepatitis viral aguda b)Ampuloma
c)Adenocarcinoma de vía biliar Colangitis d)esclerosante Coledocolitiasis

181.Son signos y síntomas de acidosis metabólica. Excepto:


a)Dolor abdominal agudo b)Respiración de Kussmaul c)Tetania
d)Bradicardia e)Desorientación

182.La dermatitis seborreica es un proceso de naturaleza:


a)Infecciosa b)Metabólica c)Constitucional d)Autoinmune e)Tumoral

183.Mejor técnica para enseñar habilidad:


a)Observación b)Simulación c)Reforzamiento
d)Reestructuración cognitiva e)Tutorías entre iguales

184.Gestante con pelvis estrecha, el ponderado fetal es de 4.3kg ¿Cuál es el manejo más adecuado?
a)Cesárea de urgencia b)Parto vaginal c)Cesárea electiva d)Observación
e)Inducción con oxitocina

185.Paciente varón de 41 años con malestar, palpitaciones, con un patrón electrocardiográfico. Ritmo no
sinusal, no hay ondas P, solo las llamadas f rápidas y polimorfas. Las distancias RR son desiguales. La morfología del QRS es normal.
A cuál de las siguientes entidades corresponde. Marque la correcta:
a)Fibrilación Auricular b)Flúter auricular
c)Bloqueo auricular – ventrículo (A-V) completo d)Bradicardia severa
e)Taquicardia ventricular

186.El Neumotórax a tensión se produce por:


a)Bullas Subpleurales b)TBC
c)Respiración mecánica d)Asma bronquial
e)Ninguno de las alternativas.

187.Reflejo corneal abolido ¿qué nervio está lesionado?


a)VII par craneal b)III par craneal c)II par craneal d)V par craneal
e)I par craneal

188.Surfista de 30 años de edad presenta una lesión verrucosa en el labio inferior, única, con eritroplaquia en el paladar ¿Cuál es el
diagnóstico más probable?
a)Queratosis actínica b)Carcinoma basocelular c)Carcinoma espinocelular d)Queratosis seborreica e)Melanoma maligno

189.¿Cuál parásito no se trasmite por via fecal oral ?


a)Ascaris lumbricode b)Enterobio vermicularis c)Necátor americano d)Himenolepis diminuta e)Taenia saginata

190.Varón de 28 años; con tumoración en región inguinal derecha hace 2 años, que aumenta con el esfuerzo físico y desaparece al
acostarse. Desde hace una semana la tumoración no desaparece al decúbito ¿Cómo se define esta condición de la hernia?
a)Estrangulación
b)Recurrencia c)Coercibilidad. D)Deslizamiento e)Incarceración
191.¿Qué alteración ácido base ocurre en cólera?
A)Acidosis metabólica hiperclorémica con HCH3 bajo b)Alcalosis metabólica hipoclorémica
c)Acidosis metabólica hipoclorémica con HCO3 bajo d)Alcalosis metabólica hiperclorémica
e)Acidosis metabólica con AGAP elevado

192.¿Cuál de las siguientes alteraciones puede dejar lesiones neurológicas?:


a)Hiperkalemia b)Hipokalemia c)Hipocalcemia d)Hipofosfatemia e)Hipoglicemia

193.Anciana postcolecis- tectomizada presenta dolor y distensión abdominal, además náuseas y vómitos ¿Cuál es el diagnóstico más
probable?
a)Pancreatitis aguda b)Suboclusión intestinal c)Intusucepción
d)Ileo etabólico e)Ileo biliar

194.¿Cuál es el área de superficie corporal de un paciente quemado a nivel de la cara y 2 manos?


a)13%
b)10%
c)15%
d)23%
e)6%

195.¿Cómo se denomina a la protrusión por fondo de saco de Douglas?


a)Enterocele b)Cistocele c)Rectocele d)Proctocele
e)Quiste de Gardner
196.¿Cuál es el objetivo de la maniobra de Pringle?
a)Detección de sangrado
b)Prevenir infecciones
c)Control del sangrado
d)Prevenir daño del tejido
e)Prevenir del hilio

197.Varón de 69 años, que luego de ingesta alcohólica presenta hematemesis, melena y debilidad. Al e xamen: PA: 80/50 mm Hg, FC:
110 x', sudoroso, pálido y ruidos hidroaéreos aumentados. ¿Cuál es el tratamiento inicial?
a)Sucralftato
b)Suero fisiológicon EV
c)Antiácidos
d)Transfusión de Sangre
e)Inhibidores de bomba de protones

198.Primigesta de 37 semanas que ingresa por RPM de 12 horas de evolución, sin evidencia de contracciones uterinas, ni signos de
infección amniótica. Al examen: feto en cefálica, índice de Bishop: 7 puntos, no signos de sufrimiento fetal. ¿Cuál es la con ducta a
seguir?
a)Corticoterapia e inducción del parto a las 48 horas b)Antibioticoterapia y esperar el comienzo espontáneo del parto
c)Cesárea electiva
d)Esperar el inicio espontáneodel parto
e)Inducción del parto con oxitócina

199.En niños mayores de 5 años. ¿Cuál el agente etiológico más frecuente de Neumonía adquirida en la comunidad?
a)Haernophilus influenza tipo 6
b)Moraxelia catarrhaiis
c)Staphylococcus aureus
)Streptococcus pneumoniae
e)Chlamydia trachomatis

200.Gestante de 34 semanas, hospitalizada 7 días por RPM, que recibió antibioticoterapia. A la fecha sin signos clínicos de i nfección.
Laboratorio: Proteína C reactiva 12 mg/L, líquido transcervical: leucocitos 7 x c y bacterias 6 x c. ¿Cuál es la conducta a seguir?
a) Continuar antibioticoterapia
b) Conducta expectante
c)Administrar corticoides
d) Culminar la gestaciòn
e) Realizar amnioinfusión
3 º EXAMEN ENAN

1¿Cuál es el medicamento inicial de elección en un cuadro convulsivo de emergencia?


a) Fenobarbital
b) Fenitoina
c) Diazepam
d) Ácido valproico
e) Tiopental
2.Una mujer de 25 años acude a consulta por poliuria y polidipsia. Hasta el momento las investigaciones excluyen las causas p sicógenas y
diabetes. Se realiza prueba de restricción de líquidos. Al final de la misma la osmolalidad urinaria es 240 mosm/kg y el nivel sérico de
hormona antidiurética(ADH) se encuentra elevado.
¿Cuál de los siguientes diagnósticos es el más probable?
a) Defecto de la corteza suprarrenal
b) Diabetes insípida nefrógena
c) Diabetes insípida central
d) Necrosis tubular aguda
e) Enfermedad de Addison
3.Mujer 54 años, G: 8 P: 8008 con antecedente de partos domiciliarios. Acude por presentar pérdida de orina cuando ríe, tose o al subir
gradas. ¿Cuál es el diagnóstico más probable?
a) Incontinencia de urgencia
b) Incontinencia de esfuerzo
c) Incontinencia oculta
d) Prolapso genital
e) Infección del tracto urinario
4.¿Qué hernias tienen mayor riesgo de estrangulamiento?
a) Femorales
b) Inguinales indirectas
c) Inguinales directas
d) Spiegelianas
e) Umbilicales
5. Mujer de 25 años sufre accidente de tránsito. En emergencia se constata: Funciones vitales estables y fractura de rama isquiopúbica
derecha que compromete vejiga en su porción intra peritoneal. ¿Cuál es el tratamiento más adecuado?a. Cirugía. b. Colocación de sonda
Foley. c. Talla vesical. d. Citostomía suprapúbica. e. Reparación endoscópica.

6.Gestante a término no controlada, VIH+. Al examen: altura uterina compatible con embarazo de 38 semanas, sin enfermedad
oportunista. Se inicia tratamiento antirretroviral. ¿Cuál sería la forma del parto y recomendación para la lactancia?
a) Parto vaginal y lactancia materna
b) Cesárea y fórmula maternizada
c) Parto vaginal y fórmula maternizada
d) Cesárea y lactancia materna
e) Cesárea y lactancia mixta
7.Los nódulos de Heberden característicos de la osteoartritis se ubican en las articulaciones...
a) Interfalángicas distales.
b) Radiocarpianas.,
c) Interfalángicas proximales.
d) Metacarpofalangicas.
E) Metatarsofalángicas
8.El antibiótico de elección en otitis media aguda es:
a) Gentamicina
b) Eritromicina
c) Clindamicina
d) Amoxicilina
e) Penicilina V
9.Varón de 72 años con fibrilación auricular y alcoholismo crónico. Es traído porque desde hace 1 año presenta deterioro cogn itivo y
algunos déficit focales. Al examen PA: 130/70 mm Hg, FC: 87 F R: 17 x', despierto, orientado parcialmente, Babinski y Hoffrnan izquierdo
positivos. ¿Cuál es la causa de demencia más probable?
a) Encefalopatia de korsakoff
b) Enfermedad de Alzheimer
c) Encefalopatia de wernicke
d) Multiinfarto
e) Alcoholismo crónico
10.Varón de 18 años, con herida cortante en muñeca derecha por asalto con arma blanca. Presenta hipoestesia y postura en
hiperextensión metacarpofalángica del 4to. y 5to. dedo. ¿Cuál es el nervio lesionado?
a) Mediano
b) Radial
c) Cubital
d) Musculocutáneo
e) Circunflejo
11.Varón de 53 años, con tuberculosis pulmonar BK positivo y en tratamiento con esquema 1, que presenta alteraciones de la visión de
colores. ¿Cuál de los siguientes fármacos produce con mayor frecuencia este efecto adverso?
a) Pirazinamida
b) Isoniacida
c) Rifampicina
d) Estreptomicina
e) Etambutol
12.¿Cuál de las siguientes leucemias es más frecuentes en niños?
a) Linfoblástica
b) Mieloblásticas
c) Monoblástica
d) Mieloide crónica
e) Todos por igual
13. La catarata en roseta es típica de: a. Espondilitis anquilosante. b. Traumatismo ocular c. Adulto mayor (senil). d. Diabético. e.
Degeneración macular

14.Paciente con infección por VIH que presenta signos meníngeos ¿Cuál es la etiología más frecuente?
a) Toxoplasmosis
b) Histoplasmosis
c) Criptococosis
d) Estreptococo pneumoniae
e) Listeria monocitógenes
15.¿Cuál de los siguientes es considerado un efecto colateral de los anticonceptivos orales?
a) Mialgia
b) Condritis
c) Hipermenorrea
d) Dismerionea
e) Mastalgia
16. Una cirugía por apendicitis en la que se encontró perforación de colon, se realiza jareta invaginante y se deja dren tubular
corresponde a una herida: a. Limpia. b. Limpia contaminada. c. Sucia. d. Infectada. e. Contaminada.

17.Varón de 71 años, hipertenso, acude por precordalgia intensa, EKG: supradesnivel del 17.segmento ST en D II, D III, aVF e infradesnivel
ST en V1, ¿Cuál es el diagnóstico más probable?
a) IMA STE diafragmático
b) IMA STNE posterior
c) IMA STE posterior
d) IMA STE inferoposterior
e) IMA cara anterior
18. La hernia indirecta que presenta anillo inguinal dilatado, sin desplazamiento de vasos epigástricos, corresponde a la clasificación de
Nyhus: a. II b. I c. IIIb d. IV e. IIIa

19.Cuando existe una correlación entre dos variables cuantitativas continuas. ¿Cuál es el modelo predictivo que debe aplicarse?
a) Regresión múltiple
b) Regresión de Cox
c) De efectos fijos
d) Dispersión
e) Regresión lineal
20.¿Cuál es la técnica que se utiliza para el control de sesgo de selección?
a) Pareamiento
b) Aleatorización
c) Estratificación
d) Enmascaramiento
e) Medidas repetidas
. 21. La fractura del recluta ocurre a nivel de: a. Primer metacarpiano. b. Segundo metatarsiano. c. Calcáneo. d. Quinta falange del pie. e.
Astrágalo.

22.El puerperio inmediato comprende: Las primeras dos


a) horas post parto Las primeras 12
b) horas post parto
c) Las primeras 24 horas post parto
d) Las primeras 48 horas post parto
e) La primera semana post parto
23.Mujer de 43 años, acude por convulsiones tónico-clónicas generalizadas en 4 oportunidades desde hace 35 minutos, sin recuperación
de la conciencia. Durante la evaluación nuevamente se repite el cuadro descrito ¿Cuál es el tratamiento?
a) Diazepam: 10 mg EV, carga de Fenitoina a 20 mg/k g peso a una infusión máxima de 50 mg/min.
b) Midazoiam 5 mg, fenobarbital EV diluido lento, infusión más de 50 mg/
d) Propofol EV, Fenitoina bolo de 25 mg/kg peso, infusión máxima de 75 mg/minuto
c) Diazepam 10 mg EV, seguido de Midazolam en infusión
e) Diazepam 5 mg EV, carga de 800 mg de Fenitoina diluida en dextrosa al 5%
24.Varón de 40 años acude por dolor anal, progresivo e invalidante, de dos días de evolución que se incrementa al sentarse y caminar. Al
examen: tumoración renitente con signos de flogosis en el margen anal, ¿Cuál es el tratamiento inicial?
a) AINEs
b) Antibloticoterapia
c) Baños de asiento
d) Corticoterapia
e) Drenaje quirúrgico
25.Varón de 22 años que en reyerta sufre agresión con cuchillo. Al examen: PA: 110/70 mm Hg, FC: 80 x', FR. 18 x', herida de 4cm en
flanco izquierdo, con dolor local y rebote negativo. ¿Cuál es la conducta inicial?
a) Laparotornia exploradora
b) Exploración de herida
c) Laparoscopia exploradora
d) Transfusión urgente
e) TAC abdominal
26.Varón de 60 años, con disnea progresiva desde hace 2 horas. Al examen: FR: 34 x', presenta cianosis perioral y distal, sibilantes
diseminados en ambos campos pulmonares, escasos roncantes. Laboratorio: Leucocitos 12,368, pH: 7,42, PCO2: 28, P02: 68, HCO3: 21.
Mejora después de administrar p2 adrenérgicos. ¿Cuál es el diagnóstico más probable?
a) Tromboembolia pulmonar
b) EPOC descompensado
c) Insuficiencia respiratoria tipo II
d) Crisis asmática
e) Alcalosis metabólica e hipoxemia
27.Mujer de 45 años, con esquizofrenia paranoide y HIV positivo, acude al hospital por convulsiones y trastorno del sensorio. Al examen:
Glasgow 9, signos meníngeos (+). LCR: levaduras, positivo a tinta china. ¿Cuál es el tratamiento a seguir?
a) Fluconazol
b) Itraconazol
c) Anfotericina B
d) Ketoconazol
e) Caspofungina
28.Varón de 66 años, diabético, acude por fiebre, malestar general, tos con expectoración verdosa. Al examen: PA: 80/50 mm Hg, FC: 110
x', FR: 35 x', MEG, pulsos débiles y oliguria. Laboratorio: leucocitosis 16000 x' lactato 1,4 U/L. Se realiza reto de fluidos y se observa
estabilización de la PA, mejora el rango diurético, ¿cuál es el diagnóstico?
a) Sepsis grave, foco respiratorio
b) SIRS e injuria renal aguda
c) Shock séptico foco respiratorio
d) Sepsis foco urinario
e) Shock séptico foco urinari o
29.En la fractura de Monteggia hay:
a) Fractura del 1/3 distal + luxación de cabeza de radio
b) Fractura del 1/3 proximal del cubito + luxación anterior de cúpula radial
c) Fractura de cubito y radio + luxación de codo
d) Fractura de 1/3 medio del radio + luxación del codo
e) Ninguna de las alternativas.
30.A una paciente de 70 años de edad se le detecta irregularidad de la frecuencia cardiaca en una evaluación de rutina. No ha
presentado nuevos síntomas en reposo o asociados a los esfuerzos. En el ECG, no se documentan ondas P y se encuentra un intervalo RR
irregular con una frecuencia de 70/ min. En su ECG de hace cuatro años tenía ritmo sinusal ¿Cuál de las siguientes conductas es la más
adecuada en el manejo de la paciente?
a) Cardioversión
b) Antiarrítmicos
c) Bloqueador Beta
d) Anticoagulación
e) Ácido acetilsalicílico
31.A los antioxidantes se les conoce como tales al Prevenir o retardan el envejecimiento:
a) Porque protege el S.N.C.
b) Porque protege al sistema hormonal
c) Porque bloquea los electrones impares de los radicales libres
d) Ninguna de las alternativas.
e) Todas las alternativas son correctas.
32.¿Cuál es el proceso inicial de la hemostasia?
a) Tapón hemostático temporal
b) Vasoconstricción de la arteriola hemostático definitivo
c)Tromboplastina hística
d) Activación de la coagulación
e)Tapón
33.Lactante de 6 meses inicia hace 7 días cuadro de resfrió común, hace dos días se agrega fiebre de 39°C. Hace un día se agr ega
dificultad respiratoria. Al examen: polipnea, crepitantes y roncantes en ambos campos pulmonares. ¿Cuál es el diagnóstico más
probable?
A) Neumonítis viral
b) Neumonía bacteriana
c) Bronquiolitis
d) Neumonía complicada
e) Neumonía atípica
34.Varón de 28 años con cuadro febril de 3 semanas de evolución, sudor nocturno, artralgias y dolor en cadera bilateral. Refiere comer
queso fresco hecho tradicionalmente. Si se sospecha de Brucellosis, ¿Cuál es la prueba de screeníng recomendada?
a) 2-Mercaptoetanol
b) Aglutinación en lamina
c) Aglutinación en tubo
d) Hemocultivo
e) Rosa de Bengala
35.Multípara de 40 años que una hora después de alumbramiento de su último parto, presenta pérdida sanguínea vaginal no muy
abundante, pero la PA cae a 80/50 y el pulso se acelera a 110/m, mostrando sudoración aunque sin perder la conciencia. No refiere dolor
al hacer la palpación del abdomen, no se encuentra útero, el examen rápido de la región vulvoperineal no evidencia desgarros. Cuál de
las siguientes presunciones diagnósticas es la más probable:
a) Desgarro perineo vaginal de 4to grado
b) Coagulación intravascular diseminada
c) Inversión uterina probablemente parcial
d) Desgarro cervical
e) Retención total de placenta
36. Una mujer de 64 años notó lesiones del tipo ampollosas en sus muslos y axilas. Eran pruriginosas, pero no dolían. No tenían otros
síntomas. En la exploración las lesiones son grandes, a tensión, en forma de ampollas de contenido seroso. La biopsia confirmó el
diagnóstico de Penfigoide ampolloso. ¿Cuál de las siguientes características histológicas es típica de esta condición?
a) Cambios inespecíficos
b) Depósitos de inmunoglobulina A (Ig
c) Lesiones dentro de la epidermis (acantólisis)
d) Lesiones de la membrana basal
e) Depósitos de inmunoglobulina M (IgM)
37.La ulcera péptica es una enfermedad multifactorial de curso crónico y periódico. De los siguientes factores diga ¿cuál es el más
importante para su desarrollo?
a) AINE
b) Infección por helicobacter pylori
c)Alcohol
d)Tabaco
e)Hiperclorhidria
38.Una mujer de 68 años con hipertensión arterial y dislipidemia presenta dolor estroesternal de 30 minutos de duración que se irradia al
cuello. Esta sudorosa y con moderada angustia. El ECG muestra una elevación del segmento ST en las derivaciones de la cara in ferior.
¿Cuál de los siguientes mecanismos es la causa más probable de esta entidad?
a) Rotura de una placa coronaria
b) Aortitis
c) Inflamación pericárdica
d) Vasculitis
e) Miocarditis
39.Un varón asintomático de 64 años, ejecutivo de profesión, que asiste a su chequeo regular anual. Tuvo un infarto de miocardio
anterior hace cuatro años y presenta actualmente en el EKG, supradesnivel del ST en V3 -V6 ¿Cuál es su diagnóstico?
a) Pericarditis calcificada
b) Aneurisma de la pared ventricular
c) Quiste hidatídico
d) Pleuropericarditis
e) Normal
40¿Cuál de los siguientes mecanismos describe mejor el efecto de los iones del calcio sobre el miocardio?
a) Inotropismo positivo
b) Inotropismo negativo
c) Cronotropismo positivo
d) Cronotropismo negativo
e) Desacoplamiento de la excitación – contracción
41¿Cuáles son los componentes de la calidad de los servicios de salud?
a) Organizacional, técnico y humano
b) Científico y técnico
c) Administrativo y financiero
d) Humano, financiero y gerencia!
e) Técnico, científico y financiero
42.¿Que significa CREST ?
a) Calcinosis, Raynaud, endocrinopatía, telagiectasia
b) Calcinosis, Raynaud, endocrinopatía y telagiectasia
c) Calcinosis, Raynaud, alteración del esófago, esclerodactilia y telagiectasia
d) Calcinosis, Raynaud, alteración del esófago, esclerodermia y telagiectasia
e) Calcinosis, Raynaud, alteración del esófago, esclerodermia y Tremor
43. Durante una laparoscopía, accidentalmente el cirujano perfora el colon transverso, perforándolo en un 20% de su circunferencia.
Inmediatamente convierte la cirugía a una laparotomía. ¿Cuál es la conducta a seguir con el segmento lesionado?: a. Debridación de
bordes y rafia. b. Resección más anastomosis. c. Drenaje. d. Colostomía. e. Ileostomía.

44.Lactante de 7 meses, con fiebre de 39 °C de 3 días de evolución, un


vómito por día y deposiciones normales. Rechaza alimentos semisólidos pero lacta bien. Al examen: T: 39 °C. Hemodinámicamente
estable, mucosa orofaringea normal. MV presentes en ambos campos pulmonares y piel sin lesiones, ¿Cuál es el diagnóstico más
probable?
a) Faringitis aguda
b) Fiebre tifoidea
c) Infección del tracto urinario
d) Mononucleosis infecciosa
e) Diarrea aguda
45. En relación a las inmunizaciones. ¿Cuál es la actitud médica frente a un niño con asplenia funcional o anatómica?
a) Sólo puede recibir vacunas inactivadas
b) Al vacunar con agentes atenuados pueden multiplicarse
c) Debe evitarse la vacuna antineumocócica
d) Están indicadas las inmunizaciones
e) No debe recibir la vacuna antimeningocócica
46.Varón de 69 años en UCI, con neumonía, que presenta al 7mo día de evolución Murphy (+) ¿Cuál' es el diagnóstico más probab le?
a) Coliecistitis aguda iitiasica
b) Coliecistitis aguda reagudizada
c) Colecistitis crónica
d) Colecístitis aguda alitiasica
e) Colecistitis crónica reagudizada
47¿Cuál es la reacción adversa más frecuente en el tratamiento de la onicomicosis?
a) Síndrome de Stevens – Johnson
b) Hepatotoxicidad
c) Nefritis intersticial
d) Hemorragia digestiva
e) Dermatitis exfoliativa
48.En qué tipo de episiotomía es más frecuente el desgarro perineo vaginal de 4to grado:
a) El medio lateral izquierdo
b) En la medio lateral derecha
c) En la mediana con peritoneo corto
d) En la medina con perineo grande y amplio
e) En la cesareada anterior
49. Los siguientes son factores de riesgo para la pre eclampsia–eclampsia, con una excepción. Márquela:
a) Historia personal de hipertensión arterial Primigravidez
b) Diabetes mellitus
c) Enfermedad trofoblástica gestacional
d) Embarazo prolongado
50.Se considera EHG (Enfermedad hipertensiva de la gestión) cuando se encuentra:
a) Hipertensión, más proteinuria después de las 20 semana gestacional
b) Hipertensión, edema y aumento exagerado de peso después de las 20 semana gestacional
c) Hipertensión, anasarca y poliuria después de las 20 semana gestacional
d) Hipertensión, cefalea, tinitus después de la 20 semana gestacional
e) Hipertensión, hipoproteinemia y trombocitopenia después de la 20 semana gestacional
51.Neonato de 15 días con lactancia materna exclusiva y diagnóstico de galactosemia
¿Cuál es la conducta a seguir?
a) Puede recibir lactancia mixta con fórmula maternizada
b) Puede recibir lactancia materna de manera relativa
c) Debe recibir fórmula maternizada e iniciar ablactancia
d) Suspender de manera absoluta la lactancia materna
e) Puede recibir una fórmula extensamente hidrolizada.
52.Mujer de 30 años acude por dolor anal intenso al defecar. Antecedente: constipación crónica. Al examen: tacto rectal doloroso,
esfínter hipertónico y hemorroide centinela. Anoscopia: úlcera lineal media posterior. ¿Cuál es el tratamiento?
a) Esfinterotomía interna lateral
b) Esfinterotomia externa lateral
c) Ligadura de paquete hemorroidal
d) Curetaje de la úlcera
e) Cauterización de la úlcera
53.El LCR en la meningoencefalitis bacteriana se caracteriza:
a) Hipoglucorraquia, hipoproteinor raquia y lactato mayor de 3.9 mEq/L
b) Hipoglucorraquia, hiperproteinor raquia y lactato menor de 1.6 mEq/L
c) Hipoglucorraquia, hiperproteinorraquia y lactato mayor de 3.9 mEq/L
d) Ninguna de las alternativas es correcta .
e) Todas las alternativas son correctas.
54¿En el caso supuesto que su respuesta hubiese sido cáncer de próstata, cuál de los siguientes análisis de laboratorio hubiese sido de
mayor utilidad?
a) Fosfatasa alcalina
b) Fosfatasa acida
c) Antígeno específico prostático
d) Antígeno carcinoembrionario
e) Gonadotropinas coriónicas
55.La transición demográfica en el Perú se relaciona con la disminución de la...
a) Razón de dependencia.
b) Densidad poblacional
c) Tasa global de fecundidad
d) Tasa de migración interna.
e) Esperanza de vida al nacer.
56. En una comunidad con servicios de agua a domicilio y alcantarillado, pero no en el colegio, se ha detectado incremento de casos de
diarrea aguda en los estudiantes, en este contexto, el personal del Centro de Salud ha planificado realizar las siguientes ac ciones:
capacitación a docentes del colegio, coordinación con la Municipalidad y el sector educación para construir servicios sanitarios
adecuados (mientras tanto, colocar letrinas). Estas actividades corresponden a:
a) Promoción
b) Prevención
c) Gestión
d) Organización
e) Recuperación
57¿Cuál de los siguientes tratamientos se considera el primer paso para el manejo inicial de las fracturas abiertas o expuest as?
a) Fijación con tutores externos
b) Desbridamiento urgente
c) Inmovilización endomedular
d) Inmovilización con placas y tornillos
e) Inmovilización con aparato de yeso
58.Varón de 70 años con dolor de espalda y extremidades al deambular, astenia de cinco meses de evolución. Al examen: PA 130/80 mm
Hg, FC: 78 x', FR: 20 x', T°: 37.5 °C. Palidez de piel, hepatoesplenomegalia, Laboratorio: anemia, hipercalcemia, azoemia,
¿Cuál es el diagnóstico más probable?
a) Leucemia mieloide crónica
b) Leucemia linfática crónica
c) Linterna de células
d) Mieloma múltiple
e) Leucemia de células plasmáticas
59.¿Cuál de las siguientes enzimas plasmáticas diagnóstica enfermedad obstructiva biliar?
a) Gamma-glutamil transferasa
b) Transaminasa glutámico-oxalacetica
c) Deshidrogenasa isocítrica
d) Acetilcolinesterasa
e) Leucina aminopeptidasa
60.La tuberculosis de primoinfección es más frecuente de
a) Piel
b) Estómago
c) Pulmón
d) Ganglio linfático
e) Intestino delgado
61.Varón de 32 años que ha presentado matarla por Plasmodium vivax, para prevenir la recaída, ¿cuál es su ind icación terapéutica?
a) Cloroquina 500 mg. X 4 semanas
b) Artesunato 1200 a 1600 mg. Vía oral x 3 días
c) Primaquina 30 mg x día x 14 dlaa
d) Quinidina 10 mg/kg, hasta un máximo de 600 mg. En solución salina x 2 semanas
e) Usar mosquiteros
62.La presencia de secreción vaginal amarillenta, espumosa y lesiones eritematosas en cérvix nos da la sospecha de infección por:
a) Trichomonas vaginalis
b) Candida albicans
c) Gardnerella
d) Chlamidya trachomatis
e) Neisseria gonorrheae
63.Mujer de 30 años con G: 3 P: 3003 y FUR hace 1 semana, acude a emergencia por dolor de gran intensidad en bajo vientre y fiebre de
39 °C a más. Al tacto vaginal: dolor a la movilización de cérvix, masa palpable en anexo izquierdo y flujo vaginal de regular volumen. ¿Cuál
es el diagnóstico más probable?
a) Embarazo ectópico
b) Absceso pélvico
c) Apendicitis aguda complicada
d) Pielonefritis aguda
e) Diverticulitis
64.Gestante de 41 semanas G: 2 P: 1001, El embarazo ha transcurrido normalmente. Al examen: feto en cefálico, test no estresante
reactivo y Bishop favorable. ¿Cuál es la conducta a seguir?
a) Esperar hasta que inicie el trab ajo de parto espontaneo
b) Maduración cervical
c) Inducir el parto
d) Administrar corticoides e inducir el parto pasadas las 48 horas
e) Hacer cesárea electiva.
65.Mujer de 25 años, acude por dolor abdominal en FID, hace 7dias, se auto medica con ciprofloxacino y paracetamol VO. Al ex amen: PA:
80/50 mm Hg, FC: 120, FR: 24, T°: 38°C. Piel pálida, sudorosa, escleras ictéricas Abdomen: distendido, RHA disminuidos, Mc Burney
positivo, rebote positivo. El diagnóstico es apendicitis aguda complicada con peritonitis generalizada con indicació n quirúrgica. En el post
operatorio continua ictérica, ¿Cuál de las siguientes patologías explica ia ictericia?
a) Hepatitis
b) Pancreatitis
c) Pi leflebitis
d) Colangitis
e) Iatrogenia
66¿Cuál es un principal criterio para endometritis?
a) Fiebre
b) Dolor pélvico
c) Secreción vaginal maloliente
d) Sangrado vaginal
e) Estreñimiento
67.¿Cómo se denomina la alteración de la fusión de los pliegues uretrales ?
a) Hipospadia
b) Agenesia del genital externo
c)Pene bífido
d) Micropene
e) Hidrocele
68.Paciente que presenta dolor precordial de 5 minutos que cede al reposo .¿Cuál es el diagnóstico más probable?
a) Angina inestable
b) Angina estable
c) Infarto de miocardio
d) Angina de Prinzmetal
e) Osteocondritis
69.Después de la penetración de un espermatozoide en el óvulo, se produce la intensificación del metabolismo del huevo. ¿Cuál es la
causa del incremento del metabolismo oxidativo?
a) Aumento de la necesidad de oxígeno y su consumo
b) Disminución del metabolismo de carbohidratos
c) Disminución del contenido de aminoácidos libres
d) Disminución de la actividad de fermentos proteolíticos
e) Aumento de la síntesis de ácidos grasos
70. Lactante que rueda de prono a supino y se sienta sin soporte. ¿Cuál es su edad en meses?
a) 4
b) 5
c) 6
d) 2
e) 3
71.Recién nacido de 14 días de edad, P: 3700 gr, con lactancia materna exclusiva, ginecoobstetra, refiere que la madre tiene BK (++). El
neonato ha ganado 120 gr de peso, luce activo, al examen físico es normal y no ha recibido BCG, ¿Cuál es su Dx y conducta con el
neonato?
a) Contacto, aplicar BCG
b) Infección, RX Tórax, Hemograma.
c) Infección, aplicar PPD
d) Contacto, INH 18.5 mg/día vo
e) Contacto, suspender lactancia materna, dar sucedáneo de la LM 13% 90cc cada 3 horas.
72. La incidencia más alta en que se producen los accidentes por cuerpo extraño en niños es:
a) A los 4 años de edad.
b) A los 36 meses de edad.
c) A los 14 meses de edad.
d) Cuando el niño diferencia el día y la noche
e) Cuando el niño deja de orinarse en la cama.
73. Lesión punzocortante a la altura de L1 paravertebral derecha ¿Qué arteria puede lesionarse?
a) Arteria hepática
b) Arteria aorta
c) Arteria renal
d) Arteria colónica
e) Arteria iliaca
74. En la enfermedad de Parkinson ¿qué neurotransmisor se altera?
a) Aspartato
b) Dopamina
c) Glutamato
d) Ácido g-aminobutírico (GABA)
e) Serotonina
75.¿Cuál es la mordedura más contaminada?
a) Gato
b) Conejo
c) Hamster
d) Humana
e) Canina
76.Recién nacido de 32 semanas y 1800 gramos de peso, a las 48 horas de vida presenta ictericia, hepatoesplenomegalia y lesiones
cutáneas de aspecto petequial en el hemograma se aprecia anemia con eritroblastosis y trombocitopenia, en la radiografía de c ráneo se
observan calcificaciones cerebrales periventriculares. Que diagnostico le parece más probable:
a) Rubeola congénita
b) Citomegalovirus congénita
c) Sepsis neonatal
d) Enfermedad hemolítica del recién nacido
e) Sífilis congénita
77.Varón de 60 años, con masa dolorosa en fosa iliaca derecha y fiebre de 7 días de evoluc ión, ¿Cuál es el diagnóstico más probable?
a) NM de ciego
b) Absceso de pared abdominal
c) Hernia de Spiguel Hernia Spiguel
d) Tumor carcinoide
e) Plastrón apendicular
78.Mujer de 56 años asintomática acude a su control mensual de diabetes mellítus tipo
2. Al examen: no anormalidades. Laboratorio: glucosa: 129mg/d1., urocultivo: E. coll
>100,000 UFC/ml, ante estos resultados. ¿Cuál es la conducta a seguir?
a) Jugo de Cramberry diario
b) Observacion
c) Norfloxacino 400mg / 2 veces al día por 5 días
d) Nitrofurantoína 100mg / d ía como profilaxis por 3 meses
e) Nitrofurantoina 100 mg cada 6 horas como tratamiento por 3 días
79.El fármaco que produce fibrosis pulmonar es:
a) Bleomicina Busulfan
b) Methotrexate
c) Ciclofosfamida
d) Todas las alternativas son correctas.
80.¿Cuál de las siguientes drogas en el tratamiento específico de actúa inhibiendo a la ARN polimerasa?
a) Rifampicina.
b) Etionamida
c) Estreptomicina
d) Isoniazida
e) Etambutol
81.Con respecto al tiempo de sangría marque lo correcto:
a) Evalúa los factores de coagulación
b) En pacientes hemofílicos se encuentra prolongado
c) En trombocitopenias severas está prolongado
d) En los defectos de agregación plaquetaria, se encuentra en valores normales
e) En pacientes anticoagulados el tiempo de sangría se encuentra prolongado
82.Con respecto al estudio del L.C.R:
a) La celularidad normal es menor a 05 leucocitos por ml
b) Normalmente encontramos predominancia mononuclear
c) En las meningitis asépticas puede existir hipoglucorraquia
d) Un valor de más de 200 mg/dl de proteínas me sugieren una meningitis séptica
e) En enfermedad hemorrágica meníngea no se encuentra aumento de proteínas en el L.C.R
83. Adolescente de 15 años con diagnóstico de depresión severa e intento de suicidio.
¿Cuál es el tratamiento inicial?
a) Lamotrigina
b) Haloperidol
c) Clorpromazina
d) Sertralina
e) Amitriptilina
84.Hijo pretérmino de madre RPM con signos de obstrucción y sangrado al defecar ¿Cuál es el diagnóstico más probable?
a) Neumoperitoneo
b) Neumoretroperitoneo
c) Enterocolitis necrotizante
d) Trombosis mesentérica
e) Aire en la vena porta
85. Niño de 3 años que vive en una institución para menores abandonados y al que se le ha diagnosticado MEC por gérmenes Gram
negativos. Usted le indicaría tratamiento ATB durante:
a) Dos semanas como máximo
b) Una semana es lo ideal.
c) 10 días
d) Seis semanas
e) Tres semanas mínimo
86. El tratamiento de mantenimiento de las convulsiones febriles es:
a) Fenobarbital por 2 años.
b) Ácido valproico por 6 meses
c) No se debe dar tratamiento generalmente
d) Fenobarbital + diazepan de acuerdo a crisis convulsiva
e) Carbamazepina + ácido valpróico por dos años
87.¿Cuál es la causa más frecuente de hipertiroidismo?
a) Adenoma tóxico
b) Bocio multinodular tóxico
c) Tiroiditis de Hashimoto
d) Tiroiditis sugaguda de De Quervain
e) Enfermedad de Graves Basedow
88.¿Por dónde pasa el impulso al ventrículo?
a) Has de Bachman
b) Has de Wenckebach
c) Has de Thorel
d) Has de Hiss
e) Fibras de Purkinje
89.Sangrado rojo vinoso en tercer trimestre, escaso, con contractilidad uterina frecuente
¿Cuál es el diagnóstico probable?
a)Rotura uterina
b)Placenta previa
c)Rotura de vasa previa
d)Desprendimiento prematuro de placenta
e)Placenta de inserción baja
90.En un hijo de madre con rubeola ¿Cuál es la cardiopatía congénita más frecuente?
a) CIV
b) CIA
c) PCA
d) Transposición de grandes vasos
e) Tetralogía de Fallot
91.Gestante multípara, con embarazo doble de 36 semanas quien llega en periodo expulsivo. Luego de atender el parto en cefálica del
primer producto se evidencia que el segundo se halla en situación transversa con el dorso superior. ¿Cuál es la actitud más apropiada?
a) Versión externa
b) Cesárea de emergencia
c) Conducta expectante
d) Tracción cefálica
e) Versión interna
92.Una mujer de 63 años presenta angina de esfuerzo y dos episodios de síncope en el pasado. En la exploración se detecta un soplo
sistólico con irradiación a las carótidas y un S2 suave, ¿Cuál de los siguientes diagnósticos es el más probable?

a) Estenosis mitral
b)Insuficiencia mitral
c)Estenosis aórtica
d)Insuficiencia aórtica
e)Estenosis tricúspidea
93.Una niña de 4 años es llevada al médico por un cuadro de inquietud, intenso prurito anal nocturno y discreto prurito vulvar. ¿Cuál es
el diagnóstico?
A. Teniasis.
B. Oxiuriasis.
C. Ascariasis.
D. Trichiuriasis.
E. Eccema del pañal.
94.Una niña de 6 meses de edad es traída donde su pediatra por presentar incremento en la frecuencia de las deposiciones durante las
últimas dos semanas. Sus heces son olorosas, “sueltas”, y flotan en el agua. Su madre refiere que después de darle de comer elimina
flatos en exceso. Tiene como antecedente un episodio de obstrucción meconial a las 24 horas de nacida. Su alimentación actual consiste
en leche en fórmula y ocasionalmente arroz. El examen físico revela un peso actual por debajo del percentil 5 para la edad. El examen de
Thevennon en heces y cultivos resultan negativos. ¿Cuál de los siguientes exámenes confirmaría el diagnóstico?
A. Biopsia guiada por endoscopía del intestino delgado
B. Análisis cualitativo de grasas en heces
C. Test de cloruro en sudor
D. Ecografía de páncreas
E. Radiografía de abdomen y pelvis con contraste
95.En un niño de 8 años previamente sano, la causa más frecuente de meningitis bacteriana es:
A. H. influenzae.
B. S. pneumoniae.
C. N. meningitidis.
D. S. aureus.
E. Estreptococos del grupo B.
96.Un paciente de 32 años infectado por VIH consulta por astenia y fiebre ocasional, de unos 2 meses de evolución. En la exploración
destaca: palidez cutánea, ausencia de adenopatías periféricas, esplenomegalia 5 cm por debajo del re- borde costal. En la analítica
destaca: hemoglobina 6,5 g/dl, volumen corpuscular medio normal, leucocitos 1.300/ml, plaquetas 40.000/ml, CD4 300/ml. En la TAC de
abdomen presenta esplenomegalia homogénea, sin adenopatías. De los siguientes procesos, ¿cuál le parece más probable?
A. Infección por M. avium-intracellulare.
B. Infección por Citomegalovirus.
C. Infección por Parvovirus B19.
D. Infección por Leishmania.
E. Toxoplasmosis diseminada.
97.Un hombre de 30 años sano se realiza una radiografía de tórax para formalizar un contrato laboral en una empresa. En la radiografía
se observa un patrón intersticial bilateral de tipo reticular, adenopatías hiliares bilaterales y medias- tínicas. Se indica una
broncofibroscopia con lavado broncoalveolar que en el recuento celular muestra los siguientes resultados: linfocitos 50%, h istiocitos
40%, eosinófilos 2%, polimorfonucleares 8% y el cociente CD4/CD8 5 ¿Cuál es el diagnóstico más probable?
A. Alveolitis alérgica extrínseca.
B. Neumoconiosis.
C. Sarcoidosis.
D. Linfangitis carcinomatosa.
E. Hemosiderosis pulmonar idiopática.
98.¿Cuál es una característica del Haemophilus influenzae?
A. La inmunización no ha reducido la incidencia de infecciones por ese agente.
B. La máxima incidencia se observa en niños mayores de 5 años.
C. Los gérmenes más virulentos no pertenecen al serotipo B.
D. Forma parte de la flora respiratoria normal en el 60-90% de niños sanos.
E. Es un coco bacilo grampositivo.
99.Un hombre de 40 años, turista extranjero, es remitido a un centro de Urgencias por bajo nivel de conciencia, tempe- ratura de 37,7 °C
y disnea. No tiene traumatismos. Se realizan varias pruebas. La más inmediata es una gasometría arterial sin oxigenoterapia q ue muestra
pH 7,33, PCO2 50 mmHg, PO2 65 mmHg, HCO3- 27mEq/l. Señale el diag- nóstico más probable entre los siguientes:
A. Tromboembolismo pulmonar.
B. Neumonía lobar.
C. Edema agudo de pulmón.
D. Hipoventilación alveolar.
E. Bronconeumonía.
100¿Cuál es el antibiótico de elección para el tratamiento de neumonía por Chlamydia pneumoniae en menores de 4 años?
A. Amoxicilina + ácido clavulánico.
B. Ceftriaxona.
C. Eritromicina.
D. Rifampicina.
E. Tetraciclina.
101. Todos los siguientes fenómenos se asocian a taponamiento cardíaco, EXCEPTO:
A. Pulso paradójico.
B. Presión venosa disminuida.
C. Presión diferencial reducida.
D. Gasto cardíaco disminuido.
E. Taquicardia sinusal.
102.Paciente de 48 años, con hipertensión pulmonar severa y cianosis, a quien en su infancia se le diagnosticó comunica- ción
interauricular. La posibilidad diagnóstica es:
A. Enfermedad de Lutembacher.
B. Hipertensión pulmonar primaria.
C. Síndrome de Eisenmeger.
D. Tetralogía de Fallot.
E. Malformación de Ebstein.
103.Una paciente de 35 años refiere presentar disnea en los últimos 10 días. A su llegada a urgencias se observa crepitantes basales y
ECG con taquicardia irregular de QRS estrecho, sin que se observen ondas P. ¿Cuál de las siguientes es la actitud más correcta?
A. Cardioversión eléctrica sincronizada.
B. Cardioversión eléctrica no sincronizada.
C. Digoxina y diuréticos, anticoagulación durante 4 semanas y cardioversión posterior.
D. Digoxina y diuréticos, anticoagulación durante 4 días y cardioversión posterior.
E. Digoxina y diuréticos, antiagregación durante 4 semanas y cardioversión posterior.
104.¿Cuál de los siguientes antiulcerosos produce quelación de otros medicamentos, impidiendo su absorción de forma significativa?
A. Cimetidina.
B. Ratinidina.
C. Misoprostol.
D. Tetraciclina.
E. Sucralfato.
105.Un paciente de 78 años de edad, previamente sano, que vive en una residencia de ancianos bastante masificada y con insuficientes
recursos higiénicos, padece un cuadro diarreico desde hace 6 semanas. Refiere molestias abdominales tipo retortijón, febrícula ocasional
y 4-6 deposiciones diarias, alguna de ellas nocturna, con mucosidad y, en ocasiones, con hebras de sangre. Entre los diagnósticos que se
enumeran a continuación seleccione el que le parece MENOS probable:
A. Cáncer de colon.
B. Infección por Clostridium difficile.
C. Enfermedad de Crohn.
D. Colitis isquémica.
E. Salmonelosis.

106.¿Cuál de los siguientes procedimientos, es el menos indicado para el tratamiento de la úlcera péptica?
A. Régimen lácteo.
B. Anticolinérgicos.
C. Antiácidos.
D. Bloqueadores H2.
E. Tranquilizantes.
107.La tomografía cerebral en los infartos cerebrales isquémicos:
A. Muestra típicamente área hipodensa rodeada de edema.
B. Muestra área hiperdensa que capta contraste.
C. Puede ser normal en la fase inicial.
D. Distingue bien la zona de infarto de la de edema.
E. Sólo distingue bien los infartos cerebelosos.
108.Gestante en la décimo cuarta semana, desde hace semanas presenta náuseas y vómitos persistentes sin respuesta al
dimenhidrinato, ha recibido en los últimos 15 días sueros glucosados por vía IV. Actualmente presenta edema de miembros infer iores,
taquicardia, diplopía, dificultad para la marcha, nistagmo y alteraciones mentales. PA: 100/70; pulso: 115 lpm. Análisis: HB: 9 g%, Na: 130
mEq/L, bilirrubina total: 2 mg%, bilirrubina directa: 1,4 mg%. ¿Cuál es la posibilidad diagnóstica?
A. Encefalopatía hepática.
B. Pre – eclampsia severa.
C. Síndrome de HELLP.
D. Hipertiroidismo de la gestante.
E. Encefalopatía de Wernicke.
109.La cefalea en racimos se caracteriza por lo siguiente:
A. El alcohol puede desencadenar los ataques ya sea en el período de cefalea o intercrítica.
B. Los ataques duran típicamente 8 – 10 horas y luego remiten rápidamente.
C. Puede haber síndrome de Horner ipsilateral.
D. Es más frecuente en el sexo femenino.
E. Todo lo anterior.
110.Un paciente diabético de 60 años consulta por primera vez respecto al tratamiento de su enfermedad, ¿qué objetivo a
alcanzar de los siguientes le recomendaría en primer lugar?
A. Mantener la tensión arterial por debajo de 110/70 mmHg.
B. Abandono del hábito tabáquico.
C. Mantener un índice de masa corporal (IMC) menor de 21.
D. Realizarse glucemia capilar basal a diario.
E. Evitar las grasas animales en la dieta.
111.En una mujer de 55 años intervenida de cáncer de mama tres años antes, con buen estado general, se comprueba una hipercalcemia
de 11,1 mg/dl. ¿Cuál es la primera prueba a realizar?
A. Determinación de PTH.
B. Determinación de 1,25(OH)2D.
C. Determinación de 25OHD.
D. Determinación de péptido relacionado con la PTH (PRPTH).
E. Gammagrafía ósea.
112.Una niña de 11 años está hospitalizada debido al aumento de nerviosismo y palpitaciones; los tranquilizantes no la han aliviado. El
apetito ha sido bueno pero registró pérdida de peso. El aprovechamiento escolar ha ido deteriorándose gradualmente. La n iña fue
enviada por el profesor a un tutor guía que sugirió ayuda psiquiátrica. Al examen se en- contró un pulso amplio, la temperatura de la piel
estaba aumentada, transpiración excesiva y ROT rápidos. Los ojos presentaban una apariencia vidriosa. El diagnóstico más probable es:
A. Tirotoxicosis juvenil.
B. Psicosis juvenil.
C. Diabetes mellitus.
D. Disautonomía familiar.
E. Síndrome de Cushing.
113.¿Cuál de los siguientes trastornos no produce vasculitis de pequeños vasos?
A. Poliangeítis microscópica
B. Púrpura de Henoch-Schonlein
C. Síndrome de Churg Strauss
D. Enfermedad de Kawasaki
E. Crioglobulinemia mixta esencial
114.En el lupus cutáneo subagudo es característica de la presencia de anticuerpos:
A. Anti-Sm.
B. Anticentrómero.
C. Anti-Ro.
D. Anti-Jo1.
E. Antihistona.
115.La segunda generación de antidepresivos incluye a todos los siguientes, EXCEPTO:
A. Amoxapina.
B. Traxodona.
C. Maprotilina.
D. Amitriptilina.
E. Fluoxetina.
116.Señalar en cuál de los siguientes trastornos de la personalidad es más IMPROBABLE la aparición de sintomatología alucinatoria:
A. Trastorno límite.
B. Trastorno esquizotípico.
C. Trastorno obsesivo-compulsivo.
D. Trastorno esquizoide.
E. Trastorno paranoide.
117.¿Qué es cierto acerca de la oliguria?
A. Se define como un volumen urinario inferior a 1.000 ml/día.
B. Es un hallazgo constante en la insuficiencia renal aguda.
C. Se asocia a dolor lumbar bilateral.
D. Hace más difícil el manejo conservador del fracaso renal agudo.
E. Mejora el pronóstico del fracaso renal agudo.
118.Mujer de 28 años de edad, consulta por hemoptisis recurrente, disnea, anemia, presión arterial: 120/85 mmHg, res- piraciones 28 x’.
Examen de orina: hematuria microscópica con cilindros hemáticos; creatinina: 3,2 mg%, urea: 65 mg%; Hb: 6 g%; anticuer pos
antinucleares: negativo; anticuerpos anticitoplasmáticos ANCA-C + 52 U. Radiografía de pulmones: lesión homogénea 1/3 medio del
hemitórax izquierdo. ¿Cuál es el diagnóstico más probable?
A. Granulomatosis de Wegener.
B. Lupus eritematoso sistémico.
C. Tuberculosis pulmonar y renal.
D. Síndrome de Goodpasture.
E. Aspergilosis pulmonar.
119.La aparición de un pliegue extra de piel por debajo del párpado inferior es una característica de:
A. Dermatitis seborreica
B. Rosácea.
C. Pitiriasis rosada.
D. Dermatitis atópica.
E. Eccema de contacto.
F. Enfermedad injerto contra huésped.
120.En Querion el tratamiento es:
A. Antihistamínicos
B. Penicilina.
C. Clotrimazol
D. Griseofulvina.
E. Anfotericina B.
121.Durante qué mes de gestación comienza la hemopoyesis en la médula ósea:
A. Tercero.
B. Quinto.
C. Sexto.
D. Séptimo.
E. Octavo.

122.El linfoma de Burkitt está asociado a la infección de los linfocitos B con el:
A. Citomegalovirus.
B. Virus delta.
C. Virus de Epstein Barr.
D. Papilomavirus.
E. Virus del polioma.
123. La histamina se encuentra en los: a. Mastocitos. b. Eosinófilos. c. Leucocitos. d. Neutrófilos. e. Linfocitos

124.Actualmente, el tratamiento de elección del carcinoma epidermoide del canal anal es:
A. Ablación local.
B. Radioterapia sola.
C. Radioterapia y quimioterapia combinadas.
D. Resección abdómino perineal.
E. Resección anterior baja.
125.En el tratamiento con inyección para esclerosar las hemorroides, es importante inyectar:
A. En el lumen de la vena varicosa.
B. En la unión mucocutánea.
C. En la muscularis del intestino.
D. En todas estas.
E. En ninguna de las anteriores.
126.¿Cuál de los siguiente GLUT'S participa en el transporte de fructuosa ?: a. GLUT 1. b. GLUT 5. c. GLUT 2. d. GLUT 4. e. GLUT 3

127. Cuál es el electrolito encargado de la fase de repolarización ventricular en el corazón:


a. Sodio. b. Potasio. c. Cloro. d. Calcio. e. Magnesio. 39.

128.El tratamiento paliativo más adecuado en caso de cáncer de cabeza de páncreas es:
A. Quimioterapia.
B. Operación de Whipple.
C. Derivación biliodigestiva sola.
D. Derivación biliodigestiva más gastroyeyunoanastomosis.
E. Radioterapia.
129.En las próximas 30 horas después de una laparotomía puede haber fiebre de causas infecciosas importantes, EXCEP - TO:
A. Lesión intestinal con filtración de líquidO intestinal.
B. Infección de tejidos blandos por Clostridium perfringes.
C. Infección de tejidos blandos por Gram negativos.
D. Infección de tejidos blandos por Streptococo B-hemolítico.
E. Infección de tejidos blandos por Klebsiella.
130. Una herida traumática ocurrida hace más de 4-6 horas se considera que está:
A. Contaminada.
B. Limpia.
C. Sucia.
D. Supurada.
E. Infectada.
131.¿Cuál es el electrolito que más se altera en las primeras 48 horas post-quemadura?
A. Sodio.
B. Cloro.
C. Zinc.
D. Magnesio.
E. Todos.
132.La hernioplastía de tipo Liechtenstein se caracteriza por el uso de:
A. Malla de polipropileno.
B. Incisiones relajantes.
C. Tapón de polipropileno en anillo inguinal.
D. Doble sutura a surget en la fascia transversalis.
E. Técnica laparoscópica.
133.En la Fase I del ATLS (Advanced Trauma Life Support), debe realizarse todo, excepto:
A. Control de la columna cervical.
B. Respiración y ventilación.
C. Inserción de sonda nasogástrica.
D. Exponer desnudo al paciente.
E. Control de la hemorragia externa.
134.¿Cuál de las siguientes suturas es de material absorbible?.
A. Seda.
B. Poliéster.
C. Nylon.
D. Polipropileno.
E. Ácido poliglicolico
135.En la fractura hepática con hemoperitoneo ¿qué estructura se tiene como referencia, para clampar y cohibir el san- grado?
A. El ligamento de Treitz.
B. La vía biliar principal.
C. La vena cava inferior.
D. El ligamento colecistoduodenal.
E. Epiplón gastrohepático
136.El dolor abdominal postprandial intenso a repetición en un paciente anciano con varios factores de riesgo cardiovas- cular, nos debe
hacer sospechar:
A. Embolia mesentérica.
B. Isquemia mesentérica crónica.
C. Colitis isquémica.
D. Rotura de aneurisma abdominal..
E. Aneurisma esplácnico.
137.¿Cuáles son las dos causas MÁS comunes de hemorragia grave del colon?
A. Diverticulosis y angiodisplasia.
B. Enfermedad hemorroidaria y poliposis.
C. Adenocarcinoma de colon y colitis ulcerativa.
D. Vólvulo del colon y amebiasis.
E. Colitis granulomatosa y prolapso rectal.
138.Mejor operación (con menor índice de recurrencia) en paciente joven con enfermedad acidopéptica:
A. Gastrectomía total.
B. Gastrectomía subtotal Billroth-II.
C. Vagotomía supraselectiva.
D. Vagotomía troncular y piloroplastia.
E. Vagotomía troncular y antrectomía.
139.Mujer de 20 años de edad, G2 P2002. Último parto por cesárea, hace 3 meses lactando. ¿Cuál es el anticonceptivo
hormonal MÁS adecuado?
A. Estrógeno oral.
B. Estrógeno más progesterona por vía oral.
C. Progestágenos orales.
D. Parche de etinilestradiol y progesterona.
E. Inyectable mensual de estrógeno y progesterona.
140.Paciente con prurito vulvar, que al examen ginecológico se encuentra secreción blanquecina maloliente. Cuello uteri- no con
punteado rojizo como fresas. La infección vaginal sería causada por:

A. Cándida albicans.
B. Trichomonas vaginalis.
C. Gardnerella vaginalis.
D. Flora mixta.
E. Chlamydia trachomatis.
141.En una gestante con condiloma genital de gran volumen, el mejor tratamiento es:
A. Podofilina local.
B. Láser de dióxido de carbono.
C. Ácido tricloroacético local.
D. 6-fluorouracilo local.
E. Interferon sistémico.
142.Paciente a quien se realizó un cono frío, cuya anatomía patológica reporta carcinoma epidermoide invasor con menos de 5 mm de
profundidad en el estroma. Según la clasificación de la FIGO, corresponde al estudio:
A. I A.
B. I B.
C. II A.
D. II B.
E. N.A.
143.Mujer de 60 años de edad. Presenta pérdida de orina al esfuerzo físico sin poder controlarlo. Al examen se corrobora Con la
maniobra de VaIsaIva. ¿Cuál es el diagnóstico MÁS probable?
A. Incontinencia urinaria de esfuerzo.
B. Incontinencia urinaria de urgencia.
C. Desgarro perineal antiguo.
D. Prolapso uretral.
E. Incontinencia urinaria por rebosamiento.

144.El tratamiento MÁS apropiado en caso de carcinoma endometrial estadío I comprende:


A. Histerectomía total abdominal.
B. Salpingooforectomía bilateral.
C. Estadiaje.
D. Sólo A y B son correctas.
E. Todas las respuestas son correctas.
145.¿Cuál de los siguientes signos NO suele aparecer en la clínica del cáncer de mama?
A. Mastodinia.
B. Telorragia.
C. Tumoración.
D. Retracción del pezón.
E. Adenopatía axilar.
146.La modificación endocrina que aparece de modo MÁS PRECOZ en la menopausia es:
A. Aumento de FSH.
B. Disminución de FSH.
C. Aumento de LH.
D. Disminución de LH.
E. Aumento de estrógenos.
147.La cianosis de la mucosa vaginal y del cérvix es un signo presuntivo de embarazo y se le denomina signo de:
A. Godell
B. Hegar
C. Noble Budin

D. Chadwick
E. Mac Donald
148.¿Qué tiempo después de la enfermedad trofoblástica molar (mola hidatiforme) generalmente hace su aparición la enfermedad
trofoblástica maligna?
A. 4 a 6 años.
B. 5 a 10 años.
C. 36 meses.
D. 2 a 28 meses.
E. 6 a 24 meses.
149.En la placenta acreta hay un defecto en la decidua que consiste en la AUSENCIA de:
A. Las Estrías de Rohr.
B. La zona esponjosa.
C. La zona compacta.
D. La zona basal.
E. El estrato de Nitabuch.
150.El embarazo gemelar que se produce en el primer día de la división celular del huevo, tiene las siguientes caracterís- ticas:
A. Monoplacentario y biamniótico.
B. Monoplacentario y monocoriónico.
C. Monocoriónico y monoamniótico.
D. Monocoriónico y biamniótico.
E. Bicoriónico y biamniótico
151.En el trabajo de parto normal, cuando la flexión se completa, el diámetro de la cabeza fetal que ingresa en el estrecho superiorde la
pelvis es:
A. Occipitofrontal.
B. Suboccipitobregmático.
C. Occipitomentoniano.
D. Biparietal.
E. Bitemporal.
152.Con respecto al uso del sulfato de magnesio, señale lo INCORRECTO:
A. La dosis de carga es de 4g /EV.
B. La dosis de mantenimiento es de 1 - 2 g /EV por hora.
C. No es necesario la colocación de sonda vesical, salvo cuando se maneja convulsiones.
D. Solo debe ser usado hasta 24 horas post parto con el fin de prevenir las convulsiones.
E. La arreflexia patelar y la depresión respiratoria son signos de intoxicación por sulfato de magnesio.
153.La complicación MÁS SEVERA de la hiperemesis gravídica es:
A. Deshidratación.
B. Hipotensión.
C. Encefalopatía de Wemicke.
D. Hiperkalemia.
E. Hipematremia.

154.Gestante de 22 años, con 18 semanas por FUR, grupo sanguíneo A, factor Rh negativo. Ant ecedente de un parto eu- tócico, donde
recibió inmunoglobulina Anti D. el embarazo actual es de segundo compromiso. ¿Cuál es la conducta adecuada?
A. Esperar porque no hay problema.
B. Inmunoglobulina anti D a las 26 y 32 semanas.
C. Inmunoglobulina Anti D dosis única.
D. Test de Liley.
E. Tipificación sanguínea de la pareja.
155. En el perfil biofísico fetal se estudia lo siguiente, EXCEPTO:
A. Situación y posición fetal.
B. Movimientos fetales y respiratorios.
C. Frecuencia y reactividad cardíaca fetal.
D. Volumen del líquido amniótico.
E. Tono fetal.
156. En relación a la valoración Apgar, señale lo CORRECTO:
A. Se toma desde el momento que sale completamente el RN del canal del parto
B. No necesariamente sirve como guía para la reanimación del RN.
C. Un puntaje de 7 nos da un buen pronóstico neurológico a largo plazo.
D. En un recién nacido grave debe evaluarse básicamente el tono muscular.
E. Existe limitación de aplicación en el RN prematuro.
157.El hijo de madre diabética puede presentar los siguientes problemas, EXCEPTO:
A. Prematuridad.
B. Macrosomía.
C. SDR I.
D. Trombosis de la vena renal.
E. Mesomelia.
158. La forma neurotóxica de la bilirrubina en el recién nacido es la:
A. Bilirrubina no conjugada o indirecta.
B. Bilirrubina conjugada o directa.
C. Urobilinógeno.
D. Biliverdina.
E. Urobilina.
159.¿Cuál de los siguientes NO es parte del síndrome de rubéola congénita?
A. Coriorretinitis.
B. Microftalmia.
C. Sordera.
D. Conducto arterioso persistente.
E. Hepatoesplenomegalia.
160.El feto postmaduro típicamente presenta todo lo siguiente, EXCEPTO:
A. Piel suelta.

B. Uñas largas.
C. Caída del pelo.
D. Disminución del vérmix.
E. Ninguna de las anteriores.
161.Una madre se queja de que su niño de 3 meses vomita frecuentemente, el control del peso del niño durante el período de 1 1/2
meses, revela que éste se mantiene dentro del percentil 70, ¿qué actitud tomaría usted?
A. Solicita Rx del tracto digestivo alto.
B. Solicita estudio de acidez gástrica.
C. Cambia la fórmula láctea.
D. Agrega cereales a su alimentación.
E. Ninguna de las anteriores.
162.¿Cuál de los siguientes NO ha sido implicado como agente causal en caso de una enfermedad parecida al coqueluche?
A. Bordetella pertusis.
B. Echovirus tipo 21.
C. Bordetella parapertussis.
D. Bordetella bronchiseptica.
E. Especies de Adenovirus.
163.En el primer trimestre de vida extrauterina, el crecimiento en los niños se caracteriza por, EXCEPTO:
A. Ganancia de peso diario de 30g.
B. Crecimiento en talla de 3.5 cm/mes.
C. Crecimiento de perímetro cefálico: 2 cm/mes.
D. Requerimiento calórico de 80 kcal/g/d.
E. B y D.
164.Los senos paranasales que se encuentran desarrollados al nacer son:
A. Maxilares y etmoidales.
B. esfenoidales y maxilares.
C. Frontales y etmoidales.
D. Esfenoidales y etmoidales.
E. Frontales y maxilares.
165.¿Cuál de las siguientes expresiones no es correcta respecto a la laringotraqueobronquitis?
A. Es la forma más común de obstrucción de las vías aéreas superiores.
B. El 75% de los casos es producida por los virus parainfluenza.
C. La budesonida inhalada es útil en el tratamiento.
D. El uso de antibióticos previene la infección bacteriana.
E. La dexametasona disminuye el edema inflamatorio.
166. Niño de 15 años con pligue epicanto, nariz en silla de montar, boca pequeña y lengua grande. ¿Cuál es el posible diagnostico que
presenta?: a. Trisomía 21. b. Síndrome de Turner. c. Síndrome de Klinefelter. d. Trisomía 16. e. Trisomía 18.
167. ¿Qué exceso de vitamina produce hipertensión endocraneal benigna?: a. A. b. D. c. B. d. C. e. K
. La glucosa durante la glucólisis en condiciones de alta tensión de oxígeno produce: a. Lactato b. Malato c. Acetato d . Piruvato e.
Succinilcolina

168.¿Cuál de las siguientes manifestaciones se presenta en la hipoglicemia neonatal?


A. Hiperreflexia.
B. Distensión abdominal.
C. Fontanela tensa.
D. Hipertonía.
E. Temblores.
169. El petit mal del niño se caracteriza por lo siguiente, EXCEPTO:
A. Puede observarse aura.
B. Por lo general las crisis duran menos de 15 segundos.
C. Las crisis no alteran la postura del niño.
D. Puede observarse contracciones de los miembros superiores.
E. La conciencia se recupera rápidamente.
170.Niño de 2 meses de edad, es llevado a Emergencia por presentar convulsiones tónico-clónicas generalizadas desde hace 45 minutos,
sin recuperar la conciencia. Antecedente de asfixia al nacer. Presenta fiebre de 39ºC. ¿Cuál es el diagnós- tico más probable?
A. Crisis convulsiva
B. Convulsión febril compleja
C. Estatus convulsivo
D. Convulsión febril simple
E. Epilepsia
171. Un niño de 10 años presenta anemia, hemorragias pulmonares y hepatoesplenomegalia, En la placa de tórax se obser - va una
masa mediastínica anterior, ¿Qué proceso descartaría en primer lugar?
A. Mononucleosis infecciosa
B. Leucemia linfoide crónica
C. Leucemia linfoblástica aguda tipo T
D. Leucemia linfoblástica aguda tipo B
E. Leucemia M2
172. Un niño de 6 años acude a consulta por un cuadro de febrícula de 3 días de evolución, con dolor a la deglución. Los datos m ás
relavantes de la exploración física son lesiones erosivas en el paladar y vesículas intraepidérmicas no agru- padas en palmas y plantas.
Entre los siguientes diagnósticos, ¿cuál es el más probable?
A. Eritema multiforme.
B. Rickettsiosis
C. Síndrome de Steven-Jonhson.
D. Enfermedad de pie, mano, boca
E. Deshidrosis.
173.El enteropatógeno parasitario que con más frecuencia causa diarrea con malabsorción intestinal en niños menores de 5 años y en
desnutridos es:
A. Isospora belli
B. Esfamoeba histolytica
C. Strongyloides stercolaris
D. Ascaris lumbricoides
E. Giardia lamblia
174.Si el valor de una canasta básica familiar en Tarma es de S/. 1500 y el valor de la canasta solo con alimentos es de S/75 0, señale
usted en que grupo se encontraría la familia “X” cuyo ingreso promedio mensual es de S/. 550 mensual:
A. Quintil 4
B. Pobre no extremo
C. Extrema pobreza
D. Pobreza
E. Clase media
175.Son factores condicionantes de la salud pública:
A. Biología
B. Medio ambiente
C. Estilo de vida
D. Sistema sanitario
E. Todas
176.¿Cuál de los siguientes documentos sobre promoción de la salud puso mayor énfasis en la perspectiva sociopolítica?
A. Informe de Lalonde (Canadá, 1974)
B. Declaración de Santa Fe De Bogotá (Colombia, 1992)
C. Declaración de Alma Ata (URSS, 1978)
D. Carta de Ottawa (Canadá, 1986)
E. Declaración de Sundsvall (Suecia, 1991)
177.¿Cuál es el programa social encargado de realizar incentivos monetarios, que promuevan y apoyen el acceso a los
1servicios en educación, salud y nutrición, para las familias muy pobres?
A. Pronaa
B. Cuna más
C. Pensión 65
D. Foncodes
E. Juntos
178.De acuerdo con las normas sectoriales, el establecimiento de Salud que cuenta con servicio de emergencia, sala de operaciones, las
cuatro especialidades básicas y mediana capacidad resolutiva, es categorizado como:
A. I-3
B. I-4
C. II-1
D. II-2
E. III-1
179. La existencia de un servicio de atención al cliente o al usuario es un indicador de:
A. De eficacia
B. De acceso
C. De estructura
D. De proceso
E. De resultado
180.Cual no es documento de gestión de un establecimiento de primer nivel:
A. Manual de organización y funciones
B. Manual de procedimientos
C. Texto único de procedimientos administrativos
D. Guía de procedimientos y trámites
E. Plan operativo anual
181.Marque la respuesta CORRECTA, de acuerdo al tema de SUNASA:
A. Es un órgano consultor adscrito al MINSA con autonomía funcional, administrativa y financiera.
B. Ejerce sus competencias y funciones desde Lima pero a un ámbito nacional, regional y local.
C. Ejerce función conciliatoria y arbitral para tramitar las controversias entre el asegurado y el seguro.
D. Uno de sus objetivos es construir un sistema de aseguramiento público sostenible.
E. Es un órgano ejecutor del Seguro Integral de Salud
182.El coeficiente de correlación de Pearson (r):
A. Varía de -1 a 0.
B. Tiene el mismo signo que la varianza de la cual procede.
C. No sirve para estudiar la relación lineal entre dos variables cuantitativas.
D. Si se eleva al cuadrado, es el coeficiente de determinación.
E. Todas son ciertas.
183. La expectativa de vida al nacer en el Perú actualmente es de:
A. 60,2 años
B. 74,1 años
C. 65,0 años
D. 63,4 años
E. 68,8 años
184.Al tratar una artritis psoriásica con azatioprina, mejora el 60% de los pacientes; la 6-mercaptopurina mejora a un 55%. La diferencia
entre ambos tratamientos es significativa (p<0,05), lo cual se debe interpretar como una de las siguientes opciones:
A. Con azatioprina mejorarán el 60% de tus pacientes.
B. La azatioprina es en un 95% mejor que la 6-mercaptopurina.
C. El nivel de significación es del 1%.
D. Hay diferencias entre los dos tratamientos, con una probabilidad de que esta afirmación sea equivocada menor al 5%.
E. No se puede concluir nada.
185.Se llama probabilidad condicionada a:
A. La probabilidad de un suceso conocido previamente.
B. La condición que debe cumplir un suceso.
C. La probabilidad que ocurra un suceso habiendo ocurrido otro.
D. La probabilidad de que suceda de que ocurra un suceso u otro.
E. El teorema de Bayes restringido.
186. Varón de 20 años de edad, que presenta convulsiones en el servicio de Emergencia. ¿Cuál de los siguientes fármacos
administraría?
A. Carbamazepina.
B. Diazepam.
C. Metilfenitoína.
D. Valproato.
E. Difenilhidantoína.
187. Entre los efectos colaterales del litio NO se incluye:
A. Aumento de peso.
B. Gastritis.
C. Polidipsia.
D. Leucocitosis.
E. Convulsiones.
188. ¿Cuál es el efecto adverso más frecuente en la administración de Clindamicina?
A. Colitis pseudomembranosa.
B. Nefrotoxicidad.
C. Ototoxicidad.
D. Anemia aplásica.
E. Artropatia.
189. Varón de 7 años, 15 días después de cumplir tratamiento por faringoamigdalitis estreptocócica presenta náuseas, vó - mitos, cólicos
abdominales e ictericia. ¿Cuál de los siguientes fármacos se asocia a estos efectos adversos?
A. Clindamicina fosfato
B. Cefradina
C. Vancomicina
D. Ampicilina
E. Eritromicina estolato
190. ¿Cuál es el hospedero definitivo de Hymenolepis diminuta?
A. Hombre.
B. Cerdo.
C. Vaca.
D. Rata.
E. Venado.
191. ¿Cuál es el tipo de estreptococo que produce estreptolisina O?
A. Faecalis.
B. Bovis.
C. Pyogenes.
D. Agalactie.
E. Pneumoniae.
192.¿Cuál de las siguientes enfermedades es causada por un parásito intracelular?
A. Asepergilosis.
B. Histoplasmosis.
C. Lepra.
D. Malaria.
E. Tuberculosis.
193.En un ensayo clínico se comparan 3 tratamientos (p.e. placebo, tratamiento establecido y un tratamiento nuevo). La variab le
respuesta es continua (p.e. nivel de glucosa en sangre). ¿Si la variable no tiene una distribución normal, el test correcto para comparar la
respuesta es?:
A.La t de Student.
B.El test de Wilcoxon.
C.Análisis de la varianza.
D.El test de Krusal-Walls.
E.El test Chi-cuadrado.
194.Si como resultado de un estudio prospectivo en el que se han comparado dos grupos de individuos, unos expuestos y otros no
expuestos a un factor de riesgo causante de una enfermedad. Después de comprobar la comparabilidad de los dos grupos, obtenem os
que de los 500 individuos expuestos al factor de riesgo 50 padecen la enfermedad de estu- dio, y que de los 500 no expuestos sólo la
padecen 10. ¿Cuál sería el riesgo de padecer la enfermedad que podríamos atribuir al factor de riesgo?:
A. 0,08.
B. 0,1.
C. 0,2.
D. 0,5.
E. 10.
195.En el estudio de Framingan (Framingan Heart Study) sobre cardiopatía coronaria, se encontró que en el examen ini- cial 17 personas
por cada 1.000 tenían síntomas evidentes de cardiopatía coronaria. Elija la tasa o razón que mejor describe este hecho:
A.Tasa de incidencia.
B.Tasa de prevalencia.
C.Razón de morbilidad estandarizada.
D.Tasa específica por edad.
E.Tasa de mortalidad ajustada.
196.En un estudio de cohortes diseñado para comprobar la relación tabaco-cáncer de vejiga, se siguieron durante un año a 1.000
personas, 500 fuman y 500 no. De los 50 cánceres producidos en ese tiempo, 45 aparecieron en fumadores.
¿Cuál es la incidencia en expuestos?:

A. 50/1.000 = 5%.
B. 50/500 = 10%.
C. 45/500 = 9%.
D. 5/500 = 1%.
E. 45/1.000 = 4,5%.
197. Niño de dos meses, sin antecedentes de importancia, presenta rinorrea hialina y fiebre de 38° C en los tres últimos días. Desde hace
24 horas, tiene tos y dificultad respiratoria progresiva. En las últimas 12 horas rechaza todas las tomas. Ha recibido la primera dosis de
DTP acelular, Hemophilus influenzae tipo B, Meningococo C, Polio oral y dos dosis de Hepatitis B. Examen físico: frec. cardíaca 135 l.p.m.,
frec.respiratoria 55r.p.m. y Sat O2: 90%. Tiraje intercostal y subcostal, subcrepitantes y sibilancias bilaterales. ¿Cuál es el diagnóstico más
probable?
A.Crisis asmática moderada - severa.
B.Bronquiolitis.
C.Neumonía bacteriana.
D.Tos ferina.
E.Neumonía por Clamidia trachomatis.
198.Paciente de 9 meses que presenta discreta fiebre, adenopatías retroauriculares grandes y dolorosas y además exante- ma
maculopapular de color rosáceo en cuerpo. ¿Cuál es el diagnóstico más probable?
a). Roséola infantil
b).Rubéola
c) Sarampión
d) Mononucleosis infecciosa
e) Varicela
199.Niño de 13 años llevado a la consulta por presentar estornudos, rinorrea acuosa y prurito nasal persistentes. Al exa- men: intenso
edema y palidez de la mucosa nasal, con secreción clara. El diagnóstico probable es:
a).Cuerpo extraño
b) Mastocitosis nasal
c) Rinitis alérgica
d) Rinitis neutrofílica
e) Rinitis vasomotora
200.A los 5 minutos de vida un neonato presenta estos signos: 130 pulsaciones por minuto, manos y pies cianóticos, buen tono muscular,
llanto poderoso, reactividad normal, frecuencia respiratoria 60 por minuto. El índice de Apgar en este niño es:
a) 6
b) 7
c) 8
d) 9
e) 10
4TO EXAMEN ENAM

1. Primigesta con 32 semanas de gestación por última regla, acude a emergencia por dolor en epigastrio e
hipocondrio derecho. PA: 180/110 mmHg, proteinuria ++, tacto vaginal: cérvix posterior, orificio externo cerrado, pelvis
ginecoide. El diagnóstico más probable es:

A. Preeclampsia severa y trabajo de parto

B. Preeclampsia superpuesta e hipertensión crónica

C. Preeclampsia severa

D. Preeclampsia severa y colecistitis

E. Preeclampsia severa y DPP

2. Puérpera de parto gemelar que presenta sangrado vaginal profuso luego del alumbramiento. El diagnóstico más
pro- bable es:

A. Laceración de cuello uterino

B. Ruptura uterina

C. Coagulación

D. Atonía uterina

E. Trombocitopenia

3. Gestante a término de 35 años acude a emergencia por contracciones uterinas cada 3 minutos y sangrado vaginal
de 10cc. PA: 120/70, P: 84x’, FR: 16x’, Ecografía: feto único en LCI, placenta previa marginal. LCF: 148x’. Se realiza TV en
condiciones de operabilidad: incorporación: 90%, dilatación: 8cm, membranas íntegras, altura de presentación: 0, variedad
de posición: OIIA, pelvis ginecoide. ¿Cuál es la conducta a seguir?

A. Acentuación del trabajo de parto

B. Esperar a que continúe el trabajo de parto espontáneo

C. Parto instrumentado

D. Preparar para cesárea inmediata

E. Transfusión de sangre

4. Paciente de 49 años, con mioma uterino de tamaño equivalente a una gestación de 12 semanas, que presenta
hiperme- norreas y hemoglobinemia de 9 g%. No existe patología asociada. La paciente está en lista de espera para la
práctica de una histerectomía programada para dentro de 3-4 meses. En esta paciente está indicado el tratamiento
preopera- torio con:
A. Estrógenos.

B. Ergóticos dopaminérgicos.

C. Inhibidores de la fibrinolisis.

D. Análogos de la GnRH.

E. Derivados del cornezuelo del centeno.

5. Mujer con tumoración que protruye por genitales externos. Al examen tiene una longitud de vagina de 11. El punto
Aa

es +3. Según la clasificación de prolapso de órganos pélv icos (POP-Q), ¿cuál es el grado de distopia genital?

A. III

B. I

C. II

D. V

E. IV

6. Paciente de 26 años con 21 semanas de gestación, presenta dolor abdominal y sangrado vaginal escaso, de 2
días de evolución. Al examen: altura uterina 20 cm, movimientos fetales presentes. Especuloscopía: se observa membranas
ovulares prominentes e integras, que protruyen por el orificio externo abierto. ¿Cuál es el diagnóstico?

A. Óbito fetal

B. Aborto incompleto

C. Aborto inevitable

D. Aborto inminente

E. Aborto frustro

7. Primigesta en la 39 semana de gestación y con contracciones de parto. Todo ha transcurrido con normalidad
hasta que ha roto la bolsa espontáneamente con 3 cm de dilatación. Nada más romper la bolsa ha comenzado a sangrar
(sangre roja en mediana cantidad) y han surgido signos de sufrimiento fetal muy grave. La causa más probable es:

A. Placenta previa.

B. Abruptio placentae.
C. Lesión de cérvix.

D. Síndrome de Hellp.

E. Rotura de vasa previa

8. Gestante en la décimo cuarta semana, desde hace semanas presenta náuseas y vómitos persistentes sin
respuesta al dimenhidrinato, ha recibido en los últimos 15 días sueros glucosados por vía IV. Actualmente presenta edema
de miembros inferiores, taquicardia, diplopía, dificultad para la marcha, nistagmo y alteraciones mentales. PA: 100/70;
pulso: 115 lpm. Análisis: HB: 9 g%, Na: 130 mEq/L, bilirrubina total: 2 mg%, bilirrubina directa: 1,4 mg%. ¿Cuál es la
posibilidad diagnóstica?:

A. Encefalopatía hepática.

B. Pre – eclampsia severa.

C. Síndrome de HELLP.

D. Hipertiroidismo de la gestante.

E. Encefalopatía de Wernicke.

9. Después de un expulsivo normal y tras 60 minutos de periodo de alumbramiento no se aprecian signos de


despren- dimiento placentario a pesar de haberse aplicado masaje uterino y de haber incrementado moderadamente la
dosis de oxitocina. Se indica una extracción manual de placenta y la pared uterina ¿Cuál es el diagnóstico más probable?

A. Placenta incarcerada

B. Engatillamiento placentario

C. Placenta adherente por acretismo placentario

D. Placenta succenturiada con cotiledón aberrante

E. Placenta circunvalata

10. Paciente de 55 años, general en su activo, con antecedentes de hipertensión, diabético y angina inestable, traído
a la urgencia tras caída montando en bicicleta. Radiográficamente presenta una fractura intracapsular desplazada del fémur
proximal. ¿Cuál será el tratamiento más adecuado?

A. Reducción y osteosíntesis con tornillo

B. Artroplastía parcial de cadera

C. Reducción y osteosíntesis con tornillos canulados

D. Reducción incruenta

E. Artroplastía total de cadera

11. Un hombre fumador de 50 años consulta por ronquera, afonía e hipo. En la exploración ORL se evidencia parálisis
de la cuerda vocal izquierda como única anomalía. ¿Dónde localizaría la lesión?

A. Esófago distal

B. Mediastino posterior

C. Pleura

D. Cavum

E. Língula
12. Un paciente consulta por pérdida de peso, dolor abdominal y trombosis venosa. En la exploración tiene
esplenomega-

lia e ictericia. En las pruebas de imagen se confirma la sospecha clínica de tumor abdominal de:

A. Hígado

B. Vesícula

C. Páncreas

D. Estómago

E. Colon

13. Si un paciente queda disfónico después de haber sido sometido a una hemitiroidectomía izquierda, es porque se
ha lesionado:

A. Nervio laríngeo superior izquierdo

B. Nervio frénico izquierdo

C. Cadena simpática izquierda

D. Nervio laríngeo inferior izquierdo

E. Nervio frénico

14. Tras hacer la historia clínica y ordenar las pruebas complementarias se llega al diagnóstico de que un paciente de
45 años tiene una apendicitis aguda sin p eritonitis que requiere una apendicectomía urgente, el paciente le pregunta sobre
la laparoscopía. Su contestación es:

A. La laparoscopía solo sirve para el diagnóstico en casos de apendicitis aguda

B. La cirugía laparoscópica solo está indicado para la co lecistectomía

C. La apendicitis aguda es una contraindicación absoluta para el abordaje laparoscópico

D. La apendicitis aguda sin peritonitis puede tratarse por laparoscopía y puede ofrecer algunas ventajas sobre la
laparotomía

E. La única indicación de cirugía laparoscópica es el plastrón inflamatorio palpable en fosa ilíaca derecha

15. Paciente de 52 años de edad que acude a consulta manifestando que desde hace tres días nota visión borrosa
con mo- lestias en ojo derecho que se acentúan cuando está en lugares muy iluminados o en la calle si hace sol. Al ser
explorado encontramos una discreta hiperemia conjuntival y tras la tinción con colirio de Fluoresceína sódica observamos
sobre la córnea central una figura lineal ramificada de unos 5mm de longitud que nos sugiere que estamos ante:

A. Queratoconjuntivitis epidémica

B. Queratitis bacteriana

C. Queratitis vírica

D. Queratitis por acanthamoeba

E. Queratitis medicamentosa

16. Paciente de 37 años que desde hace 4 días presenta dolor en región anal y desde hac e 2 días fiebre. En urgencia
se le observa una zona indurada tumefacta y enrojecida en la zona perianal derecha. A la presión es muy dolorosa. Se le
prescribe antibióticos y se le recomienda consulta en Cirugía a las 48 horas, ¿Cuál es el origen más probable de su
patogenia actual?

A. Fisura anal
B. Prolapso rectal

C. Fístula anorrectal

D. Hemorroides internos

E. Rectocele

17. Una paciente diabética de 65 años acude a urgencias por fiebre de 38.5°C y malestar general junto con síndrome
mic- cional. PA: 90/60 mmHg, FC: 105x’ y FR: 22x’ con SO2: 89%. En la exploración física destacaba PPL (+). En el labo -
ratorio sobresalía leucocitosis de 15700/mm3 (87%: neutrófilos) y creatinina de 1.4 mg/dL. Plaquetas: 90000; señale aquella
medida inicial cuyo retraso tenga una mayor influencia negativa en la evolución del cuadro:

A. Ingreso en UCI para ventilación mecánica asistida

B. Administración precozmente de bicarbonato

C. Administración de tratamiento inotrópico con dobutamina

D. Administración de antibióticos y estabilización hemodinámica

E. Administración precozmente de corticoides y proteína C activada

18. Mujer de 51 años de edad que consulta por debilidad generalizada y visión doble de 3 meses de evolución. Se le
realiza un test de edrofonio y una electromiografía que permiten un diagnóstico de miastenia gravis. En Rx. y TAC torácico
de localización anteroposterior de 5x4x4cm, y densidad homogénea. ¿Qué patología mediastínica debemos sospechar
como más probable?

A. Bocio endotorácico

B. Linfoma

C. Timoma

D. Teratoma

E. Tumor neurogénico

19. Un paciente de 65 años diagnosticado de EPOC, es trasladado al hospital por disnea progresiva y expectoración
purulenta. Durante el traslado en la ambulancia se le administra oxígeno y fluidoterapia. A su llegada el paciente está
obnubilado y tembloroso. Se auscultan crepitantes en la base derecha. La gasometría arterial muestra un pH: 7.08, PO2:
106. El tratamiento inicial más adecuado sería:

A. Reducir flujo de oxígeno a 1L/min. y administrar bicarbonato sódico

B. Mantener flujo de oxígeno y administrar acetazolamida

C. Intubación y ventilación mecánica

D. Retirar oxígeno y administrar bicarbonato sódico

E. Administrar epinefrina i.v. y bicarbonato sódico

20. Mujer de 35 años sin factores de riesgo cardiovascular, con historia de una muerte fetal de 20 semanas, que
presenta un infarto de la cerebral media derecha. En el hemograma presenta plaquetopenia y en el estudio de coagulación
se observa un alargamiento del TTPa que no corrige con plasma fresco congelado. ¿Cuál es el diagnóstico más probable?

A. Poliarteritis nodosa

B. Micropoliangeítis

C. Síndrome antifosfolípido

D. Hemofilia
E. Esclerosis sistémica

21. Mujer de 62 años que acude a la consulta por presentar dolor en el borde interno del antebrazo y a nivel del dedo
anular y meñique de la mano derecha, también explica hormigueos, frialdad y pérdida de sensibilidad. ¿Qué tipo de
neuropatía por compresión tiene el paciente?

A. Síndrome canal carpiano

B. Atrapamiento de nervio mediano y cubital a nivel de la muñeca

C. Compresión proximal del nervio cubital

D. Sección nerviosa cubital

E. Compresión del nervio radial

22. Mujer de 72 años, con debilidad marcada que le impide caminar, diarreas de 3 meses de duración, pérdida de
peso 15 kg en un año. Antecedentes no contributorios. Al examen: palidez severa de piel y mucosas, glositis, edema de
miem-bros inferiores. Hb: 4g%; VCM: 130, hcm: 37; leucocitos: 2,500; plaquetas: 25.000; creatininemía: 1mg%; glucosa:
160 mg%; lámina periférica: macrocitosis y polisegmentación de neutrófilos. ¿Cuál es su posibilidad diagnóstica?:

A. Leucemia aguda aleucémica.

B. Anemia megaloblástica por déficit de folatos.

C. Aplasia medular.

D. Anemia megaloblástica.

E. Púrpura trombótica trombocitopénica.

23. Una niña de 12 años presenta como mo lestiaprincipal ptosis palpebral y visión doble. El examen revela debilidad
de los músculos extraoculares, de los músculos del cuello y de los músculos faciales. Se observa debilidad progresiva ante
las contracciones musculares sostenidas o repetidas. Cuando se solicita a la paciente que mantenga la mirada dirigida
hacia arriba, se observa ptosis progresiva. La debilidad muscular mejora con el reposo. Los estudios de tiroides son
normales. El diagnóstico más probable es:

A. Parálisis de Bell.

B. Miastenia gravis juvenil.

C. Ptosis congénita.

D. Parálisis del nervio facial congénita.

E. Miositis osificante progresiva.

24. Un paciente con cerebro esclerosis que presenta disturbios mnésicos, hipomimia, bradicinesia, hipertonía,
Hiperre-

flexia profunda, Babinski bilateral, temblor de actitud, voz “monótona”, corresponde a la forma clínica siguiente:

A. Pseudobulbar.

B. Parkinson arteriopático.

C. Demencia arteriopática.

D. Forma lacunar.

E. Neurasténica.

25. Mujer de 19 años que en los últimos 15 días presenta febrícula, sudoración nocturna, astenia y dolor torácico
derecho. En la Rx de tórax se aprecia derrame pleural derecho importante, sin otras alteraciones. Se realiza toracocentesis
diagnóstica, obteniéndose un líquido serohemático con: pH 7,36, proteínas 4,5 g/dl (sangre 7,5), LDH 850 U/l (sangre 150),
glucosa 40 mg/dl y amilasa en rango de normalidad. El Gram no muestra gérmenes. La citología muestra pre- dominio de
linfocitos. Ante la sospecha diagnóstica, ¿cuál sería el siguiente paso a dar?

A. Repetir de nuevo la toracocentesis, para descartar que sea un empiema.

B. Realizar biopsia pleural.

C. Realizar toracoscopía.

D. Iniciar tratamiento antibiótico y esperar respuesta.

E. Esperar el resultado del cultivo de Lowenstein.

26. Paciente de 25 años de edad, hace dos semanas presentó cuadro de influenza. Desde ayer parestesias en
miembros infe- riores y debilidad progresiva que imposibilita deambular. Hoy pérdida de fuerza en miembros superiores y
dificultad respiratoria. Al examen: cuadriparesia hipotónica e hiporreflexia. El diagnóstico probable es:

A. Polimiositis aguda.

B. Síndrome de Guillain- Barré.

C. Polineuropatía carencial.

D. Mielitis transversa.

E. Botulismo.

27. Paciente de 40 años con disnea progresiva de esfuerzo hasta disnea de decúbito, p ulso con arritmia completa,
frecuen- cia cardiaca 156pm, soplo diastólico de tonalidad grave en area mitral PA 115/80 mmHg, crepitantes en ambas
bases pulmonares ¿cuál es el diagnóstico más probable?

A. Insuficiencia aortica

B. Coartación de aorta

C. Estenosis mitral

D. Insuficiencia mitral

E. Estenosis aortica

28. Un varón de 35 años lleva varios años trabajando de pintor y ahora presenta cefalea de varios meses de
evolución, difi- cultad de concentracion y artralgias. En la exploración física neuropatía periférica. Laboratorio: anemia
normocitica

¿cuál de estas pruebas de laboratorio confirmaría el diagnostico?

A. Concentración de plomo en el suero

B. Concentración de plomo en la sangre

C. Concentración de arsénico en la sangre

D. Concentración de arsénico en el suero

E. Concentración de cadmio en el suero

29. Paciente mujer de 45 años con enfermedad de Crohn y una pequeña fistula intestinal desarrolla tetania durante la
2da semana de nutrición parenteral. Los resultados de laboratorio incluyen del Ca 8.2 meq/L; Na 135 meq/L ; K 3.2 meq/L;
Cl 103 meq/L ; PO4 2.4 meq/L ; albumina 2.4 ; pH 7.48 ; bicarbonato 25 meq/L. La causa más probable de la tetania del
paciente es:

A. Hipomagnesemia
B. Hiperventilación

C. Hipocalcemia

D. Deficiencia esencial del ácido graso

E. Convulsiones focales

30. Un varón de 30 años ha experimentado dificultades para la deglución tanto de sólidos como de liquidos en los
últimos

6 meses. ¿Cuál es el diagnóstico más probable?

A. Carcinoma esofágico

B. Anillos de Schatzki

C. Acalasia

D. Estenosis esofágica benigna

E. Esófago de Barret

31. Un recién nacido postmaduro con el antecedente de asfixia intraparto presenta en el momento del nacimiento una
atelectasia pulmonar asociada a neumotórax y neumomediastino, presentando en la radiología condensaciones algo-
donosas diseminadas. ¿Cuál sería su diagnóstico?

A. Persistencia de circulación fetal

B. TTRN

C. SALAM

D. Neumonía estafilocócica

E. EMH

32. Un recién nacido varón presenta una malformación congénita que consiste en agenesia de timo y paratiroides,
arcoaórtico derecho, atresia esofágica y anomalías faciales. ¿Qué síndrome polimalformativo sospecharía?

A. Martin-Bell

B. Potter

C. Pierre-Robin

D. Prune-Belly

E. Di George

33. Un varón de 10 meses presenta desde hace unos días signos de coriza y febrícula, hace unas horas comienza
con un episodio agudo de dificultad respiratoria con tiraje intercostal y sibilancias en la auscultación. Los padres nos comen-
tan que nunca había sufrido un episodio similar. ¿Cuál es el agente etiológico que con más frecuencia relaciona este cuadro
clínico?

A. Virus sincitial respiratorio

B. Parainfluenzae

C. Adenovirus

D. Mycoplasma

E. Idiopático
34. Varón de 3 años de edad que, encontrándose previamente bien, comienza de forma brusca con fiebre, dol or de
gargan- ta, estridor, disnea, acúmulo de secreciones orales y emisión de saliva por la boca. Un hermano de 2 años lleva 72
horas ingresado por una meningitis bacteriana. El agente etiológico de ambos cuadros clínicos será con más probabilidad:

A. Providencia rettgeri

B. Haemophilus influenzae

C. Klebsiella pneumoniae

D. Streptococcus pneumoniae

E. Neisseria meningitidis

35. En relación a las etapas de desarrollo mamario, según Tanner, la presencia de montículos mamarios, incremento
de la areola y desarrolla de pezón en grado variable, corresponde a:

A. Etapa 5

B. Etapa 3

C. Etapa 2

D. Etapa 0

E. Etapa 1

36. Un recién nacido presenta un área edematosa de contenido seroso en la zona de presentación. Presenta alguna
pete- quia aislada, pero no existe colección de sangre. Se observa también cabalgamiento de parietales. Este cuadro
desapa- rece en 24 horas. ¿Cómo se denomina dicho cuadro?

A. Cefalohematoma

B. Caput venoso

C. Necrosis adiposa del tejido subcutáneo

D. Hemorragia subaponeurótica difusa

E. Caput succedaneum

37. Un bebé a término tiene una clínica de vómitos tardíos, retraso en la eliminación de meconio, episodios
alternantes de diarrea-estreñimiento y enterocolitis. Se le practica un tacto rectal observándose hipertonía con recto vacío.
Usted decide hacer una biopsia de colon con el hallazgo de ausencia de células ganglionares de los plexos mioentéricos en
un segmento colónico. ¿Cuál es el segmento que con más frecuencia se ve afectado en esta enfermedad?

A. Colon transverso

B. Recto-sigma

C. Colon descendente

D. Recto

E. Colon ascendente

38. Una madre acude con su hijo de 2 semanas al pediatra ya que le encuentra irritable, rechaza el alimento, tiene
diarrea mucosanguinolenta y tinte ictérico. A la exploración presenta distensión abdominal con palpación dolorosa y aspecto
séptico. En una radiografía descubre una perforación intestinal. ¿Cuál será la causa más frecuente de este hallazgo
teniendo en cuenta que el niño todavía pertenece al período neonatal?

A. Invaginación intestinal

B. Estenosis pilórica
C. Enterocolitis necrotizante

D. Celíaca precoz

E. Apendicitis aguda

39. Un niño de 7 meses deshidratado, con vómitos, diarrea y fiebre; nos llega la siguiente analítica: Hb 11,6 g %; Hto:
39%; Leucocitos: 14.900 mm3 (Neutrófilos: 63%; Linfocitos: 30%; Monocitos: 7%); Plaquetas: 322.000 mm3; Os- molaridad:
295 mOs/l; Na: 137 mEq/l; K: 5.6 mEq/l; Ca iónico: 1,2 mmol/l; pH: 7,20; pCO2: 25 mmHg; HCO3: 11 mEq/l; E.B.: -19 mEq/l;
Lactato: 5,3 mmol/l, Creatinina: 4,2 mg %. ¿Cuál es la valoración metabólica de esta deshi- dratación?

A. Deshidratación isotónica con acidosis mixta.

B. Deshidratación hipotónica con acidosis metabólica.

C. Deshidratación isotónica con acidosis metabólica.

D. Deshidratación hipertónica con acidosis metabólica.

E. Deshidratación isotónica con acidosis respiratoria.

40. Un niño de 2 años, previamente saludable, desarrolla, fiebre de 40,6º C y presenta convulsión generalizada que
dura 60 seg. El examen físico revela una infección respiratoria alta y los estudios de laboratorio son normales, mostrándose
asintomático al día siguiente. Ud. Explica a los padres que la convulsión es febril y que tiene buen pronóstico, no ne -
cesitando quimioprofilaxis con fenobarbital. Su opinión está basada en:

1) Estuvo previamente sano hasta el inicio de la convulsión.

2) La convulsión ocurrió luego del año de edad.

3) El episodio agudo duró menos de 15 minutos.

4) Al día siguiente no presentó manifestación neurológica residual

5) La convulsión estuvo asociada a una infección respiratoria al ta benigna.

A. La respuesta correcta es: B. 1,2 y 3.

C. 1,3 y 5.

D. 2,4 y 5.

E. 2,3 y 4.

F. 1,4 y 5.

41. ¿A los cuántos meses de edad se recomienda colocar la vacuna contra la hepatitis A?

A. 4

B. 9

C. 12

D. 2

E. 6

42. La asociación apropiada de antibióticos para tratar la sepsis en un recién nacido es:

A. Vancomicina + Amoxicilina

B. Amikacina + Penicilina

C. Amikacina + Ampicilina
D. Gentamicina + Cefalotina

E. Cloranfenicol + Ampicilina

43. ¿Cuál de los siguientes cambios es un fenómeno de adaptación?

A. Autolisis

B. Metaplasia

C. Apoptosis

D. Cariolisis

E. Cariorrexis

44. ¿Cuál de las siguientes vitaminas se comporta como hormona esteroidea?

A. A

B. B5

C. E

D. K

E. D

45. ¿Cuál de las siguientes alteraciones caracteriza a las células malignas?

A. Atrofia

B. Hipertrofia

C. Anaplasia

D. Metaplasia

E. Hiperplasia

46. Un paciente ha tenido un resultado positivo en un Mantoux que se la ha realizado por sospecha de tuberculosis.

¿Cómo se denomina la probabilidad de que este individuo tenga realmente la enfermedad cuando el test realizado ha
resultado positivo?

A. Especificidad

B. Riesgo atribuible

C. Valor predictivo positivo

D. Sensibilidad

E. Valor predictivo negativo

47. Entre los factores de riesgo modificables de la aterosclerosis se encuentran el tabaquismo, la obesidad, la vida
sedenta- ria, los trastornos de los lípidos, la hipertensión, la resistencia a la insulina, entre otros; como factores no
modificables están la edad, el sexo masculino y factores genéticos. Un varón de 35 años sufre un infarto agudo de
miocardio. Es fumador desde hace 20 años por lo que se le aconseja dejar de fumar. Con esto usted realiza una:

A. Promoción de la salud

B. Atención curativa

C. Prevención terciaria
D. Prevención primaria

E. Prevención secundaria

48. Se determina la sensibilidad de la prueba a partir de la siguiente fórmula

Resultado de la prueba Enfermos

SI NO

Positiva a b

Negativa c d

A. a/a+c

B. c/a+c

C. d/c+d

D. b/b+d

E. a/a+b

49. ¿Qué tipo de estudio epidemiológico emplearía en el estudio de una enfermedad rara, crónica, de larga evolución,
y de larga inducción?

A. Transversal

B. Cohortes

C. Test de Welch

D. Casos y controles

E. Incidencia

50. Un médico se encuentra realizando SERUMS en Cieneguilla y desea realizar una actividad de promoción en
salud. Su equipo de salud le envía el siguiente listado de actividades. Considerando el concepto de promoción. ¿Cuál es la
actividad que debe elegir?

A. Consejería grupal de adolescentes sobre la transmisión del VIH.

B. Charla a la comunidad en nutrición saludable.

C. Campaña de vacunación en menores de 5 años.

D. Búsqueda en la comunidad de sintomáticos respiratorios.

E. Control vectorial para disminuir casos de enfermedades metaxénicas.

51. Si el valor de una canasta básica familiar en Tarma es de S/. 1500 y el valor de la canasta sólo con alimentos es
de S/750, señale usted en que grupo se encontraría la familia “X” cuyo ingreso promedio mensual es de S/. 550 mensual:

A. Quintil 4.

B. Pobre no extremo.

C. Extrema pobreza.

D. Pobreza.

E. Clase media.
52. En el análisis FODA de un hospital, “los permanentes conflictos entre la dirección y el cuerpo médico”
corresponde a:

A. Debilidad.

B. Amenaza.

C. Fortaleza.

D. Oportunidad.

E. Recurrencia.

53. Varón de 18 años de edad, presenta un forúnculo en la cara y a los pocos días aparece edema con signos de
inflama- ción en la rodilla izquierda, asociado a fiebre. A los 3 días aparece tos, disnea, dolor torácico, fiebre elevada y
signos de toxicidad sistémica. Una radiografía de tórax muestra múltiples infiltrados nodulares y neumatocele. El agente
etiológico más probable es:

A. Staphylococcus aureus

B. Peptostreptococcus magnus

C. Streptococcus pyogenes

D. Pseudomonas aeruginosa

E. Mycobacterium tuberculosis

54. Paciente varón de 18 años que presenta limitación funcional y dolor en región de columna dorsal, tiene como
ante- cedente haber sufrido de “pleuresia” y enfermedad venérea .Es consumidor de queso fresco. Radiográficamente se
evidencia lesión en columna D12 con destrucción de las vértebras contiguas preservación de pedículos. El diagnóstico más
probable seria

A. Brucelosis

B. Neoplasia

C. Espondilitis anquilosante

D. Mal de pott

E. Sd de Reiter

55. Paciente con antecedente de haber tenido impétigo luego hipertensión, edema de párpados, oliguria, ictericia,
hígado aumentado de tamaño ¿Cuál es el diagnóstico?

A. Enfermedad de Berger

B. Glomerulonefritis post estreptocócica

C. GMN rápidamente progresiva

D. Glomerulopatía membranosa

E. Cambios mínimos

56. Paciente de 54 años de edad, procedente de Pucallpa. Acude a hospital de la capital porque tiene fiebre
intermitente y palidez. ¿Cuál es el diagnóstico más probable?

A. Hepatitis C

B. Fiebre tifoidea

C. Malaria
D. Brucelosis

E. Bartonelosis

57. Mujer de 32 años llega a consulta por cefalea intensa, vómitos y rigidez de nuca. El LCR presenta pleocitosis
linfoci- taria. ¿Cuál es el diagnóstico más probable?

A. Meningitis por hongos

B. Meningitis viral

C. Meningitis bacteriana

D. Hemorragia subaracnoidea

E. Meningitis por VIH

58. Paciente portador de p rótesis cardiaca metálica, alérgico a la penicilina, al que se le va a realizar broncoscopia
diag-nóstica. ¿Qué profilaxis antibiótica utilizaría?

A. Cefalexina

B. Eritromicina

C. Clindamicina

D. Vancomicina

E. Gentamicina

59. Anciana de 70 años sin antecedente de DM es llevada a emergencia por presentar estado mental alterado, cuadro
de deshidratación severa, el familiar solo informa que días antes presentó polidipsia, poliuria. El diagnóstico planteado ser ia.

A. DM tipo 1

B. DM tipo 2

C. Cetoacidosis diabética

D. Coma hiperosmolar no cetósico

E. Hipoglicemia

60. Un hombre asintomático presenta en dos análisis de sangre rutinarios glucemias basales de 132 y 130mg/dl.
¿Cuál será la actitud correcta?

A. Se le pide una curva de glucemia para diagnostico

B. Se inicia tratamiento con antidiabéticos orales

C. Se inicia tratamiento con dosis de insulina NPH

D. Se le pide nuevos análisis dentro de un año

E. Se le diagnostica diabetes mellitus tipo 2 y se le inicia tratamiento con dieta

61. Paciente varón de 65 años con antecedente de diabetes mellitus en control regular que acude a su cita con los
siguientes resultados: glicemia, creatinina sérica y electrolitos en límites normales, en el examen de orina se aprecia una
cruz de proteínas. En el mes previo sus presiones arteriales han fluctuado entre 150-160 / 70-80. ¿Cuál sería la mejor
terapia farmacológica para su hipertensión arterial?

A. Calcio antagonistas

B. Beta bloqueadores
C. Inhibidores de la enzima convertidora de angiotensina

D. Diuréticos tiazídicos

E. Diuréticos ahorradores de potasio

62. Paciente varón fumador de 68 años, que presenta tos con expectoración matutina esporádica desde hace 15 años
y que desde los últimos 3 la tos y la expectoración se han incrementado añadiéndose disnea al realizar grandes esfuerzos,
las molestias ceden parcialmente con el uso de broncodilatadores. ¿Cuál es la prueba que Ud. le solicitaría primero para
completar el diagnóstico?

A. Tomografía axial computarizada de tórax

B. Espirometría

C. Prueba de difusión de PCO2

D. Búsqueda de bacilos de Koch en el esputo

E. Test de hiperreactividad bronquial con metacolina

63. Paciente mujer de 65 años que acude a la consulta con dolor en rodilla derecha de 15 días de evolución, que se
incre- menta al caminar y cede al reposo, sin historia de trauma previo; tiene un índice de masa corporal en 32, aumento de
volumen de la rodilla derecha pero sin enrojecimiento ni incremento de temperatura; a la movilización pasiva se percibe
crujido. ¿Cuál es el diagnóstico más probable de la paciente?

A. Monoartritis infecciosa

B. Artropatía por cristales

C. Osteoartritis de rodilla

D. Artritis reumatoide de rodilla

E. Meniscopatía de rodilla derecha

64.Ante un varón con antecedentes de tabaquismo y alcoholismo que consulta porque desde hace meses nota una sen-
sación de cuerpo extraño al tragar, parestesias faríngeas, pinchazos en los oídos y una adenopatía cervical. ¿Qué debemos
sospechar?

A. Cáncer de las cuerdas vocales

B. Cáncer subglótico

C. Laringitis aguda

D. Cáncer de cavum

E. Cáncer de supraglotis

65. Ante un niño de 5 años con un cuadro de hipertensión endocraneal, alteraciones visuales e hipotalámicas, que
pre- senta una radiografía lateral del cráneo con calcificaciones en forma de paréntesis a nivel supraselar. ¿Cuál será su
diagnóstico presuntivo?

A. Meduloblastoma

B. Craneofaringioma

C. Adenoma hipofisario

D. Glioma del nervio óptico

E. Pinealoma productor de hidrocefalia


66. Paciente de 65 años, con cuadro de ictericia progresiva, coluria y dolor abdominal. En las últimas 58 horas se
agregan fiebre y escalofríos. ¿Cuál es el diagnóstico más probable?

A. Absceso hepático.

B. Colangitis aguda.

C. Hepatitis aguda.

D. Neoplasia de la vesícula biliar.

E. Ninguna de las anteriores.

67. Varón de 56 años de edad, desde hace 3 horas presenta dolor en la fosa ilíaca izquierda, fiebre, vómitos. Niega
moles- tias urinarias. ¿Cuál es la primera impresión diagnóstica?:

A. Intoxicación aguda por plomo

B. Apendicitis aguda

C. Cólico ureteral izquierdo

D. Crisis hemolítica

E. Diverticulitis colónica aguda

68. Paciente de 24 años de edad, que sufre un politraumatismo por caída del segundo piso y a las 6 horas fallece.
¿Cuál cree usted que es la causa más probable de muerte?:

A. Pancreatitis aguda

B. Edema cerebral

C. Lesión de colon

D. Perforación ileal

E. Hemorragia

69. Paciente de 50 años de edad que consulta por dolor en la fosa renal, poliaquiuria, disuria y hematuria. En el
análisis de orina se observa piuria y pH ácido con cultivos repetidamente negativos. ¿Cuál sería la primera posibilidad diag -
nóstica, de entre las siguientes?

A. Pielonefritis aguda.

B. Síndrome nefrítico.

C. Tuberculosis genitourinaria.

D. Prostatitis aguda.

E. Carcinoma renal de células claras.

70. A una mujer de 49 años de edad se le realiza un Papanicolaou y el resultado se informa como neoplasia
intracervical III o lesión intraepitelial escamosa de alto grado, usted indicaría el siguiente procedimiento :

A. Colposcopia y biopsia dirigida

B. Cono frio diagnostico

C. Histerectomía total

D. Inspección visual con ácido acético

E. Aplicación del test de Schiller


71. Ingresa una primigesta de 18 años de edad con 32 semanas de gestación refiriendo cefalea intensa y
epigastralgia tipo opresivo. Al examen tiene presión arterial 180/120 mm Hg. No hay dinámica uterina, presenta
movimientos fetales y los latidos del feto fluctúan entre 130 y 150, la altura uterina es de 25 cms. con feto en LCI. Los
reflejos patelares están en 3+/4+. ¿Cuál es el manejo farmacológico inmediato?

A. Sulfato de magnesio 4 gms EV en 20’, nifedipino 10 mg VO

B. Diazepam 10 mg EV, labetalol 200 mg EV

C. Sulfato de magnesio 10 gms IM, alfa metil dopa 1 gm VO

D. Fenitoina 900 mg VO, hidralazina 10 mg EV

E. Diazepam 5 mg EV, nifedipino 10 mg Sublingual

72. Gestante a término en fase activa de trabajo de parto, presenta ruptura de membranas hace 30 minutos, y usted
de- tecta frecuencia cardiaca fetal basal de 132 latidos por minuto. En 3 oportunidades ha presentado latidos fetales en 110
latidos por minuto durante 30 segundos fuera de las contracciones y asociado a movimiento del bebe. ¿Cuál es la causa
más probable de estas desaceleraciones en la frecuencia cardiaca fetal?

A. Compresión de cabeza fetal

B. Acidemia fetal

C. Compresión de cordón umbilical

D. Hipoxia fetal

E. Desprendimiento de placenta

73. Paciente en su tercer día de puerperio de cesárea por expulsivo prolongado, la intervención quirúrgica discurrió
sin complicaciones. El día de hoy presenta temperatura 38.50 C, pulso 120 por minuto, frecuencia respiratoria 20 por mi-
nuto, presión arterial 120/70 mmHg, llenado capilar < 2 segundos. Luce en regular estado, al examen muestra mamas
induradas y dolorosas, útero doloroso a la palpación con altura 3 cms. por encima del ombligo, la herida no muestra
flogosis, loquios escasos sin mal olor, el resto del examen es normal. ¿Cuál es el diagnóstico más probable?

A. Atelectasia pulmonar

B. Flebitis

C. Congestión mamaria

D. Endometritis

E. Infección urinaria

74. Primigesta de 34 semanas diagnosticada de infección por VIH a las 18 semanas iniciándose tratamiento con
zidovudi- na y lamivudina. Acude a control prenatal. ¿Cuál de las siguientes alternativas sería la más apropiada para el
manejo del parto?

A. Amniocentesis a las 38 semanas para madurez pulmonar

B. Cesárea segmentaria a las 40 semanas

C. Inducción de parto a las 39 semanas

D. Medir carga viral a las 36 semanas

E. Medir leucocitos CD4 a las 36 semanas

75. Primigesta de 35 semanas acude a emergencia refiriendo hace 30 minutos dolor abdominal y sangrado vaginal
oscuro de 300 cc. Al examen detecta Presión arterial 90/60 mm de Hg, pulso 120 por minuto, respiraciones 24 por minuto,
afebril. Abdomen con altura uterina de 40 cm, contracciones uterinas cada 2 minutos, 50 segundos de duración, intensidad
4 cruces, latidos fetales 100 a 120 por minuto; en el examen con especulo sangrado a través del cervix, el cual se encuentra
cerrado y largo. Se indica vía EV, hemograma grupo y Rh, prueba cruzada y perfil de coagulación.

¿Cuál debe ser la siguiente conducta?

A. Ecografia transvaginal

B. Perfil biofísico

C. Inducción de parto con oxitocina

D. Amniotomia y estimulación con oxitocina

E. Cesárea

76. Paciente mujer de 27 años de edad, con IMC de 30, presenta oligomenorrea, acné, hirsutismo y desde hace 3
años busca salir gestando. ¿Cuál sería el diagnóstico de la paciente?

A. Hipotiroidismo

B. Hiperplasia Suprarrenal

C. Síndrome de Ovario Poliquistico

D. Deficiencia de la Fase Lútea

E. Hiperprolactinemia

77. Mujer de 53 años de edad, quien cursa con sequedad vaginal, disminución del deseo sexual, disuria y poliaquiuria
con urocultivos negativos. Estos síntomas están asociados a disminución de:

A. Progesterona

B. FSH

C. Estrógenos

D. LH

E. Prolactina

78. Paciente mujer de 21 años de edad, con pareja estable, quien tuvo su segundo parto vaginal hace 3 días con
recién nacido sano. Planea amamantar exclusivamente. ¿Qué anticonceptivo seria el apropiado en este caso?

A. Anticonceptivo Oral Combinado

B. Ligadura Tubárica Bilateral

C. Preservativos y espermicida

D. Progestágenos Orales

E. Anticonceptivos Inyectables mensuales

79. Si un niño de 8 años hace shock anafiláctico por penicilina, el medicamento de elección es:

A. Clorfenamina

B. Suero fisiológico

C. Epinefrina

D. Dexametasona

E. Dopamina
80. La medicación que permite mejoría rápida de un lactante con crup infeccioso grave es:

A. Adrenalina en nebulización

B. Antibióticos

C. Dexametasona vía endovenosa

D. Ambiente húmedo

E. Salbutamol en inhalación

81. El diagnóstico de desnutrición aguda se hace en base a la relación:

A. Peso/talla2

B. Peso/edad

C. Peso/talla

D. Talla/edad

E. Talla/peso

82. La panencefalitis esclerosante subaguda de Dawson tiene como etiología al vi rus de:

A. Rubéola

B. Sarampión

C. Influenza

D. Varicela

E. Hepatitis B

83. En las niñas, el agente causal más frecuente de infección urinaria es:

A. Escherichia coli.

B. Ureoplasma.

C. Parvovirus.

D. Campilobacter.

E. Chlamydia trachomatis.

84. Un niño de 4 años de edad con un cuadro de fiebre, conjuntivitis, adenopatía cervical, lengua de fresa y edema
con descamación membranosa de los dedos de manos y pies, orientará nuestro diagnóstico hacia:

A. Enfermedad de Behcet

B. PAN

C. Forma juvenil del lupus discoide

D. Enfermedad de Still

E. Enfermedad de Kawasaki

85. La saturación de oxígeno media en la arteria pulmonar es aproximadamente de:

A. 0.33

B. 0.75
C. 0.5

D. En personas sanas es superior al 95%

E. Depende de varios factores, pero se mueve en el intervalo 80-95%

86. Su alteración es muy sensible en la fase inicial del trastorno tiroideo a veces el único cambio:

A. TSH

B. T3 libre

C. T3

D. T4 libre

E. T4

87. ¿Cuál de las siguientes acciones corresponde a la vitamina E?

A. Cicatrización de las heridas

B. Anitoxidante

C. Síntesis de algunos factores de la coagulación

D. Síntesis de colágeno

E. Regularización de los niveles de calcio y fósforo en sangre

88. Al nacer, el volumen sanguíneo es:

A. 65 cc. x Kg.

B. 80 cc. x Kg.

C. 110 cc. x Kg.

D. 125 cc. x Kg.

E. 150 cc. x Kg.

89. El contenido de hierro corporal total en el adulto normal está entre:

A. 5 y 10 mg.

B. 50 y 100 mg.

C. 25 y 50 mg. D. 0.5 y 1.5 mg.

E. 2 y 5 g.

90. El hierro molecular, Fe, es:

A. Almacenado primariamente en el bazo.

B. Excretado en la orina como Fe++.

C. Almacenado en el cuerpo en combinación con ferritina.

D. Absorbido en el intestino por la transferrina.

E. Absorbido en la forma férrica, Fe+3.

91. La vitamina B12, se absorbe en el:


A. Íleon distal.

B. Yeyuno proximal.

C. Duodeno.

D. Colon.

E. Yeyuno medio.

92. El sistema nervioso simpático que sale de la médula espinal:

A. Cesa de funcionar después de sección del bulbo.

B. Contiene solamente fibras adrenérgicas.

C. Contiene solamente fibras colinérgicas.

D. Incluye una sinapsis gangliónica.

E. Ninguna de las anteriores.

93. Se dice que el sistema extrapiramidal está conformado por estratos o niveles superpuestos. Estos niveles son:

1. Cuerpo estriado.

2. Núcleo óptico.

3. Hipotálamo.

4. Núcleos de la región subopto-estriada.

5. Sustancia reticular. Es cierto solamente: A. 1, 2.

B. 2, 3.

C. 3, 4.

D. 1, 4.

E. 1, 3.

94. En un centro de salud la enfermera da una charla sobre control prenatal a un grupo de madres, la semana
siguiente la obstetriz da la misma charla al mismo grupo de madres. La enfermera ordena a una técnica de enfermería que
realice una visita domiciliaria, mientras que el médico manda a la misma técnica que lo ayude en consultorio externo.
¿Estos problemas a que componentes de la administración corresponden?:

A. Planificación.

B. Organización.

C. Control.

D. Dirección.

E. Evaluación.

95. Un centro de salud programó para el año 2009 realizar 2 000 controles de crecimiento y desarrollo, para 1 500
niños menores de 5 años, utilizando 800 horas enfermera. AI finalizar el año atendió 200 consultantes nuevos, 300 reingre-
santes y 500 continuadores, utilizando 600 horas enfermera. El porcentaje de avance de meta en atenciones fue:

A. 50.

B. 85.
C. 25.

D. 75.

E. 100.

96. Se quiere graficar un conjunto de datos cuantitativos continuos, trabajados en una escala de razón. ¿Qué tipo de
grá- fico sería más adecuado?:

A. Gráfico de barras.

B. Gráfico circular.

C. Histograma.

D. Pictograma.

E. Gráfico semi-logarítmico.

97. En una muestra de 50 estudiantes la estatura promedio es de 1,65 m. Luego, considerando un intervalo de
confianza

del 95%, el límite inferior es de 1,56 m. y el superior:

A. 91,74 m.

B. 1,68 m.

C. 1,70 m.

D. 1,77 m.

E. No se puede calcular con la información disponible.

98. Una encuesta determina que las mujeres consumen bebidas alcohólicas unas 0,65 veces menos que los varones.
En este caso, la medición usada es un(a):

A. Proporción.

B. Fracción aritmética.

C. Porcentaje.

D. Mediana.

E. Razón.

99. En un país “X”, durante los últimos cinco años, el número total de casos de malaria mantiene una tendencia
ascen- dente, superior a la del crecimiento de la población. Este cambio anual en la frecuencia total de la malaria, tiene su
origen en la modificación de su:

A. Incidencia.

B. Letalidad.

C. Mortalidad.

D. Riesgo atribuible.

E. Riesgo relativo.

100. La tasa de incidencia de leucemia, en población expuesta a radiación ionizante es de 4 en mil y la población no
expues- ta de 0,5 en mil. ¿Cuál es el riesgo atribuible al factor de exposición?:

A. 8,0.
B. 4,5.

C. 3,5.

D. 4,0.

E. 0,5.

101.Son las células encargadas de la contracción de la herida, durante el proceso de cicatrización:

A. Neutrófilos.

B. Polimorfonucleares.

C. Fibroblastos.

D. Macrófagos.

E. Fibrocitos.

102. La cirugía moderna del cáncer gástrico, tiene fundamento en la disección de las diferentes estaciones
ganglionares. La extirpación del grupo N° 9, corresponde a:

A. Arteria coronaria estomáquica.

B. Arteria esplénica.

C. Arteria hepática.

D. Pedículo hepático.

E. Tronco celíaco

103. La pepsina es inactivada por:

A. La secreción acuosa del páncreas.

B. La concentración gástrica de hidrogeniones.

C. Las prostaglandinas gástricas.

D. El ácido desoxicólico.

E. La concentración de gastrina.

104. La mayor absorción de sodio se realiza en:

A. La porción ascendente del asa de Henle

B. La porción descendente del asa de Henle

C. El túbulo proximal

D. El túbulo distal

E. El túbulo colector

105. ¿Cuál es el antibiótico que inhibe la DNA girasa?

A. Ciprofloxacino

B. Penicilina

C. Amikacina
D. Clindamicina

E. Ceftriaxona

106. En relación a las fases del potencial de acción cardíaco:

A. La fase 4 en las células marcapaso es estable.

B. El nodo sinusal es el marcapaso cardíaco en condiciones normales porque la pendiente de la fase 4 es la menor.

C. La pendiente de la fase 0 de una célula marcapaso es mayor que en una célula del sistema de conducción.

D. En el nodo sinusal, el ion responsable de la fase 0 es el calcio.

E. El período refractario relativo engloba la fase 1 del potencial de acción.

107. Normalmente la válvula ileocecal se cierra en qué circunstancias:

A. Ocurre una onda peristáltica

B. Incrementa la presión ileal

C. Incrementa la presión colonica

D. Existe estimulación vagal

E. Se contrae el ciego

108. ¿Cuál de los siguientes cambios es un fenómeno de adaptación?

A. Autolisis

B. Metaplasia

C. Apoptosis

D. Cariolisis

E. Cariorrexis

109. ¿Cuál de las siguientes alteraciones caracteriza a las células malignas?

A. Atrofia

B. Hipertrofia

C. Anaplasia

D. Metaplasia

E. Hiperplasia

110. ¿Cuál de las siguientes acciones corresponde a la vitamina E?

A. Cicatrización de las heridas

B. Anitoxidante

C. Síntesis de algunos factores de la coagulación

D. Síntesis de colágeno

E. Regularización de los niveles de calcio y fósforo en sangre


111. Un centro de salud realizó la evaluación de sus servicios correspondientes al año 2009. Población total 12,000
habitan- tes. Atendidos 3000 personas, consultas médicas realizadas 6000 ¿cuál es la intensidad de uso o concentración
(%)?

A. 1.3

B. 4

C. 25

D. 2

E. 1

112. Se realizó un estudio para relacionar la ingesta rica/pobre en grasas y la existencia / no existencia posterior de
cáncer de próstata en adultos mayores de 40 años. La observación duró 10 años. ¿Cuál es el tipo de estudio de la
investiga- ción?

A. Casos y controles

B. Ecológica

C. Cohortes

D. Experimental

E. Transversa

113. En una localidad determinada, durante los recientes 3 años, el número total de casos de fiebre amarilla mantiene
una tendencia ascendente, superior a la del crecimiento de la población. Este cambio anual en la frecuencia total de fiebre
amarilla, tiene su origen en la modificación de su:

A. Incidencia

B. Letalidad

C. Mortalidad

D. Riesgo atribuible

E. Riesgo relativo

114. Un estudio observacional en el que se comparan 110 individuos con cáncer de senos paranasales con 220
individuos sin la enfermedad, corresponde a un estudio:

A. Longitudinal

B. Caso control

C. De cohortes concurrente

D. Transversal

E. Cohortes no concurrente

115. En un distrito rural del país se notificaron durante la primera semana de Octubre del 2005, dos casos locales o
autóc- tonos de rabia humana, enfermedad que no se observó́ en las últimas tres décadas. Esta situación se define como
una:

A. Endemia

B. Epidemia

C. Epizootia
D. Hiperendemia

E. Holoendemi

116. Se desarrolla un estudio con la finalidad de buscar la asociación entre el polvo de una mina y la presencia de
proble- mas respiratorios en la población. Se estudian dos poblaciones: Una expuesta al polvo de la mina y la otra de agri-
cultores; se hace seguimiento por un año y se registra la aparición de los casos en ambas poblaciones. ¿A qué tipo de
diseño epidemiológico corresponde el estudio?

A. Cohortes

B. Casos y controles

C. Cuasi experimental

D. Longitudinal

E. Transveral

117. Durante su ejercicio profesional, en el centro de salud se presenta un elevado número de casos de diarrea
acuosa, neumonía, desnutrición infantil y parasitosis. De acuerdo a su observación, ¿qué acciones preventivas
recomendaría?

A. Incrementar la compra de medicamentos para el centro de salud

B. Solicitar más personal profesional y técnicos de salud

C. Mejorar los servicios de emergencia del establecimiento

D. Ampliar el número de camas del establecimiento

E. Acceso a agua potable, mejoramiento de la vivienda y nutrición

118. En el análisis situacional de salud de un establecimiento se encuentra que su comunidad de referencia carece de
agua potable, el año anterior se presentaron 40 casos de cólera con 8 fallecidos, y el establecimiento no tiene protocolos de
atención. Según el análisis FODA institucional, la situación mencionada corresponde a:

A. Oportunidades y amenazas

B. Oportunidades y fortalezas

C. Debilidades y fortalezas

D. Amenazas y debilidades

E. Debilidades y oportunidades

119. En un centro de salud la enfermera da una charla sobre control prenatal a un grupo de madres, la semana
siguiente la obstetriz da la misma charla al mismo grupo de madres. La enfermera ordena a una técnica de enfermería que
realice una visita domiciliaria, mientras que el médico manda a la misma técnica que lo ayude en consultorio externo.
¿Estos problemas a que componentes de la administración corresponden?

A. Planificación

B. Organización

C. Control

D. Dirección

E. Evaluación
120. En un estudio para comprobar la relación entre el consumo de tabaco y el cáncer de vejiga, se siguió durante un
año a 1.000 personas, de las cuales 500 fumaban, y otras 500 no; de las 50 personas que padecieron cáncer de vejiga
durante este tiempo, 45 fumaban. ¿Cuál es el riesgo individual de enfermar entre los expuestos al tabaquismo?:

A. 0,09.

B. 0,10.

C. 0,01.

D. 1,00.

E. 1,11.

121. Niño preescolar presenta bruscamente tos intensa. Al examen físico pulmonar: timpanismo y sibilantes
unilaterales. Debe sospecharse:

A. Epiglotitis

B. Síndrome de escape de aire

C. Cuerpo extraño

D. Neumonitis intersticial

E. Crisis asmática.

122. Un niño de 3 años de edad presenta hace 3 días un cuadro catarral leve, con fiebre leve, adenopatías
retroauriculares y exantema maculopaular rosado de inicio en cara. El diagnóstico más probable es:

A. Eritema infeccioso.

B. Escarlatina.

C. Exantema súbito.

D. Rubéola.

E. Sarampión

123. Un lactante de seis meses, amamantado con leche materna, presenta desde hace 5 horas episodios repetitivos
de llanto intenso y vómitos. Presenta también desde hace 2 horas dos deposiciones semilíquidas y sanguinolentas con
moco. No se refiere fiebre. El diagnóstico más probable es:

A. Diarrea por Shigella.

B. Diarrea por Campylobacter.

C. Divertículo de Meckel.

D. lnvaginación intestinal.

E. Alergia a la proteína de leche.

124. Un recién nacido, de seis horas de vida, presenta párpados edematizados y conjuntivas enrojecidas con secreción
ocu- lar clara. El diagnóstico más probable es:

A. Conjuntivitis química

B. Dacriocistitis

C. Oftalmia neumococica

D. Oftalmia gonocócica
E. Conjuntivitis por clamydia

125. Un lactante de 1700 gr sufrió asfixia al nacimiento y después de maniobras de reanimación presentó numerosos
episo- dios apneicos. Al tercer día de vida empezó a vomitar y presentó distensión abdominal con evacuaciones
sanguinolen- tas. El diagnóstico más probable es:

A. Intususcepción

B. Enterocolitis necrotizante

C. Vólvulo

D. Aganglionosis

E. Gastroenteritis.

126. RN pretérmino que, a las 6 h de vida, presenta quejido espiratorio a la auscultación, ausencia de retracción
esternal, tiraje intercostal leve con movimientos toraco-abdominales sincrónicos y sin aleteo nasal. Su puntuación en el
Score de Silverman será:

A. 2

B. 3

C. 4

D. 5

E. 6

127. Un recién nacido de parto vaginal y peso al nacer de 4 kilos, presenta al examen: deformación de la cabeza que
sobre- pasa las suturas craneales y que deja fóvea a la presión. ¿Cuál es el diagnóstico más probable?

A. Caput succedaneum

B. Céfalohematoma

C. Fractura de cráneo

D. Rodete

E. Hidrocefalia

128. Un lactante de seis meses y sin antecedentes de importancia, presenta coriza y fiebre y tos desde hace tres días.
Desde hace 12 horas se agrega respiración rápida y al examen se auscultan roncantes y subcrepitantes. ¿Cuál es el
diagnós- tico más probable?

A. Neumonía

B. Bronquiolitis

C. Laringotraqueitís

D. Rinofaringitis

E. Crisis asmática

129. Un niño de 3 años traído a emergencia por haber presentado convulsiones tónico-clónicas generalizadas que
duraron 5’. Al examen: T: 40°C, se auscultan roncantes en ambos campos pulmonares y presenta déficit motor en miembro
in- ferior derecho y somnolencia post convulsión. No tiene antecedentes relevantes. ¿Cuál es el diagnóstico más probable?

A. Convulsión febril compleja

B. Episodio de escalofríos
C. Meningoencefalitis bacteriana aguda

D. Convulsión febril simple

E. Convulsión asociada a fiebre

130. Niño de 4 años que presenta tos y dificultad respiratoria todas las noches y durante la actividad física. ¿Cuál es el

tratamiento a seguir?

A. Corticoide inhalado a altas dosis y beta 2 de acción prolongada

B. Corticoide inhalado a baja dosis y adrenalina

C. Corticoide inhalado a baja dosis y cromonas

D. Bromuro de ipratropio y fluticasona

E. Corticoide inhalado de acción corta y teofilina

131. Después de haber sufrido un traumatismo craneal, un niño de 10 años tiene 4 puntos en la escala de Glasgow y
pre- senta respiración irregular, en este momento está indicado:

A. Practicarle intubación endotraqueal.

B. Administrarle naloxona.

C. Administrarle Ringer lactato.

D. Administrarle manitol.

E. Administrarle dexametasona.

132. La persistencia del sexto arco aórtico izquierdo y que se manifiesta en el periodo neonatal por dificultad
respiratoria,

precordio hiperdinámico y pulsos periféricos amplios es la:

A. Atresia pulmonar con septum interventricular intacto.

B. Persistencia del conducto arterioso.

C. Comunicación interventricular.

D. Conexión anómala de las venas pulmonares.

E. Tetralogía de Fallot.

133. En un neonato que en las primeras horas de vida presenta vómitos de contenido biliar, y cuya radiografía de tórax
y abdomen muestra el signo de la “doble burbuja”, el diagnóstico más probable es:

A. Estenosis congénita del píloro.

B. Atresia duodenal.

C. Invaginación intestinal.

D. Hernia diafragmática.

E. Malrotación intestinal.

134. Un escolar de seis años que presenta temperatura de 39 grados C., movimientos tónicos del miembro superior
dere- cho, midriasis y pérdida del estado de alerta, tiene una crisis convulsiva de tipo:

A. Tónico.
B. Mioclónico.

C. Parcial simple.

D. Parcial compleja.

E. Febril compleja.

135. Una lactante de 6 meses que tiene potasio sérico de 7.5 mEq/L y que además presenta arritmia cardiaca, debe
ser tratada con:

A. Bicarbonato de sodio, glucosa e insulina.

B. Gluconato de calcio, salbutamol inhalado y glucosa con Insulina.

C. Glucosa y furosemida.

D. Bicarbonato de sodio, sulfato de magnesio y furosemida.

E. Resinas de intercambio iónico, glucosa y furosemida.

136. Un niño de 10 años ingresa al hospital por presentar un cuadro clínico de tres semanas de evolución
caracterizado por edema generalizado. Las pruebas de laboratorio informan: hemoglobina 10.5 g/dl, leucocitos: 3 500/mm3,
Urea 15 mg/dl, creatinina 2 mg/dl, albúmina sérica 2.2 g/dl, globulinas séricas 4 g/dl, y colesterol total 280 mg/dl. El
diagnóstico más probable es:

A. Síndrome nefrótico.

B. Glomerulonefritis aguda.

C. Lupus eritematoso.

D. Hipotiroidismo.

E. Insuficiencia renal.

137. Un paciente de 7 años de edad presenta gingivorragia y lesiones petequiales en tronco y extremidades, sin otras
alte- raciones al examen físico. Hma: 8 940/mm3, Hb: 11.5 g/dl, Plaquetas: 60 000/mm3. El diagnóstico más probable es:

A. Púrpura Trombocitopénica idiopática

B. Linfoma de Hodking

C. Leucemia linfoblástica aguda

D. Síndrome de Wiskott Aldrich

E. Púrpura de schonlein Henoch

138. ¿Cuántos gramos de proteína le corresponden a un escolar de 30 Kg?

A. 30 gr/día

B. 15 gr/día

C. 20 gr/día

D. 1.5 gr/día

E. 2 gr/día

139. En una paciente que se le realiza una amniografia y se aprecia imagen en panal de abeja. A que patología nos
estamos refiriendo.

A. Aborto espontáneo
B. Embarazo ectópico

C. Pseudociesis

D. Down

E. Mola

140. Femenina de 22 años de edad, casada hace 6 meses, la cual refiere amenorrea de 16 semanas de evolución. A la
explo- ración se observa, signo de Chadwick, Godell y útero entre la sinfisis del pubis y cicatriz umbilical. Con estos datos
usted puede hacer el diagnóstico de

A. Posible embarazo

B. Embarazo confirmado

C. Embarazo positivo

D. Posibilidad de embarazo

E. Probable embarazo

141. Femenina de 23 años la cual cursa con su 38 semana de gestación. Refiere que hace 2 horas arrojo una masa
muco- sanguinolenta. A la exploración se observa cuello con 3 cm de dilatación, presentación cefálica en OIA, FCF 138.
Con estos datos usted haría el diagnostico de:

A. Placenta previa

B. DPPNI

C. Embarazo a término

D. Trabajo de parto en fase pasiva

E. Trabajo de parto en fase activa

142. En una paciente de 38 semanas de gestación la cual usted le pide un eco y este le reporta todo normal con líquido
am- niótico de 800 ml. Con estos datos usted hace el diagnostico de:

A. Polihidramnios

B. ECO normal

C. Embarazo normal

D. Insuficiencia útero placentaria

E. Oligohidramnios

143. Gestante a término de 35 años acude a emergencia por contracciones uterinas cada 3 minutos y sangrado vaginal
de 10cc. PA: 120/70, P: 84x’, FR: 16x’, ecografía del día informa: feto único en LCI, placenta previa marginal. LCF: 148x’, se
incida vía, análisis y se realiza TV en condiciones de operabilidad: incorporación: 90%, dilatación: 8cm, membranas
íntegras, altura de presentación: 0, variedad de posición: OIIA, pelvis ginecoide. ¿Cuál es la conducta a seguir?

A. Acentuación del trabajo de parto

B. Esperar a que continúe el trabajo de parto espontáneo

C. Parto instrumentado

D. Preparar para cesárea inmediata

E. Transfusión de sangre
144. Puérpera de parto gemelar que presenta sangrado vaginal profuso luego del alumbramiento. El diagnóstico más
pro- bable es:

A. Laceración de cuello uterino

B. Ruptura uterina

C. Coagulación

D. Atonía uterina

E. Retención de restos placentarios

145. Primigesta con 32 semanas de gestación por última regla, acude a emergencia por dolor en epigastrio e
hipocondrio derecho. PA: 180/110 mmHg, proteinuria ++, tacto vaginal: cérvix posterior, orificio externo cerrado, pelvis
ginecoide. El diagnóstico más probable es:

A. Preeclampsia severa y trabajo de parto

B. Preeclampsia superpuesta e hipertensión crónica

C. Preeclampsia severa

D. Preeclampsia severa y colecistitis

E. Preeclampsia severa y DPP

146. Una gestante a término de 18 años, sin antecedentes de importancia, con 180/100 mm Hg de presión arterial, en
2 tomas, y con una proteinuria de 350 mg en orina de 24 horas su diagnóstico más correcto sería:

A. Preeclampsia leve.

B. Hipertensión transitoria leve.

C. Preeclampsia severa.

D. Hipertensión transitoria severa

E. HTA crónica con PE severa

147. En una paciente que cursa su 32 semana d e gestación usted le pide un ECO ya que la altura del fondo uterino es
mayor a la esperada y el ECO le reporta que el líquido amniótico es de 2100 ml. Con esto usted puede decir que la paciente
tiene

A. Diabetes gestacional

B. Producto con anencefalia

C. Oligohidramnios

D. Producto con atresia esofágica

E. Polihidramnios

148. Mujer obesa en edad reproductiva, con oligomenorrea, infertilidad e hirsutismo. El diagnóstico más probable es:

A. Síndrome de ovario poliquístico

B. Tumor funcionante de ovario

C. Hiperprolactinoma

D. Hiperplasia suprarrenal

E. Síndrome de Kallman
149. Mujer de 35 años, acude a consulta por metrorragia de dos meses, sin otro antecedente de importancia. Examen
clínico: pálida, peso 61 kg, PA 110/76, pulso 68 X’, útero se palpa dos cm por encima del pubis. Al examen pélvico: útero
anterior con nódulo de 4 cm a nivel fúndico, anexos normales. Hemoglobina 9 g/dL. ¿Cuál es el diagnóstico más probable?

A. Endometrioma

B. Leiomioma uterino

C. Quiste ovárico

D. Sarcoma uterino

E. Teratoma

150. Una mujer de 53 años de edad que no ha menstruado durante un año ha comenzado con tratamiento de
restitución hormonal cíclico. Tiene ligero sangrado vaginal que dura dos días, mientras comienza su segundo ciclo de
restitución. Está sana, su índice de masa corporal (BMI) es de 21, su presión arterial es normal, y usó anticonceptivos
orales hasta los 42 años. Se rehúsa a una muestra endometrial. El siguiente paso más apropiado para tratar su sangrado
es:

A. Comenzar un calendario menstrual.

B. Hacer un Papanicolaou, lo que incluye una muestra endocervical.

C. Insistir en una muestra endometrial.

D. Realizar una colposcopia.

E. Realizar una ecografía transvaginal para medir el grosor endometrial.

151. Una mujer de 33 años no puede sentir el hilo de su dispositivo intrauterino. Su última menstruación fue hace una
semana. Una prueba sérica de embarazo es negativa. La mejor acción a seguir es:

A. Insertar otro DIU para reemplazar el que se perdió

B. Hacer una radiografía abdominal

C. Hacer una ecografía pélvica

D. Realizar una histerosalpingografía

E. Palpar el cuello uterino cuidadosamente para jalar el hilo

152. Una mujer de 29 años de edad G 6 P 1041 se presenta con retraso menstrual de 2 semanas; su régimen
catamenial es 3/28 regular, tiene un examen de gonadotrofina en orina positivo. Ha tenido su primer embarazo a término
normal, luego 2 abortos espontáneos, un aborto provocado y el último fue un embarazo ectópico. No refiere enfermedades
previas, tuvo salpingectomía derecha. ¿Cuál de las siguientes alternativas sería la primera medida a adoptar?

A. Dosaje sérico cuantitativo de HCG Beta.

B. Test triple (HCG, Alfa feto proteína, estriol)

C. Anticuerpos anticardiolipinas

D. VDRL

E. Ecografía transvaginal

153. Ingresa una primigesta de 18 años de edad con 32 semanas de gestación refiriendo cefalea intensa y
epigastralgia tipo opresivo. Al examen tiene presión arterial 180/120 mm Hg. No hay dinámica uterina, presenta
movimientos fetales y los latidos del feto fluctúan entre 130 y 150, la altura uterina es de 25 cms. con feto en LCI. Los
reflejos patelares están en 3+/4+. ¿Cuál es el manejo farmacológico inmediato?
A. Sulfato de magnesio 4 g EV en 20’, nifedipino 10 mg VO

B. Diazepam 10 mg EV, labetalol 200 mg EV

C. Sulfato de magnesio 10 g IM, alfa metil dopa 1 g VO

D. Fenitoína 900 mg VO, hidralazina 10 mg EV

E. Diazepam 5 mg EV, nifedipino 10 mg Sublingual

154. Una paciente de 47 años acude refiriendo sensación de bulto en vagina, no hay incontinencia urinaria, su última
regla fue hace dos años. Al examen de acuerdo a la clasificación POPQ presenta las siguientes medidas: Aa +2; Ba +3; C

-5; Hiato Genital (gH) 5; Cuerpo perineal (pB) 3; longitud total de vagina (Tvl) 10; Ap -1; Bp -2; D -6.

+2 +3 -5
5 3 10
-1 2 -6
¿Cuál es el grado de distopia de la paciente?

A. 0

B. 1

C. 2

D. 3

E. 4

155. Paciente mujer de 30 años de edad, que acude por dolor pélvico de una semana de evolución que se inicia dos
días después de haber sido sometida a una histerosalpingografía por infertilidad. Hace tres días refiere fiebre de 38 0C. Al
examen presenta dolor en abdomen bajo y se insinúa rebote, el examen bimanual del útero y anexos es difícil de evaluar
por el dolor, el resto del examen es normal. ¿Cuál es el diagnóstico más probable en este caso?

A. Apendicitis aguda

B. Endometriosis

C. Enfermedad pélvica inflamatoria

D. Ruptura folicular

E. Infección urinaria

156. Paciente varón de 19 años de edad con fractura de fosa orbitaria derecha, al examen presenta: Ptosis palpebral y
mirada hacia adentro y abajo, que nervios podrían encontrarse afectados:

A. IV nervio craneal y rama interna del oftálmico

B. VI nervio craneal y rama interna del oftálmico

C. III nervio craneal y rama interna del oftálmico

D. III y IV nervios craneales

E. III y VI nervios craneales

157. Paciente que viene a consultar por sufrir de tumoración en la parte lateral superior e inferior derecha del hígado,
evidenciada por tomografía hepática, se pregunta: ¿Cuáles son los segmentos hepáticos afectados?

A. I Y II
B. II Y IV

C. V Y VIII

D. V Y VI

E. VI Y VII

158. Paciente que sufre un accidente de tránsito por impacto o choque de vehículo con otro, por detrás. El conductor
del vehículo impactado resulto muerto asido al timón, se pregunta: ¿Qué articulación(es) vertebral resultaron compro-
metidos?

A. Las 3 primeras articulaciones interapofisarias cervicales

B. Articulación atloidoaxoidea media

C. Articulación interapofisaria de la prominente

D. Sexta articulación interapofisaria cervical

E. Articulación odontoatloidea

159. Ante un niño de 5 años con un cuadro de hipertensión endocraneal, alteraciones visuales e hipotalámicas, que
pre- senta una radiografía lateral del cráneo con calcificaciones en forma de paréntesis a nivel supraselar. ¿Cuál será su
diagnóstico presuntivo?

A. Meduloblastoma

B. Craneofaringioma

C. Adenoma hipofisario

D. Glioma del nervio óptico

E. Pinealoma productor de hidrocefalia

160. Para un paciente con antecedentes de nefropatía crónica y apendicitis aguda perforada con peritonitis localizada.

¿Cuál de las siguientes alternativas de manejo sería la más efectiva y segura?

A. Apendicectomía, drenaje y una asociación de Amikacina y Metronidazol por 48 horas.

B. Apendicectomía, drenaje y una asociación de Amikacina y Metronidazol por siete dÍas.

C. Apendicectomía, lavado de cavidad y una asociación de Cefuroxima y Clindamicina por siete dias.

D. Apendicectomía, drenaje y una asociación de Cefuroxima y Metronidazol por siete días.

E. Apendicectomía, drenaje y antibiótico-profilaxis con cualquiera de los esquemas mencionados.

161. Un paciente de 60 años acude a emergencia con quemaduras de 2do grado en la región anterior del tronco, los
genita-les y el dorso de la mano derecha. ¿Qué porcentaje de superficie corporal quemada presenta?

A. 20%.

B. 18%.

C. 11%.

D. 36%.

E. 38%.
162. Chofer que sufre accidente de tránsito (choque) es llevado a emergencia porque presenta dolor abdominal. Al
examen clínico presenta PA 80/40 mmHg. Pulso 105 por minuto, despierto, dolor abdominal difuso a la palpación, reacción
peritoneal dudosa, ruidos hidroaereos escasos. Su conducta a seguir será:

A. Hospitalizar al paciente y programar a sala de operaciones por tratarse de un traumatismo abdominal cerrado.

B. Enviar al paciente a su domicilio e indicarle que regrese si empeora.

C. Mantener al paciente en observación con hidratación parenteral, sonda Foley, control de hematocrito seriado.

D. Colocar vía parenteral, aplicar antiespasmódico y solicitar TAC abdominal.

E. Realizar paracentesis diagnóstica para determinar si es traumatismo.

163. Al ser atropellado, un hombre de 52 años recibe una contusión directa en la pelvis. En la exploración física se
encuen- tra sangre en la uretra; al efectuar el tacto rectal se percibe que la próstata está desplazada. El diagnóstico clínico
más probable es:

A. Hematoma prostático

B. Lesión vesical.

C. Laceración de la uretra.

D. Laceración del trígono.

E. Desgarro perineal.

164. Para un paciente de 16 años, con diagnóstico de torsión testicular de ocho horas de evolución, el tratamiento más
adecuado consiste en practicar la:

A. Orquiectomía.

B. Destorsión manual.

C. Orquidopexia.

D. Destorsión quirúrgica.

E. Resección del área afectada.

165. Una mujer de 20 años recibió un golpe contuso en la región lumbar izquierda. Acude al hospital 8 días después
con ictericia generalizada, dolor en el hombro izquierdo, temperatura de 38.5 grados C., frecuencia cardiaca de 90 por
minuto y presión arterial de 90/60 mmHg. El procedimiento quirúrgico más adecuado en este momento es:

A. Esplenectomía.

B. Nefrectomía.

C. Drenaje del hematoma.

D. Laparotomía exploradora.

E. Lavado peritoneal.

166. Varón de 78 años, con antecedente de hiperplasia prostática, que presenta elevación del PSA (100 ng/ml) y
lumbago. En el tacto rectal se encuentra una próstata grande, de consistencia dura en ambos lóbulos, de superficie nodular
y de bordes no bien definidos. AL realizarle los procedimientos que corresponden se realiza el diagnostico de adeno-
carcinoma de próstata pobremente diferenciado que afecta ambos lóbulos y que infiltra las vesículas seminales, con
metástasis ósea. ¿Cuál es el tratamiento más adecuado?

A. Extirpación total de la próstata


B. Quimioterapia enérgica

C. Radioterapia pelviana externa

D. Braquiterapia prostática.

E. Hormonoterapia

167. Una mujer de 24 años que acusa visión borrosa y enrojecimiento del ojo derecho de aproximadamente 15 días de
evo- lución. Hace 5 días se agrega dolor en el mismo ojo acompañado de lagrimeo. El examen revela AV de 20/400 en ese
ojo, inyección conjuntival periquerática y una pupila irregular aparentemente arreactiva. En un caso como éste, ¿cuál sería
la causa más probable del síndrome?

A. Glaucoma agudo congestivo

B. Escleritis necrotizante

C. Neuritis óptica

D. Iridociclitis aguda

E. Queratoconjuntivitis por cuerpo extraño

168. Paciente post operada de apendicitis + peritonitis. Presenta cuadro respiratorio con tos y bulto que protruye a
través de herida operatoria con asas intestinales exteriorizadas. Se trata de:

A. Eventración

B. Evisceración

C. Fuga intestinal

D. Estrangulación

E. Hernia inscional

169. Paciente mujer de 52 años que acude a la emergencia por presentar dolor abdominal y tumoración en región
umbilical de 12 horas de evolución, se trata de reducir y con mucho esfuerzo se consigue pero protruye nuevamente de
inmedia- to, se trata de una hernia:

A. Indirecta

B. Inguino-escrotal

C. Incoercible

D. Encarcelada

E. Por deslizamiento

170. Paciente mujer de 64 años con cuatro días de enfermedad, caracterizado por dolor abdominal tipo cólico,
náuseas, vómitos fecaloideos, dificultad para defecar y no eliminación de flatos; al examen: distensión abdominal, ausencia
de ruidos hidroaéreos, poco depresible. Ecografía: Vesícula pequeña, de paredes engrosadas, con cálculos, colédoco 6
mm; Radiografía de abdomen: niveles hidroaéreos, edema de pared i ntestinal, neumobilia. ¿Cuál es la causa más probable
de obstrucción intestinal?

A. Hernia crural complicada.

B. Íleo biliar.

C. Cáncer de ciego.

D. Bridas y adherencias.
E. Hernia interna.

1 71. Mujer de 62 años que acude a la consulta por presentar dolor en el borde interno del antebrazo y a nivel del dedo

anular y meñique de la mano derecha, también explica hormigueos, frialdad y pérdida de sensibilidad. ¿Qué tipo de

neuropatía por compresión tiene el paciente?

A. Síndrome canal carpiano

B. Atrapamiento de nervio mediano y cubital a nivel de la muñeca

C. Compresión proximal del nervio cubital

D. Sección nerviosa cubital

E. Compresión del nervio radial

172. Un hombre asintomático presenta en dos análisis de sangre rutinarios glucemias basales de 132 y 130mg/dl
¿Cuál será la actitud correcta?

A. Se le pide una curva de glucemia para diagnóstico

B. Se inicia tratamiento con glibenclamida

C. Se inicia tratamiento con dosis de insulina NPH

D. Se le pide nuevos análisis dentro de un año

E. Se le diagnostica diabetes mellitus tipo 2 y se le inicia tratamiento con dieta

173. Paciente adulto traído a la Emergencia por los Bomberos, quienes lo encontraron inconsciente en la calle. Al
examen: pálido sudoroso, sin signos de agresión. PA: 120/60 mmHg, FC: 100 por minuto, FR: 12 por minuto, saturación de
oxígeno 95%, pupilas centrales de 3,5 mm, fotorreactivas lentas. Su diagnóstico más probable es coma…:

A. Hiperglicémico

B. Hipoglicémico

C. Barbitúrico

D. Hipoxémico

E. Vigil

174. Mujer de 38 años de edad, obesa, con trastorno del sensorio, sequedad de piel, caída del cabello, bradipsiquia e
hipo- tensión arterial. Dosaje de T4 disminuida y TSH elevada. ¿Cuál es el tratamiento de elección?:

A. Hormona tiroidea de inicio sin corticoides

B. Hormona tiroidea a dosis altas y corticoides a dosis bajas

C. Corticoides a dosis altas, luego hormona a tiroidea a dosis bajas

D. Corticoides y luego adicionar hormona tiroidea

E. Hormonas tiroideas, corticoides y diuréticas

175. Varón de 56 años, presenta dolor retroesternal súbito e intenso de 6 horas de evolución. El EKG revela onda Q y
seg- mento ST elevado en DII, DIII, AVF ¿Cuál es el diagnóstico y que fármaco usaría?

A. Tromboembolismo pulmonar / heparina.

B. Infarto agudo de miocardio / heparina.


C. Pleurodinea / antiinflamatorios no esteroideos.

D. Trombosis pulmonar / trombolítico.

E. Infarto agudo de miocardio / trombolítico.

176. Paciente de 13 años, 15 días antes viajó a Piura. Presenta fiebre desde hace 5 días, artralgia, cefalea, dolor retro
ocular y erupción cutánea. ¿Cómo se clasifica el caso?

A. Probable dengue con signos de alarma

B. Probable dengue grave

C. Probable dengue sin signos de alarma

D. Dengue confirmado

E. Dengue descartado

177. Mujer de 24 años con varios días de fiebre, de mayor intensidad por las tardes. Recibe tratamiento sintomático sin
mejoría; hace 3 días se agrega dolor a nivel sacro iliaco, persistiendo el cuadro febril. ¿Cuál es el diagnóstico más
probable?

A. Salmonelosis

B. Malaria

C. Pielonefritis aguda

D. Leptospirosis

E. Brucelosis

178. Una mujer de 64 años notó lesiones del tipo ampollosas en sus muslos y axilas. Eran pruriginosas, pero no dolían.
No tenían otros síntomas. En la exploración las lesiones son grandes, a tensión, en forma de ampollas de contenido seroso.
La biopsia confirmo el diagnóstico de penfigoide ampolloso. Cuál de las siguientes características histológicas es típica de
esta condición:

A. Cambios inespecíficos

B. Depósitos de inmunoglobulina A (igA)

C. Lesiones dentro de la epidermis (acantolisis)

D. Depósitos de inmunoglobulina M (igM)

E. Lesiones de la membrana basal

179. Es una causa de acidosis con hiato aniónico elevado:

A. Cetoacidosis diabética

B. Diarrea

C. Acidosis tubular renal

D. Inhibidores de la enzima convertidora de angiotensina

E. Hipopotasemia

180. Una mujer de 85 años tiene lesiones ampollosas extensas en el abdomen que aparecen y desaparecen de
manera espon- tánea, sin ningún tratamiento, El signo de Nikolsky es negativo. Cuál de los siguientes diagnósticos es el
más probable
A. Pénfigo vulgar

B. Dermatitis herpetiforme

C. Herpes gestacional

D. Eritema multiforme

E. Penfigoide ampolloso

181. Masculino de 40 años con antecedente de ingesta crónica de esteroides y salicilatos, que ingresa a urgencias por
dolor abdominal que inicio 6 horas antes, el cual se inició de manera súbita en epigastrio. Se difundió a todo el vientre y que
describe como muy intenso. Está en posición de gatillo y no quiere movilizarse; aumenta su dolor a la palpación. El
abdomen se palpa en madera. A la percusión hay ausencia de la matidez hepática y se ausculta silencio abdominal.

T.A. 140/90, pulso 100, respiración 20 y temperatura 37.5, su diagnóstico más probable es:

A. Oclusión intestinal

B. Apendicitis aguda

C. Pancreatitis

D. Cólico renal

E. Ulcera péptica perforada

182. Cuando al explorar la presión venosa yugular, se encuentra que el paciente tiene una onda “v” gigante, ¿cuál es el
diagnóstico más probable?

A. Estenosis aórtica

B. Insuficiencia mitral

C. Insuficiencia tricuspídea

D. Estenosis mitral

E. Insuficiencia aórtica

183. Un hombre de 72 años, oxigenodependiente, con antecedente de tabaquismo intenso desde su juventud, inicia su
pade- cimiento actual hace 4 días con disnea, tos en accesos, expectoración purulenta, fiebre y dolor torácico. La
radiografía de tórax muestra infiltrado basal y broncograma aéreo. El agente etiológico más probable en este caso es:

A. Klebsiella pneumoniae.

B. Haemophilus influenzae.

C. Mycoplasma pneumoniae.

D. Estreptococcus pneumoniae.

E. Moraxella catarrhalis.

184. Varón de 58 años quien tiene diagnóstico desde hace 6 años de colitis ulcerativa, presenta desde hace 5 días
fiebre, dia- rrea inflamatoria, y dolor abdominal. Al realizarle rectosigmoidoscopía el resultado fue úlceras y exudado
purulento y hemorrágico. .Luego de 72 horas de tratamiento con antibióticos y corticoides se asocia hipotensión, taquicardia
y dolor abdominal intenso. En la radiografía de abdomen se encuentra dilatación de 9 cm del colon ¿Cuál es el diagnós - tico
más probable y el tratamiento más adecuado?
A. Colitis pseudomembranosa asociada / Imipenem

B. Perforación de colon / Colectomía de urgencia

C. Brote severo de colitis ulcerativa / Corticoides y antibióticos

D. Sobreinfección / Moxifloxacino

E. Megacolon tóxico / Colectomía si no hay mejoría en 24 h

185. Paciente varón de 59 años, fumador, con diagnóstico de carcinoma broncogénico en pulmón izquierdo. Se le
realizan los estudios correspondientes encontrándose un nódulo ganglionar en la región paratraqueal baja derecha .Si
aplica- mos la clasificación TNM ¿Cuál sería el resultado?

A. T4.

B. N1.

C. N3.

D. N2.

E. M1.

186. Ante la sospecha de quilotórax en un paciente que tiene un dosaje de triglicéridos en el líquido pleural de
100mg/dl

¿Cuál es el estudio que se de realizar en el líquido pleural?

A. LDL en el líquido pleural

B. VLDL en el líquido pleural

C. Quilomicrones en el líquido pleural

D. Gradiente de albúmina sérico .líquido pleural

E. IDL en el líquido pleural

187. Paciente masculino de 19 años de edad, quien acude a consultar porque presenta episodios intermitentes de
disnea, la cual llega a ser de pequeños esfuerzos, el cuadro se acompaña de tos irritativa y de ¨silbidos¨; es
predominantemente nocturno, y este último episodio ha durado más de 24 horas. A la exploración física se encuentra con
frecuencia res- piratoria de 28 por minuto, taquicárdico 120 por minuto, con aleteo nasal y tiros intercostales, y en la
auscultación de tórax se encuentran sibilancias difusas bilaterales. Con estos datos su sospecha clínica es:

A. Neumotórax espontáneo primario

B. Fibrosis pulmonar idiopática

C. Enfisema pulmonar

D. Asma bronquial

E Neumonía

188. Mujer de 42 años acude por consulta por prurito generalizado. Ella niega otros síntomas. En los exámenes de
labora- torio se encuentra una fosfatasa alcalina aumentada en 10 VNS. Para confirmar una etiología hepatobiliar se solicita
gammaglutamiltransferasa la cual esta aumentada 5 VNS ¿cuál es el marcador serológico que se debería solicitar?

A. Anticuerpos antinucleares

B. Antimusculo liso
C. Microsomico hepatorrenal

D. Anticuerpos antimitocondriales

E. Alfafetoproteina

189. Hombre de 53 años acude a Emergencia por dolor abdominal severo, náuseas y vómitos alimentarios. Exámenes
au- xiliares: Leucocitos: 18 000/ mm3 y amilasa sérica marcadamente elevada. ¿Qué otra anormalidad en los exámenes de
laboratorio podría encontrarse?

A. Hipoglicemia

B. Hipercolesterolemia

C. Hiperglicemia

D. Hipercalcemia

E. Hipercalemia

190. Varón de 70 años presenta lesiones eritematosas y ampollares pruriginosas que predominan en las axilas, en la
ingle y muslos. Hay algunas lesiones en los antebrazos y en los miembros inferiores, además lesiones orales
moderadamente dolorosas. El signo de Nikolsky es negativo. No hay compromiso de los ojos. El diagnóstico más probable
es:

A. Dermatitis herpetiforme

B. Pénfigo vulgar

C. Penfigoide buloso

D. Penfigoide cicatricial

E. Epidermolisis bulosa

91. Paciente varón de 40 años presenta desde hace 3 meses lesiones papulosas poligonales, localizadas en la cara
anterior de las muñecas y tobillos, acompañadas de prurito .El estudio histopatológico de una de las lesiones muestra una
der- matitis de interfase con degeneración vacuolar de las células basales. ¿Cuál es diagnóstico?

A. Pitiriasis rosada

B. Papulosis linfomatoide

C. Psoriasis eruptiva

D. Liquen plano

E. Acantosis nigricans

192. Paciente de 75 años que consulta por una mácula pigmentada heterocroma , de contorno irregular, de 2 x 3 cm de
diámetro, localizada en la mejilla derecha, que ha tenido un crecimiento muy lento en los últimos años ¿Cuál sería el
diagnóstico más probable?

A. Melanoma lentiginoso acral

B. Lentigo maligno

C. Carcinoma basocelular pigmentado

D. Nevo de Ota
E. Eritema fijo pigmentario

193. Paciente mujer de 40 años de edad, que acude a Emergencia por presentar hace dos semanas dolor intenso en la
cara anterior del cuello, que aumenta con la deglución, y malestar general. Al examen físico: dolor a la palpación en la re -
gión anterior del cuello, con leve crecimiento asimétrico del lado izquierdo de la tiroides. El diagnóstico más probable es
tiroiditis:

A. Aguda

B. Subaguda

C. De Hashimoto

D. Tuberculosa

E. De Riedel

194. Mujer de 20 años, presenta confusión, somnolencia, convulsiones tónico-clónicas generalizadas, al examen
rigidez de nuca, brudzinski positivo, vibraciones vocales abolidas y egofonía en base de hemitórax derecho. Punción
lumbar: pleocitosis, glucosa: 15, proteínas 150, ADA: 7. Toracocentesis: exudado. Western blot para cisticercosis en sangre:
negativo. HIV: negativo. TAC cerebral: múltiples imágenes heterogéneas hipo e hiperdensas redondeadas en el parén-
quima cerebral. El diagnóstico más probable es:

A. Neurocisticercosis.

B. Tuberculosis cerebral.

C. Toxoplasmosis cerebral.

D. Criptococosis cerebral.

E. Melanoma metastásico.

195. Una paciente de 25 años, sin antecedentes de interés acude a su consulta por presentar molestias precordiales
inter- mitentes, a veces prolongadas, que no son claramente opresivas y que en ocasiones se modifican con la respiración y
los movimientos. La exploración física es anodina salvo que en la auscultación cardíaca se pone de manifiesto un click
mesosistólico con un soplo suave telesistólico. Teniendo en cuenta la sospecha diagnóstica, cuál sería el tratamiento más
indicado para esta paciente:

A. AAS, nitratos y calcioantagonistas.

B. AAS a dosis antinflamatorias y reposo en cama al menos 2 semanas.

C. Corticoides a dosis altas y, si no hay respuesta, añadir inmunosupresores.

D. Hielo, AINEs y vendaje compresivo.

E. Propranolol.

196. Mujer de 35 años, que desde hace 6 meses refiere dolor y edema en ambas manos. Examen físico: Dolor con
aumento de volumen en articulaciones metacarpofalángicas proximales y desviación cubital. ¿Cuál es el diagnóstico más
pro- bable?

A. Artritis reumatoide

B. Fiebre reumática

C. Lupus eritematoso sistémico

D. Artritis gotosa

E. Artritis infecciosa
197. Paciente mujer de 20 años que presenta debilidad muscular en miembros inferiores; hiporeflexia, disminución del

tono muscular y se queja de parestesias, disestesias y allodinea. Cuál es el diagnóstico más probable:

A. Polineuropatía.

B. Síndrome piramidal.

C. Síndrome de motoneurona del asta anterior.

D. Síndrome de placa mioneural.

E. Mielitis transversa.

198. Mujer diabética de 42 años es llevada a emergencia por presentar un cuadro de sopor alteración del sensorio.
Glucosa en sangre: 350mg/dl. ¿Cuál de los siguientes hallazgos confirma el diagnóstico de cetoacidosis?

A. Polidipsia, polifagia y poliuria.

B. Sudoración, enfriamiento generalizado y trastorno de conciencia.

C. Deshidratación severa hiperosmolaridad y convulsiones.

D. Dolor abdominal, respiratorio de Kussmaul y cetonuria.

E. Fiebre, decaimiento y malestar general.

199. ¿Qué medicamento elegiría en primer lugar para prevenir nuevos episodios maníacos en un paciente
diagnosticado de trastorno maníaco-depresivo y sin otros trastornos somáticos acompañantes?:

A. Carbamazepina.

B. Propanolol.

C. Carbonato de litio.

D. Diazepán.

E. Clonidina.

200. ¿Cuál de las siguientes actuaciones es la más importante en la prevención del fracaso renal agudo?:

A. Administración de manitol

B. Administración de furosemida

C. Evitar los aminoglucósidos

D. Valorar y corregir la hipovolemia

E. Vigilar la diuresis
SIMULACRO 5A
1. Mujer de 28 años, vida sexual activa, presenta hiporexia, fiebre, pCO2: 19 mmHg; PaO2: 68 mmHg. SaO2: 80% con FiO2: 0.21.
escalofríos, disuria, polaquiuria y dolor lumbar. Examen: febril, ¿Cuál es el tratamiento farmacológico inicial?
puño percusión lumbar bilateral positivo. ¿Cuál es el A. Enoxaparina 1mg/Kg SC cada 12 horas
diagnóstico probable? B. Fondaparinux 7.5 mg IM cada 24 horas
A. Pielonefritis C. Tinzaparina 15 U/Kg cada 24 horas
B. Cistitis D. Warfarina 5 mg VO cada 24 horas
C. EPI E. Daltaparina 50 Ul VO cada 24 horas
D. Anexitis 5. Varón de 50 años, refiere tos con expectoración blanquecina,
E. Endometritis diaforesis vespertina y pérdida de peso desde hace un mes.
2. Mujer de 25 años diabética mal controlada, acude a emergencia Examen: murmullo vesicular pasa bien en ambos campos
deshidratada y somnolienta. Examen: FC: 90 X´, FR: 28 X´, PA: pulmonares, no estertores patológicos. Radiografía de tórax:
110/70 mmHg. Laboratorio: glicemia: 280 mg/dL, glucosuria y infiltrado apical derecho. ¿Cuál es el diagnóstico más probable?
cetonuria. ¿En qué segmento de la nefrona se ha superado el A. Aspergilosis pulmonar
mecanismo de transporte máximo de glucosa? B. Tuberculosis pulmonar
A. Túbulo distal C. Neumonía atípica
B. Asa de Henle D. EPOC
C. Túbulo proximal E. Silicosis
D. Túbulo colector 6. Mujer de 45 años, obesa, dos horas después de haber sido
E. Glomérulo operada de histerectomía total, súbitamente presenta dolor
3. Mujer de 65 años, consulta por pérdida de peso, astenia y intenso en el pecho, disnea, lipotimia y sensación de muerte
parestesias en manos y pies desde hace un año. En los últi- mos inminente. Examen: no ingurgitación yugular, reforza- miento
dos meses se añade palpitaciones y ageusia. Antecedente de del segundo ruido en foco pulmonar, cianosis y taquicardia. Rx
gastritis crónica. Examen: palidez con ictericia leve, mucosa oral de pulmones: normal. ¿Cuál es el diagnóstico más probable?
pálida, lengua roja y depapilada. Hemoglobina: 6.5 g/dL, A. Tromboembolismo pulmonar
leucocitos: 4200 x mm3, plaquetas: 120000 x mm3; aumento de B. Coartación de aorta
LDH y de bilirrubina indirecta. ¿Qué examen se solicita para C. Infarto de miocardio
determinar la etiología de la anemia? D. Aneurisma de aorta
A. Test de Coombs E. Neumonía aspirativa
B. Ferritina sérica 7. Varón de 20 años con diagnóstico de asma bronquial desde
C. Ácido fólico hace cinco años y en tratamiento con salmeterol de larga data.
D. Vitamina B 12 ¿Cuál de los siguientes efectos adversos está relacionado a su
E. Reticulocitos administración?
4. Mujer de 42 años, luego de viaje prolongado, refiere dolor e A. Bradicardia
inflamación de las pantorrillas y bruscamente dificultad B. Hiperkalemia
respiratoria; por lo que acude a emergencia. Examen: PA: C. Hipocalcemia
100/60 mmHg, FC: 118 X’, FR: 26 X’, peso: 88 kg. Pulmo- nes D. Hiperglicemia
normales. Extremidad inferior derecha con aumento de E. Disminución de ácidos grasos libres
volumen y signo de Homans positivo. Gases arteriales: pH: 7.20;
8. Varón de 18 años presenta tos seca, sensación de opresión en E. Benzodiazepinas
el tórax y disnea intermitente que empeora en el invierno desde 15. Varón de 25 años con fiebre reciente de cinco días de evolución
hace 2 años. Examen: sibilantes en ambos campos pulmonares. y antecedente epidemiológico de exposición a dengue, refiere
Se sospecha de asma bronquial. ¿Qué paráme- tro de la función artralgias, mialgias, cefalea, dolor retro-ocular, dolor lumbar y
pulmonar se encuentra más disminuido en el paciente? se evidencia erupción cutánea. ¿A qué tipo de caso de la
A. Volumen espiratorio forzado al primer segundo clasificación de dengue corresponde?
B. Volumen de reserva inspiratoria A. Descartado
C. Volumen de ventilación pulmonar B. Sospechoso con signos de alarma
D. Capacidad inspiratoria C. Confirmado por laboratorio
E. Capacidad vital D. Probable sin signos de alarma
9. Mujer de 53 años, acude por cansancio, sequedad de piel, caída E. Grave
de cabello y aumento de peso. Se sospecha hipotiroi- dismo 16. Mujer de 30 años, VIH positivo, hace 15 días presenta fiebre y
primario. ¿Cuál de los siguientes hallazgos confirma el cefalea. Examen: signos meníngeos presentes. Cultivo de
diagnóstico? líquido cefalorraquídeo positivo a Cryptococcus neoformans.
A. TSH baja y T4 baja ¿Cuál es el tratamiento de inducción para la menin- gitis?
B. TSH elevada y T4 normal A. Anfotericin B + Flucitosina
C. TSH normal B. Trimetroprim + Sulfametoxazol
D. TSH normal y T4 baja C. Ganciclovir + Corticoides
E. TSH elevada y T4 baja D. Fluconazol + Corticoides
10. Mujer de 60 años, consulta por debilidad muscular, poliuria, E. Sulfadiazina + Pirimetamina
polidipsia, dolores articulares y estreñimiento. Antece- dente: 17. Varón de 35 años, con cefalea diaria de varias semanas de
úlcera péptica. Examen: PA: 150/95 mmHg. EKG: QT corto y evolución que le impiden conciliar el sueño, se acompaña de
onda T ensanchada. Laboratorio: calcio sérico y paratohormona congestión nasal, rinorrea y epífora. Examen: ptosis palpebral,
elevadas. ¿Cuál es el diagnóstico? miosis, exoftalmos y anhidrosis derecha. ¿Cuál es el diagnóstico
A. Hiperparatiroidismo secundario más probable?
B. Diabetes mellitus tipo II A. Cefalea tensional
C. Hiperparatiroidismo primario B. Cefalea en racimos
D. Osteodistrofia renal C. Migraña
E. Síndrome pluriglandular D. Arteritis temporal
11. Mujer de 35 años, quien desde hace aproximadamente 7 meses E. Tumor cerebral
se encuentra constantemente preocupada por la enfer- medad 18. Varón de 48 años con antecedente de tabaquismo, refiere
de su madre, se muestra intranquila, nerviosa y sufre de cefaleas esporádicas desde hace 5 años, se automedica con
insomnio, en la última semana se levanta varias veces durante analgésicos. Desde hace 3 meses las cefaleas son constantes y
la noche. Se siente triste y evita las reuniones sociales. ¿Cuál es se acompañan de mareos e insomnio. Examen: lúcido,
el diagnóstico más probable? colaborador, IMC: 32, PA: 150/100 mmHg, FC: 85 X’.
A. Ansiedad generalizada Cardiovascular: RC: rítmicos regulares y no soplos. Resto de
B. Trastorno fóbico examen físico sin alteraciones. ¿Cuál es la conducta terapéutica
C. Estrés traumático inicial?
D. Trastorno obsesivo A. Cambios en los estilos de vida
E. Trastorno bipolar B. Solo monoterapia anti hipertensiva
12. Varón de 40 años con historia personal donde prima la C. Iniciar terapia combinada anti hipertensiva
búsqueda de su propia satisfacción a costa del sufrimiento de D. Uso de antiagregantes plaquetarios
los demás, sin capacidad de cambio ni culpa a pesar de las E. Uso de hipolipemiantes
consecuencias. ¿Qué trastorno de personalidad presenta? 19. Mujer de 65 años, consulta por dolor en columna dorso lumbar
A. Limítrofe de dos años de evolución. Se plantea el diagnóstico de
B. Antisocial osteoporosis. ¿Qué examen solicita para confirmar el
C. Obsesivo diagnóstico de osteoporosis?
D. Paranoide A. Densitometría ósea
E. Esquizoide B. Resonancia magnética con secuencia espectroscópica
13. Varón de 70 años, presenta: halitosis, ronquera, disfagia a C. Resonancia magnética con secuencia de difusión
sólidos y líquidos, regurgitaciones de alimentos sin digerir y mal D. Resonancia magnética funcional
olientes. Antecedente de faringitis crónica, niega otra E. Tomografía axial computarizada ósea
enfermedad. ¿Cuál es el diagnóstico más probable? 20. Varón de 57 años, obeso, consulta por polidipsia y poliuria.
A. Divertículo de Zenker ¿Cuál es un criterio que contribuye al diagnóstico de diabetes?
B. Acalasia A. Glicemia al azar de 160 mg/dL
C. Candidiasis esofágica B. Glicemia postprandial menor a 160 mg/dL
D. Cáncer de estómago C. Hemoglobina glucosilada menor a 6.5%
E. Esclerodermia D. Test de tolerancia a la glucosa menor de 140 mg/dL
14. Adolescente de 15 años, es traído por presentar euforia y E. Glicemia en ayunas superior a 126 mg/dL
ansiedad. Examen: FC: 120 X´, FR: 28 X´, PA: 160/110 mmHg, T: 21. Paciente de 26 años, con amenorrea primaria, desarrollo
38°C, ojos: midriasis bilateral. Antecedente: epistaxis y rinorrea mamario normal, con escaso vello púbico y axilar. Tiene informe
crónica. ¿Cuál es la sustancia causante de este cuadro? de cariotipo XY. ¿Cuál es el diagnóstico?
A. Heroína A. Aplasia cervical
B. Marihuana B. Septum vaginal transverso completo
C. Ácido lisérgico C. Insensibilidad a los andrógenos
D. Cocaína D. Síndrome de Turner
E. Síndrome de Mayer Rokitansky A. Paratohormona
22. ¿Cuál de los siguientes, es un criterio que define el síndrome B. Tiroxina
nefrótico? C. Eritropoyetina
A. Disminución de las globulinas séricas D. Renina
B. Proteinuria mayor de 3.5 g / 24 horas E. Grelina
C. Presencia de cilindros hemáticos en orina 32. La mayor parte de hierro en el organismo se encuentra como…
D. Disminución de los triglicéridos A. Hemosiderina.
E. Disminución de eritropoyetina B. Mioglobina.
23. En pacientes con insuficiencia renal crónica. ¿Cuál de los C. Ferritina.
siguientes hallazgos es indicación de diálisis de emergencia? D. Hemoglobina.
E. Apoferritina.
A. Hiperkalemia severa 33. ¿Cuál de las siguientes sustancias aumenta la secreción de
B. Derrame pleural gastrina?
C. Alcalosis metabólica A. Glucagon
D. Hiponatremia severa B. Somatostatina
E. Hipoxemia severa C. Secretina
24. Para el diagnóstico de anemia ferropénica. ¿Cuál es el marcador D. Adrenalina
de laboratorio más sensible? E. Péptido intestinal vasoactivo
A. Hemosiderina 34. ¿En qué tipo de célula se produce principalmente la
B. Transferrina testosterona?
C. Hierro sérico A. Germinativa
D. Ferritina sérica B. Sertoli
E. Saturación de hierro C. Paneth
25. ¿Cuál es el trastorno más frecuente asociado al síndrome de D. Leydig
dificultad respiratoria aguda (SDRA)? E. Intercalada
A. Embolia grasa 35. ¿Cuál es el tipo de epitelio que constituyen los folículos
B. Pancreatitis tiroideos?
C. Sepsis A. Simple cúbico
D. Tuberculosis miliar B. Estratificado plano
E. Edema pulmonar neurogénico C. Simple cilíndrico
26. ¿Qué segmentos pulmonares compromete típicamente la D. Estratificado cúbico
tuberculosis post primaria o del adulto? E. Seudoestratificado
A. Apicales y posteriores de lóbulos superiores 36. ¿Dónde se sintetiza la hormona cardionatrina?
B. Anteriores y laterales de lóbulos inferiores A. Miocardio
C. Superiores y basales de lóbulos medios B. Epicardio
D. Superiores y basales medios de lóbulos inferiores C. Endocardio
E. Anteriores y lingulares de lóbulos inferiores D. Subendotelio
27. En el tratamiento de la crisis asmática. ¿Qué grupo E. Pericardio
farmacológico se indica inicialmente? 37. ¿Cuál de los siguientes músculos de la pared torácica contribuye
A. Beta 2 agonistas de acción larga a determinar el volumen de reserva espiratoria?
B. Beta 2 agonistas de acción corta A. Intercostal externo
C. Metilxantinas B. Intercostal interno
D. Corticosteroides C. Subcostal
E. Anticolinérgicos D. Serrato anterior
28. ¿Cuál es la causa más frecuente de embolia de origen cardíaco? E. Escaleno
A. Infarto agudo de miocardio 38. ¿Cuál de los siguientes enunciados caracteriza la ubicación de la
B. Fibrilación auricular arteria pulmonar derecha?
C. Miocardiopatía dilatada A. Posterior a la vena pulmonar inferior
D. Endocarditis infecciosa subaguda B. Anterior a la aorta descendente
E. Insuficiencia aórtica C. Posterior a la aorta ascendente
29. ¿En qué derivaciones del EKG se observan las alteraciones de un D. Posterior al bronquio principal derecho
infarto lateral de miocardio? E. Anterior a la aorta ascendente
A. DI, aVL 39. ¿A partir de qué estructura se desarrollan las células
B. V1,V2 yuxtaglomerulares?
C. V3,V4 A. Túbulo proximal
D. DI,V1 B. Mesangio
E. DII, aVF C. Arteriola aferente
30. ¿Cuál es el método ideal para el diagnóstico de insuficiencia D. Capilar peritubular
venosa periférica? E. Asa de Henle
A. Prueba de Perthes 40. ¿Cómo se encuentra la presión oncótica plasmática en los
B. Pletismografía tejidos quemados?
C. Flebografía A. Estable
D. Doppler manual B. Elevada
E. Prueba de Brodie-Trendelenburg C. Disminuida
31. ¿Cuál de las siguientes hormonas interviene directamente en la D. Inestable
excreción renal de fosfato? E. Muy elevada
41. Mujer de 23 años, acude a la consulta por presentar congestión B. III-B
ocular bilateral, que se corrobora al examen, sin ante- cedente C. I
de traumatismo ocular. ¿Cuál es la causa más frecuente de ojo D. II
rojo que presenta la paciente? E. IV
A. Iritis aguda 48. Varón de 20 años, llega a emergencia presentando herida por
B. Uveítis anterior aguda arma blanca a nivel sub escapular izquierda y dificultad
C. Glaucoma agudo respiratoria. Se diagnostica hemoneumotórax izquierdo. ¿Cuál
D. Conjuntivitis es el procedimiento a seguir?
E. Abrasión de la córnea A. Toracotomía mínima con drenaje cerrado
42. En el ojo que ha sufrido una lesión penetrante y después de B. Toracotomía en sala de operaciones
tomar las medidas locales oftalmológicas. Es aconsejable C. Toracocentesis evacuatoria
prevenir un probable traumatismo posterior. ¿Qué fármaco se D. Videotoracoscopía
administra? E. Observación + oxigenoterapia
A. Antiemético 49. Mujer de 60 años con estreñimiento crónico, dolor y sangrado
B. Antihipertensivo rojo rutilante a la defecación, acompañado de protru- sión de
C. Diurético de asa un “bulto” por el ano que la reduce con la mano. ¿Cuál es el tipo
D. Antiinflamatorio de hemorroides que presenta?
E. Diurético osmótico A. Interna de primer grado
43. Mujer de 80 años llega a la consulta con dolor post traumático B. Interna de cuarto grado
de hombro derecho y limitación de movimiento. Exa- men: C. Interna de tercer grado
brazo con hematoma en cara interna del 1/3 superior con cierta D. Externa trombosada
angulación. ¿Cuál es el diagnóstico? E. Interna de segundo grado
A. Fractura del cuello del húmero 50. Mujer de 60 años politraumatizada, con diagnóstico de fractura
B. Fractura de clavícula de pelvis estable. Examen: PA: 90/60 mmHg, abdo- men
C. Luxación acromio clavicular ligeramente doloroso sin reacción peritoneal. Ecografía FAST
D. Fractura de escápula negativa. ¿Cuál es el manejo inicial?
E. Luxación escápulo humeral A. Laparoscopia diagnóstica
44. Varón de 48 años, consulta por presentar progresivamente B. Tomografía abdominal
pérdida de audición en el oído derecho, acompañado de C. Reposición de volumen
vértigos, acúfenos pulsátiles, dolor tenebrante profundo, D. Laparotomía exploradora
parálisis facial y supuración. ¿Cuál es el diagnóstico probable? E. Lavado peritoneal diagnóstico
A. Tumor de oído medio o interno 51. Varón de 30 años, politraumatizado por accidente de tránsito,
B. Síndrome vestibular es evaluado en emergencia. Examen: confuso, pálido, pulso:
C. Otitis media supurada >140 X’, hipotenso, FR: 40 X’ y diuresis: 5 ml/h. Abdomen
D. Laberintitis distendido y doloroso; se sospecha hemoperitoneo.
E. Otitis externa supurada ¿Qué clase de shock hemorrágico presenta?
45. Durante una colecistectomía laparoscópica a un hombre de 40 A. III
años por colelitiasis, antes de concluir la cirugía se per- cata la B. I
presencia de bilis en el campo operatorio. Al revisar se C. II
encuentra sección total del colédoco que mide 8 mm de D. IV
diámetro, razón por la que se convierte la cirugía y se decide E. V
reparar la vía biliar. ¿Cuál de las siguientes alternativas 52. Mujer de 45 años, acude por escape involuntario de orina
quirúrgicas es la más apropiada? desencadenada por actividad física y grandes esfuerzos desde
A. Hepato-yeyunoanastomosis en Y de Roux término lateral hace 2 años. G:5 P:5005. En la evaluación no es posible realizar
B. Colédoco-duodenoanastomosis término lateral pruebas urodinámicas. Examen: maniobra de Valsalva positiva.
C. Colédoco-duodenoanastomosis latero lateral ¿Qué tipo de incontinencia urinaria presenta?
D. Reparación del colédoco + dren de Kerh A. Rebosamiento
E. Colédoco-gastroanastomosis B. Neurogénica
46. Varón de 24 años, que hace 3 horas sufre accidente de tránsito. C. Urgencia
Hemodinámicamente estable. Refiere dolor abdominal y D. Mixta
nauseas. Examen: dolor abdominal en epigastrio contractura y E. De esfuerzo
reacción peritoneal generalizada. Leucocitos: 18.000 x mm3. 53. Varón de 75 años que refiere estar en tratamiento por
¿Cuál es el órgano lesionado? hiperplasia benigna de próstata; sin embargo nota hematuria e
A. Hígado infecciones urinarias repetidas. ¿Qué solicita inicialmente para
B. Intestino delgado descartar cáncer de próstata?
C. Bazo A. Ecografía prostática
D. Riñón B. Citología de la orina
E. Páncreas C. Cistoscopía
47. Varón de 70 años, es operado con el diagnóstico de hernia D. Dosaje de antígeno específico de la próstata
inguinal izquierda, en el hallazgo operatorio se encuentra que la E. Tomografía
hernia es de tipo indirecto, con anillo interno dilatado, saco 54. Mujer de 70 años con diagnóstico de colecistitis aguda.
herniario que comprime medialmente la fascia transversal del Antecedente: insuficiencia cardíaca congestiva y ortopnea.
triángulo de Hesselbach, con contenido deslizado de vísceras en ¿Cuál es la escala ASA que le corresponde desde el punto de
forma de pantalón. ¿Cuál es el tipo de hernia dentro de la vista anestesiológico?
clasificación de Nyhus? A. III
A. III-A B. II
C. IV C. Gris
D. V D. Verde
E. VI E. Rojo
55. La anestesia raquídea se logra mediante la inyección del 62. Un Médico interviene un brote de enfermedad transmitida por
anestésico a nivel: alimentos (ETA) en trabajadores de una fábrica; identificando
A. Epidural como factor común el almuerzo consumido el día anterior en el
B. Subaracnoideo comedor institucional. Para lo cual estudió a 20 trabajadores
C. Local afectados asignando un testigo para cada uno; encontrando
D. Sitio operatorio que de 5 posibles causas, el alimento asociado fue la mayonesa
E. Plexo lumbar con un OR de 3.5 y un IC: 1.9 a 7.2. ¿Qué tipo de estudio realizó
56. En el Centro de Salud San Camilo se está elaborando el Plan de para encontrar el alimento asociado a la ETA?
Salud Local requiriéndose la asistencia técnica (apoyo técnico, A. Casos y controles
recursos y ejecución de proyectos). Según el MAIS Basado en B. Cohortes
Familia y Comunidad. ¿En cuál de los paquetes está C. Transversal
comprendida la asistencia técnica? D. Ecológico
A. Promoción de comunidades saludables E. Correlacional
B. Producción de servicios 63. ¿Cuál es el propósito de la prevención primaria?
C. Metas de desarrollo sanitario local A. Evitar estados graves de enfermedad
D. Desarrollo institucional B. Evitar surgimiento de patrones de vida social
E. Apoyo técnico de los servicios C. Limitar incidencia mediante control de factores de riesgo
57. El médico jefe de un establecimiento de salud realiza un taller D. Aumentar curación de pacientes
con su equipo de gestión. En base al ASIS identifican y priorizan E. Reducción del progreso de complicaciones
problemas. Programan tareas y recursos que garanticen la 64. Al proceso de valorización razonada de factores: sociales,
ejecución de actividades para el logro de objeti- vos y metas del económicos, demográficos, políticos, sanitarios y otros; con el
siguiente año fiscal. ¿Qué tipo de planificación han fin de tomar decisiones sobre la cantidad, calidad y tipo de
desarrollado? recursos de salud para cubrir las demandas y problemas de
A. Táctica salud en un periodo determinado. ¿Cómo se le denomina?
B. De recursos A. Organización de sistemas de salud
C. Estratégica B. Análisis situacional
D. Operativa C. Planificación estratégica
E. Contingencial D. Planificación en salud
58. En un establecimiento I-4, la evaluación de indicadores del E. Salud pública
último quinquenio muestra cambio de las primeras causas de 65. ¿Cuál de las siguientes alternativas caracteriza a la “Distribución
morbilidad y de las prioridades sanitarias de la demanda, sin Estándar”?
incremento de la población total. ¿Qué decisión es- tratégica A. Media 1 y desviación estándar 1
debe tomar e implementar el equipo de gestión? B. Mediana 0 y desviación estándar 0
A. Replantear la misión del establecimiento C. Media 0 y desviación estándar 1
B. Solicitar cambio del establecimiento a nivel II D. Media 1 y desviación estándar 0
C. Implementar servicio de hospitalización E. Media 0 y moda 1
D. Realizar mercadeo social para incremento de partos 66. Mujer de 30 años acude a emergencia por presentar sangrado
E. Mejorar las técnicas de diagnóstico de laboratorio vaginal abundante hace 3 días, tomó anticonceptivo oral de
59. En un centro de salud se utiliza un mapa epidemiológico emergencia hace 6 días. RC: 3/28. FUR: hace 15 días. Al examen:
consignando el tipo de aprovisionamiento de agua en los útero de 7x4x3 cm. Resto del examen normal.
hogares. Mensualmente se ubica en el mapa los casos de EDA, ¿Cuál es el examen a solicitar para definir la probable causa del
observándose la relación de los hogares con suministro de agua sangrado?
inadecuado, lo cual facilita la planificación local para el control A. Histerosalpingografía
del EDA. Esta información está disponible en… B. Ecografía abdominal
A. El monitoreo situacional C. Dosaje de progesterona
B. Las estadísticas de gestión sanitaria. D. Ecografía transvaginal
C. El avance de metas sanitarias. E. Histerosonografía
D. La sala situacional. 67. Mujer de 29 años, con 10 semanas de embarazo por FUR,
E. El monitoreo epidemiológico integral G:2 P:0010. Citología de cuello uterino: células glandulares
60. El alcalde de Oxapampa se reúne con todos los sectores y atípicas de significado incierto. ¿Cuál es la conducta a seguir?
organizaciones representativas del distrito, para elaborar el A. Conización
presupuesto participativo y el plan de desarrollo local. ¿Qué B. Control citológico posparto
dimensión de la promoción de la salud aplicó? C. Determinar ADN-VPH
A. Cultural D. Biopsia de cérvix
B. Social E. Colposcopía
C. Económica 68. Mujer de 42 años, casada sin hijos, acude por presentar
D. Psicosocial irregularidad en su ciclo menstrual hace 5 meses, sofocos,
E. Política sudoración nocturna y cambios de carácter. FUR: hace 2 meses,
61. En vigilancia de Aedes, al elaborar un mapa de riesgo no usa métodos anticonceptivos. Subunidad beta: negativo.
entomológico, localidad en escenario II, mediano riesgo, con ¿Cuál es el diagnóstico probable?
índice aédico: 1 - < 2%. ¿Qué color le corresponde? A. Perimenopausia
A. Blanco B. Hipotiroidismo
B. Amarillo C. Menopausia
D. Post menopausia D. Uso de misoprostol
E. Hipertiroidismo E. Legrado uterino
69. Mujer de 30 años acude por hipermenorrea de un año de 76. En un estudio de colposcopía. ¿Cuál es un hallazgo anormal?
evolución, además de cansancio fácil, refiere que es usuaria de A. Epitelio cilíndrico
anticonceptivos orales combinados. ¿Cuál es el diagnóstico? B. Epitelio escamoso original
A. Hiperplasia endometrial C. Epitelio captador de yodo
B. Síndrome de ovario poliquístico D. Zona de transformación tipo I
C. Hipertiroidismo E. Zona de transformación tipo II
D. Endometriosis pélvica 77. De los siguientes hallazgos. ¿Qué criterios mayores son
E. Miomatosis uterina considerados para el diagnóstico de enfermedad inflamatoria
70. Mujer de 38 años acude por presentar leucorrea y sangrado pélvica?
post coital hace 6 meses. FUR: hace 8 días. Primera re- lación A. Dolor pélvico y dolor a la palpación anexial
sexual: 16 años. N° de parejas sexuales: 4. Método B. Fiebre (T>38°C) y dolor pélvico
anticonceptivo: ritmo. PAP: LIE de alto grado. Biopsia por C. Sensibilidad hipogástrica y hemograma con leucocitosis
colposcopia: Ca in Situ. ¿Cuál es la conducta a seguir? D. PCR aumentada y dolor anexial a la movilización del cérvix
A. Histerectomía ampliada a vagina E. Líquido libre en fondo de saco por ecografía y dolor pélvico
B. Histerectomía total 78. ¿Cuál es la etiología más frecuente del aborto espontáneo?
C. Histerectomía radical A. Endometriales
D. Braquiterapia B. Constitucionales
E. Conización fría del cérvix C. Infecciones
71. Gestante de 25 años, con embarazo a término, feto vivo, D. Ambientales
ponderado fetal 3800 g, presenta contracciones uterinas E. Cromosómica
irregulares durante 3 días; los tactos vaginales repetidos en 79. ¿Cuál de las siguientes alternativas es indicación absoluta de
periodos de 6 horas demuestran que la dilatación cervical no se cesárea?
ha iniciado. ¿Qué tipo de dinámica uterina presenta? A. Macrosomía fetal
A. Hipodinamia secundaria B. Cesárea anterior
B. Hipodinamia primaria C. Miomatosis múltiple
C. Inercia verdadera D. Condilomatosis perineal
D. Inercia mixta E. Placenta previa total
E. Hipoinercia 80. En una deflexión de III Grado. ¿Cuáles son las estructuras
72. Paciente de 23 años, G2 P1102, acude al consultorio anatómicas palpables al tacto vaginal?
manifestando presentar secreción vaginal abundante verdosa e A. Arcos supraorbitarios y mentón
irritación vulvar desde hace 7 días. Examen: cérvix uterino en B. Sutura frontal y lambdoidea
fresa. El diagnóstico es……………y el mejor manejo terapéutico C. Dorso de la nariz y sutura bregmática
es con…………… D. Fontanela anterior y borde orbitario
A. Tricomoniasis/ metronidazol. E. Dorso de la nariz y fontanela lambdoidea
B. Vaginosis bacteriana / metronidazol. 81. Durante el trabajo de parto, la presentación más común es la
C. Candidiasis / fluconazol. variedad occipito iliaca…
D. Vulvovaginitis / clindamicina. A. Izquierda posterior.
E. Vaginitis atrófica / estrógenos. B. Derecha posterior.
73. Gestante de 33 años, G4 P2012, de 34 semanas por FUR acude C. Izquierda anterior.
a su primer control. Tiene grupo y factor O negativo y Coombs D. Derecha anterior.
indirecto positivo, nunca se ha colocado la inmunoglobulina anti E. Transversa derecha.
D. Para determinar la anemia fetal por ecografía el párametro a 82. Mujer de 65 años que presenta una masa tumoral en la mama
evaluar, es la velocidad de… izquierda, fija a la pared torácica, y ganglios palpables en la
A. Las arterias uterinas. región axilar ipsilateral. En la evaluación inicial de la paciente se
B. La arteria umbilical. debe incluir...
C. La arteria cerebral media. A. Ultrasonido
D. El ductus venoso. B. Biopsia incisional
E. Las arterias mesentéricas. C. Biopsia excisional
74. Gestante de 38 semanas por FUR, G6 P5005, acude a D. Aspiración con aguja fina
emergencia por cefalea, trastornos visuales y oliguria. Examen: E. Tomografia axial computarizada
no trabajo de parto, FCF: 136 X’, PA: 152/112 mmHg en dos 83. En la Rx de tórax de un neonato prematuro con dificultad
controles de 6 horas de intervalo. Trombocitopenia, pro- respiratoria se describe: parénquima con granulación reti- cular
teinuria 2+ y creatinina sérica elevada. ¿Cuál es el diagnóstico? fina y broncogramas aéreos. ¿En qué patología se sospecha?
A. Eclampsia A. Hernia de Bochdalek
B. Hipertensión gestacional B. Síndrome de aspiración meconial
C. Hipertensión crónica C. Enfermedad de membrana hialina
D. Preeclampsia con signos de severidad D. Secuestro pulmonar
E. Preeclampsia sobreimpuesta E. Taquipnea transitoria del RN
75. Gestante de 12 semanas por FUR, usuaria de DIU hace 6 meses. 84. Recién nacido a término, con diagnóstico de incompatibilidad
Examen preferencial: se visualiza guía de DIU por el cérvix. ¿Cuál Rh, a las primeras 12 horas de vida presenta ictericia hasta
es la conducta a seguir? plantas. Madre es A negativa y el niño es O positivo. El paciente
A. Retiro de DIU tiene indicación de exanguinotransfusión. ¿Qué grupo
B. Antibioticoterapia sanguíneo y factor debe indicar para el procedimiento?
C. AMEU A. O negativo
B. O positivo adenopatías palpables, móviles no dolorosas y de consistencia
C. A positivo blanda. ¿Cuál es la etiología probable?
D. B positivo A. Parvovirus
E. AB negativo B. Virus de la gripe
85. Lactante de 11 meses, presenta durante 20 minutos C. Estreptococcus grupo A
convulsiones en hemicuerpo derecho, que luego se generalizan D. Adenovirus
con estado post ictal de recuperación rápida. Sin antecedentes E. Staphylococcus aureus
de importancia. Al examen T: 39°C. FC: 110 X´. Resto del examen 92. Lactante de 2 meses, inicia a las cuatro semanas de vida cuadro
normal. ¿Cuál es el diagnóstico probable? catarral, afebril y dificultad respiratoria progresiva.
A. Síndrome de Lennox Gastaut Antecedente: conjuntivitis aguda bilateral a los 10 días de vida.
B. Crisis febril simple Examen: Sat O2: 91%, FR: 65 X’, leve tiraje subcos- tal,
C. Crisis febril compleja estertores inspiratorios y crepitantes. Leucocitos: 19.600/mm3,
D. Epilepsia primaria IgG: 13.262/mm3. TAC tórax: patrón en mosaico con áreas en
E. Convulsión asociada a fiebre vidrio deslustrado y zonas de hiperinflación. ¿Cuál es el
86. Lactante de 3 meses presenta cuadro de 24 horas de evolución probable germen etiológico?
con 15 deposiciones líquidas sin moco ni sangre. Exa- men: FC: A. Mycoplasma pneumoniae
200 X´, FR: 62 X´, ojos hundidos, ausencia de lágrimas, mucosa B. Legionella pneumophila
oral seca, cutis marmórea y llenado capilar 4 seg. ¿Cuál es el C. Bordetella pertussis
tratamiento inicial? D. Chlamydia trachomatis
A. Solución isotónica en bolo E. Chlamydia pneumoniae
B. Albúmina 5% en bolo 93. Para un lactante de 8 Kg con cuadro de deshidratación severa
C. Plasma en 2 horas por diarrea aguda infecciosa. ¿Qué volumen de suero fisiológico
D. Dextrosa 5% en bolo en ml debe infundir?
E. Solución hipertónica 3% EV A. 160
87. Lactante varón de 6 meses presenta desde hace 2 días: 4 B. 80
deposiciones líquidas al día, fiebre de 39°C, vómitos, hiporexia C. 240
e irritabilidad. Antecedente: Infección urinaria a los 3 meses. D. 40
¿Qué examen de imagen indica inicialmente? E. 100
A. Cistografía 94. Recién nacido de 42 semanas, producto de parto vaginal,
B. Gammagrafía distocia por sufrimiento fetal agudo, con líquido amniótico
C. Radiografía meconial. APGAR: 6 al minuto 1 y 8 al minuto 5, recibió
D. Tomografía ventilación por máscara. A las 2 horas de vida, reportan ta-
E. Ecografía quipnea, Sat O2: 70%, tiraje subcostal y cianosis. Rx tórax:
88. Niño de 1año, es llevado a la emergencia por presentar hace 5 infiltrado pulmonar en “parches”. ¿Cuál es el diagnóstico
días rinorrea y tos seca. Hace 2 días fiebre de 39 °C y dificultad probable?
respiratoria. Examen: aleteo nasal, FR: 58X´, matidez en tercio A. Quiste pulmonar congénito
superior derecho, disminución del murmu- llo vesicular, B. Asfixia neonatal
vibraciones vocales aumentadas y subcrepitantes diseminados C. Neumonía neonatal
en ambos campos pulmonares. ¿Cuál es el diagnóstico D. Síndrome de aspiración meconial
probable? E. Taquipnea transitoria neonatal
A. Asma 95. En la atención inmediata de un recién nacido flácido,
B. Neumonía impregnado en líquido amniótico meconial y no respira espon-
C. Derrame pleural táneamente, con FC: 60 X’. ¿Cuál es la primera acción que se
D. Atelectasia debe realizar?
E. Bronquitis A. Intubación endotraqueal y aspiración
89. Lactante de 9 meses, con deshidratación hiponatrémica, recibe B. Ventilación a presión positiva
tratamiento de reposición de Na >12 mEq/L en las primeras 24 C. Ventilación y masaje cardíaco
horas. Dos días después presenta confusión, agitación y D. Posicionar, estimular y secar
tetraparesia flácida. ¿Cuál es el diagnóstico pro- bable? E. Dar oxígeno a flujo libre
A. Hemorragia intracerebral 96. Recién nacido de madre sin antecedentes de importancia,
B. Trombosis venosa central presenta apnea y FC: <100 X’. Se realiza inmediatamente
C. Infarto cerebral intubación endotraqueal; a la reevaluación se encuentra FC: <60
D. Mielinólisis pontínica central X’. ¿Cuál es el siguiente paso en la reanimación?
E. Disgenesia cerebral A. Naloxona EV
90. Preescolar de 3 años presenta hace tres días fiebre de 39°C, B. Adrenalina EV
hace un día ojos inflamados con secreción amarillenta y eritema C. Masaje cardíaco
faríngeo. ¿Cuál es el agente causal probable? D. Atropina EV
A. Adenovirus E. Adenosina EV
B. Streptococcus pneumoniae 97. Neonato de 18 horas de vida que presenta vómitos biliosos. El
C. Virus sincitial respiratorio estudio radiográfico muestra el signo de doble burbuja.
D. Haemophilus influenzae ¿Cuál es el diagnóstico?
E. Chlamydia trachomatis A. Atresia esofágica
91. Niño en edad pre-escolar sin antecedentes de importancia, B. Atresia duodenal
presenta tumoraciones cervicales bilaterales a lo largo del C. Páncreas anular
borde del músculo esternocleidomastoideo. Examen: pequeñas D. Malrotación intestinal
E. Estenosis pilórica hipertrófica
98. Recién nacido a término, peso 3500 g, APGAR 9 al minuto, en A. Caput succedaneum
alojamiento conjunto, recibe lactancia materna, funcio- nes B. Cefalohematoma
biológicas normales. Madre hace 15 horas ha presentado C. Hemorragia subgaleal
hemoptisis con resultados BK++. ¿Cuál es la conducta en el D. Hematoma temporal
recién nacido? E. Hematoma frontal
A. Radiografía de tórax 100. Lactante de 10 meses con vacunas incompletas, desde hace 2
B. PPD días presenta vómitos en 8 oportunidades, 10 deposicio- nes
C. Quimioprofilaxis líquidas sin moco ni sangre. ¿Cuál es el agente más frecuente?
D. BCG A. Coronavirus
E. Solicitar BK B. Adenovirus
99. En un recién nacido se encuentra una colección de sangre por C. Calicivirus
debajo de la aponeurosis que cubre el cuero cabelludo a lo largo D. Rotavirus
de todo el músculo occipitofrontal. ¿Cuál es el diagnóstico? E. Norwalk

SIMULACRO 6A
1. El diagnóstico de gastritis crónica se realiza fundamentalmente mediante:
A. Valoración de Helicobacter pylori.
B. Ecoendoscopia.
C. Determinación de la vitamina B12.
D. Estudio histológico.
E. Estudios radiológicos.
2. Una mujer de 46 años de edad, diabética en tratamiento con insulina desde hace unos 12 años, presenta sensación de saciedad, náuseas,
vómitos y pérdida de unos 3 kg de peso en el último mes. Se sospecha una gastroparesia diabética, siendo la mejor prueba para este diagnóstico
una de las siguientes:
A. Estudio de vaciamiento gástrico de sólidos con isótopos radiactivos.
B. Manometría gástrica.
C. Gastroscopia.
D. Electrogastrografía.
E. Rx. con contraste baritado (tránsito gastrointestinal).
3. Hombre de 20 años, previamente sano y sin antecedentes personales de interés, que es llevado al servido de urgencias por presentar agitación
durante las últimas 24 horas. Durante la semana previa sus familiares le han notado colora - ción amarillenta de piel y de conjuntivas.
Consumidor ocasional de cocaína y éxtasis y, los fines de semana, de alcohol (20 gramos). Mantiene relaciones sexuales de riesgo sin protección
adecuada. Temperatura 37°C. Presión arterial 110/60 mmHg. En la exploración física se observa agitación psicomotriz y desorie ntación en
tiempo y en espacio, as- terixis e ictericia en piel y mucosas, gingivorragias y epistaxis. El resto de la exploración física sin datos patológicos.
Las pruebas de laboratorio demuestran hiperbilirrubinemia (25 mg/dl), ALT (GPT) y AST (GOT) mayores de 1000 UI/L; prolongación del tiempo
de protrombina (actividad < 40%, INR 15). Indique el diagnóstico más probable del cuadro que presenta el paciente:
A. Síndrome hemolítico urémico.
B. Hepatitis alcohólica (esteatohepatitis alcohólica grave).
C. Hepatitis aguda fulminante (insuficiencia hepática aguda grave).
D. Intoxicación por éxtasis.
E. Síndrome de abstinencia de cocaína.
4. Una mujer de 55 años ingresa en el hospital con el diagnóstico de pancreatitis aguda. ¿Cuál de las siguientes pruebas o deter minaciones
analíticas NO es útil para predecir la gravedad de esta enfermedad?:
A. Tomografía axial computarizada (TAC) del abdomen.
B. Creatinina en sangre.
C. Niveles de amilasa y lipasa en sangre.
D. Hematocrito.
E. Nitrógeno ureico en sangre (BUN).
5. De acuerdo a las técnicas de planificación del sector público, el enunciado: “ El Centro de Salud tiene que proteger la digni dad personal,
promoviendo la salud, previniendo las enfermedades y garantizando la atención integral de salud de toda la población residente en su ámbito
de influencia”, corresponde a una definición de la:
A. Imagen-horizonte
B. Imagen-objetivo
C. Meta-estrategia
D. Misión
E. Visión
6. Niño de año y medio de edad (peso al nacer 3200 gr) con lactancia materna exclusiva los primeros 6 mes es de vida y ablactancia normal. Hace
6 meses presenta cuadros repetitivos de diarrea e infecciones respiratorias. En su valoracion nutricional se encuentra P/E 62 %, P/T 75%, T/E
95% ¿Cuál es el diagnostico nutricional?
A. Eutrófico
B. Desnutrición aguda
C. Desnutrición crónica
D. Obeso
E. Desnutrición crónica reagudizada
7. Respecto al insulinoma, señale la respuesta correcta:
A. Se asocia a MEN tipo IIa.
B. Es un tumor endocrino pancreático cuya resección quirúrgica supone la curación en la mayoría de casos.
C. Suele ser un tumor multifocal y maligno en la mayoría de los casos.
D. Suele ser un tumor de localización extrapancreática.
E. El tratamiento de elección es la radiofrecuencia percutánea.
8. Las complicaciones preoperatorias de mayor interés de la hernia inguinal son la estrangulación y la incarceración.
¿Qué hecho la diferencia?:
A. La hernia estrangulada se reduce con la manipulación y la incarcerada no.
B. La hernia incarcerada se caracteriza por cursar con interrupción del tránsito intestinal y la estrangulada no.
C. La hernia estrangulada cursa con alteración de la circulación sanguínea del contenido herniario.
D. La estrangulación es más frecuente en las hernias inguinales directas y la incarceración en las indirectas.
E. La hernia incarcerada se caracteriza por cursar con dolor intenso y permanente.
9. Un paciente con EPOC grave agudizado acude a Urgencias y presenta una gasometría arterial extraída con Fi02 del 31% a nivel del mar con una
Pa02 de 86 mm Hg, PaC02 65 mm Hg, pH 7.13 y Bicarbonato 27 mmol/litro. ¿Cuál de las siguientes afirmaciones es FALSA?:
A. El paciente está hiperventilando.
B. El gradiente alveolo-arterial de oxígeno está elevado.
C. El paciente está en acidosis respiratoria.
D. El nivel de bicarbonato es normal.
E. Se debería considerar iniciar ventilación mecánica.
10. En relación al TORCH en RN, quien desarrolla la tétrada de SABIN:
A. Toxoplasma
B. Sífilis
C. Rubeola
D. Citomegalovirus
E. Herpes
11. En la auscultación respiratoria se producen una serie de sonidos básicos que debemos reconocer, y por ello propongo una serie de parejas
(sonidos / posibles patologías) que se relacionan, salvo en un caso que no existe ninguna congruen- cia:
A. Crepitantes finos - fibrosis intersticial.
B. Estridor - obstrucción bronquiolar.
C. Crepitantes gruesos - bronquitis aguda.
D. Una respiración superficial con difícil audición de los ruidos respiratorios - enfermedad neuromuscular.
E. Crepitantes que recuerdan al frote de dos trozos de cuero - inflamación pleural.
12. Teniendo en cuenta sus conocimientos de los tumores en pediatría. Marque la alternativa INCORRECTA:
A. El tumor cerebral más frecuente es el astrocitoma
B. El tumor de Wilms se asocia a una delección del brazo corto del cromosoma 12
C. El tumor abdominal primitivo más frecuente es el neuroblastoma
D. El meduloblastoma es el segundo tumor más frecuente infratentorial
E. El signo de la capas de cebolla es de Osteosarcoma
13. En un paciente con parkinsonismo, ¿cuál de las siguientes situaciones le parece MENOS probable que ocurra en la enfermedad de Parkinson?:
A. Ausencia de respuesta a la levodopa.
B. Disquinesias coreicas bajo tratamiento con levodopa.
C. Distonía en el pie.
D. Alucinaciones visuales bajo tratamiento.
E. Antecedentes familiares de parkinsonismo.
14. Ante un paciente que presenta problemas para la comprensión del lenguaje tanto hablado como escrito, incapacidad para denominar objetos
y repetir palabras que se le dicen, habla fluida incomprensible con parafasias semánticas y fonémicas, se trata de una:
A. Afasia global.
B. Afasia de Wernicke.
C. Afasia de Broca.
D. Afasia transcortical sensitiva.
E. Afasia transcortical motora.
15. El 90% de los aneurismas del ventrículo izquierdo, como complicación de un infarto de miocardio transmural, son secundarios a una oclusión
aguda de la arteria coronaria:
A. Tronco común de la arteria coronaria izquierda.
B. Arteria descendente anterior.
C. Arteria circunfleja.
D. Arteria descendente posterior.
E. Arteria coronaria derecha.
16. Mujer de 45 años que presenta hipertensión arterial (190/120 mm Hg ) acompañada de K 2.5 mEq/l. Se realiza ecogra- fía abdominal que
muestra estenosis de ambas arterias renales. Indique qué tratamiento está contraindicado:
A. Enalapril.
B. Propanolol.
C. Amiloride.
D. Prazosin.
E. Amlodipino.
17. Si en un paciente con insuficiencia cardiaca crónica detectamos unas ondas v prominentes en el pulso venoso yugular y en la auscultación
cardiaca se ausculta un soplo holosistólico en el área del apéndice xifoides que se acentúa con la inspiración profunda, ¿cuá l es la valvulopatía
responsable de esta exploración física?:
A. Insuficiencia mitral.
B. Insuficiencia pulmonar.
C. Insuficiencia tricúspide.
D. Insuficiencia aórtica.
E. Estenosis aórtica.
18. Niño de 18 meses de edad, con calendario vacunal completo hasta la fecha, que consulta en el Servicio de urgencias por tumefa cción de la
rodilla derecha tras jugar en el parque, sin traumatismo evidente. En la anamnesis dirigida, la madre refiere que un t ío de ella tenía problemas
similares. La exploración ecográfica es compatible con hemartros y en la analítica que se realiza sólo destaca un alargamient o del TTPA de 52”
(normal 25-35”). ¿Cuál es la hipótesis diagnóstica más probable?:
A. Síndrome de Marfan.
B. Enfermedad de Von Willebrand.
C. Enfermedad de Ehlers-Danlos.
D. Hemofilia A.
E. Enfermedad de Bernard-Soulier.
19. Un hombre de 58 años, no fumador y sin antecedentes personales relevantes, es ingresado en la planta de neurología por un acc idente
cerebrovascular agudo isquémico. Su hemograma muestra 18.5 g/dl de hemoglobina con un hemato- crito de 60%. Todos los siguientes datos
concuerdan con el diagnóstico de Policitemia vera, EXCEPTO uno. Señálelo:
A. Niveles de eritropoyetina séricos elevados.
B. Presencia de la mutación V617F del gen JAK-2.
C. Esplenomegalia moderada.
D. Presencia de prurito “acuágeno” y eritromelalgia.
E. Presencia de leucocitosis neutrofílica y trombocitosis.
20. Mujer de 32 años con clínica de una semana de evolución consistente en febrícula, eritema nodo-so, inflamación pe- riarticular de tobillos y
uveítis anterior. En la radiografía de tórax presenta adenopatías hiliares bilaterales. ¿Cuál es el diagnóstico?:
A. Síndrome de Sjögren.
B. Tuberculosis ganglionar mediastínica.
C. Sarcoidosis tipo síndrome de Löfgren.
D. Lupus eritematoso sistémico.
E. Poliarteritis nodosa.
21. Un paciente de 27 años es incapaz de impedir el desplazamiento anterior del fémur sobre la tibia cuando la rodilla está
flexionada. ¿Cuál de los siguientes ligamentos está dañado?:
A. Cruzado anterior.
B. Colateral peroneo.
C. Rotuliano.
D. Colateral tibial.
E. Cruzado posterior.
22. Paciente de 28 años, que tras sufrir un accidente de tráfico frontal de gran energía, presenta deformidad del miembro inferior derecho en
aducción y rotación interna de la cadera. Llama la atención que cualquier intento de manipulación le produce intenso dolor. Debemos pensar
que el paciente tiene:
A. Fractura subtrocantérea de fémur.
B. Fractura subcapital de fémur.
C. Luxación coxofemoral anterior.
D. Luxación coxofemoral posterior.
E. Fractura pertrocantérea de fémur.
23. Con relación a la técnica de amamantamiento , el signo que indica que el niño hace un buen agarre es:
A. La nariz del bebe está pegado a la areola
B. La boca del bebe esta semiabierta
C. Se observa gran parte de la areola libre
D. El mentón del bebe está separado del pecho de su madre.
E. El labio inferior del bebe esta evertido.
24. En el trabajo de parto normal, cuando la flexion se completa, el diametro de la cabeza fetal que ingresa en el estrecho
superior de la pelvis es:
A. Occipitofrontal
B. Suboccipitobregmatico
C. Occipitomentoniano
D. Biparietal
E. Bitemporal
25. Un paciente con infección VIH se presentó con 40 linfocitos CD4/μL y una carga viral de 2 millones de copias/mL en el momento del diagnóstico.
Inició tratamiento antirretroviral con efavirenz, tenofovir y emtricitabina, y 3 semanas más tarde desarrolló un cuadro de fiebre, malestar
general y adenopatías cervicales bilaterales. El Mantoux fue negativo, en la Rx de tórax se observaban múltiples adenopatías mediastínicas y
en los análisis más recientes presentaba 77 linfocitos CD4/μL y una carga viral de VIH -1 de 1000 copias/mL. ¿Cuál es el diagnóstico más
probable?:
A. Tuberculosis ganglionar como síndrome de reconstitución inmune.
B. Linfoma de alto grado.
C. Reacción adversa al tratamiento antirretroviral.
D. Criptococosis sistémica.
E. Infección por Pneumocystis jirovecii.
26. Hombre de 35 años que presenta hematuria tras infecciones respiratorias desde hace varios años. En la analítica de sangre pre senta creatinina
1 mg/ dl sin otras alteraciones y en la orina aparecen hematíes 50/campo, siendo el 80% di smórficos, con proteinuria de 0.8 gramos en 24
horas. ¿Cuál es el diagnóstico más probable?:
A. Nefropatía de cambios mínimos.
B. Glomerulonefritis membranosa.
C. Nefropatía Ig A.
D. Glomerulonefritis proliferativa difusa.
E. Glomeruloesclerosis focal y segmentaria primaria.
27. Paciente de 48 años que consulta por un cuadro de fiebre, cefalea frontal intensa y sensación nauseosa de 48 horas de evoluci ón. En la
exploración física presenta un estado general conservado. No tiene lesiones cutáneas. Se aprecia una discreta rigidez de nuca con signo de
Kernig positivo. Ante la sospecha de meningitis se practica una punción lumbar que da salida a un líquido de aspecto claro, c on proteínas 170
mg/dl, glucosa 54 mg/dl (glucosa plasmática 98 mg/dl) y células 280 con un 89% de linfocitos. ADA 4 Ul/L. ¿Cuál es la causa más probable de
la meningitis de este paciente?:
A. Neisseria meningitidis.
B. Mycobacterium tuberculosis.
C. Streptococcus pneumoniae.
D. Enterovirus.
E. Virus herpes 6.
28. De acuerdo a la convulsión febril en niños. Marque la alternativa INCORRECTA:
A. Es más frecuente en el segundo año de vida
B. La compleja es cuando tiene paresias postictal
C. Aumentan el riesgo de epilepsia en un 10%
D. Suelen aparecer el primer día de la fiebre
E. Las que duran más de 30 minutos son catalogadas como status convulsivo
29. En cuanto a los índices urinarios en el diagnóstico del fracaso renal agudo prerrenal, indique la afirmación INCO - RRECTA:
A. La osmolalidad urinaria es superior a 400 mOsm/ Kgr.
B. El sodio urinario es inferior a 20 mEq/1.
C. El índice de fallo renal (IFR) es superior a 1.
D. El cociente entre la urea urinaria y la urea plasmática es superior a 10.
E. La excreción fraccional de Sodio (EFNa) es inferior al 1 %.
30. Un paciente con insuficiencia renal crónica estadio IV (filtrado glomerular renal 25 ml/min) presenta una hemoglobi- na de 8.6 g/dL. El estudio
de anemia muestra un volumen corpuscular medio de 78 fl y los niveles de ferritina en sangre son de 48 ng/ mL (valor normal 3 0-300 ng/mL).
¿Cuál de las siguientes opciones es más adecuada?:
A. Iniciar tratamiento sustitutivo con hemodiálisis.
B. Administrar darbopoyetina alfa 0.70 microgamos vía subcutánea cada 2 semanas.
C. Indicar realización de un aspirado medular para completar el estudio.
D. Reponer el déficit de hierro y, si persite la anemia, iniciar tratamiento con un agente eritropoyético.
E. Realizar una endoscopia digestiva para descartar sangrado gastrointestinal.
31. Un paciente de 80 años, con historia de hipertensión y en tratamiento con enalapril y espirono-lactona, acude al hos- pital por astenia y
debilidad muscular severa. La presión arterial es de 110/70 mm Hg. En el ECG destacan ondas T picudas y elevadas, extrasístol es ventriculares
y QT corto. ¿Cuál es el diagnóstico más probable?:
A. Hipercalcemia.
B. Hiperpotasemia.
C. Hipomagnesemia.
D. Hipocalcemia.
E. Hipernatremia.
32. Los fármacos finasteride y dutasteride, inhibidores de la 5-alfa-reductasa, están indicados en el tratamiento único o combinados con alfa-
bloqueantes de los síntomas producidos por:
A. Adenocarcinoma de próstata.
B. Adenocarcinoma renal de células claras.
C. Hiperplasia prostática benigna.
D. Litiasis urinaria infectiva.
E. Infección urinaria recidivante.
33. De los tumores renales, el de mayor agresividad es:
A. Variedad papilar.
B. Variedad cromófoba.
C. Variedad de células claras.
D. Variedad sarcomatoide.
E. Variedad oncocítica.
34. Un hombre de 58 años ingresa inconsciente en Urgencias en situación de parada cardiorrespiratoria, procediéndose de forma inm ediata a la
aplicación de maniobras de RCP avanzada. Se objetiva en el monitor Fibrilación ventricular (FV), por lo que se realiza desfibrilación monofásica
con carga de 300 Julios. La FV persiste, motivo por el que se reinician nuevos bucles de masaje cardiaco-ventilación. ¿Cuándo considera que
está indicada la administración de Amiodarona en este paciente?:
A. En caso de continuar la FV después del 3º choque desfibrilatorio.
B. Debe aplicarse desde el inicio en RCP avanzada, cuando se detecte FV.
C. Tras el primer choque desfibrilatorio, en caso de persistencia de la FV.
D. No está indicada su administración en RCP avanzada.
E. Sólo debe administrarse si se objetiva taquicardia ventricular polimorfa.
35. Con respecto a las quemaduras, ¿qué manifestaciones clínicas presentan las quemaduras de se-gundo grado?:
A. Eritema, dolor intenso y sequedad.
B. Superficie dura y dolor escaso o ausente.
C. Dolor intenso, formación de ampollas y exudado.
D. Dolor escaso o ausente, exudado y ampollas.
E. Eritema, dolor y superficie costrosa y seca.
36. Mujer de 56 años que presenta en los últimos años clínica de vértigo rotatorio, recurrente, en forma de episodios que se inic ian con sensación
de plenitud ótica derecha, que duran entre 2 y 3 horas y que posteriormente, durante unos días, cursa con inestabilidad. Asimismo la paciente
refiere acúfenos en oído derecho e hipoacusia fluctuante. A la pa- ciente se le ha practicado una RM craneal informada como normal y una
audiometría que evidencia una hipoacusia neurosensorial moderada en oído derecho. ¿Cuál es el diagnóstico más probable?:
A. Vértigo de origen central.
B. Enfermedad de Ménière.
C. Vértigo posicional paroxístico benigno.
D. Neuritis vestibular.
E. Schwannoma del nervio vestibular.
37. En el proceso de coordinación para desarrollar entornos y estilos de vida saludables en los municipios, instituciones educativas y otras
instituciones a nivel local, ¿Cuál de los lineamientos de política de promoción de la salud debe aplicarse?:
A. Desarrollar alianzas intra e intersectoriales para la promoción de la salud
B. Empoderar a la ciudadanía, la participación comunal y la interculturalidad
C. Promover la participación comunitaria conducente al ejercicio de la ciudadanía.
D. Reorientar la inversión hacia la promoción de la salud y el desarrollo
E. Reorientar los servicios de salud, con enfoque de promoción de la salud.
38. Mujer de 30 años y antecedentes patológicos de esclerosis múltiple que acude por miodesopsias en ambos ojos de una semana de evolución.
La exploración de fondo de ojo muestra vitritis, conglomerados inflamatorios en forma de “bo- las de nieve” en cavidad vítrea inferior y
periflebitis periférica en ambos ojos. ¿Cuál es su diagnóstico?:
A. Uveítis intermedia.
B. Síndrome de manchas blancas.
C. Neuritis óptica.
D. Uveítis anterior.
E. Coroiditis punteada interna.
39. En relación al síndrome de ovario poliquístico, es cierto que:
A. Es una endocrinopatía muy poco frecuente en mujeres en edad reproductiva.
B. Existe un mayor riesgo a largo plazo de desarrollar diabetes mellitus y carcinoma endometrial.
C. Clínicamente, es típico en este síndrome la polimenorrea, obesidad e hirsutismo.
D. Habitualmente la concentración sérica de FSH es mayor que la de LH.
E. No existe una imagen ecográfica ovárica característica.
40. 40. Gestante de 36 semanas, primigesta, es trasladada al hospital para valoración tras accidente de coche en cadena en la autopista,
presentando dolor cervicodorsal. Durante la exploración, la paciente inicia dolor abdominal intenso, leve sangrado vaginal os curo y aumento
mantenido del tono uterino. ¿Qué diagnóstico le parece el más probable?:
A. Rotura esplénica con hemoperitoneo.
B. Desprendimiento prematuro de placenta normoinserta.
C. Amenaza de parto prematuro.
D. Rotura uterina.
E. Rotura de vasa previa.
41. 41. Mujer de 26 años, primigesta de 32 semanas, acude a urgencias por dolor cólico y lumbalgia. No refiere pérdida de líquido. En la exploración
no se observan sangrado ni líquido amniótico en vagina. Se confirma latido cardiaco fetal normal y la ecografía vaginal nos i nforma de un
acortamiento cervical del 60%. En los primeros 10 minutos de vigi- lancia cardiotocográfica presenta 3 contracciones. ¿Qué pensaría como
primera opción en esta paciente?:
A. Antibióticos.
B. Sulfato de Magnesio.
C. Hidratación intravenosa.
D. Corticoesteroides y tratamiento tocolítico.
E. Ecografías seriadas.
42. 42. ¿Cuál es el tratamiento antibiótico ambulatorio de elección en la enfermedad inflamatoria pélvica leve/moderada?:
A. Clindamicina y gentamicina.
B. Metronidazol.
C. Azitromicina.
D. Amoxicilina-clavulánico y doxicilina.
E. Ceftriaxona y doxiciclina.
43. En relación a la Neoplasia Trofoblástica Gestacional, NO es cierto que:
A. Engloba una serie de neoplasias que tienen en común una producción aumentada de Beta-HCG.
B. El síntoma más frecuente de la mola total es la hemorragia genital.
C. En la etiología de la mola completa, el origen es paterno.
D. En la mola parcial, el cariotipo embrionario es diploide en el 90% de los casos.
E. En la mola total es frecuente la aparición de hiperemesis gravídica muy precoz y severa.
44. A un adolescente asintomático se le practica un ECG que muestra ritmo sinusal y bloqueo de rama derecha. En la exploración fí sica se ausculta
un segundo ruido desdoblado y en la radiografía de tórax se objetiva un arco pulmonar prominente. ¿Cuál sería su sospecha diagnóstica?:
A. Tetralogía de Fallot.
B. Estenosis pulmonar.
C. Comunicación interauricular.
D. Persistencia de ductus arterial.
E. Comunicación interventricular.
45. 45. La leche humana es el alimento natural y apropiado durante el primer año de vida. ¿Cuál de las siguientes asevera- ciones al respecto NO
es cierta?:
A. La lecha humana contiene anticuerpos bacterianos y víricos.
B. La lactancia natural se asocia con una menor incidencia de alergia o intolerancia a la leche de vaca.
C. Están bien reconocidas las ventajas psicológicas de la lactancia al pecho tanto para la madre como para el niño.
D. No se ha documentado la transmisión de infección por VIH por la leche materna.
E. La leche humana contiene lactoferrina, que tiene un efecto inhibitorio en el crecimiento de E. coli.
46. Acude de urgencia al centro de salud un niño de 15 meses de edad que, durante la cena, tras ingerir un bocado de tortilla, presenta de forma
súbita: enrojecimiento facial de predominio perioral, lesiones habonosas en tronco y extre- midades, y tos. A su llegada al centro se encuentra
consciente y se objetiva, además de lo descrito: tiraje supraesternal, rinorrea acuosa abundante, hipoventilación bilateral sin sibilancias, y
relleno capilar inferior a 2 segundos. De las siguientes afirmaciones, señale la respuesta CORRECTA:
A. Lo prioritario es canalizar una vía venosa.
B. La metilprednisolona por vía intramuscular es el tratamiento de elección.
C. Se trata de un cuadro de urticaria asociado a asma, y debe ser tratado con antihistamínicos y broncodilatadores inhalados.
D. Se debe recomendar a los padres su traslado a un Servicio de Urgencias hospitalario.
E. Se debe administrar sin más dilación adrenalina por vía intramuscular.
47. ¿Cuándo se aconseja intervenir quirúrgicamente la hernia umbilical infantil?:
A. Se debe intervenir cuanto antes para evitar el riesgo de estrangulación.
B. A partir de los 3 ó 4 años de vida, al ser frecuente su cierre espontáneo antes de esa edad.
C. Con el fin de utilizar prótesis en la cirugía, se recomienda esperar a que el paciente sea mayor.
D. Si el niño presentara vómitos durante los accesos febriles.
E. Ante la sospecha de que se trate de un onfalocele.
48. En un contraste de hipótesis estadístico, ¿a qué definición corresponde con más exactitud el valor “p”?:
A. La probabilidad de observar los resultados del estudio, u otros más alejados de la hipótesis nula, si la hipótesis nula fuera cierta.
B. La probabilidad de que la hipótesis nula sea cierta.
C. La probabilidad de observar los resultados del estudio si la hipótesis nula fuera cierta.
D. La probabilidad de que los resultados observados sean debidos al azar.
E. La probabilidad de observar los resultados del estudio, u otros más alejados de la hipótesis nula, si la hipótesis alternativa fuera cierta.
49. Un grupo de 1000 pacientes diagnosticados de Síndrome del Aceite Tóxico (SAT) fueron segui-dos desde 1981 hasta 1995 junto con un número
similar de vecinos sin dicho diagnóstico. Entre los pacientes con SAT se observó en 1995 un 20% con signos de neuropatía peri férica frente a
un 2% en los vecinos. Según el diseño descrito, ¿de qué tipo de estudio se trata?:
A. Estudio transversal.
B. Estudio de cohortes.
C. Estudio de casos y controles.
D. Estudio cuasiesperimental.
E. Ensayo clínico controlado.
50. Llega a emergencia un paciente que sufrió un accidente con traumatismo directo de abdomen. El cirujano comenta que probablemente tenga
hematoma retroperitoneal por un órgano lesionado de la zona I. ¿Cuál de los siguientes pertenecen a dicha zona?
A. Vejiga
B. Riñones
C. Huesos pélvicos
D. Duodeno
E. Colon descendente
51. ¿Qué es un estudio de casos y controles anidado?:
A. Es el tipo de estudio de casos y controles en el que la serie de controles está apareada con los casos en posibles factores de confusión.
B. Es el tipo de estudio de casos y controles en el que la serie de controles está muestreada aleatoriamente de la cohorte que da origen a los
casos.
C. Es el tipo de estudio de casos y controles en el que tanto los casos como los controles se extraen del mismo hospital o centro de estudio.
D. Es el tipo de estudio de casos y controles que se realiza para estudiar los factores etiológicos de las malformaciones congénitas y que se
llevan a cabo en las unidades de neonatología.
E. Es el tipo de estudio de casos y controles que se realiza en poblaciones estáticas o cerradas en las que no se permita la entrada o salida
de la misma.
52. De entre las modificaciones gravídicas, ¿cuál es INCORRECTA?:
A. Además de los 2 litros de volemia extra, existe un aumento de 5 litros de volumen extravascular.
B. El aumento de la volemia no conlleva hipertensión arterial al disminuir las resistencias vasculares periféricas.
C. Existe un aumento del sistema renina-angiotensina- aldosterona, aunque su efecto quede bloqueado.
D. Los uréteres pueden dilatarse desde la semana 8 hasta tres meses después del parto, siendo más evidente en el uréter izquierdo.
E. A nivel bucal, puede aumentar las pérdidas dentarias debido a un aumento de la salivación y a una disminución del pH de dicha saliva.
53. ¿Cómo se denomina al ensayo clínico en el que los pacientes, los investigadores y los profesionales sanitarios implica - dos en la atención de
los pacientes desconocen el tratamiento asignado?:
A. Enmascarado.
B. Triple ciego.
C. Abierto.
D. Simple ciego.
E. Doble ciego.
54. Una prueba diagnóstica tiene una sensibilidad del 95%. ¿Qué nos indica este resultado?:
A. La prueba dará, como máximo, un 5% de falsos negativos.
B. La prueba dará, como máximo, un 5% de falsos positivos.
C. La probabilidad de que un resultado positivo corresponda realmente a un enfermo será alta.
D. La probabilidad de que un resultado negativo corresponda realmente a un sano será alta.
E. La prueba será muy específica.
55. La vena porta está formada por la confluencia de varias venas. De las respuestas que se ofrecen, ¿cuál es la verdade- ra?:
A. Vena mesentérica superior, vena gástrica izquierda y vena gastro-omental izquierda.
B. Vena mesentérica inferior, vena gástrica izquierda y vena renal.
C. Vena esplénica, vena mesentérica superior y vena mesentérica inferior.
D. Vena esplénica, venas pancreato-duodenales y vena omental izquierda.
E. Venas pancreato-duodenales, vena mesentérica superior y vena mesentérica inferior.
56. En una nefrona, el 60% del cloruro de sodio es reabsorbido en:
A. Túbulo proximal.
B. Rama descendente del asa de Henle.
C. Rama ascendente del asa de Henle.
D. Túbulo contorneado distal.
E. Conducto colector.
57. Carmen y Pedro tienen 3 hijos: Enrique, de 5 años, Isabel, de 4 años, y Pablo, de 1 año. Recientemente han notado que Enrique tiene ciertos
problemas para subir las escaleras y se cansa mucho cuando corre. Después de unas pruebas mé- dicas, le han diagnosticado una enfermedad
genética llamada distrofia muscular de Duchenne. Señale la alternativa correcta sobre el tipo probable de herencia de esta enfermedad:
A. La madre es la que le ha transmitido la enfermedad.
B. El padre es el que ha transmitido la enfermedad.
C. Ambos padres le han trasmitido la enfermedad.
D. Si tienen una nueva hija, puede presentar la enfermedad.
E. Si tienen un nuevo hijo, no puede heredar la enfermedad.

58. Paciente primigesta, de 31 semanas, con un embarazo gemelar monocorial biamniótico con el primer gemelo en ce - fálica y un peso fetal
estimado de 1.300 g y el segundo gemelo en cefálica, con un peso fetal estim ado de 1.150 g, que acude a Urgencias con dinámica uterina
regular y una dilatación cervical de 3 cm. La vía del parto más adecuada será:
A. Cesárea por edad gestacional inferior a 32 semanas.
B. Cesárea por pesos fetales estimados inferiores a 1.500 g.
C. Cesárea por gestación gemelar monocorial.
D. Parto vaginal de ambos gemelos.
E. Parto vaginal con versión interna y gran extracción del segundo gemelo.
59. ¿Qué tipo de linfocitos son los más abundantes en sangre periférica?:
A. Linfocitos B CD5+ (B la).
B. Linfocitos T colaboradores CD4+.
C. Linfocitos T citotóxicos CD8+.
D. Linfocitos T gamma/delta.
E. Linfocitos NK (Natural Killer o asesinos naturales).
60. Paciente politraumatizado ingresado en la UCI que presenta neumonía asociada a ventilación mecánica. El hemocul- tivo es positivo a Acineto-
bacter baumannii resistente a carbapenem y ampicilina/ sulbactan. ¿Cuál de los siguientes antibióticos sería de elección?:
A. Cefepima.
B. Vancomicina.
C. Linezolid.
D. Amikacina.
E. Colistina.
61. Mujer de 55 años, fumadora habitual, a la que se le descubre en una revisión anual un nódulo de 1,5cm, duro, fijo e indoloro, en el cuadrant e
superoexterno de la mama. En la exploración física no se palpan adenopatías axilares ni su- praclaviculares. La mamografía muestra imágenes
sugestivas de malignidad. Se realiza biopsia intraoperatoria, cuyo resultado es carcinoma intraductal in situ. La actitud ter apéutica más
adecuada es:
A. Mastectomía simple.
B. Tumorectomía + radioterapia + biopsia del ganglio centinela.
C. Quimioterapia + radioterapia local + hormonoterapia, si receptores estrogénicos positivos.
D. Quimioterapia + hormonoterapia si receptores estrogénicos positivos.
E. Mastectomía radical (Halsted).
62. ¿Qué fármaco NO estaría indicado en el tratamiento preventivo de la migraña?:
A. Sumatriptan.
B. Topiramato.
C. Propranolol.
D. Flunarizina.
E. Ácido Valproico.
63. ¿Qué hallazgo, de los siguientes, en la exploración que se realiza en las primeras 24 horas a un recién nacido obliga a reali zar exámenes
complementarios para aproximar su diagnóstico?:
A. Acabalgamiento de sutura parieto-occipital.
B. Edema palpebral bilateral.
C. Eritema tóxico en tronco y extremidades.
D. Mancha mongólica en espalda y extremidades superiores.
E. Mechón de pelo en zona de columna lumbosacra.
64. Un paciente presenta unas lesiones eritematoescamosas en la cara extensora de los brazos y manos, con áreas de piel respetada dentro de
algunas de ellas, y descamación fina en áreas seborreicas de cara y cuello. El eritema es más acen- tuado en las palmas, donde además usted
aprecia una llamativa queratodermia. En las uñas, observa hiperqueratosis y estriación longitudinal. ¿Qué diagnóstico haría?:
A. Psoriasis.
B. Pitiriasis rubra pilaris.
C. Pitiriasis rosada.
D. Pitiriasis liquenoide.
E. Dermatosis seborreica y eczema de contacto en las manos.
65. Un paciente es diagnosticado de tuberculosis pulmonar e inicia tratamiento con isoniacida, rifampicina y pirazina - mida. A las tres semanas de
tratamiento se objetiva en los datos analíticos hiperuricemia. ¿Cuál considera que es la causa más probable?:
A. Toxicidad por isoniacida.
B. Toxicidad por rifampicina.
C. Toxicidad por pirazinamida.
D. Reactivación de su cuadro de tuberculosis.
E. Destrucción del microorganismo.
66. Varón de 35 años que acude por presentar enrojecimiento ocular, fiebre y mialgias de 7 días de evolución. Sus familia - res han tenido síntomas
similares, por lo que han sido diagnosticados de síndrome gripal. En los análisis practicados destaca CPK 1.200 U/l, LDH 1.80 0 U/ml, 7.200
leucocitos con 30% de eosinófilos. En la biopsia muscular practicada como prueba diagnóstica, espera encontrar:
A. Cisticercosis.
B. Taenia solium.
C. Fasciola hepatica.
D. Triquinosis.
E. Giardia lamblia.
67. ¿Cuál es el tratamiento de elección de la brucelosis aguda?:
A. Doxiciclina 6 semanas.
B. Doxiciclina 6 semanas, más rifampicina las primeras dos semanas.
C. Doxiciclina 6 semanas, más estreptomicina las primeras dos semanas.
D. Cotrimoxazol 4 semanas.
E. Eritromicina.
68. Todos los datos de laboratorio pertenecen a las anomalías producidas por infección VIH en el sistema inmune, EX- CEPTO:
A. Depleción de linfocitos T4.
B. Activación policlonal de células B.
C. Producción de inmunoglobulinas incrementada.
D. Aumento de la producción de interferón gamma en respuesta a los antígenos.
E. Aumento de beta-2-microglobulina.
69. Sobre la estructura del hueso, ¿cuál de las siguientes afirmaciones NO es cierta?:
A. El hueso cortical tiene como unidad el sistema Haversiano, compuesto por un canal rodeado de láminas óseas con osteo- citos.
B. La función de los osteoclastos es la reabsorción del hueso.
C. La matriz orgánica está compuesta principalmente por colágeno tipo II.
D. El crecimiento óseo viene determinado por el platillo epifisario (crecimiento longitudinal) y el periostio (creci- miento concéntrico).
E. El esqueleto tiene un origen mesenquimal.
70. Señale la afirmación INCORRECTA en relación con la clínica del asma:
A. Radiológicamente, lo habitual es encontrar una Rx tórax normal, salvo en las crisis graves.
B. El neumotórax y el neumomediastino pueden ser complicaciones de las crisis asmáticas.
C. El dato auscultatorio más típico son las sibilancias inspiratorias.
D. La PaCO2 suele estar disminuida durante las crisis.
E. En crisis asmáticas graves puede aparecer pulso paradójico.
71. ¿Qué tipo de carcinoma mamario es especialmente difícil de detectar por mamografía?:
A. Tumor filodes.
B. Tumor escirro.
C. Tumor medular.
D. Comedocarcinoma.
E. Carcinoma inflamatorio.
72. En una paciente de parto que se encuentra con 3 cm de dilatación y, coincidiendo con una contracción, expulsa líquido amniótico teñido de
sangre, al mismo tiempo que se objetiva un deterioro en el estado fetal, el diagnóstico más pro- bable será:
A. Una hipertonía uterina.
B. Una placenta baja o previa.
C. Una aplopejía uteroplacentaria o hemorragia retroplacentaria.
D. Una rotura de “vasa previa”.
E. Un desgarro de cuello uterino.
73. ¿Cuál de los siguientes patógenos es el causante de la enfermedad por arañazo de gato?:
A. Bartonella henselae.
B. Mycobacterium tuberculosis.
C. Mycobacterium avium complex.
D. Toxoplasma gondii.
E. Virus de Epstein-Barr.
74. ¿Cuál de los siguientes emparejamientos entre fármaco anticoagulante y mecanismo de acción es INCORRECTO?:
A. Heparina - cofactor de la antitrombina III.
B. Acenocumarol - inhibe la vitamina K epóxido reductasa.
C. Dabigatrán - inhibe la trombina.
D. Rivaroxabán - inhibe el factor Xa.
E. Warfarina - inhibe la absorción de la vitamina K.
75. La placenta es un derivado de:
A. La zona pelúcida.
B. El trofoblasto.
C. El ectodermo.
D. La línea primitiva.
E. El mesodermo.
76. El signo de Rovsing, característico en las apendicitis agudas, consiste en:
A. Dolor a la presión en epigastrio al aplicar una presión firme y persistente sobre el punto de McBurney.
B. Dolor agudo que aparece al comprimir el apéndice entre la pared abdominal y la cresta ilíaca.
C. Sensibilidad de rebote pasajera en la pared abdominal.
D. Pérdida de la sensibilidad abdominal al contraer los músculos de la pared abdominal.
E. Dolor en el punto de McBurney al comprimir el cuadrante inferior izquierdo del abdomen.
77. En una mujer de 29 años se hallan, en el curso de una analítica de rutina, los siguientes parámetros: Hb 11,5 g/dL, VCM 70 fl , HCM 28 pg,
ferritina 10 ng/mL, leucocitos 5.200/mm3, plaquetas 335.000/ mm3. La exploración física es normal. La exploración más indicada en esta
situación es:
A. Exploración ginecológica.
B. Estudio de sangre oculta en heces.
C. Estudio radiológico de aparato digestivo.
D. Electroforesis de hemoglobina.
E. Test de Coombs.
78. Un hombre de 62 años, con una diabetes mellitus tipo 2 de 10 años de evolución, realiza tratamiento con metformina y sitaglip tina. Hace
ejercicio físico escaso y realiza una dieta adecuada. En los últimos 6 meses ha perdido peso y tiene más astenia. Sus controles glucémicos se
han deteriorado, pasando de glucemias basales de 110-140 mg/dl a glucemias de 170-200 mg/dl, así como su hemoglobina glicosilada, que ha
pasado de 7,1 a 8,5%. La medida terapéutica más adecuada a realizar es:
A. Aumentar la ingesta de proteínas e hidratos de carbono de cadena larga en la dieta para mejorar la astenia y la pérdida de peso.
B. Asociar al tratamiento una dosis de insulina basal.
C. Asociar al tratamiento acarbosa.
D. Sustituir la sitagliptina por pioglitazona.
E. Sustituir la metformina por glimepirida.
79. NO es cierto, respecto a la artritis reumatoide:
A. Es fundamental el diagnóstico precoz, un tratamiento intensivo y un control estrecho de la enfermedad.
B. El metotrexato es el fármaco de primera elección para controlar la actividad de la enfermedad.
C. El factor reumatoide es típico de la enfermedad y es un criterio obligado para el diagnóstico.
D. La especificidad de los anticuerpos anti péptidos citrulinados es muy elevada y tiene valor pronósti co.
E. Los fármacos biológicos, como los anti TNF, han revolucionado el tratamiento.
80. Una mujer de 56 años presenta, tras caída casual al suelo, dolor, deformidad y aumento de volumen en el brazo iz -
quierdo, con imposibilidad para la flexión dorsal de la mano. ¿Cuál es el diagnóstico más probable?:
A. Fractura de troquíter en húmero izquierdo.
B. Fractura diafisaria de húmero izquierdo con lesión del nervio radial.
C. Fractura supraintercondílea de paleta humeral con lesión del nervio mediano.
D. Fractura de cuello anatómico de húmero con lesión del nervio radial.
E. Fractura de epitróclea desplazada con lesión del nervio cubital.
81. Paciente de 26 años con 21 semanas de gestación, presenta dolor abdominal y sangrado vaginal escaso, de 2 días de evolución. Al examen:
altura uterina 20 cm, movimientos fetales presentes. Especuloscopia se observa membranas ovulares prominentes e integras, que protuyen
por el orificio externo abierto. ¿Cuál es el diagnostico?
A. Óbito fetal
B. Aborto incompleto
C. Aborto inevitable
D. Aborto inminente
E. Aborto frustro
82. ¿Cuál de las siguientes asociaciones (enfermedad - síntoma o signo clínico) es INCORRECTA?:
A. Sarampión - manchas de Koplik.
B. Exantema súbito - fiebre.
C. Eritema infeccioso - anemia por aplasia medular.
D. Varicela - adenopatías occipitales.
E. Escarlatina - fiebre y disfagia.
83. Ante la sospecha de estenosis hipertrófica de píloro, ¿cómo iniciaría los exámenes complementarios?:
A. Tránsito digestivo.
B. pHmetría gástrica.
C. Radiología simple.
D. Ecografía.
E. Estudio isotópico.
84. Mujer de 16 años que consulta por amenorrea primaria. Presenta un desarrollo femenino normal de los caracteres sexuales secundarios. Los
niveles de estradiol y testosterona son normales. En la exploración ginecológica se aprecia ausencia de vagina. Se realiza ecografía y se aprecia
ausencia de útero. Los ovarios son normales ecográficamente. No se observa riñón izquierdo. El diagnóstico más probable es:
A. Síndrome de Rokitanski.
B. Síndrome de Morris.
C. Hiperplasia suprarrenal congénita.
D. Síndrome de ovarios poliquísticos.
E. Síndrome de Kallmann.
85. Joven de 24 años que, a los 3 días de un contacto sexual de riesgo, presenta numerosas lesiones pustulosas, pequeñas, muy pruriginosas, y
que evolucionan a diminutas erosiones, afectando todo el glande y cara interna del prepucio. Señale cuál es, entre las siguientes, la orientación
diagnóstica más probable:
A. Candidiasis genital.
B. Chancroide.
C. Sífilis secundaria.
D. Balanitis por Trichomonas.
E. Infección fúngica por dermatofitos.
86. Un paciente con artritis simétrica que compromete más de 10 articulaciones por más de 3 meses, ¿cuál marcador serológico positivo a lto le
daría el diagnóstico de artritis reumatoide?
A. ANA
B. Anti DNA
C. Factor reumatoideo
D. leucocitosis
E. RNP
87. ¿Cuál de las siguientes medidas es más adecuada para combatir la hiperbilirrubinemia (11.2mg/dl) de un lactante de 3 semanas, con un
desarrollo y crecimiento normal por recibir adecuada técnica de lactancia?
A. Fototerapia
B. Exanguinotransfusión
C. Fenobarbital
D. Observación
E. Suspender lactancia materna por 2 días y dar leche maternizada
88. Paciente de 25 años, en la cual se colocó un dispositivo intrauterino hace 20 días. Presenta dolor pélvico constante y crecie nte. Al examen
físico: cérvix doloroso a la movilización y anexos dolorosos. La complicación más frecuente es:
A. Colpocervicitis
B. Infeccion urinaria
C. Perforacion uterina
D. Expulsion del DIU
E. Enfermedad inflamatoria pelvica
89. Mujer de 40 años, con carcinoma de cervix, presenta sangrado postcoital y dolor pélvico mal definido. Al examen: cérvix con eritroplasia
perioficial de consistencia dura y sangrante. Tacto rectal: parametrio derecho se encuentra infiltrado en un tercio interno ¿ Cuál es el estado
clínico?
A. II a
B. I a
C. II b
D. I b
E. III
90. Gestante de 36 ss por amenorrea. Ingresa por emergencia por dolor abdominal, sangrado vaginal oscuro de aproxi - madamente 100 ml, PA:
90/60 mmHg, pulso 120 x, FR 24 x, afebril. Examen respiratorio y cardiovascular sin alte- raciones, AU 37 cm, contracciones uterinas cada 2
min, de 50 segundos de duración, intensidad (+++), LCF: 100-120 x, examen vaginal con especulo confirma sangrado oscuro a través del cérvix,
el cual está cerrado y presenta una longitud de 3.5 cm ¿Cuál es el diagnóstico más probable?
A. Placenta previa
B. DPPNI
C. Vasa previa
D. Rotura de seno marginal
E. Cáncer de cérvix
91. Según Lalonde y los Campos de la Salud, ¿Cuál no es una de ellas?
A. Medio ambiente
B. Estilos de vida
C. Biología humana
D. Organización de los servicios de salud
E. La familia
92. Según la historia natural de la enfermedad y sus niveles de prevención, ¿Cuál define mejor las acciones para evitar que apare zcan los factores
de riesgo?
A. Prevención primordial
B. Prevención primaria
C. Prevención esencial
D. Prevención inicial
E. Prevención incipiente
93. Con respecto a las hernias, marque la alternativa INCORRECTA:
A. La hernia femoral es muy frecuente en mujeres
B. La técnica de Bassini es un tipo de técnica con tensión
C. La hernia por deslizamiento es de órganos retroperitoneales
D. Pueden existir hernias lumbares por ejemplo de Petit
E. La Técnica de Shouldice es un tipo de reparación tisular
94. Un paciente acude a consulta con una evidente paresia de la musculatura facial derecha que le apareció tres días antes.
¿Cuál de los siguientes datos sugiere que la lesión causante no es periférica y afecta al sistema nervioso central?
A. Oye los sonidos por el oído derecho con más intensidad
B. No nota el sabor de la comida por el lado derecho de la lengua
C. Tiene nistagmus bilateral en la desviación de la mirada hacia la derecha
D. Tiene una acusada debilidad del musculo orbilcular del ojo derecho
E. Tiene erupción y dolor en el conducto auditivo externo derecho
95. Un aloinjerto se define como:
A. El que se da entre individuos con antígenos idénticos
B. Trasplante a un sitio anormal
C. El que se da entre especies diferentes
D. El que ocurre entre miembros de la misma especie genéticamente diferentes
E. Un injerto donde el donante es también receptor
96. En relación con la atención inmediata del RN, señale lo correcto:
A. Secado inmediato para prevenir pérdidas por radiación
B. Aspiración primero de ambas fosas nasales
C. Debe tomarse la temperatura rectal
D. Administrar 1ml de Vitamina K para prevenir enfermedad hemorrágica
E. El Capurro A se evalúa a las 12h de vida
97. Con respecto a la glicemia e hipoglicemia del recién nacido. Marque lo INCORRECTO:
A. Las glucogenolisis dependen del aumento de adrenalina, glucagón y disminución de la insulina
B. El menor valor de glicemia que un RN puede presentar se da a las 2 horas
C. El VIG máximo por vía endovenosa es de 12.5
D. El minibolo consiste en 2g glucosa por cada kilo de peso
E. Luego del minibolo requiere darle dextrosa a un VIG de 6-8
98. En el manejo de RCP neonatal, si un recién nacido a los 30 segundos de manejo tiene una FC<100x´. ¿Qué medida debemos instaurar?
A. Ventilación a presión positiva
B. Intubación
C. Uso de inotrópicos
D. Iniciar compresiones
E. Oxígeno a flujo libre
99. Durante la evaluación de atención inmediata de un RN se detecta maniobra de Barlow y Ortolani positivas. Ante su sospecha diagnostica que
examen solicitaría y que tratamiento instauraría.
A. Ecografía y reducción abierta y osteosíntesis
B. Ecografía y reducción cerrada
C. Ecografía y arnes de Pavlik
D. Radiografía y arnes de Pavlik
E. Radiografía y reducción abierta y osteosíntesis
100. Una medida cuantitativa para determinar la extensión de un brote epidémico es la tasa de...
A. Ataque
B. Incidencia
C. Prevalencia
D. Letalidad
E. Transmisión
Examen 7-A

1. Con respecto a la terapia antitrombótica en el síndrome coronario, señale lo FALSO:


A. Se pueden emplear agentes fibrinolíticos

B. La Hirudina es un inhibidor directo de la Trombina

C. El efecto de la Aspirina es consecuencia del bloqueo de formación de Tromboxano A2

D. El Abciximab no es un inhibidor directo de la trombina

E. La ticlopidina y los inhibidores de receptores de glicoproteína IIb, IIIa so n agentes antiplaquetarios

2. Varón de 16 años, con meningoencefalitis de inicio brusco, LCR muy turbio. Se inicia terapia con penicilina. ¿Con qué anti microbiano
sustituiría a la penicilina?:

A. Dicloxacilina

B. Clindamicina

C. Ceftriaxona

D. Eritromicina

E. Metronidazol

3. ¿Cuál es la etiología más frecuente de artritis séptica en la población general?:

A. Staphylococcus aureus.

B. Pseudomonas aeruginosa.

C. Chlarnydia trachomatis.

D. Neisseria gonorrhoeae.

E. Haemophylus influenzae.

4. Con respecto a las células de Langerhans, marque lo incorrecto:

A. Proceden de la médula ósea

B. Se ubican en el estrato granuloso

C. Poseen enzimas hidrolíticas

D. Poseen gránulos de Birbeck

E. Son captadoras de antígenos cutáneos

5. En una ITU en el niño, marque la incorrecta:

A. En el periodo neonatal las niñas la padecen con más frecuencia.

B. El E. coli es el más frecuente.

C. Se recomienda una ecografía renal en el primer episodio de una ITU

D. Los antibióticos indicados son Cefalosporinas de tercera

E. La gammagrafía con Tc 99 es la mejor exploración para detectar cicatrices renales.

6. Un niño de 10 años presenta, desde hace 4 horas, un cuadro de fiebre elevada, tos, disnea y un estridor perfectamente audi bles en ambos
tiempos respiratorios. El diagnóstico probable es:

A. Absceso retrofaríngeo.

B. Traqueítis bacteriana.

C. Laringitis aguda.

D. Epiglotitis.

E. Aspiración de cuerpo extraño.

7. Adolescente de 15 años de edad, sometido desde hace 4 meses a disminución de peso mediante dietas restrictivas por aludir sobrepeso.
Expresa miedo intenso a volverse obesa, ausencia de 3 ciclos menstruales consecutivos. Al examen: pálida, piel fría, IMC: 13, FC: 50 por
minuto. ¿Cuál es el diagnóstico más probable?

A. Anorexia nerviosa

B. Hipertiroidismo

C. Síndrome de malabsorción

D. Bulimia
E. Síndrome depresivo

8. El Cáncer de pulmón que suele producir síndrome de Pancoast:

A. Carcinoma microcítico de pulmón.

B. Adenocarcinoma.

C. Carcinoma epidermoide.

D. Carcinoma de cèlulas grandes.

E. Carcinoma bronquioloalveolar.

9. ¿Cuál de las siguientes suturas es de material absorbible?:

A. Seda

B. Poliéster

C. Nylon

D. Polipropileno

E. Ácido poliglicolico

10. En apendicitis aguda perforada, los gérmenes más frecuentemente aislados en el cultivo bacteriológi co de líquido peritoneal son:

A. Pseudomonas y Bacteroides fragilis

B. Peptostreptococcus y Bacteroides esplanchnicus

C. Bacteroides fragilis y Escherichia coli

D. Lactobacillus y Escherichia coli

E. Pseudomonas y Escherichia coli

11. Varón de 17 años de edad, contacto de TBC. Acude por dolor progresivo en hemitórax derecho desde hace 10 días y sensación de alza
térmica. La radiografía de TÓRAX muestra derrame pleural y el examen del líquido: proteínas 5g/ dL (sérico: 7 g/dL), DHL 300 mg/ dL. ¿Qué
tipo de líquido es y cuál es la conducta más adecuada?

A. Exudado / buscar etiologia.

B. Trasudado / buscar etiologia.

C. Exudado / drenaje toracico percutaneo.

D. Trasudado / drenaje toracico percutaneo.

E. Exudado / colocacion de tubo de drenaje.

12. Una de las siguientes afirmaciones referidas a las osteomielitis y artritis sépticas causadas por Staphylococus aureus NO es correcta:

A. La osteomielitis por S. aureus puede ser resultado de diseminación hematógena, traumatismos o infección estafilocócica sobreyacente.

B. En los niños, la diseminación hematógena suele afectar las metáfisis de los huesos largos, un área de crecimiento óseo mu y vascularizada.

C. En los adultos la osteomielitis hematógena suele afectar a las vértebras, y rara vez aparece en los huesos largos.

D. La evidencia radiográfica de osteomielitis es previa al inicio de los síntomas clínicos.

E. El absceso de Brodie es un foco aislado de osteomielitis estafilocócica en el área metafisaria de los huesos largos.

13. La etiología más frecuente de la mediastinitis aguda es:

A. CA de pulmón.

B. Neumopatia necrotizante.

C. Perforación esofágica.

D. Traumatismo torácico abierto.

E. Traqueostomia.

14. La diferenciación clínica fundamental y característica entre la desnutrición de tipo marasmático y la de tipo Kwas- hiorkor está dada por:

A. La deshidratación.

B. Los edemas.

C. La coloración y consistencia del pelo.


D. La anemia.

E. Los signos tróficos de la piel.

15. Un niño que presenta signos de urticaria aguda en consulta externa, no tiene antecedentes y tampoco presenta ningún signo de
gravedad. Lo más importante es:

A. Una cuidadosa historia clínica

B. Una descripción adecuada de las lesiones

C. Pedir exámenes de ayuda diagnostica, como: IgE y pruebas cutáneas de alergia.

D. El estudio anatomopatológico por biopsia

E. El manejo con antihistamínicos y corticoides.

16. En un niño de un año cuatro meses con el siguiente hemograma: leucocitos: 2.400, abastonados: 0, segmentados: 20, linfoci tos: 74,
monocitos: 6, Hb: 7,6 g/dL y plaquetas: 30.00 x mm3. El cuadro hematológico corresponde a:

A. Eritroblastopenia transitoria

B. Leucopenia con neutropenia moderada

C. Pancitopenia con neutropenia severa

D. Púrpura trombocitopénica

E. Síndrome anémico con neutropenia leve

17. Son vacunas de microorganismos vivos atenuados. Excepto:

A. Polio oral

B. Hepatitis B

C. Sarampión

D. Rubéola

E. Parotiditis

18. Afectación muscular en la distrofia muscular de Duchenne:

A. Comienza en los grupos musculares proximales de las extremidades

B. Comienza en los grupos musculares distales de las extremidades

C. Afecta exclusivamente a la cintura pélvica

D. Afecta exclusivamente a la cintura escapular

E. Los músculos intercostales se afectan precozmente

19. Señale cuál sería el patrón radiológico que más frecuentemente encontraríamos en un RN que ha sufrido un SAM:

A. Patrón reticulogranular con broncograma

B. Aumento de marcas vasculares con derrame

C. Infiltrados dispersos con atrapamiento aéreo.

D. Patrón en esponja

E. Normalidad radiológica

20. Recién nacido a término de 18 horas de vida, madre refiere que su bebe no succiona bien, se encuentra hipoactivo, poco reacti vo al
estímulo, con temperatura de 38° C. De los siguientes enunciados ¿Cuál estaría en relación como fac- tor de riesgo para sepsis neonatal?

A. Toxemia materna

B. Ruptura de membranas menor a 8 horas

C. Recién nacido grande para edad gestacional

D. Infección de tracto urinario materno en el III Trimestre

E. Rinofaringitis

21. Varón de 20 años de edad, que presenta convulsiones en el servicio de Emergencia. ¿Cuál de los siguientes fármacos administraría?:

A. Carbamazepina
B. Diazepam

C. Metilfenitoína

D. Valproato

E. Difenilhidantoína

22. De los siguientes parásitos, ¿Cuál produce diarrea y síndrome de mala absorción con más frecuenci a?:

A. Oxiuro

B. Áscaris

C. Tricocéfalos dispar

D. Giardia lamblia

E. Hymenolepis nana

23. Respecto a los tratamientos habituales o hábitos de un paciente programado para una intervención quirúrgica bajo anestesi a general,
una de las respuestas siguientes es CORRECTA:

A. Los bloqueantes beta adrenérgicos deben mantenerse durante todo el periodo preoperatorio.

B. Los antiparkinsonianos deben suspenderse 48 hrs antes.

C. La heparina de bajo peso molecular a dosis profilácticas se asocia a hemorragia intraoperatoria significativa.

D. La ticlopidina no se asocia a riesgo de hemorragia intraoperatoria significativa.

E. Evitar fumar cigarrillos durante las 24 hrs previas carece de beneficio intraoperatorio.

24. La localización ideal para hacer una traqueotomía es:

A. La membrana cricotiroidea

B. La membrana miotiroidea.

C. Primer anillo traqueal.

D. Segundo o tercer anillo traqueal.

E. Cuarto o quinto anillo traqueal.

25. Una lesión purpúrica que no palidece a la vitropresión, nos indica:

A. La presencia de edema dérmico.

B. La presencia de exocitosis

C. La presencia de extravasación hemática.

D. La presencia de una malformación venosa.

E. La presencia de una malformación capilar

26. ¿Cuál de las siguientes entidades está asociada con herpes virus tipo 6?

A. Exantema súbito

B. Enfermedad febril de la infancia

C. Síndrome de fatiga crónica

D. Síndrome de Gianotti- Crosti

E. Todas las anteriores.

27. Micosis superficial que afecta el estrato córneo y se manifiesta con lesiones discrómicas con descamación fina:

A. Tiña pedis.

B. Tiña corporis

C. Pitiriasis versicolor

D. Exofialosis

E. Tiña imbricada

28. ¿Cuál es el andrógeno fundamental en la mujer?:


A. Androstenediona.

B. Dehidroepiandrosterona.

C. Androstenediol.

D. Androsterona.

E. Testosterona.

29. En un ciclo menstrual de 30 días, con una gráfica de temperatura corporal basal considerada ovulatoria; la duración de la fase luteínica o
secretoria será de:

A. 21 ± 2 días

B. 07 ± 3 días

C. 18 ± 1 días

D. 14 ± 2 días

E. 16 ± 2 días

30. Los siguientes factores pueden influir en la reserva de hierro al nacer, EXCEPTO:

A. Transfusión feto-materna

B. Transfusión de gemelo a gemelo

C. Rotura de vasos placentarios

D. Anemia de la madre

E. Todas las anteriores

31. En los neonatos normales a término, la capacidad gástrica es:

A. 5 a 9 mL

B. 10 a 15 mL

C. 25 a 30 mL

D. 20 a 45 mL

E. 50 a 90 mL

32. La arteria radial:

A. Es más grande que la arteria cubital

B. Pasa a lo largo del lado cubital de antebrazo

C. Pasa entre el tendón del musculo abductor largo del pulgar y el ligamento radial colateral de la muñeca

D. Comienza a lo largo de la extremidad de la diáfisis del humero

E. Pasa sobre el musculo supinador largo

33. El musculo flexor común profundo esta inervado por los nervios:

A. Mediano e interóseo anterior

B. Mediano y ulnar

C. Musculocutáneo y cubital

D. Cubital y radial

E. Radial y mediano

34. En la pubertad, el cuerpo uterino crece bajo la influencia directa de:

A. Hormonas hipofisiarias

B. Hormonas ováricas

C. Hormonas tiroideas

D. Factores hipotalámicos

E. Hormonas adrenales
35. Un investigador pretende determinar si existe una asociación entre las cifras de PA diastólica (medida en mmHg) y los niv eles de
colesterol (medidos en mg/mL). Para ello, ha realizado estas mediciones a 230 voluntarios. ¿Qué prueba estadística es la MÁS apropiada
pare examinar esta asociación?:

A. Regresión logística.

B. Prueba de la t de Student.

C. Prueba de Chi cuadrado.

D. Correlación de Pearson.

E. Prueba de Fisher.

36. Un estudio analiza la relación entre la PA sistólica (PAS) y la edad en una muestra de mujeres adultas. Los autores presentan los
resultados como la siguiente ecuación de regresión lineal: PAS = 81,5 + 1,2 x edad. ¿Cuál de las siguientes afirmaciones acer ca de este
análisis es FALSA

A. La edad se ha utilizado como variable independiente

B. La pendiente de la recta es de 1,2 mmHg/año de edad

C. El valor 81,5 corresponde a la media de PAS en la muestra de mujeres

D. Por cada año más de edad de las mujeres de la muestra, su PAS se incrementa en 1,2 mmHg de promedio

E. Se ha asumido que la relación entre la edad y la PAS es lineal

37. El test de Capurro para la edad gestacional no incluye:

A. Forma de la oreja

B. Pliegues plantares

C. Textura de la piel

D. Nódulo mamario

E. Ventana cuadrada

38. ¿En cuál de los siguientes casos suele indicarse la prueba de estrógenos para evaluar el origen de una ameno - rrea?:

A. Cuando la prueba de la progesterona es positiva.

B. Cuando la prueba de la progesterona es negativa.

C. Cuando se detectan niveles elevados de gonadotropinas.

D. Cuando se detectan niveles disminuidos de gonadotropinas.

E. En todos los casos de amenorrea primaria.

39. ¿Cuál de los siguientes tendones se inserta en el astrágalo?:

A. Aquiles

B. Peroneo lateral corto

C. Peroneo lateral largo

D. Tibial anterior

E. El astrágalo carece de inserción tendinosa

40. Respecto al desarrollo psicomotor normal del niño, señale la afirmación correcta:

A. Presiona pinzando el pulgar con el índice a los 12 meses

B. Junta las manos en la línea media a los 6 meses

C. Vuelve las páginas de un libro a los 8 meses

D. Usa 4-6 palabras a los 9 meses

E. Se sienta estable y sin apoyo a los 6 meses

41. La edad recomendada para la introducción de la ablactancia es:

A. Entre los 15 días y los 2 meses

B. Entre los 2.3 meses

C. A los 4 meses
D. A los 6 meses

E. A los 8 meses

42. La leche materna, para evitar la sobrecarga osmótica renal en el lactante, contiene menos:

A. Sodio

B. Hierro

C. Potasio

D. Cloro

E. Fósforo

43. De la inmunización activa del sarampión que se utiliza en la actualidad, ¿Cuál es la más eficaz?

A. Gammaglobulina

B. Virus muerto

C. Virus vivo con gammaglobulina

D. Virus vivo atenuado

E. Ninguna de las anteriores.

44. Niño de 6 años de edad con riesgo de tétanos. Si sufre una herida contaminada la conducta correcta es, asumiendo que tiene completas
las inmunizaciones y refuerzo de vacuna antitetánica:

A. Vacuna antitetánica

B. Antitoxina tetánica

C. Inmunoglobulina

D. Antitoxina + vacuna antitetánica

E. No se aplica ni vacuna ni inmunoglobulina

45. La desviación de la atención hacia otros puntos del pensamiento, se denomina…. Y la afectividad inadecuada respecto al co ntexto en
que se produce, se denomina………:

A. Hiperprosexia/Aprosodia

B. Hipoprosexia/Paratimia

C. Paraprosexia/Alexitimia

D. Paratimia/Afasia

E. Disprosexia/Distimia

46. Con respecto a la vacuna triple (DPT), señale lo falso:

A. Debe conservarse en refrigeración entre 4 y 8° C

B. Se aplica por vía IM en una dosis de 0,5 mL

C. Se puede administrar desde los 3 meses hasta los 72 meses de edad

D. La duración de la inmunidad conferida es de 10 años

E. Puede causar reacción febril en las primeras 24 – 48 horas

47. Estructuras que forman el plexo de Kiesselbach, EXCEPTO:

A. Esfenopalatina.

B. Palatina mayor.

C. Etmoidal anterior.

D. Etmoidal posterior.

E. Palatina menor.

48. Las pruebas no paramétricas:

A. Únicamente se utilizan para comparar distribuciones de variables continuas.

B. Requieren la comprobación del requisito de normalidad.


C. Originan unos valores de error alfa similares a los que se calculan mediante pruebas paramétricas.

D. Deben utilizarse siempre que manejemos muestras de gran tamaño.

E. No realizan asunciones sobre el tipo de distribución de la variable

49. Uno de los siguientes NO es un test no paramétrico:

A. U de Mann-Whitney

B. Test de Kruskal-Wallis

C. Test de Wilcoxon

D. Análisis de varianza.

E. Coeficiente de correlación de Spearman

50. Cuál de las siguientes afirmaciones es falsa:

A. En una paciente perimenopáusica y deseos de descendencia cumplidos, que no responde al tratamiento con gestágenos es candidata a una
ablación endometrial o histerectomía

B. Durante los 18 meses posteriores a la menarquia son frecuentas los ciclos anovulatorios, a intérvalos irregulares.

C. Debido a lo comentado en el apartado anterior, son frecuentes el flujo menstrual irregular entre las adolescentes jóvenes

D. Cuando el sangrado menstrual excesivo, comienza con la menarquia y como consecuencia del sangrado produce una a nemia severa, hay
que descartar un trastorno de la coagulación (deficiencia del factor de Von Willebrand)

E. En la valoración de la HUD es imprescindible la realización de un estudio hormonal.

51. El agente causal del impétigo ampollar en lactante y niños pequeños es:

A. Aeromona hydrophila

B. Estafilococo aureus coagulasa positivo

C. Estreptococo piógeno

D. Estreptococo pneumoniae

E. Legionella

52. Varón de 32 años, tiene sensación de pesadez escrotal, con tumoración difusa de consistencia lisa y firme di stal a la cabeza del
epidídimo. No signos inflamatorios, marcadores tumorales negativos, transiluminación negativa. El diag - nóstico más probable es:

A. Hidrocele.

B. Seminoma.

C. Carcinoma embrionario.

D. Hernia inguinal directa.

E. Hernia inguinal indirecta.

53. Niño nacido de parto normal hace 2 días. Desde hoy presenta edema palpebral y abundante secreción purulenta en ambos ojos . El
diagnóstico más probable es conjuntivitis neonatal por:

A. Gonococo

B. Clamidia

C. Estreptococo

D. Herpes simple

E. Haemophilus

54. La hemorragia intracraneal neonatal es más frecuentemente encontrada en los siguientes casos, EXCEPTO:

A. Prematuros

B. Neonatos nacidos por cesárea

C. Neonatos con presentación podálica

D. Neonatos nacidos con ayuda mecánica, excepto el fó rceps bajo

E. Neonatos que han sufrido asfixia

55. ¿Cuál es el principal tratamiento psicofarmacológico del trastorno obsesivo compulsivo?


A. Antipsicóticos

B. Benzodiacepinas

C. Antidepresivos inhibidores de la recaptación de noradrenalina

D. Antidepresivos inhibidores de la recaptación de serotonina

E. Estimulantes

56. Cuál es la finalidad de la administración entendida como una ciencia:

A. Explicar el comportamiento y la conducción de la organización

B. Uso de técnicas para la conducción de la organización

C. Comprender el comportamiento del personal

D. Explicar el porqué del éxito de la organización

E. Explicar el funcionamiento y evolución de la organización

57. La secuencia lógica del proceso administrativo es:

A. Organización-dirección-control-planificación

B. Dirección-planificación-organización-control

C. Planificación-organización-dirección –control

D. Planificación-dirección-organización-control

E. Organización-planificación-dirección-control

58. Respecto a trastornos de la refracción, señale la respuesta INCORRECTA:

A. La presbicia se corrige con lentes positivas.

B. Las ametropías esféricas se corrigen con lentes cónicas.

C. En las lentes negativas, el movimiento de la imagen acompaña al movimiento de cristal.

D. En la hipermetropía la imagen de la esquiascopÍa acompaña el movimiento del retinoscopio.

E. El tratamiento del astigmatismo en la actualidad es la cirugía láser (LASIK, etc.).

59. El reflejo de acomodación consta de los siguientes componentes:

A. Convergencia ocular, contracción pupilar y acomodación.

B. Convergencia ocular, dilatación pupilar y aumento de la concavidad de la superficie retiniana

C. Contracción pupilar, aumento del diámetro AP del cristalino y aplanamiento retiniano.

D. Miosis, enoftalmos y reducción de la presión de la cámara anterior.

E. Contracción pupilar y contracción del musculo ciliar.

60. La vena porta generalmente se forma por la confluencia de las venas:

A. Mesentérica superior y esplénica

B. Mesentérica superior y mesentérica inferior

C. Mesentérica superior y coronaria estomáquica

D. Mesentérica inferior y esplénica

E. Esplénica y coronaria estomáquica

61. El ectropión puede ocasionar:

A. Blefaritis

B. Triquiasis

C. Blefaroespasmo

D. Distiquiasis

E. Queratopía por exposición

62. ¿Cuál de los siguientes pólipos tiene mayor potencial maligno?:


A. Pólipo juvenil

B. Pólipo hamartamatoso.

C. Pólipo hiperplásico.

D. Pólipo adenomatoso.

E. Ninguna de las anteriores.

63. ¿Cuál es el procedimiento quirúrgico curativo en el cáncer localizado en el tercio distal del recto?:

A. Resección abdominoperineal

B. Íleo transverso anastomosis

C. Extirpación mesorectal total

D. Colostomía sigmoidea en asa

E. Colectomía izquierda

64. Paciente varón de 75 años que presenta prurito, ictericia y dolor leve en cuadrante superior derecho del abdomen. Al examen
hepatomegalia. En la ecografía se muestra dilatación de vías biliares intra y extrahepaticas sin cálculos en vía biliar. La T AC de abdomen no
muestra masas en páncreas. Tiene BT de 10, FA de 400 y amilasa normal. En el diagnóstico diferencial del paciente debe incluirse lo
siguiente:

A. Cirrosis biliar primaria.

B. Colestasis inducida por drogas.

C. Colangiocarcinoma intrahepático.

D. Ampuloma.

E. Enfermedad de Caroli.

65. Paciente de 65 años, con cuadro de ictericia progresiva, coluria y dolor abdominal. En las últimas 58 horas se agregan fiebre y
escalofríos. ¿Cuál es el diagnóstico más probable?

A. Absceso hepático.

B. Colangitis aguda.

C. Hepatitis aguda.

D. Neoplasia de la vesícula biliar.

E. Ninguna de las anteriores.

66. ¿Cuál es el vaso sanguíneo que permite diferenciar una hernia inguinal directa de la indirecta?:

A. Femoral

B. Epigástrica inferior

C. Folicular

D. Deferente

E. Del epidídimo

67. Multípara de 40 años de edad, presenta vómitos y distensión abdominal. Al examen físico de la región inguinocrural se encuentra
tumoración dolorosa no reductible por debajo del ligamento iliopubiano. ¿Cuál es el diagnóstico más probable?:

A. Hernia inguinal indirecta

B. Hernia crural

C. Hernia inguinal directa

D. Várices del cayado de la safena

E. Hernia obturatriz

68. El patógeno intestinal que puede asociarse al síndrome de Guillan Barré es:

A. Criptosporidium.

B. Shigella.

C. Salmonella.
D. Yersinia.

E. Campylobacter.

69. La alteración del equilibrio ácido-básico producida por los vómitos repetidos por el síndrome pilórico es:

A. Acidosis hiperclorémica.

B. Acidosis hipernatremica

C. Acidosis Hipoclorémica.

D. Alcalosis hiperpotasémica.

E. Alcalosis hipoclorémica.

70. El seguimiento postoperatorio para detectar la recurrencia del cáncer de colon se realiza con:

A. Marcadores tumorales

B. Resonancia magnética

C. Radiografía abdominal

D. Estudio de la función hepática

E. Ecografía abdominal

71. El tratamiento de elección de las hemorroides externas trombosadas es:

A. Dieta rica en fibra

B. Ablación quirúrgica

C. Baños de asiento

D. Ligadura con banda de caucho

E. Reducción manual

72. Mujer de 72 años de edad, con 7 días de dolor en el cuadrante superior derecho, ictericia progresiva y temperatura de 39 ºC recibió
tratamiento con cefalexina vía oral, sin mejoría. Ingresa a Emergencia hipotensa, con compromiso del sensorio. Leucocitos: 19 .000 x mm2,
bilirrubinas, transaminasas y fosfatasa alcalina elevadas, concentración de amilasa sérica normal. ¿Cuál es su diagnóstico?

A. Hepatitis.

B. Colelitiasis.

C. Enfermedad de Wilson

D. Colecistitis aguda alitiásica.

E. Colangitis supurativa aguda.

73. Señale lo CORRECTO en relación al carcinoma de la vesícula biliar:

A. La frecuencia es menor en pacientes con “vesícula en porcelana”

B. El tratamiento adyuvante con quimio o radioterapia es muy eficaz

C. El adenocarcinoma no es el más frecuente

D. Se asocia a cálculo de vesícula

E. El coeficiente varón/mujer igual al 2/1

74. En el caso de un paciente con hiperplasia prostática benigna, ¿Cuál, entre las siguientes circunstancias, NO establece po r sí misma
indicaciones de cirugía?:

A. Hematuria severa recurrente.

B. Retención urinaria que requiere sondaje vesical permanente.

C. Infecciones de orina de repetición.

D. Nicturia de dos veces.

E. Hidronefrosis retrograda.

75. Varón de 30 años de edad con fractura cerrada transversal de tercio medio de fémur, tras sufrir un accidente de trá - fico, sin otras
lesiones asociadas. ¿Cuál es la conducta a seguir?:

A. Tracción transesquelética.
B. Clavo intramedular.

C. Placa y tornillos.

D. Fijador externo

E. Yeso.

76. En la fractura de Monteggia hay:

A. Fractura del 1/3 distal del cúbito + luxación de cabeza de radio

B. Fractura de 1/3 proximal del cúbito + luxación de la cabeza radial

C. Fractura de cúbito y radio + luxación de codo

D. Fractura de 1/3 medio del radio + luxación del codo

E. Ninguno de ellos

77. Paciente varón 24 años, cae mientras jugaba fútbol en la calle golpeando directamente la rodilla derecha contra el pavimento,
inmediatamente presenta intenso dolor, edema e incapacidad para extender la pierna derecha. ¿Qué me- didas se deben tomar
inmediatamente?

A. Frotación y que siga jugando.

B. Hielo, ferulización y traslado a emergencia.

C. Traccionar la rodilla hasta lograr extensión.

D. Analgésicos y que descanse en su casa.

E. Traccionar la rodilla hasta lograr flexión.

78. Los siguientes signos sugieren el diagnóstico de displasia de cadera:

A. Asimetría de pliegues glúteos

B. Signo de Barlow positivo

C. Limitación en la abducción

D. Signo de Ortolani positivo

E. Todas las anteriores son correctas

79. En sala de recuperación Usted sospecha sobredosis de anestesia por lo siguiente:

A. Paciente con hipotensión y taquicardia.

B. Paciente con hipertensión y bradicardia.

C. Paciente con hipertensión y taquicardia.

D. Paciente con hipotensión y bradicardia.

E. Paciente con excitación psicomotriz.

80. El tumor primario más frecuente en mediastino posterior es:

A. Linfoma.

B. Fibroma.

C. Quiste esofágico.

D. Tumor neurogénico.

E. Quiste broncogénico.

81. ¿Cuál es el test que tiene por objetivo comparar la audición de un sonido transmitido por vía aérea, poniendo el dia - pasón delante del
pabellón auricular, con la audición del mismo sonido transmitido por vía ósea, situado el diapasón sobre la mastoides?

A. Test de Weber.

B. Test de Rinné.

C. Test de Schwabach.

D. Test de Fowler.

E. Test de Sisi.
82. Hombre joven con dolor facial y congestión nasal, descarga nasal amarilla después de infección de vías respiratoria alta hace 10 días. T°
38.2°C y sensibilidad del seno maxilar a la palpación y mucosas nasales pálidas con algo de dre- naje amarillento. Obnubilación del seno
maxilar a transiluminación. ¿Cuál es el diagnóstico MÁS probable?:

A. Sinusitis aguda.

B. Sinusitis crónica.

C. Angina de Vincent.

D. Angina de Ludwig.

E. Celulitis orbitaria.

83. El raquitismo se caracteriza por:

A. Craneotabes.

B. Hendiduras de Harrison.

C. Retardo en el cierre de la fontanela.

D. Tórax de paloma

E. Todas las anteriores.

84. Señale lo correcto con respecto al tratamiento de la bronquiolitis en el niño:

A. Administrar un antibiótico que tenga una menor resistencia.

B. Incluir corticoides por ser beneficiosos.

C. La Ribavirina es útil en todos los casos

D. Los fármacos broncodilatadores se aplican empíricamente.

E. En casos severos la traqueotomía no es beneficiosa.

85. Lactante de 6 meses que hace tres días inicia con rinorrea y tos progresiva. Al examen: tiraje subcostal, taquipnea, espiración
prolongada, sibilancias y cianosis perioral. La radiografía de pulmones revela hiperinflación. ¿Cuál es la terapia inmediata inicial?

A. Salbutamol

B. Amoxicilina

C. Ribavirina

D. Oxígeno húmedo frío

E. Eritromicina

86. En un centro de salud la enfermera da una charla sobre control prenatal a un grupo de madres, la semana siguiente la obstetriz
da la misma charla al mismo grupo de madres. La enfermera ordena a una técnica de enfermería que realice una visita domicilia ria, mientras
que el médico manda a la misma técnica que lo ayude en consultorio externo. ¿Estos problemas a que componentes de la administr ación
corresponden?:

A. Planificación.

B. Organización.

C. Control.

D. Dirección.

E. Evaluación.

87. El eczema atópico usualmente aparece a la edad de:

A. 2 semanas

B. 3 meses

C. 2 años

D. 6 años

E. 12 años.

88. Varón de 5 años presenta de forma súbita dolor en teste derecho, sin antecedente de traumatismo previo. Exploración: afebril,
teste tumefacto, doloroso a la exploración, con ausencia del reflejo cremastérico. Señale la afirmación falsa:

A. En el Eco-Doppler se observaría disminución del flujo sanguíneo en ese testículo.


B. El tratamiento consiste en antibióticos, antiinflamatorios, y tras la fase aguda, cirugía.

C. La mayor incidencia se produce en la niñez tardía y adolescencia temprana.

D. En mayores de 13 años, se debe hacer diagnóstico diferencial con una epididimitis.

E. En el periodo neonatal, generalmente tiene mal pronóstico.

89. Niño pálido, con parpados hinchados y orinas cargadas. Padres refieren antecedente de cuadro faringoamigdaliano quince días
antes, tratado medicamente. ¿Cuál es el diagnóstico más probable?

A. Glomerulonefritis.

B. Infección del tracto urinario.

C. Litiasis renal.

D. Nefritis intersticial.

E. Síndrome nefrótico.

90. Un lactante de 7 meses presenta, presenta fiebre 38ºC. No existe ningún otro síntoma y la exploración física es nor- mal, salvo el
hallazgo de un cuadro catarral leve, conjuntivitis y adenopatías cervicales posteriores. El niño parece encontrarse bastante bien. ¿Cuál de
las siguientes afirmaciones describe mejor los factores diagnósticos que deben considerarse en este caso?:

A. Si apareciera una erupción cutánea 24 horas después de comenzar la fiebre, sería probable el diagnóstico de roséola o exantema súbito.

B. En la roséola infantil, el diagnóstico diferencial puede ser difícil, puesto que en las primeras 36 horas cursa con leucocitosis con predominio
de neutrófilos.

C. La fiebre podría corresponder a los pródromos de una Enfermedad de Kawasaki.

D. Si la fiebre dura 2 días, y luego aparece un exantema confluyente en cara de 3 días después, es probable que sea una Rubeola

E. Es probable una escarlatina, pues es una enfermedad propia de niños de 6 a 18 meses.

91. Establezca la relación entre las siguientes columnas:

A. Sarampión

B. Rubéola

C. Varicela

D. Mononucleosis infecciosa

E. Herpes simple

1. Linfadenopatía

2. Lesiones simultáneas en distintos estadíos generalizada + esplenomegalia

3. Vesículas pequeñas sobre base eritematosa

4. Manchas de Forchheimer

5. Manchas de Koplick

A. (a–5) (b–4) (c–2) (d–1) (e–3)

B. (a–2) (b–5) (c–4) (d–3) (e–1)

C. (a–1) (b–4) (c–2) (d–3) (e–5)

D. (a–3) (b–4) (c–2) (d–5) (e–1)

E. (a–4) (b–2) (c– 5) d–1) (e–3)

92. En Emergencia se recibe a una niña de 10 años de edad que ha presentado durante 40 minutos 8 crisis convulsivas, entre las cuale s no
ha recuperado la conciencia. El familiar refiere que las convulsiones han tenido características tónico -clónicas generalizadas. Su
diagnóstico es:

A. Convulsiones tónico-clónicas simples

B. Epilepsia

C. Convulsión febril

D. Síndrome de West

E. Estado epiléptico
93. ¿Cuál es el tratamiento más efectivo en el espasmo infantil o síndrome de West?

A. Fenobarbital

B. Etosuximida

C. Difenilhidantoína

D. Vigabatrina

E. Carbamazepina

94. Según el calendario de inmunizaciones, ¿Qué vacuna debe recibir el niño a los 12 meses de edad?

A. AMA (AntiamarílicA)

B. Refuerzo DPT

C. Polio oral

D. SPR (Triple víricA)

E. 3ra dosis Pentavalente

95. Niña de 9 meses que inicia vómitos y dolor abdominal hace 12 horas, minutos antes de ir a emergencia, presento de - posiciones color
rojo oscuro. El abdomen se encuentra distendido y con dolor discreto a la palpación. El diagnóstico más probable es:

A. Estenosis pilórica

B. Apendicitis aguda

C. Infección urinaria

D. Invaginación

E. Enfermedad ulcero péptica

96. ¿Cuál de las siguientes afirmaciones es correcta respecto a la talla baja asociada a déficit de hormona de crecimiento (G H)?

A. La deficiencia de GH es la causa más frecuente de hipocrecimiento armónico patológico.

B. En las formas congénitas de deficiencia de GH, el crecimiento prenatal suele ser normal.

C. La maduración ósea en la deficiencia de GH, al contrario de lo que ocurre en otras endocrinopatías, se encuentra muy elevada.

D. Los valores séricos de IGF-I se encuentran claramente elevados en la deficiencia de GH.

E. El tratamiento con GH debe iniciarse precozmente, pero debe suspenderse antes de la pubertad, por el riesgo de desarrollo de leucemia en este
periodo de la vida.

97. Por lo que hace referencia al tratamiento del asma bronquial, una de las siguientes respuestas es FALSA:

A. El salbutamol se indica como medicación de rescate.

B. La prednisona oral a pequeñas dosis puede estar indicada en el asma inestable moderada.

C. Los glucocorticosteroides inhalados se metabolizan en el hígado.

D. Los agonistas adrenérgicos-beta2 pueden emplearse en combinación con los glucocorticosteroides inhalados en el asma persistente leve-
moderada.

E. El efecto del salmeterol dura 24 horas.

98. Señale lo correcto en relación a la edad gestacional por examen físico del RN:

A. Test de Capurro evalúa el color del nódulo mamario y tiene mucha correlación estadística con la edad gestacional

B. El test de Ballard considera el examen físico y neurológico del RN y puede ser aplicado a prematuros

C. La edad gestacional por examen físico es más exacta que la edad gestacional por FUR

D. La edad gestacional por Capurro tiene una variación de +/– 6 semanas y puede hacerse máximo hasta las 12 horas de vida.

E. El método de Usher añade evaluaciones neurológicas

99. Niño de 4 semanas de edad, recibe lactancia materna exclusiva, gana peso 30 g x día, ictérico, hasta la raíz del muslo, l uce saludable,
Grupo sanguíneo: O (+), Hto: 48%, Reticulocitos 1%, Bilirrubinas totales: 12 mg/dL, Bilirrubina di- recta: 0.5 mg/dL, TSH normal. En relación a
la lactancia materna exclusiva. ¿Cuál es la conducta a seguir?

A. Suspender por una semana

B. Suspender por 48 horas


C. Continuar e iniciar fototerapia

D. Suspender e iniciar fototerapia

E. Proseguir con la lactancia materna.

100. La causa más frecuente de síndrome de dificultad respiratoria o enfermedad de membrana hialina en el recién nacido prema turo es:

A. El defecto de surfactante pulmonar por inmadurez.

B. Un neumotórax a tensión.

C. Un síndrome de aspiración meconial.

D. Una infección respiratoria.

E. La escasa fuerza muscular

Examen 8-A

1. Paciente varón de 60 años con antecedente de HTA e Insuficiencia renal crónica en terapia dialítica regular, ingresa a emergencia por
disnea, SatO2 79%, PA 210/110mmHg, crépitos en ACP y alteración de conciencia. Marque la FALSA:

A. Se trata de una emergencia hipertensiva.

B. Requiere descartar evento cerebrovascular.

C. Probablemente presente edema agudo de pulmón.

D. Requiere control de PA inmediato con objetivo 120/80mmHg en 24 horas.

E. Está contraindicado el uso de Betabloqueadores EV y VO.

2. ¿Cuál es el agente etiológico de la endocarditis infecciosa en un usuario de drogas endovenosas?

A. Klebsiella pneumoniae.

B. Enterococos faecalis.

C. Estreptococos viridans.

D. Neisseria gonorrhoeae.

E. Staphylococcus aureus.

3.¿Cuál de las siguientes es la manifestación más frecuente en la depresión?:

A. Suicidio

B. Bajo rendimiento

C. Alcoholismo

D. Pseudodemencia

E. Agresividad

4. En relación a la diabetes insípida es falso:

A. La prueba de deshidratación se suspenderá cuando pierda un 10% del peso corporal inicial.

B. Ha sido descrito un origen autoinmune en algunos casos de diabetes insípida central.

C. El tratamiento de la diabetes insípida nefrogenica comporta la utilización de diuréticos.

D. La lisina vasopresina no es la hormona antidiuretica natural humana.

E. La poliuria de la diabetes insípida no respeta el sueño.

5.El ERGE puede relacionarse con cualquier de las entidades siguientes, EXCEPTO una:

A. Fibrosis pulmonar.

B. Broncoespasmo.

C. Neumonía recurrente.

D. Sinusitis.

E. Laringotraqueitis.

6. El esquema de elección para el tratamiento de la tuberculosis pulmonar en el adulto no tratado previamente es:
A. 2HRSE/4R2H3

B. 2RHRZSE/4R2H2E2

C. 2HRZ/4R2H2

D. 2HRZE/4H3R3.

E. HRZE/7R2H2

7.El síndrome nefrítico consiste en:

A. Hematuria, proteinuria, hiperazoemia y retención renal de agua y sal

B. Proteinuria de unos 4g/d con hematuria e HTA

C. Disuria y leucocituria

D. Hematuria e HTA con función renal normal

E. Microhematuria recidivante en niños

8. Es un trastorno neuromuscular que se caracteriza por debilidad muscular especialmente de los labios y extraoculares (diplopía y ptosis)
así como debilidad proximal y fatigabilidad de los músculos esqueléticos, con reflejos osteotendino- sos profundos conservados:

A. Miastenia Gravis.

B. Miopatía por corticoides.

C. Miopatía por hipertiroidismo.

D. Neurastenia.

E. Botulismo.

9. Un chico de 17 años, deportista y sin antecedentes de interés, acude al médico por presentar ictericia conjuntival oca - sionalmente. Niega
ingesta de fármacos y dolor abdominal. En los análisis realizados se observan cifras de ALT, AST, GGT y FA normales, con bilirrubina total
de 3,2 mg/dl y bilirrubina directa de 0,4 mg/dl. No tiene anemia y el hígado es ecográficamente normal. ¿Cuál es su diagnósti co?

A. Síndrome de Rotor.

B. Coledocolitiasis.

C. Síndrome de Dubin-Johnson.

D. Hepatitis aguda.

E. Síndrome de Gilbert.

10.El nervio laríngeo recurrente inerva todos los músculos intrínsecos de la laringe, EXCEPTO:

A. Cricotiroideo.

B. Cricoaritenoideo lateral.

C. Aritenoideo oblicuo.

D. Cricoaritenoideo posterior.

E. Tiroaritenoideo.

11.Mujer de 13 años con dolor en mesogastrio que se irradia, hacia fosa iliaca derecha, se acompaña de náuseas, y vómi - tos. Al examen:
temperatura de 38,4 C, abdomen doloroso en hemiabdomen inferior derecho. Laboratorio: Sedimento urinario: 6 -8 hematíes x campo.
Hemograma: leucocitosis de 14 000 x mm3, FUR hace 30 días. Tiene como presunción diagnóstica apendicitis aguda. ¿Cuál de las siguientes
patologías se considera en el diagnóstico diferencial?

A. Poliposis intestinal

B. Vólvulo de ciego

C. TBC entero peritoneal

D. Litiasis vesical

E. Diverticulitis de Meckel

12.En la anestesia epidural, ¿Cuáles de los signos clínicos se presentan como complicación por punción e inyección inad - vertida de la
duramadre?

A. Hipertensión arterial y endocraneana

B. Hipotensión grave, parálisis respiratoria


C. Hipertensión arterial, fiebre

D. Fiebre, hipotensión

E. Hipertensión, oliguria

13.Frente a un paciente varón de 10 años de edad, con 72 horas de enfermedad, con fiebre, dolor ósea localizado en tibia, eri tema de partes
blandas, con sospecha de osteomielitis, el método de diagnóstico precoz más útil es:

A. Proteína C reactiva

B. Hemocultivo

C. Velocidad de sedimentación

D. Radiografía de tibia

E. Gammagrafía ósea

14.Paciente varón de 70 años de edad, con diagnóstico clínico de vólvulo de sigmoides, en mal estado general, tóxico, taquicárdico,
hipotenso y con reacción peritoneal. ¿Qué tipo de cirugía es la MÁS ADECUADA?:

A. Colostomía.

B. Resección más colostomía.

C. Cecostomia más pexia.

D. Resección más anastomosis.

E. Ileostomía.

15.Paciente de 72 años, que acude a Emergencia por hematoquezia. Durante la evaluación presenta hipotensión y shock hipovolemico que
responde al tratamiento médico. Se coloca sonda nasogástrica no evidenciándose sangrado. Por frecuencia, ¿cuál sería su primera
posibilidad diagnóstica?:

A. Cáncer de colon derecho.

B. Colitis ulcerativa hemorrágica.

C. Hemorragia diverticular de colon.

D. Hemorroides.

E. Malformación arteriovenosa.

16.Con respecto al Sistema de Redes y Microrredes de Salud, marcar lo que no corresponda:

A. Los establecimientos de salud del primer de salud se ordenan principalmente en base a Redes y Microrredes de Salud.

B. La unidad básica de gestión lo constituyen las microrredes de salud.

C. Los puestos de salud por lo general son cabecera de redes de salud

D. Los centros de salud pueden contar o no con internamiento.

E. La conformación y/u organización de las Redes y Microrredes está a cargo de los Gobiernos Regionales.

17.Un centro de salud programó para el año 2009 realizar 2000 controles de crecimiento y desarrollo, para 1 500 niños menores de 5 años,
utilizando 800 horas enfermera. AI finalizar el año atendió 200 consultantes nuevos, 300 reingre- santes y 500 continuadores, utilizando 600
horas enfermera. El porcentaje de avance de meta en atenciones fue:

A.50

B.85

C.25

D.75

E. 100

18.La causa más frecuente de síndrome de Asherman es:

A. Infección endouterina posparto.

B. Infección endouterina postaborto.

C. Miomectomía.

D. Tuberculosis genital.
E. Legrado uterino diagnóstico.

19.El legrado como medio diagnostico puede ser omitido antes del tratamiento de una HUD en el siguiente caso:

A. Adolescencia.

B. Postmenopausia.

C. Premenopausia.

D. Todas las anteriores.

E. Ninguna de las anteriores.

20.¿Cuál de las siguientes alternativas NO corresponde a endometriosis moderada a severa?

A. CA 125 disminuido

B. Dismenorrea secundaria

C. Dispareunia

D. Dolor pélvico crónico

E. Subfertilidad

21.¿Cuál de los siguientes criterios se usa para establecer el diagnostico de Vaginosis bacteriana?

A. Aumento de células indicio y aumento de leucocitos.

B. Secreción blanquecina adherente a la pared vaginal

C. pH de secreción vaginal menor de 4,5

D. Test de aminas o KOH positivo

E. Incremento de leucocitos en el frotis vaginal

22.Mujer de 30 años de edad, presenta retraso menstrual de 7 días. Para diagnosticar un posible embarazo, ¿Cuál es el examen
confirmatorio?:

A. Estrógenos

B. Progesterona

C. HCG-beta

D. LH

E. FSH

23.¿Cuáles son los signos y síntomas de la ruptura uterina consumada?

A. Acinturamiento del útero, dinámica uterina intensa, sufrimiento fetal agudo

B. Cese de contracciones uterinas, palpación fetal fácil, sufrimiento fetal agudo

C. Descenso del útero, sangrado silencioso, dinámica uterina irregular

D. Sangrado vaginal, crecimiento uterino, hipertonía

E. Tono uterino aumentado, sangrado escaso, taquicardia, hipotensión

24.Una primigesta de 37 semanas de edad gestacional ingresa por rotura prematura de membranas de 24 horas de evo - lución. No hay
trabajo de parto ni signos de infección amniótica El test de Bishop (de maduración cervical) es de 6 puntos. El feto esta en cefálico y no hay
signos de sufrimiento fetal. ¿Cuál es la conducta obstétrica indicada?:

A. Cesárea.

B. Inducción del parto con Oxitocina.

C. Administrar Antibióticos y corticoides e inducir el parto 48 horas después.

D. Administrar Antibióticos y esperar el comienzo espontáneo del parto.

E. Esperar el comienzo espontáneo sin tratamiento y hacer cesárea si aparecen signos de i nfección amniótica.

25.En un examen, a un RN se le comprueba ausencia de murmullo vesicular en un lado del tórax. La causa más probable es:

A.

B. Bronconeumonía.
C. Neumotórax.

D. Atelectasia

E. Síndrome de distrés respiratorio.

F. Enfermedad de membrana hialina.

26.RN de 38 semanas de edad gestacional presenta distensión abdominal y vómitos biliosos durante el primer día de vida. En el examen
físico se palpan cordones duros que siguen el marco cólico. Una radiografía en bipedestación muestra una masa en pompa de jabón en
cuadrante inferior derecho. El tratamiento inmediato más apropiado es:

A. Antibioterapia.

B. Supositorios de glicerina.

C. Enema de solución hiperosmolar.

D. Estimulación rectal.

E. Cirugía.

27.Un recién nacido, con peso al nacer de 4300 gramos, hijo de madre con diabetes gestacional, doble circular ajustada a cuello; se muestra
muy inquieto a los 30 minutos de vida; quince minutos más tarde, sufre una convulsión tónico clónica. El diagnóstico más prob able es :

A. Hipoglucemia.

B. Hipocalcemia.

C. Hiponatremia.

D. Hiperviscosidad.

E. Encefalopatía hipoxico isquémica

28.¿Cuál es el corticoide más utilizado en el tratamiento del Crup?:

A. Dexametasona

B. Fluticasona

C. Hidrocortisona

D. Metilprednisolona

E. Prednisona

29.Niño de 1 año y 9 meses con diarrea mucosanguinolenta y fiebre alta. Hace 1 semana recibió de regalo un perro de pocos días de nacido.
¿Cuál es el germen causal más probable?

A. Campilobacter Yeyuni

B. Entamoeba Histolytica

C. E. Coli

D. Salmonella No tiphica

E. Yersinia enterocolitica

30.Niña de 2 años de edad, que desde los 9 meses presenta episodios de lesiones eritematosas descamativas en mejillas, abdome n y partes
extensoras de miembros, con prurito intenso. Actualmente aparecen lesiones similares después de comer maní. Al examen se evidencia
pliegues redundantes infraorbitarios. ¿Qué tipo de dermatitis consideraría en su presunción diagnóstica?:

A. Por ingesta de alimentos

B. Seborreica

C. Alérgica de contacto

D. Eccematoide infecciosa

E. Atópica

31.Las anastomosis entre el sistema portal y el de las venas cavas se encuentran en los siguientes sitios, EXCEPTO:

A. Recto.

B. Bazo.

C. Retroperitoneo.
D. Cardias.

E. Ombligo.

32.La sensibilidad de una prueba diagnóstica mide:

A. La proporción de casos de pacientes sin la enfermedad que presentan un resultado negativo de la prueba diagnóstica.

B. La proporción de casos de pacientes sin la enfermedad que presentan un resultado positivo de la prueba diagnóstica.

C. La proporción de pacientes que se someten a la prueba que tienen la enfermedad.

D. La proporción de casos de enfermos con resultado positivo de la prueba diagnóstica.

E. La proporción de casos con resultado positivo de la prueba que son verdaderamente enfermos.

33.En una comunidad hay abundante rabia animal y se notifican algunos casos humanos todos los años. Con el fin de fijar las bases para
iniciar un programa de control, entre las cuales se señalan las siguientes medidas: ¿Cuál a su juicio, puede dar resultados M ÁS
permanentes?:

A. Campaña para exterminar perros vagabundos.

B. Campaña masiva de educación sanitaria en la población.

C. Campaña masiva para hacer tratamiento antirrábico a todas las personas mordidas.

D. Establecer un programa de vacunación canina (80% de los perros estimados) en un año.

E. Sistema de inspección de carne de los mataderos.

34.Paciente de 38 años que, cuando se encuentra paseando por el campo e inmediatamente después de la picadura de una abeja, p resenta:
urticaria, eritema, disnea y shock. ¿Cuál es el tratamiento MÁS resolutivo para este paciente?:

A. La administración de un corticoide inhalado.

B. La administración endovenosa de teofilina.

C. La administración de un antihistamínico.

D. La administración de corticoides intramusculares.

E. La administración de adrenalina intramuscular

35.Los oligosacáridos unidos a proteínas o lípidos de la membrana plasmática se encuentran:

A. Atravesando la bicapa lipídica.

B. A ambos lados de la membrana.

C. En la cara citoplasmática de la membrana.

D. En la cara extracelular de la membrana.

E. No existen oligosácaridos unidos a la membrana.

36.Si la tasa de natalidad es de 10 por mil habitantes y la población de 100.000 habitantes, ¿Cuántos nacimientos se pro - dujeron?:

A.10

B. 100

C. 1.000

D. 5.000

E. 10.000

37.Las complicaciones del infarto transmural incluyen a todo lo siguiente, EXCEPTO:

A. Endocarditis marántica.

B. Taponamiento cardiaco.

C. Arritmia.

D. Ruptura de músculo papilar.

E. Ninguna de las anteriores.

38.Un paciente acude al servicio de urgencia con una historia de hematemesis. El pulso arterial es de 130x´. La PA es de 100/75mmHg.
Frialdad y sudoración de manos y pies. ¿Qué tipo de acción es prioritaria?:

A. Realizar endoscopia.
B. Colocar vías intravenosas y restablecer el volumen circulante.

C. Determinar hemoglobina y valor hematocrito.

D. Practicar exploración radiológica con bario.

E. Administrar somatostatina.

39.La aerobilia es un signo radiológico que puede observarse en uno de los siguientes procesos patológicos:

A. Colecistitis aguda alitiásica.

B. Peritonitis biliar no traumática.

C. Paciente colecistectomizado hace dos meses.

D. Colecistitis crónica calculosa no complicada.

E. Paciente con cáncer vesicular no invasivo

40.En el caso de que el enfermo fuera intervenido quirúrgicamente, los hallazgos quirúrgicos que encontraría más pro- bablemente serían:

A. Apendicitis perforada.

B. Úlcera duodenal perforada encubierta con epiplón.

C. Diverticulitis perforada.

D. Isquemia mesentérica.

E. Diverticulitis de Meckel.

41. El procedimiento consistente en colocar un tubo en el estómago que salga a la piel, con la intención de utilizarlo para alimentar a un
paciente, se llama:

A. Gastrotomía.

B. Gastrostomía.

C. Gastrectomía.

D. Tubulización gástrica.

E. Gastroplastia.

42.En el hiperparatiroidismo se ve todo lo siguientes, EXCEPTO:

A. Ausencia de lámina dura.

B. Hiperreflexia.

C. Incremento en la fosfatasa alcalina.

D. disminución del fósforo sérico.

E. Osteítis fibrosa quística.

43.¿En cuál de las siguientes afecciones no está recomendada la dieta hiposódica?:

A. En la insuficiencia cardíaca.

B. En la nefritis intersticial.

C. En la insuficiencia renal crónica.

D. En el síndrome nefrótico.

E. En la hipertensión.

44.Un paciente es portador de un nódulo tiroideo. El estudio citológico mediante punción con aguja fina revela la existen- cia de “cuerpos
de Psamoma”. Este hallazgo es DEFINITORIO del diagnóstico de:

A. Linfoma tiroideo.

B. Carcinoma papilar tiroideo.

C. Tiroiditis de Riedel.

D. Carcinoma medular tiroideo.

E. Tiroiditis de Hashimoto.

45.¿Cuál de los siguientes factores NO es causa de macrocitosis?:


A. Hipotiroidismo.

B. Alcoholismo.

C. Anemia perniciosa.

D. Hepatopatía crónica.

E. Intoxicación por aluminio.

46. Señale cuál de estas afirmaciones es la VERDADERA, respecto al empleo de pruebas serológicas de la sífilis:

A. Para confirmar el diagnóstico de sífilis en un enfermo con prueba de anticuerpo reagínico positiva o con sospecha clínica de sífilis, se debe utilizar
el VDRL.

B. Para medir cuantitativamente el título de anticuerpos reagínicos a fin de valorar la actividad clínica de la sífilis, se

debe utilizar el RPR.

C. Para vigilar el título de reagina en respuesta al tratamiento de la sífilis, se debe utilizar el FTA-Abs o el MHA-TP.

D. Para analizar gran número de sueros con miras diagnósticas o de detección selectiva, se debe utilizar el MHA-TP.

E. Si un enfermo tiene una prueba reagínica positiva falsa, no podemos excluir la sífilis aunque obtengamos una prueba treponémica negativa.

47.Las cepas de Staphylococcus aureus resistentes a meticilina presentan las siguientes características EXCEPTO:

A. Son resistentes a todos los antibióticos betalactámicos.

B. La resistencia está mediada por las PBP (Proteínas Fijadoras de Penicilina).

C. Las pruebas de sensibilidad convencionales pueden dar resultados equívocos para las cefalosporinas.

D. Son sensibles a Vancomicina.

E. El mecanismo de resistencia esta codificado por un plásmido.

48.La causa MÁS frecuente de síndrome nefrótico idiopático en adultos es:

A. Nefropatía membranosa.

B. Glomerulosclerosis focal.

C. Nefropatía mesangial IgA.

D. Glomerulonefritis mesangiocapilar.

E. Síndrome nefrótico con cambios mínimos.

49.Los valores aumentados de la enzima Adenosindeaminasa (ADA) en un líquido pleural sugieren diagnóstico de:

A. Embolismo pulmonar.

B. Empiema metaneumónico.

C. TBC pleural.

D. Hemotórax.

E. Artritis reumatoide.

50.Señale cuál es el tratamiento de elección en las distonías focales cervicofaciales:

A. Anticolinérgicos.

B. Benzodiacepinas.

C. Toxina botulínica.

D. Barbitúricos.

E. Tenotomía.

51.En la osteomielitis, el absceso subperióstico emigra a la articulación vecina en los siguientes casos, EXCEPTO:

A. Cadera.

B. Hombro.

C. Rodilla.

D. Codo.
E. Tobillo.

52.¿Cuál es el método diagnóstico de elección para una lesión de menisco?:

A. Radiografía simple.

B. Artrografía.

C. Ecografía.

D. TAC.

E. RNM.

53.El principal factor pronóstico de un melanoma maligno sin metástasis en tránsito, ganglionares ni hematógenas es:

A. La edad del paciente.

B. Su desarrollo sobre un nevus previo.

C. Su localización en zonas acras.

D. El traumatismo previo de la lesión.

E. El espesor de la lesión medido en milímetros.

54.De acuerdo con las maniobras de Leopold, la relación que existe entre el dorso fetal con los flancos maternos se den o- mina:

A. Situación fetal.

B. Posición fetal.

C. Actitud fetal.

D. Presentación fetal.

E. Rotación interna.

55.Señale la respuesta FALSA en relación con la endometriosis:

A. Es una causa frecuente de esterilidad.

B. Su origen está ligado al flujo menstrual retrógrado.

C. El órgano más frecuentemente afectado es la trompa.

D. En su patogenia cada día se da más importancia al factor inmunológico.

E. La laparoscopia es la prueba más definitiva para el diagnóstico de la endometriosis pélvica.

56.Una mujer postmenopáusica fue tratada de un carcinoma mamario infiltrante de 1 cm mediante tumorectomía y lin - fadenectomía. Esta
última fue negativa. ¿Qué tratamiento añadiría:

A. Ovariectomía bilateral.

B. Monoquimioterapia con taxol.

C. Poliquimioterapia.

D. Radioterapia.

E. Radioterapia y tamoxifeno.

57.Una de las siguientes NO es complicación frecuente del abruptio:

A. CID.

B. Insuficiencia renal.

C. Shock.

D. Síndrome de Sheehan.

E. Pielonefritis.

58.Un maestro de escuela de 50 años, fumador, le consulta por disfonía y tos desde 3 semanas antes. No refiere síndrome constitucional. No
presenta adenopatías. En la laringoscopia indirecta se ve una lesión blanquecina e irregular en tercio anterior de cuerda voc al izquierda. ¿En
qué debe pensar preferentemente?:

A. Laringitis aguda.

B. Edema de Reinke.
C. Carcinoma de laringe.

D. Divertículo hipofaríngeo.

E. Nódulo glótico.

59.Todos los siguientes procesos se manifiestan frecuentemente como colestasis neonatal intrahepática, EXCEPTO :

A. Hepatitis B.

B. Sepsis.

C. Hipotiroidismo.

D. Galactosemia.

E. Tirosinemia.

60.La cistoureterografía de micción está indicada, principalmente, en el diagnóstico de:

A. Cáncer de vejiga urinaria.

B. Reflujo vésicoureteral.

C. Obstrucción ureteral.

D. Adenoma postático.

E. Ureterocele.

61.Indique la respuesta CORRECTA:

A. El 2º ruido cardíaco corresponde con el cierre de las válvulas auriculoventriculares.

B. Los soplos continuos pueden auscultarse tanto en estenosis como insuficiencias valvulares severas.

C. En el pulso venoso yugular la onda “a” se debe a la distensión venosa producida por la contracción de la aurícula izquierda.

D. El pulso alternante es típico de la miocardiopatía hipertrófica.

E. El signo de Kussmaul es un aumento de la presión venosa yugular durante la inspiración.

62.Según el informe internacional Lalonde, los factores determinantes de la salud son:

A. El control del medio ambiente.

B. Sistemas de asistencia sanitaria.

C. La biología humana.

D. Solo A y C

E. Todas las anteriores

63.¿Cuál es la conducta más apropiada en una gestante de 38 semanas con VIH confirmado recientemente, sin tratamien- to antirretroviral y
una carga viral de 2000 copias?

A. Parto vaginal.

B. Cesárea programada.

C. Cesárea de urgencia.

D. Parto instrumentado.

E. Parto inducido.

64.En el análisis situacional de salud de un establecimiento se encuentra que su comunidad de referencia carece de agua potab le, el año
anterior se presentaron 40 casos de cólera con 8 fallecidos, y el establecimiento no tiene protocolos de atenc ión. Según el análisis FODA
institucional, la situación mencionada corresponde a:

A. Oportunidades y amenazas.

B. Oportunidades y fortalezas.

C. Debilidades y fortalezas.

D. Amenazas y debilidades.

E. Debilidades y oportunidades.

65.Mujer de 50 años de edad, casada hace 5 meses presenta casi todos los días: disminución del apetito y la concentración, insomnio
terminal, lentificación psíquica y motor, irritabilidad, bajo estado de ánimo, ideas de muerte. El diagnóstico más probable e s:
A. Melancolía.

B. Trastorno efectivo bipolar.

C. Depresión mayor.

D. Depresión con síntomas psicóticos.

E. Esquizofrenia.

66.Paciente de 45 años colecistectomizado hace un mes. Refiere continuar con la sintomatología previa a la operación. Dolor c ólico
abdominal, vómitos e ictericia. ¿Cuál es el diagnóstico más probable?

A. Coledocolitiasis residual.

B. Hepatitis.

C. Estenosis biliar.

D. Ulcera péptica complicada.

E. Pancreatitis aguda.

67.Varón de 52 años, obeso, diabético, con diagnóstico reciente de HTA. En relación a su terapéutica es cierto:

A. Debe iniciar manejo con Beta bloqueadores.

B. Puede iniciar manejo con IECA por ser diabético.

C. Como la gran mayoría de hipertensos podría requerir tres drogas.

D. Las terapia combinada IECA + tiazida ha demostrado ser superior a IECA + Bloqueador de canal de calcio.

E. Se sugiere iniciar metildopa por su mayor potencia.

68.¿Cuál de las siguientes afirmaciones acerca de los criterios de Ranson NO es correcta?:0

A. Predice la severidad de la pancreatitis aguda.

B. Pacientes con 2 signos positivos tienen una mortalidad elevada.

C. Se basa en 11 criterios.

D. Los parámetros son evaluados al ingreso del paciente.

E. Los parámetros son evaluados a las 48 horas del ingreso del paciente.

69.Un niño capaz de correr, subir escaleras solo, sin alternar los pies, hacer una torre de seis cubos y elaborar frases de tres palabras,
presenta un desarrollo psicomotor propio de la siguiente edad:

A. 15 meses.

B. 18 meses.

C. 24 meses.

D. 36 meses.

E. 48 meses.

70.Mujer de 45 años presenta dolor abdominal episódico y refiere, que el dolor aumenta posteriormente a la ingesta de comidas ricas en
grasas ¿La acción de cuál de las siguientes hormonas es responsable de la intensificación postpran - dial de sus síntomas?

A. Gastrina.

B. Secretina.

C. Colecistocinina.

D. Pepsina.

E. Somatostatina.

71.Paciente de 50 años de edad que consulta por dolor en la fosa renal, poliaquiuria, disuria y hematuria. En el análisis de orina se observa
piuria y pH ácido con cultivos repetidamente negativos ¿Cuál sería la primera posibilidad diag- nóstica, de entre las siguientes?

A. Pielonefritis aguda.

B. Síndrome nefrítico.

C. Tuberculosis genitourinaria.

D. Prostatitis aguda.
E. Carcinoma renal de células claras.

72.Los estudios transversales son llamados:

A. Ecológicos.

B. Cohortes.

C. Ensayo de campo.

D. Estudios analíticos.

E. De prevalencia.

73.Son signos de corioamninitis, EXCEPTO:

A. Taquicardia materna.

B. Útero doloroso.

C. Líquido amniótico meconial.

D. Taquicardia fetal persistente.

E. Leucocitosis materna.

74.Un paciente de 60 años acude por historia de 2 días de melenas al hospital. Tiene un hematocrito en 25. Esta hemodi - namicamente
estable. Se le realiza una endoscopia la cual es normal. ¿Cuál es el siguiente paso?

A. Una nueva endoscopía alta.

B. Una colonoscopía con ileoscopía.

C. Gammagrafía con glóbulos rojos marcados.

D. Una arteriografía.

E. Cápsula endoscópica.

75.Los Documentos de Gestión Institucional requeridas a las instituciones de salud son, excepto:

A. Reglamento de Organización y Funciones.

B. Guías de prácticas clínicas.

C. Manual de Organización y Funciones.

D. Manuales de Procedimientos.

E. Texto Único de Procedimientos Administrativos.

76.¿Qué aspectos clínicos se evalúan principalmente para realizar el plan preoperatorio y el desarrollo de la aneste- sia?:

A. Edad del paciente, perfil hepático, sistema hematopoyético

B. Sistema digestivo, edad del paciente, sistema nervioso central

C. Aparato urinario, estado dermatológico y respiratorio

D. Edad del paciente, estado cardiovascular y estado respiratorio

E. Masa corporal, estudio inmunológico y cardiovascular

77.Respecto a los tratamientos habituales o hábitos de un paciente programado para una intervención quirúrgica bajo anestesia general, una
de las respuestas siguientes es CORRECTA:

A. Los bloqueantes beta adrenérgicos deben mantenerse durante todo el periodo preoperatorio.

B. Los antiparkinsonianos deben suspenderse 48 hrs antes.

C. La heparina de bajo peso molecular a dosis profilácticas se asocia a hemorragia intraoperatoria significativa.

D. La ticlopidina no se asocia a riesgo de hemorragia intraoperatoria significativa.

E. La evitación de fumar cigarrillos durante las 24 hrs previas carece de beneficio intraoperatorio.

78.La localización ideal para hacer una traqueotomía es:

A. La membrana cricotiroidea

B. La membrana miotiroidea.

C. Primer anillo traqueal.


D. Segundo o tercer anillo traqueal.

E. Cuarto o quinto anillo traqueal.

79.Una lesión purpúrica que no palidece a la vitropresión, nos indica:

A. La presencia de edema dérmico.

B. La presencia de exocitosis

C. La presencia de extravasación hemática.

D. La presencia de una malformación venosa.

E. La presencia de una malformación capilar

80.¿Cuál de las siguientes afirmaciones referentes a la leucoplasia vellosa oral se asocia a la infección por VIH es INCO- RRECTA?

A. Suele ser asintomática

B. Existen células balonizadas en la histopatología

C. Existe un marcador clínico de progresión de la infección por el VIH

D. Puede encontrarse en otras situaciones de inmunodeficiencia

E. El primer implicado en su patogenia parece ser Candida albicans

81.¿Cuál de las siguientes entidades está asociada con herpes virus tipo 6?

A. Exantema súbito

B. Enfermedad febril de la infancia

C. Síndrome de fatiga crónica

D. Síndrome de Gianotti- Crosti

E. Todas las anteriores.

82.Micosis superficial que afecta el estrato córneo y se manifiesta con lesiones discrómicas con descamación fina:

A. Tiña pedis.

B. Tiña corporis

C. Pitiriasis versicolor

D. Exofialosis

E. Tiña imbricada

83.¿Cuál es el andrógeno fundamental en la mujer?:

A. Androstenediona.

B. Dehidroepiandrosterona.

C. Androstenediol.

D. Androsterona.

E. Testosterona.

84.En un ciclo menstrual de 30 días, con una gráfica de temperatura corporal basal considerada ovulatoria; la duración de la fase luteínica o
secretoria será de:

A. 21 ± 2 días

B. 07 ± 3 días

C. 18 ± 1 días

D. 14 ± 2 días

E. 16 ± 2 días

85.Los siguientes factores pueden influir en la reserva de hierro al nacer, EXCEPTO:

A. Transfusión feto-materna

B. Transfusión de gemelo a gemelo


C. Rotura de vasos placentarios

D. Anemia de la madre

E. Todas las anteriores

86.En los neonatos normales a término, la capacidad gástrica es:

A. 5 a 9 mL

B. 10 a 15 mL

C. 25 a 30 mL

D. 20 a 45 mL

E. 50 a 90 mL

87.La arteria radial:

A. Es más grande que la arteria cubital

B. Pasa a lo largo del lado cubital de antebrazo

C. Pasa entre el tendón del musculo abductor largo del pulgar y el ligamento radial colateral de la muñeca

D. Comienza a lo largo de la extremidad de la diáfisis del humero

E. Pasa sobre el musculo supinador largo

88.El musculo flexor común profundo esta inervado por los nervio:

A. Mediano e interóseo anterior

B. Mediano y ulnar

C. Musculocutáneo y cubital

D. Cubital y radial

E. Radial y mediano

89.En la pubertad, el cuerpo uterino crece bajo la influencia directa de:

A. Hormonas hipofisiarias

B. Hormonas ováricas

C. Hormonas tiroideas

D. Factores hipotalámicos

E. Hormonas adrenales

90.Un investigador pretende determinar si existe una asociación entre las cifras de PA diastólica (medida en mmHg) y los niveles de
colesterol (medidos en mg/mL). Para ello, ha realizado estas mediciones a 230 voluntarios. ¿Qué prueba estadística es la MÁS apropiada
pare examinar esta asociación?:

A. Regresión logística.

B. Prueba de la t de Student.

C. Prueba de Chi cuadrado.

D. Correlación de Pearson.

E. Prueba de Fisher.

91.Un estudio analiza la relación entre la PA sistólica (PAS) y la edad en una muestra de mujeres adultas. Los autores presen tan los
resultados como la siguiente ecuación de regresión lineal: PAS = 81,5 + 1,2 x edad. ¿Cuál de las siguientes afirmaciones acerca d e este
análisis es FALSA?

A. La edad se ha utilizado como variable independiente

B. La pendiente de la recta es de 1,2 mmHg/año de edad

C. El valor 81,5 corresponde a la media de PAS en la muestra de mujeres

D. Por cada año más de edad de las mujeres de la muestra, su PAS se incrementa en 1,2 mmHg de promedio

E. Se ha asumido que la relación entre la edad y la PAS es lineal

92.El test de Capurro para la edad gestacional no incluye:


A. Forma de la oreja

B. Pliegues plantares

C. Textura de la piel

D. Nódulo mamario

E. Ventana cuadrada

93.Los hongos dermatofitos que causan cuadros clínicos con mayor componente inflamatorio son:

A. Antropofílicos

B. Geofílicos

C. Candidiasis

D. Pytrosporum

E. Zoofílicos

94.¿En cuál de los siguientes casos suele indicarse la prueba de estrógenos para evaluar el origen de una ameno - rrea?:

A. Cuando la prueba de la progesterona es positiva.

B. Cuando la prueba de la progesterona es negativa.

C. Cuando se detectan niveles elevados de gonadotropinas.

D. Cuando se detectan niveles disminuidos de gonadotropinas.

E. En todos los casos de amenorrea primaria.

95.¿Cuál de los siguientes tendones se inserta en el astrágalo?:

A. Aquiles

B. peroneo lateral corto

C. peroneo lateral largo

D. tibial anterior

E. El astrágalo carece de inserción tendinosa

96.Respecto al desarrollo psicomotor normal del niño, señale la afirmación correcta:

A. Presiona pinzando el pulgar con el índice a los 12 meses

B. Junta las manos en la línea media a los 6 meses

C. Vuelve las páginas de un libro a los 8 meses

D. Usa 4-6 palabras a los 9 meses

E. Se sienta estable y sin apoyo a los 6 meses

97.La edad recomendada para la introducción de la ablactancia es:

A. Entre los 15 días y los 2 meses

B. Entre los 2.3 meses

C. A los 4 meses

D. A los 6 meses

E. A los 8 meses

98.La leche materna contiene menos..... lo cual es para evitar la sobrecarga osmótica renal en el lactante.

A. Sodio

B. Hierro

C. Potasio

D. Cloro

E. Fósforo

99.De la inmunización activa del sarampión que se utiliza en la actualidad, ¿Cuál es la más eficaz?
A. Gammaglobulina

B. Virus muerto

C. Virus vivo con gammaglobulina

D. Virus vivo atenuado

E. Ninguna de las anteriores.

100.Niño de 6 años de edad con riesgo de tétanos. Si sufre una herida contaminada la conducta correcta es, asumiendo que tien e completas
las inmunizaciones y refuerzo de vacuna antitetánica:

A. Vacuna antitetánica

B. Antitoxina tetánica

C. Antibiótico tipo penicilina

D. Antitoxina + vacuna antitetánica

E. No se aplica ni vacuna ni inmunoglobulina

Parte 9A

1.La educación para la salud de la población constituye una estrategia de:

A. Promoción de la salud

B. Prevención de la enfermedad

C. Protección de la salud

D. Reducción de la enfermedad

E. Participación en salud

2.El establecimiento de sector salud de categoría I-2 corresponde a:

A. Puesto de salud con médico

B. Puesto de salud sin médico

C. Puesto de salud con 10 médicos

D. Puesto de salud con hospitalizacion

E. Puesto de salud con quirófano.

3.¿Cuál es la secuencia lógica en el proceso administrativo?

A. Planificación, organización, dirección y control.

B. Planificación dirección, organización, y control

C. Organización, dirección control Planificación

D. Planificación, control organización, dirección

E. Dirección Planificación, organización, y control.

4.Es la relación que existe entre los resultados con respecto a los recursos

A. Efectividad

B. Eficacia

C. Eficiencia

D. Costo – Beneficio

E. Aceptación.
5.De acuerdo a las técnicas de planificación del sector público, el enunciado: “ El Centro de Salud tiene que proteger la dignidad
personal, promoviendo la salud, previniendo las enfermedades y garantizand o la atención integral de salud de toda la población
residente en su ámbito de influencia”, corresponde a una definición de la:

A. Imagen-horizonte

B. Imagen-objetivo

C. Meta-estrategia

D. Misión

E. Visión

6.La mediana, la moda y la media aritm ética coinciden cuando la distribución de datos asume una curva:

A. Multimodal

B. Bimodal

C. Normal

D. Asimétrica con cola a la izquierda

E. Asimétrica con cola a la derecha.

7.Se cuenta con 31 valores de nivel de bilirrubinas en pacientes con ictericia y se desconoce la varianza poblacional para es ta
variable. En la construcción del intervalo de confianza al 95% para el valor promedio de bilirrubina en este grupo de paciente s, la
prueba estadística requerida es:

A. ANOVA

B. Binomial

C. Chi-cuadrado

D. Exacta de Fisher

E. T de Student.

8.En la prueba de hipótesis, el investigador comete un error tipo II o beta cuando:

A. No establece el nivel de significancia

B. No rechaza la hipótesis nula siendo falsa

C. Rechaza la hipótesis alterna, siendo falsa

D. Rechaza la hipótesis alterna, siendo verdadera

E. Rechaza la hipótesis nula siendo verdadera.

9.Cuando tenemos un conjunto de datos agrupados y ordenados y la variable en estudio es discontinua o discreta, la mejor forma de
representar gráficamente los datos es:

A. Gráfico de barras

B. Histograma

C. Ojiva

D. Polígono de frecuencia

E. Tendencias.

10. Hablamos de un estudio observacional teniendo en cuenta una de las presente s aseveraciones:

A. El investigador controla pero no manipula las variables.

B. El investigador interviene pero no controla ni manipula las variables.

C. El investigador ni interviene, ni controla, ni manipula las variables.

D. El investigador interviene y controla, pero no manipula las variables.


E. El investigador manipula pero no controla las variables.

11.En una región A de un país se contabilizaron 5.8 muertos por accidente de tráfico por 100.000 habitantes y año. ¿A qué indicador
se refiere dicha expresión?

A. A un índice global de mortalidad específica

B. A una tasa bruta de mortalidad

C. A una tasa de mortalidad específica

D. A un índice de mortalidad específica por causa

E. A la tasa bruta de accidentabilidad

12.Paciente de 30 años de edad con transtornos del ciclo menstrual, obesidad e hirsutismo. Examen de laboratorio: aumento de
resistencia a la insulina y de la relación LH:FSH. ¿Cuál es el diagnóstico m ás probable y su tratamiento indicado?:

A. Insuficiencia ovárica secundaria / TRH

B. Síndrome de ovarios poliquísticos / metformina, baja peso, ACO

C. Diabetes mellitus y SOP / metformina y etinilestradiol

D. Androgenismo hipofisiario / ciproterona acetato

E. Hipogonadismo hipogonadotrópico / citrato de clomifeno

13.El tipo de sangrado m ás característico que producen los leimiomas uterinos:

A. Hipermenorrea

B. Metrorragia

C. Menorragia

D. Menometrorragia

E. Amenorrea

14. Paciente de 40 años de edad, que acude a la consulta por sangrado postcoital de 2 s emanas de evolución al examen
pélvico:cérvix ulcerado hipertróficos, sangrante, poco m óvil. Cuerpo uterino 6cm, anexos no palpables. Al tacto rec- tal: útero duro con
parametrios tomados en sus 2/3 internos. El diagnóstico clínico es:

A. Cáncer de cérvix IB

B. Cáncer de cérvix IIIA

C. Cáncer in situ

D. Cáncer de cérvix IIB

E. Ectropión sangrante

15.Mujer de 64 años, G:1, P:1001, menarquia: 10 años. FUR: 55 años, obesa, diab ética e hipertensa desde hace 10 años. Acude por
ginecorragia. Ecografía: endometrio 20 mm y volumen uterino 110 cc. PAP: AGUS ¿cuál es el diagnóstico más probable?

A. Adenocarcinoma endometroide

B. Carcinoma de células claras

C. Carcinoma indiferenciado

D. Carcinoma mucinoso

E. Carcinoma seroso papilar

16.Una de las siguientes NO suele cursar con una hemorragia uterina:

A. Los pólipos endocervicales.

B. Miomas.

C. Hiperplasia endometrial.
D. Cáncer de ovario.

E. Portadoras de DIU.

17.¿Cuál viene a ser probablemente un hallazgo tardío en una paciente con cáncer de mama?

A. Secreción gris – verdosa.

B. Mamas caídas.

C. Oscurecimiento de la areola.

D. Asimetría del tamaño de la mama.

E. Retracción del pezón o de la piel.

18.Mujer de 24 años con hirsutismo, llega por amenorrea. En la analítica destaca un cociente LH / FSH de 3,45. Diag- nóstico más
correcto:

A. Endometriosis.

B. Déficit de 21 alfa hidroxilasa.

C. Déficit de 11 alfa hidroxilasa.

D. Síndrome del ovario poliquístico.

E. Disfunción cerebral.

19.En una evaluación ecográfica ginecológica, de un tumor ovárico de 3 x 4 cm de borde regular, hipo ecogénicos, homo- géneos,
avascular y de aspecto vidrio esmerilado ¿Cuál es la tumoración ovárica más probable?

A. Endometrioma

B. Quiste mucinoso

C. Cistoadenoma

D. Absceso

E. Mioma

20.Gestante de 38 semanas, acude a Emergencia por presentar sangrado vaginal profuso, asociado a rotura de membra - nas durante
el trabajo de parto ¿Cuál es el diagnóstico m ás probable?

A. Desprendimiento prematuro de placenta

B. Placenta previa

C. Ruptura uterina

D. Vasa previa

E. Inversión uterina

21.Mujer de 20 años, con cesárea por inducción fallida. Al tercer día de puerperio presenta malestar general, fiebre y loquios f étidos.
Al Examen clínico: T: 39°C, útero subinvolucionado y doloroso a la palpación, secreción fétida. ¿Cuál es el tratamiento recomendable?

A. Ampicilina mas ceftazidima

B. Ampicilina mas gentamicina

C. Clindamicina mas amoxicilina

D. Clindamicina mas cefalexina

E. Ampicilina mas eritromicina

22.Gestante nulípara en trabajo de parto se le realiza episio tomía mencione que musculo secciona:

A. Puborrectal

B. Transverso superficial de perine

C. Isquiococcigeo

D. Transverso profundo de perine


E. Puborrectal

23. ¿Cuál es el último movimiento cardinal durante el de parto?

A. Expulsión

B. Rotación externa

C. Rotación interna

D. Extensión

E. Flexion

24.Primigesta añosa de 34 semanas de gestación no controlada, acude por presentar cefalea, tinnitus, dolor epig ástrico. Examen
físico: PA: 150/90 mm Hg, FC: 90 x ,́ FR: 20 x ́, T: 36.5°C, edema en miembros inferiores. Altura uterina: 30 cm, Feto LCI, FCF:140 x ́.
Exámenes de Laboratorio: Hb:10 g/dL, plaquetas 110,000/mm3, Creatinina: 1 mg/dL, TGO: 70 UI/L, LDH:1200 UI/L, Bilirrubina:1.4
mg/dL a predominio indirecto. ¿Cuál es el estudio que ayudaría al diagnóstico?

A. Frotis de sangre periferica

B. Perfil hepático completo

C. Reticulocitos

D. Radiografia de torax

E. Perfil biofísico fetal

25.En un parto vaginal con presentación cefálica la cabeza está a nivel del borde inferior de la sínfisis del pubis. El plano

de Hodge es:

A. I.

B. II.

C. III.

D. IV.

E. V.

26.Una mujer madre de 3 hijos, presenta en su cuarto embarazo, próximo al parto, un sangrado vaginal. La exploración ecográfica nos
demuestra una placenta que ocluye de una manera total, el orificio cervical interno. La paciente ha dejado de sangrar en pocos
instantes y actualmente tanto el niño como ella se encuentran asintom áticos. Nuestro plan a seguir con esta embarazada será:

A. Cesárea electiva.

B. Rotura de membranas, para que el polo cef álico comprima el punto sangrante, al bajar la presentación.

C. El tratamiento anterior unida a una perfusión de oxitócicos.

D. Inducción del parto.

E. Fórceps electivo para acelerar el periodo expulsivo.

27. Luego de expulsivo normal y tras 60 min. de periodo de alumbramiento, no se aprecian signos de desprendimiento
placentario, a pesar de haberse aplicado masaje uterino e incrementado moderadamente la dosis de oxitocina. Se

indica extracción manual de placenta, que resulta imposible por no existir plano de separaci ón entre la placenta y la

pared uterina. Diagnóstico m ás probable:

A. Placenta incarcerada.

B. Engatillamiento placentario.

C. Placenta succenturiata con cotiledón aberrante.

D. Placenta adherente por acretismo placentario.

E. Placenta circunvalata.
28.Una mujer gestante de 31 semanas presenta pérdida de líquido amniótico. A los 3 días presenta fiebre de 39ºC en 2 oca- siones,
taquicardia fetal, taquicardia materna y alteraciones analíticas (leucocitosis de 16500 y aumento de la proteína C reactiva). Dinámica
importante. Pérdida de líquido claro no maloliente y modificación cervical. ¿Qué actitud toma?

A. Administración de betamiméticos.

B. Administración de betamiméticos y corticoides.

C. Betamiméticos, corticoides y antibióticos.

D. Evolución espontánea del parto, sin otros tratamientos.

E. Finalizar gestación inmediatamente, bajo cobertura antibiótica.

29.Señale que cambios ocurren en el embarazo normal

A. disminucion del tamaño renal

B. Dilatación uretereal bilateral

C. disminucion del índice de filtración glomerular

D. Aumento del índice de flujo plasm ático renal

E. Pronunciada proteinuria

30.¿Cuál es el gran componente inmunológico de la leche materna?

A. IgE

B. IgM

C. IgG

D. IgA

E. IgD

31.¿A qué edad aproximadamente un niño duplica su talla de nacimiento?

A. 1 año

B. 2 años

C. 3 años

D. 4 años

E. 5 años

32.¿Cuál es el diagnostico nutricional de un lactante de 11 meses que tiene P/T 94%, T/E 94%?

A. Desnutrición aguda

B. Desnutrición crónica

C. Desnutrición crónica reagudizada

D. Desnutrición global

E. Eutrófico

33.Acude a consulta un paciente varon de 4 años de edad con antecedente de tene cuadro respiratorio alto, ahora viene por ten er
fiebre de 39-40°, asociado a tos productiva, respiración rápida. Al examen se encuentra crepitantes y soplo tubarico. Según su
sospecha diagnostica, ¿Cuál de las siguientes no es una indicación para solicitar radiografia de torax?

A. Duda diagnostica

B. Sospecha de complicación

C. Descarte de tuberculosis

D. Paciente que será dado de alta

E. Busca de foco en el contexto de una sepsis

34.¿Con que prueba se realiza el diagnostico de asma bronquial en un niño mayor de 5 años?
A. Espirometria simple

B. Flujometro

C. Espirometria forzada

D. Pletismografia

E. Radiografia de torax

35.Niño de 6 años de edad, cursó con diarrea acuosa que recibió antibióticos. Luego presenta fiebre, irritabilidad, pete- quias,
equimosis y disminución de la diuresis. Hemograma: Anemia hemolítica. ¿Cuál es la conducta a seguir?

A. Hidratación rápida, usar antibióticos, colocar oxígeno, hospitalizar si empeora.

B. Manejo ambulatorio, usar diuréticos, indicar antibióticos, hidratación.

C. Hospitalización, hidratación amplia, transfusión, colocar antibióticos.

D. Hospitalización, manejo de insuficiencia renal, transfusión, restringir antibióticos.

E. Hospitalización, usar antibióticos de amplio espectro, transfusión.

36.Niño de 13 años presenta súbito dolor y aumento del volumen del testículo derecho. No tiene historia de traumatismos, vida sexual
activa ni secreción transuretral. Escroto aumentado de tamaño y dolor a la palpación. No reflejo cremas- térico. Gammagrafía
testicular: “un área fría” o ausencia de flujo en el lado afectado. El diagnostico mas probable es:

A. Hernia inguinal.

B. Hidrocele.

C. Epididimitis.

D. Torsión testicular.

E. Torsión del apéndice testicular.

37.Niño de 7 meses padece intensa diarrea y adelgazamiento. Apetito conservado. No presenta hepatomegalia. Posee un panículo
adiposo reducido, con una piel seca y una fascies arrugada. Los ojos son vivos y el cabello normal. ¿Cuál es su diagnóstico m ás
probable?

A. Histidinemia.

B. Leucinosis.

C. Enfermedad de Hartnup.

D. Marasmo.

E. Kwashiorkor.

38.La malnutrición crónica se identifica especialmente por…

A. Déficit de peso para la talla.

B. Déficit de peso para la edad.

C. Déficit de talla para la edad.

D. Disminución pliegue bicipital.

E. Peso por debajo del percentil 25.

39.Ante una fractura supracondilea de un niño que ha requerido intensas maniobras para conseguir la reducci ón por el
desplazamiento de los fragmentos y a la que se ha aplicado inmovilizaci ón enyesada, es fundamental vigilar en el postoperatorio
inmediato:

A. Una posible miositis osificante.

B. El desarrollo de un síndrome de Volkmann.

C. Una deformidad en varo del cubito.


D. La aparición de defectos rotatorios.

E. La posible aparición de una parálisis cubital.

40.RNT de parto domiciliario, con llanto inmediato y cuya madre durante la gestación presentó infección urinaria en el tercer trimestre.
La madre lo lleva a Emergencia porque a las 12 horas de nacido se encuentra hipoactivo y con pobre succi ón. Hemograma: leucocitos
4,500/ml, neutrófilos 78%, plaquetas 125,000 y PCR negativo. ¿Cuál es el diagnóstico más probable?

A. Sepsis neonatal.

B. Trastorno metabólico.

C. Error innato del metabolismo.

D. Depresión neonatal.

E. Lactancia inadecuada.

41.El agente m ás frecuente de sepsis de comienzo precoz en el recién nacido es:

A. Virus Herpes Simple

B. Estreptococo del grupo B

C. Candida albicans

D. Staphylococcus aureus

E. Haemophilus influenzae

42.Un varón de 5 horas de vida presenta una salivación abundante tipo espuma desde su primera hora de vida. Reflejo de succión
normal, pero no realiza deglución; crisis de tos, sofocación y cianosis al ponerlo al pecho. Abdomen abom - bado, genitales normales.
Ano permeable, extremidades normales. Diagnóstico más probable:

A. Atresia de coanas.

B. Estenosis congénita de píloro.

C. Atresia esofágica congénita.

D. Estomatitis aguda.

E. Infección neonatal.

43.¿Cuál es la complicación más frecuente de la fototerapia en el neonato?

A. Hipotemia.

B. Erupción macular.

C. Sobrehidratación.

D. Hipoglicemia.

E. Arritmias

44.Un RNPT a los 20 minutos de nacimiento presenta respiraci ón rápida y superficial con cianosis y aleteo nasal. Ester- tores finos
basales inferiores. En la Rx de tórax un fino patrón reticulogranular y broncograma aéreo. Diagnóstico más probable:

A. Enfermedad de membrana hialina.

B. Hemorragia suprarrenal.

C. Sepsis neonatal.

D. Neumonía neonatal.

E. Encefalitis herpética.

45.¿Cuál de estas causas de anemia hemolítica no da manifestaciones clínicas en el período de RN?

A. Esferocitosis hereditaria

B. Déficit de glucosa-6-fosfato-deshidrogenasa

C. Alfa-talasemias
D. Beta-talasemias

E. N.A.

46.Neonato con sospecha de infección congénita por toxoplasmosis; la prueba de mayor rentabilidad diagnostica es:

A. Ecografia cerebral y fondo de ojo

B. Test de Tzank

C. Cultivo de orina

D. Estudio de potenciales evocados auditivos

E. a y b son ciertas

47.Un recién nacido de parto eutócico, peso al nacer: 4 000gr. Al examen: Piel seca descamativa, llanto ronco, macro - glosia,
abdomen distendido con hernia umbilical, reflejos osteotendinosos disminuidos. ¿Cu ál es el diagnóstico más probable?

A. Xerosis congénita.

B. Síndrome de Down.

C. Hipotiroidismo congénito.

D. Trisomía XVIII.

E. Gastrosquisis.

48.Una mujer de 48 años presenta erupción cutánea crónica constituida por eritema, telangiectasias y episodios de infla - mación
aguda con pápulas, pústulas y quistes. No hay comedones. Diagnóstico más probable:

A. Acné conglobata.

B. Acné fulminans.

C. Acné por fármacos.

D. Rosácea.

E. Pioderma facial.

49.¿Cuál es la característica que corresponde al sarcoma de Kaposi?

A. Es una neoplasia maligna

B. Las lesiones son dolorosas y pruriginosas.

C. No invaden la cavidad oral.

D. Es infrecuente en el tracto digestivo.

E. La etiología es el VIH.

50.Un nadador de 30 años de edad presenta una lesión verrucosa en el labio inferior, único, con eritroplaquia en el pala- dar. ¿Cuál es
el diagnósticomás probable?

A. Queratosis actínica.

B. Carcinoma basocelular.

C. Carcinoma espinocelular.

D. Queratosis seborreica.

E. Melanomamaligno.

51.Un paciente muestra parálisis de la visión hacia arriba, dilatación pupilar con déficit de respuesta a la luz y conserva- ción de la
acomodación. Señale el diagnóstico más probable:

A. Pinealoma

B. Meningioma en la convexidad superior.

C. Meduloblastoma.

D. Astrocitoma quístico cerebeloso.


E. Oligodendroglioma del lóbulo temporal

52.Paciente quien presenta cefalea constante, náuseas, vómitos, somnolencia, diplopía, parálisis del sexto par craneal, midriasis,
edema de papila, distrés respiratorio. ¿Cuál es el diagnóstico más probable?

A. Hemorragia subaracnoidea

B. Hipertensión endocraneal

C. Trombosis venosa cerebral

D. Meningoencefalitis aguda

E. Hemorragia intracerebral

53.Varón de 25 años de edad, diagnosticado hace cinco años de tuberculosis pulmonar con abandono del tratamiento. Desde hace 2
semanas presenta cefalea y vómitos explosivos. Examen físico: rigidez de nuca, Kernig y Brudsinsky positivos. Punción lumbar con
presión elevada e hipoglucorraquia. Test de ELISA para VIH negativo. ¿Cu ál es el diagnóstico m ás probable?:

A. Meningitis tuberculosa

B. Encefalitis rábica

C. Absceso cerebral

D. Meningitis criptococócica

E. Meningitis viral.

54.Agente etiológico del Chancroide y de la cervicitis respectivamente:

A. Haemophilus vaginalis / clamidia trachomatis

B. Calymmabacterium Granulomatosis / gonococo

C. Treponema pallidum / H. ducreyi

D. Chlamydia trachomatis / H. ducreyi

E. Hemophilus ducrey / C. trachomatis

55.Ante los resultados serológicos siguientes: VDRL positivo, FTA-Abs negativo, su diagnóstico será:

A. Sífilis no tratada.

B. Sífilis recientemente tratada.

C. Falso positivo.

D. Sífilis antigua tratada.

E. Ninguna de las anteriores.

56.Paciente de 56 años, trasplantado renal desde hace tres meses, comienza con fiebre, malestar general, anorexia, sudo - res
nocturnos, artralgias y tos. En la analítica observamos 2800 leucocitos y ligera alteraci ón de las pruebas de función hepática. En la Rx
de tórax observamos infiltrados retículos nodulares bilaterales en la periferia de l óbulos inferiores. El diagnóstico m ás probable sería:

A. Infección por Pneumocystis jiroveci.

B. Brucelosis.

C. Infección por Citomegalovirus.

D. Infección por virus de Ebstein-Barr.

E. Infección por Aspergillus.

57.De las siguientes afirmaciones referidas al VIH. Señale la respuesta INCORRECTA:

A. La proteína CD4 de las células T es el receptor para el virus.

B. Los anticuerpos frente al virus se producen sólo después del período de ventana.
C. El Western blotes m ás específico que el ELISA para el diagnóstico de infección por el VIH.

D. Un paciente totalmente asintomático puede estar desarrollando la fase SIDA.

E. La InmunofluorescenciaIndirecta (IFI) es una prueba confirmatoria.

58.Una mujer de 30 años de edad se presenta por haberse palpado un n ódulo en el tiroides. ¿Cuál de los siguientes es

menos sugestivo de carcinoma tiroideo?

A. Nódulo solitario.

B. Consistencia dura.

C. Nódulo no doloroso.

D. Fijación a estructuras vecinas.

E. Lesión puramente quística a la ecografía.

59.La alteración del sistema nervioso periférico más frecuente en la diabetes mellitus es:

A. Mononeuropatía

B. Polineuropatía distal simétrica mixta de predominio sensitivo.

C. Mononeuropatía múltiple de predominio sensitivo.

D. Neuropatía vegetativa.

E. Amiotrofia diabética.

60.El síndrome de Cushing debido a producción de ACTH fuera de la pituitaria se puede ser debido a:

A. Tumores adrenales indolentes

B. Incidentalomas

C. Cáncer de pulmón células pequeñas

D. Cáncer de pulm ón de células grandes

E. Iatrogenia.

61.¿Cómo se realiza el diagnostico de enfermedad celiaca?

A. Vista por endoscopia

B. Tránsito intestinal

C. Ecografía abdominal

D. Biopsia intestinal

E. Coprofuncional

62.¿Cuál es el tratamiento de elección para el manejo de diarrea causada por campilobacter jejuni?

A. Amoxicilina

B. Cotrimoxazol

C. Ciprofloxacino

D. Azitromicina

E. Clindamicina

63.Mujer de 50 años de edad, que refiere beber “sólo lo normal”, con esteatosis, necrosis hepatocelular e infiltrado infla- matorio de
polimorfonucleares de predominio centrolobulillar en la biopsia hep ática. Además, en el estudio anatomo- patológico, se observan
megamitocondrias. El diagnóstico de esta paciente es:

A. Cirrosis alcohólica.
B. Esteatosis hepática.

C. Hepatitis alcohólica.

D. Enfermedad de Wilson.

E. Esteatohepatitis no alcohólica.

64.¿Cuál es la complicación más frecuente de la luxación traumática de la cadera?

A. La parálisis del ciático.

B. La tromboflebitis.

C. La rotura de la arteria femoral.

D. Shock hemorrágico postraumático.

E. Luxación recidivante.

65.¿Cuál será el tratamiento de una fractura de tercio medio de cubito y radio, desplazada y acortada, en un adulto?

A. Reducción incruenta y manguito de yeso.

B. Reducción incruenta y braquial de yeso.

C. Reducción cruenta y yeso.

D. Tracción continua y yeso.

E. Reducción cruenta y osteosíntesis.

66.Fractura conminuta no desplazada del primer MTC:

A. BENETT

B. ROLANDO

C. JONES

D. MURPHY

E. MALLAMPATI.

67.Las fracturas que se producen por microtraumatismos, ya sea por intensidad o por repetici ón que alteran las propie- dades
fisicoquímicas del hueso, se denominan y la m ás frecuente es la fractura:

A. Por fatiga / del recluta

B. Patológicas / colles

C. Por fatiga / cargador

D. Osteoporoticas / colles

E. Patologicas / metatarsiano 2

68.¿Cuál es la fractura m ás grave de la columna vertebral?

A. Flexión.

B. Torsión.

C. Distracción.

D. Flexo torsión.

E. Flexo extensión.

69.La presencia de midriasis regular y reacción lenta a la acomodación y a la luz indican uno de los siguientes procesos:

A. Pupila de Argyll – Robertson.

B. Pupila de Holmes – Adie.

C. Pupila de Marcus – Gunn.


D. Pupila de Wernicke.

E. Pupila de Anderson.

70.Mujer de 28 años presenta cuadro de acúfenos e hipoacusia bilateral, refiere que comenzó primero en el lado derecho. No hay
vértigo. La hipoacusia presenta Rinne negativo, Weber sin lateralizaci ón y Schwabach alargado. ¿Qué pato- logía sospecha?

A. Tumor glómico.

B. Neurinoma del acústico.

C. Otoesclerosis.

D. Enfermedad de Meniére.

E. Neuronitis vestibular.

71.Un varón presenta tras un sondaje urinario, dolor lumbar y tiritona de inicio brusco, al tacto rectal se halla una pr ós- tata agrandada,
caliente y dolorosa. Diagnostico mas probable:

A. Prostatitis aguda.

B. Prostatitis crónica.

C. Prostatitis no bacteriana.

D. Prostatodinia.

E. Uretritis.

72.¿Qué puede producir la evacuación brusca de la vejiga mediante cateterismo en un paciente con retención urinaria

aguda?

A. Hematuria macroscópica severa.

B. Falsa vía uretral.

C. Uretrorragia.

D. Dolor suprapúbico.

E. Infección urinaria.

73.¿Cuál de los siguientes hallazgos de pulso venoso yugular concuerda con una insuficiencia tric úspide?

A. Onda “a” aplanada.

B. Seno “x” profundo.

C. Seno “y” borrado.

D. Onda “v” gigante.

E. Onda “a” gigante.

74.¿Dónde buscaría Ud. los panadizos de Osler en la exploración de un paciente afecto de endocarditis infecciosa?

A. Sobre las eminencias óseas.

B. En los pulpejos de los dedos.

C. En las uñas

D. En la mucosa oral.

E. En el cuero cabelludo.

75.¿Qué se encuentra en la espirometría de un niño de 10 años con diagn óstico de asma aguda?

A. VEF1 aumentado.

B. VEF1 normal.

C. VEF1 disminuido.

D. Volumen residual disminuido.


E. Capacidad pulmonar total aumentada.

76.Mujer de 75 años diabética e hipertensa en tratamiento irregular desde hace 5 años. Acude a Emergencia por disnea en forma
súbita, sudoración profusa y palpitaciones. Examen físico: pálida sudorosa y polipneica. PA: 90/60 mm Hg. FC: 98 x’. Cardiovascular:
ruidos cardiacos arrítmicos, extrasístoles ventriculares aislados. Pulmones: sin altera - ciones. Examen de laboratorio: hemograma
normal, glucosa: 150 mg/dL. ¿Qué exámenes solicita inicialmente para confirmar el diagnóstico?

A. Electrocardiograma y troponina T.

B. Radiografía de tórax y dimero D.

C. Interleucina-6 y Procalcitonina.

D. TAC torácico y CPK totales.

E. Ecocardiografía y AGA.

77.Paciente con cuadro asm ático que tiene síntomas nocturnos 2 o menos en el mes, se refiere a asma:

A. Asma persistente leve

B. Asma persistente severa

C. Asma persistente moderada

D. Asma intermitente

E. Crisis aguda de Asma

78.¿Cuál es la manifestación clínica más frecuente de la estenosis m itral?

A. Disnea

B. Hemoptisis

C. Embolismo sistémico.

D. Arritmias

E. Sincope.

79.La triada de un síndrome de condensación pulmonar se caracteriza por:

A. Subcrepitantes – soplo tubárico – vibraciones vocales aumentadas.

B. Crepitantes - soplo cavitario - vibraciones vocales aumentadas.

C. Subcrepitantes – soplo en “O” – vibraciones vocales disminuidas.

D. Crepitantes – soplo en “O” – vibraciones vocales disminuidas.

E. Crepitantes - soplo tubárico - vibraciones vocales aumentadas.

80.Fumador presenta desde hace años, tos y expectoración matutina y disnea a medianos esfuerzos. Discreta hipoventi - lación
generalizada con roncus que se modifican con la tos. Rx de tórax normal. Disminución de FEV1 y del índice de FEV/FVC. ¿Cuál es su
diagnóstico?

A. Apnea del sueño.

B. Bronquitis crónica.

C. Asma.

D. Carcinoma broncogénico.

E. Fibrosis pulmonar.

81.La vía más frecuente de ingreso de gérmenes para la producción de absceso hepático piógeno es:

A. Árbol biliar.

B. Sistema portal.
C. Hematógena.

D. Por contigüidad.

E. Criptogénica.

82.En un paciente con obstrucción intestinal. ¿Cuál es la medida inicial m ás adecuada?

A. Hidratación parenteral.

B. Sonda nasogástrica.

C. Antibióticos.

D. Observación.

E. Cirugía.

83.Mujer de 82 años, con vómitos verdosos, distensión abdominal, ausencia de flatos y deposiciones, sensación de alza térmica.
Examen: abdomen con ruidos intestinales de timbre metálico, distendido, doloroso a la palpación superficial y profunda. En la región
inguinal derecha se palpa tumoración inguinal no reductible y muy dolorosa a la palpación.¿Cuál es su diagnóstico?

A. Obstrucción intestinal por hernia complicada.

B. Hernia inguinal encarcelada.

C. Hernia inguinal estrangulada.

D. Impactación fecal.

E. Adenomegalia inguinal.

84.¿Cuál de las siguientes técnicas operatorias para hernia inguinal se asocia a baja recidiva?

A. Lichtenstein.

B. Bassini.

C. Mac Vay.

D. Nyhus.

E. Marcy.

85.Varón de 65 años, historia relevante por ictericia obstructiva y perdida de peso. El seguimiento revela una masa de 2.5 cm en la
cabeza del páncreas; PAAF: adenocarcinoma. Seguimiento metastásico negativo y al momento de la cirugia el tumor está localizado
en la cabeza del páncreas. El tratamiento óptimo en este punto es:

A. Procedimiento de Whipple.

B. Pancreatectomía total.

C. Bypass biliar.

D. Bypass biliar y gástrico.

E. División de los nervios esplácnicos.

86.¿Cuál de las siguientes es correcta en relación al divertículo faringoesofágico o divertículo de Zenker?

A. Se trata de un divertículo por tracción.

B. Suele diagnosticarse en pacientes jóvenes.

C. El tratamiento incluye la miotom ía del músculo cricofaríngeo.

D. La pirosis es su síntoma principal.

E. Se localiza siempre en la cara anterior de la hipofaringe.


87.Varón en su 5º día postoperatorio por perforación traumática de colon y peritonitis generalizada. La herida operato- ria presenta
vesículas hemorrágicas en la piel, con aponeurosis edematizada de color gris y tejido celular subcutáneo esfacelado. Diagnostico
probable:

A. Flemón.

B. Fasceítis necrotizante.

C. Absceso por grampositivo.

D. Gangrena gaseosa.

E. Celulitis postestreptocócica.

88.¿Qué tipo de cilindros están constituidos por las proteínas Tamm –Horsfall y se ven en gran cantidad luego del ejer- cicio?

A. Hialinos.

B. Granulosos.

C. Leucocitarios.

D. Grasos.

E. De células tubulares.

89.Paciente con diabetes insípida. Aunque la vasopresina aporta significativamente al equilibrio electrolítico y de fluido, a l final no
parece regular el volumen sanguíneo. El volumen sanguíneo se mantiene en casi niveles normales porque:

A. El sistema renina-angiotensina periférico es estimulado

B. La ingestión de agua esta apropiadamente regulada

C. Aumenta la presión oncótica plasmática

D. Los reflejos simpáticos disminuyen la filtración glomerular

E. Disminuye el flujo sanguíneo renal

90.Mujer de 52 años con diagnóstico de úlcera pilórica, presenta vómitos a repetición de una semana de evolución. In- gresa a
emergencia deshidratada, PA 100/58 mmHg. Laboratorio: Na: 140 mmol/L, K: 2.8 mmol/L, Cl: 86 mmol/L, Bicarbonato: 42 mmol/L, pH:
7.53, pCO2: 53 mmHg y creatinina: 2,9 mg/dL. En orina: Na: 2 mmol/L, Cl: 21 mmol/L.

¿Cuál es el diagnóstico m ás probable?

A. Alcalosis metabólica.

B. Acidosis hipoclorémica.

C. Alcalosis mixta.

D. Acidosis metabólica con Anión-Gap normal.

E. Bicarbonaturia paradójica.

91.¿Cuál de las siguientes anemias puede ser tratada con cianocobalamina (vitamina B12)?

A. Anemia en los infantes que experimentan un rápido crecimiento.

B. Anemia asociada con queilosis, disfagia, gastritis e hipoclorhidria.

C. Anemia asociada con células pequeñas y bizarras pobremente llenas de hemoglobina.

D. Anemia asociada con infestación por diphyllobothrium latum.

E. Hemorragia por una úlcera gástrica.

92.Una mujer de 61 años llega recientemente por un infarto miocardico. Su historia m édica muestra hiperlipidemia, hipertension y
asma. ¿Qué medicación antiplaquetaria debería haber sido iniciada para su enfermedad coronaria aguda?

A. Aspirina.

B. Ticlopidina.

C. Clopidogrel.

D. Abciximab.
E. Eptifibatida.

93.¿Cuál de estas terapias apoya la producción de granulocitos en los pacientes neutropénicos?

A. Rituximab.

B. Gentuzumab.

C. Interferón alfa.

D. Trastuzumab.

E. G-CSF. (Factor estimulante de colonias de granulocitos)

94.Un tratamiento es desaconsejable en la artritis crónica juvenil:

A. Salicilatos.

B. Antiinflamatorios no esteroideos.

C. Sales de oro.

D. Imunosupresores.

E. Reposo prolongado en cama.

95.En el tratamiento de la AR, la ciclosporina:

A. Se considera un DMARD (fármaco antirreum ático modificador de la enfermedad).

B. Aumenta los niveles de la interleucina 2 (IL 2) y el factor de necrosis tumoral alfa (TNF - ).

C. Es un inhibidor de la ciclooxigenasa con toxicidad considerable.

D. Su uso se basa en la mayor acción antiinflamatoria que los AINEs.

E. Generalmente actúa m ás o menos rápidamente (1 semana).

96.Señale la respueta verdadera:

A. Los nódulos de Heberden son consecuencia de la afección de las articulaciones interfalángicas proximales.

B. La coxartrosis es m ás frecuente en mujeres.

C. El dolor de la articulación coxofemoral puede referirse a rodilla o nalga.

D. La gonartrosis suele ser unilateral.

E. La espondiloartrosis afecta prefrentemente a la columna dorsal.

97.¿En cuál de las siguientes arterias se localiza con mayor frecuencia el aneurisma cerebral?

A. Comunicante anterior.

B. Comunicante posterior.

C. Basilar.

D. Cerebral media.

E. Cerebral posterior.

98.¿Cuál de los siguientes nervios se encarga del control motor del m úsculo masetero?

A. Facial.

B. Cérvico-facial.

C. Maxilar superior.

D. Maxilar inferior.

E. Oftálmico.

99.La triada portal está conformada por la vena porta:

A. La vena hepática y el ligamento hepatoduodenal.


B. El epiplon mayor y el ligamento hepatoduodenal.

C. La arteria hepática y el conducto biliar.

D. La arteria hepática y el conducto colédoco.

E. El epiplon menor y el ligamento hepatoduodenal.

100.¿De cuál de los siguientes arcos deriva la parte proximal de la arteria carotida interna?

A. Arco aórtico 1.

B. Arco aórtico 2.

C. Arco aórtico 3.

D. Arco aórtico 4.

E. Arco aórtico 6.

Parte 10 A

1. EI pulmón derecho está formado por:

A. Dos lóbulos y nueve segmentos broncopulmonares.

B. Dos lóbulos y diez segmentos broncopulmonares.

C. Tres lóbulos y diez segmentos broncopulmonares.

D. Tres lóbulos y ocho segmentos broncopulmonares.

E. Tres lóbulos y seis segmentos broncopulmonares.

2. La aorta se bifurca a nivel de la vértebra

A. L2

B. L4

C. L6

D. S2

E. Ninguna de las anteriores

3. Los músculos en ejercicio con intensa actividad glicolítica anaeróbica producen grandes cantidades de:

A. Lactato.

B. Glucosa.

C. Piruvato.

D. Glicerol.

E. Fosfonal piruvato.

4. La fecundación humana generalmente ocurre en:

A. Istmo tubárico.

B. Fondo uterino.

C. Porción intersticial tubaria.

D. Ámpula tubaria.

E. Bursa de ovulación.

5.¿En cuál de los siguientes órganos se realiza el mayor porcentaje de glucosa metabolizada mediante glicolisis?
A. Hígado

B. Musculo esquelético

C. Tejido adiposo

D. Tiroides

E. Gónadas

6.La metaplasia escamosa informada en un PAP de cérvix implica:

A. Hallazgo normal

B. Inflamación

C. Infección bacteriana

D. Infección por papiloma virus

E. Lesión pre maligna

7.En un adulto de 50 años, de 70 Kg de peso, el agua intracelular aproximadamente es:

A. 21 litros.

B. 14 litros.

C. 35 litros.

D. 28 litros.

E. 40 litros.

8.La principal acción de la hormona paratiroidea es aumentar la reabsorción de calcio en los riñones a nivel de:

A. Porción gruesa ascendente del asa de Henle y en el túbulos proximal y distal

B. Túbulo proximal

C. Túbulo distal, conducto colector

D. Conductos colectores

E. Túbulo distal

9.¿Cuál de los siguientes fármacos NO es un antipsicótico?:

A. Amisulprida.

B. Risperidona.

C. Buspirona.

D. Clorpromacina.

E. Olanzapina

10.Los canales iónicos

A. Son dianas moleculares para los fármacos

B. Accionan el paso de iones a través de las membranas nucleares

C. Accionan el paso de iones a través de las bicapas lipídicas

D. Son enzimas

E. No son proteínas.

11.Paciente de 48 años G: 2 Para: 1-0-1-1 que reporta coitorragia. El examen con espéculo revela una lesión cervical que
compromete el tercio superior de vagina; el examen bimanual no revela engrosamiento parametrial. Asumiendo que el resto del
examen es normal, señale el estadío que correspondería este Ca. de cérvix:

A. IA2
B. IIA

C. IIB

D. IIIA

E. IIIB

12.El uso de ACO combinados protege contra el cáncer de:

A. Mama

B. Ovario

C. Endocervix

D. Hígado

E. Trompa

13.Una mujer de 38 años consulta por una secreción vaginal anormal con mal olor, como a “pescado podrido”, que se hace más
intenso después del coito. No tiene prurito genital. En la exploración se observa una abundante secreción blanco -grisácea, que no se
adhiere a las pareces vaginales. Al mezclar una muestra de la secreción con una gota de hidróxido potásico al 10% se aprecia
claramente el mal olor referido. ¿Cuál de los siguientes es el tratamiento de elec - ción?:

A. Amoxicilina con Ácido Clavulamico por vía oral

B. Clotrimazol por vía intravaginal

C. Doxiciclina por vía oral

D. Clindamicina por vía intravaginal

E. Fluconazol por vía oral

14.Mujer de 24 años, con antecedente de haberse insertado un DIU hace 2 años, luego de su primer parto. Acude a Emergencia
quejándose de dolor intenso en la parte inferior del abdomen. El examen pélvico revela un aumento del volumen anexial bilateral. El
DIU es retirado y enviado al servicio de Patología. La paciente comienza a recibir trata - miento con penicilina. En su criterio, ¿cuál
sería el microorganismo que con mayor probabilidad este causando este cuadro de EIP?:

A. Staphylococcus aureus

B. Bacteroides fragilis

C. Actinomyces israelii

D. Neisseria gonorrhoeae

E. Chlamydia trachomatis

15.¿Cuáles son las causas de los bochornos y sofocos en la menopausia?

A. Disminución de niveles de estrógenos

B. Disminución de niveles de progestágenos

C. Disminución de niveles de FSH

D. Disminución de niveles de LH

E. Aumento de niveles de PRL

16.Mujer de 45 años: G:5 P:5005 de partos vaginales. Con incontinencia urinaria de esfuerzo y sensación de bulto en genitales. Según
la clasificación de Prolapso de Órganos Pélvicos (POP-Q), la longitud total de vagina (LTV) es 7 cm, con el punto Ba es (+ 4) ¿Cu ál es
el estadío del cistocele?:

A. I

B. II

C. III

D. IV
E. V

17.Cuando el ciclo menstrual dura menos de 3 semanas se denomina:

A. Polimenorrea

B. Oligomenorrea

C. Amenorrea

D. Criptomenorrea

E. Hipermenorrea

18.Paciente de 38 años, G5 P3013, en trabajo de parto, presenta contracciones intensas y dolorosas, estado de ansiedad y
abombamiento del segmento uterino, con dolor supra púbico, latidos cardiacos fetales en 136 por minuto. Estos sín - tomas
corresponden a:

A. Gestante con feto en transversa en trabajo de parto

B. Desprendimiento prematuro de placenta

C. Corioamnionitis

D. Síntomas premonitorios de rotura uterina

E. Gestante con feto en podálica en trabajo de parto

19.Paciente de 30 años, acude a emergencia refiriendo: sangrado vaginal leve, dolor abdom inal tipo cólico de baja inten- sidad y el
antecedente de haber eliminado “carnosidad”. Al examen, se encuentra cuello uterino cerrado, útero blando y más pequeño que e l
correspondiente al UPM. El diagnóstico más probable será:

A. Aborto inevitable.

B. Amenaza de aborto.

C. Aborto frustro.

D. Aborto completo.

E. Aborto incompleto.

20.¿En qué caso haríamos un RCTG estresante?

A. Frecuencia cardíaca fetal de 115 lpm.

B. No existen aceleraciones en la frecuencia cardíaca fetal.

C. Si hay escasa variabilidad y deceleraciones.

D. En una deceleración, coincidiendo con un decúbito supino.

E. Frecuencia cardíaca basal de 170 lpm.

21.¿Cuál es el tratamiento del aborto tubárico sin hemorragia intraabdominal ni rotura tubárica?

A. Observación y control seriado de beta-HCG.

B. Tratamiento con metotrexate.

C. Laparotomía exploradora urgente.

D. Laparoscopia y salpinguectomía bilateral.

E. Laparoscopia y salpingostomía unilateral.

22.El tratamiento antimicrobiano en caso de Ruptura prematura de membranas ha demostrado, indique (V) verdadero y (F) falso:

( ) Menos mujeres desarrollan corioamnionitis. ( ) Menos niños desarrollan sepsis.


( ) El embarazo se prolonga por más de 7 días en muchos casos.

( ) La supervivencia neonatal definitivamente mejora.

( ) Incidencia de enterocolitis necrotizante y de dificultad respiratoria neonatal disminuye.

A. VFVFV

B. FVFVF

C. VVFFV

D. VFFVV

E. VVVFF

23.Primigesta de 28 años, en trabajo de parto, AU: 35cm, D: 6 cm, presentación cefálica, membranas rotas, altura de presentación -2
se palpa cordón umbilical, diámetro conjugado diagonal es de 11 cm. promedio ponderal fetal es de 3970 g, el diagnóstico más
probable es:

A. DCP por macrosomía fetal

B. Estrechez pélvica

C. DCP y prolapso de cordón

D. Estrechez pélvica y prolapso de cordón

E. DCP y procúbito de cordón

24.Paciente multípara con 39 semanas de gestación, con contracciones de Braxton Hicks, cuello central y dehiscente 2 -3 cm.
Presentación en C-3. El diagnóstico m ás apropiado es:

A. No trabajo de parto.

B. Fase latente de trabajo de parto.

C. Fase activa del trabajo de parto.

D. Falso trabajo de parto.

E. Ninguno de los mencionados.

25.Nulípara de 18 años, que consulta por amenorrea de 7 semanas, con historia de p resentar síntomas de irritación ve- sical. El
examen revela una paciente algo pálida, con hiperpigmentación facial, mamas congestionadas y dolorosas. Al exa -men ginecológico
impresiona ablandamiento del cuello uterino. ¿Cómo se llama el signo descrito, den tro del diagnós-tico probable de embarazo?:

A. Signo de Godell

B. Signo de Chadwick

C. Signo de Noble-Budin

D. Signo de Hegar

E. Signo de Ladin

26.¿Cuál de los siguientes es signo de certeza de la gestación?:

A. Auscultación de los latidos fetales

B. Prueba biológica de embarazo positiva

C. Sensación subjetiva de movimientos fetales

D. Coloración violácea de la vagina

E. Hiperpigmentación de la areola mamaria

27.La mastitis puerperal es causada con más frecuencia por:

A. Estreptococo b-hemolítico

B. Peptoestreptococo
C. Peptococo

D. Estreptococo grupo B

E. Staphylococcus aureus

28.En cuál de las siguientes parasitosis se recomienda tratamiento familiar:

A. Faciolasis

B. Ascaridiasis

C. Oxiuriasis

D. Strongiloidiosis

E. Ancylostomiasis

29.De las siguientes localizaciones de tuberculosis extra pulmonar, ¿cuál tiene mayor probabilidad de contagiar la enfer - medad?:

A. Laringe

B. Piel

C. Riñón

D. Pulmón

E. Hueso

30.Paciente varón de 20 años, consulta por cuadro de fiebre no cuantificada cada 3 d ías, cefalea frontal, escalofríos de predominio
vespertino. Al examen físico T: 40 °C, FC 90xmin, FR 20xmin, PA 110/70 mmHg. Peso 60 kilos, abdomen blando, dolor a la palpación
con hepatoesplenomegalia. Los exámenes de laboratorio muestran hematocrito 27.9% Hb: 9 .3 g/dl, Frotis de sangre periférica y Gota
Gruesa: POSITIVA con presencia de Gametocitos de Malaria (++), de acuerdo a la historia del paciente se puede presumir que se
trata de la especie:

A. Flavivirus, transmitida por el Aedes Aegypti

B. Leishmania Donovani, transmitida por Lutzomia

C. Tripanosoma Cruzi, transmitida por el Triatoma

D. Plasmodium Vivax, transmitida por el Anopheles

E. Plasmodium Malariae, transmitido por Lutzomia

31.Con respecto a vectores de la Leishmania, marque la respuesta CORRECTA:

A. Parte de los estadios inmaduros de las Lutzomia se desarrollan en aguas estancadas.

B. La distribución altitudinal de las Lutzomia en los valles occidentales es desde los 0 hasta los 2,800 metros sobre el nivel d el
mar.

C. Las Lutzomia hembras inoculan los estadios de Tripomastigotes metacíclicos a los vertebrados.

D. Las cepas de las subespecies de Leishmania mexicana se desarrollan en la sección supraphylaria del intestino de las Lut -
zomia.

E. Las Lutzomia son muy activas cuando hay viento y sus vuelos son largos y silenciosos.

32.Una mujer de 22 años con antecedentes de sibilancias intermitentes durante el ejercicio acude al servicio de urgencias con disnea.
Esta vez la crisis se produjo en la clase de aeróbico. En este momento, la enferma muestra una clara difi- cultad respiratoria y
sibilancias difusas en la auscultación pulmonar. La saturación de O2, por pulsoximetría, es de un 95%. El tratamiento m ás eficaz
consiste en:

A. Aminofilina por vía intravenosa

B. Cromoglicato sódico inhalado

C. Salbutamol inhalado

D. Hidrocortisona por vía intravenosa


E. Beclometasona inhalada

33.Varón de 50 años de edad, con derrame pleural. En la toracocentesis se encuentra: l íquido amarillo citrino, proteínas 4g/dl, glucosa
50mg/dl, polimorfos nucleares 10%, linfocitos 90% y ausencia de células neoplasicas. ¿Qué procedi- miento solicitaría?

A. Biopsia pulmonar transtorácica.

B. Biopsia pleural percutánea.

C. Biopsia transtorácica con guía tomográfica.

D. Broncofibroscopía

E. Biopsia pleural por toracotomía

34.Un paciente hospitalizado por pancreatitis aguda comienza con taquipnea, taquicardia, sudoracción y cianosis pro - gresivas. La
PaO2 es de 55 mmHg, la Rx de tórax muestra infiltrados alveolares bilaterales y la presión de enclava- miento capilar pulmonar es
normal. El aporte de oxígeno suplementario no mejora la situación. ¿Qué diagnóstico, entre los siguientes, es el más probable?:

A. Neumonía nosocomial.

B. Insuficiencia cardíaca.

C. Distress respiratorio del adulto.

D. Tromboembolismo pulmonar.

E. Embolia grasa.

35.Si llega un paciente a emergencia con infarto de miocardio ST elevado de dos horas de evolución, ¿Cuál de los siguien - tes
fármacos administraría usted?:

A. Estreptoquinasa.

B. Inhibidores de la enzima convertidor a de angiotensina.

C. Calcioantagonistas.

D. Bloqueadores adrenérgicos.

E. Aspirina.

36.¿Cuál de los siguientes fármacos reduce la mortalidad de los enfermos con insuficiencia cardiaca congestiva?:

A. Digital

B. Furosemida

C. Enalapril

D. Procainamida

E. Ácido acetilsalicílico

37.¿Cuál de las siguientes es una emergencia hipertensiva?

A. PA por encima de 220/120 mmHg

B. PA por encima de 220/120 mmHg con mareo y cefalea

C. PA por encima de 220/120 mmHg y disección de aorta torácica

D. Todas las anteriores

E. Ninguna de las anteriores

38.El colapso X profundo, en el pulso venoso es típico de:

A. Ductus arteriosus
B. Estenosis mitral

C. Insuficiencia aórtica

D. Tetralogía de Fallot

E. Taponamiento pericárdico

39.En relación al manejo de infección aguda por Hepatitis A, no complicada. Se recomienda:

A. Se debe observar y evitar medicamentos

B. Se debe utilizar la gammaglobulina hiperinmune a la brevedad

C. Es mejor el dosaje sérico de Anticuerpos para inicio de tratamiento

D. Se debe usar vitamina K vía subcutánea

E. Uso de corticoides

40.NO es habitual en el síndrome del intestino irritable:

A. Gran cantidad de moco en las heces.

B. Alteración en la consistencia de las heces.

C. Dolor que alivia con la defecación.

D. Sensación de evacuación incompleta.

E. Diarrea que no cede tras 48 h de ayuno.

41.El reflujo gastroesofágico afecta al 30 a 50% de gestantes. Este desorden se debe a:

A. Aumento de la presión intrabdominal por el crecimiento uterino

B. Un esfínter incompetente desenmascarado por el embarazo

C. Un efecto del estrógeno y progesterona sobre los músculos del esfínter inferior

D. Una manifestación inicial de ansiedad

E. Ay C

42.¿Cuál de los siguientes hallazgos es normal en el LCR?:

A. Menos de 30mg de glucosa

B. 0-4 leucocitos por mm3

C. 70mg de proteínas

D. Presión mayor de 300cm de agua

E. Hematíes en poca cantidad

43.Hombre de 24 años con antecedentes de gastroenteritis aguda tres semanas antes, que consulta por pérdida progresi - va de
fuerza en 4 miembros de 3 días de evolución, habiéndose i niciado a nivel distal en miembros inferiores asociando parestesias en
manos y pies. A la exploración presentaba tetraparesia de predominio en miembros, signos piramidales.

¿Qué resultados NO esperaría encontrar en el estudio diagnóstico realizado a este paciente?

A. Estudio electroneurográfico compatible con polineuropatía desmielinizante.

B. Bioquímica de líquido cefalorraquídeo con 250 células con 90% de mononucleares e hiperproteinorraquia.

C. Determinación de anticuerpos antigangliosidos (Anti-GM1) de clase IgG positivos.

D. Resonancia magnética nuclear craneal normal.

E. Potenciales de denervación en el estudio electromiográfico.

44.Un hombre de 45 años de edad es traído a la consulta por deterioro cognitivo progresivo. La exploración muestra movimi entos
involuntarios irregulares de las extremidades. El padre del paciente falleció a los 60 años de edad en un centro psiquiátrico , y también
presentaba dichos movimientos involuntarios de extremidades. ¿Cuál es el diagnóstico más probable?:

A. Corea de Sydenham.
B. Enfermedad de Huntington.

C. Enfermedad de Lafora.

D. Enfermedad de Hallervorden-Spatz.

E. Parálisis supranuclear progresiva.

45.En una mujer de 20 años, portadora de una hiperfunción tiroidea, el estudio complementario ha revelado tratarse de un adenoma
tóxico con supresión funcional del resto del parénquima. ¿Cuál sería la actitud terapéutica a seguir?:

A. Drogas antitiroideas.

B. Yodo radiactivo.

C. Suplemento dietético de yodo.

D. Exéresis quirúrgica.

E. Seguimiento sin terapéutica.

46.El hallazgo de una pigmentación de color pardo claro u oscuro, en forma de parches, bilateral en la mucosa de reves - timiento oral
y las encías debe hacer pensar en:

A. Diabetes.

B. Enfermedad de Addison.

C. Leucoplasia.

D. Lupus eritematoso.

E. Carcinoma epidermoide.

47.Señalar qué dato de los siguientes NO esperaría encontrar en un paciente con SIADH:

A. Aumento de los niveles plasmáticos de vasopresina.

B. Hiponatremia.

C. Hipoosmolaridad urinaria.

D. Volumen extracelular normal.

E. Hipoosmolaridad plasmática.

48.En una paciente con artritis reumatoide de 9 meses de evolución, que requiere dosis bajas de corticoides para contro - lar
parcialmente sus síntomas articulares, ¿cuál sería la medida más adecuada?:

A. Agregar Inhibidores del TNF alfa.

B. Corticoides en dosis altas.

C. Añadir Ciclofosfamida.

D. Adicionar Metotrexate.

E. Suspender corticoides.

49.¿Cuál de las siguientes enfermedades del tejido conectivo está asociada con más frecuencia con el fenómeno de Ray - naud?:

A. Artritis reumatoide.

B. Dermatomiositis.

C. Esclerosis sistémica difusa.

D. Espondilitis anquilosante.

E. Lupus eritematosos sistemático.

50.Un individuo que nunca ha sentido angustia por exponerse al público, presenta una crisis de angustia en una conferen - cia y le
aterroriza el pensar en tener otra conferencia. Si presenta angustia únicamente frente a la exposición al público

¿cuál es el diagnostico apropiado?


A. Agorafobia sin crisis de angustia

B. Trastorno de pánico

C. Trastorno de angustia generalizada

D. Fobia social

E. Trastorno de angustia con agorafobia

51.Una necesidad general y excesiva de que se ocupen de uno, que ocasiona un comportamiento de sumisión adhesión y temores
de separación, que empieza al inicio de la edad adulta y se da en varios contexto s ¿a qué nos referimos?:

A. Trastorno de personalidad por dependencia.

B. Trastorno de personalidad por evitación

C. Trastorno pasivo agresivo de personalidad.

D. Trastorno límite de personalidad.

E. Trastorno histriónico de personalidad.

52.Respecto al fracaso renal agudo, señale lo INCORRECTO:

A. Durante la fase de recuperación puede existir hipopotasemia.

B. La nefrotoxicidad por cisplatino se complica a veces con Hipomagnesemia.

C. Los procesos infecciosos son responsables del 75% de los fallecimientos.

D. La anemia es leve y de etiología multifactorial.

E. La existencia de niveles de ácido úrico de 10 obliga al tratamiento con inhibidores de su síntesis.

53.Mujer de 30 años, consulta por fiebre leve, edema de miembros inferiores dolor de garganta. Ex de orina: Proteinuria 3+, hematíes
40 por C, algunos dism órficos. PA: 160/110. BUN: 48 mg%, creatinina: 2mg%. Histopatolog ía: Leve prolifera-ción mesangial, depósito
de C3, M.E: Deposito denso intramenbranoso. La posibilidad diagnostica:

A. Nefropatía Lupica tipo IV

B. Glomérulo nefritis membrana proliferativa tipo 2

C. Glomérulo nefritis post infecciosa

D. Síndrome de Goodpasture

E. Granulomatosis de Wegener

54.Las lesiones dérmicas en la lepra lepromatosa presentan gran cantidad de los siguientes elementos celulares:

A. Células plasmáticas

B. Neutrófilos

C. Linfocitos

D. Células gigantes multinucleadas

E. Macrófagos

55.Varón de 20 años, consulta por dolor y aumento de volumen de articulaciones de ambas rodillas, codo derecho, mu - ñeca
izquierda, con dificultad para la marcha, adem ás equimosis en piel de muslos y brazos de hace varios meses. Hace 30 d ías sangra 2
días post extracción dentaria. Hb: 12 gr., leucocitos: 9,000, Plaquetas: 250,000, L átex: 2 +, T. de Pro-trombina: 13”, T. parcial de
Tromboplastina: 70”, creatinina: 1 mg%, T. sangría 3’. Posibilidad diagnóstica es:

A. Hemofilia A

B. Tromboastenia de Glassman

C. Síndrome Antifosfolípido

D. Coagulación intravascular diseminada

E. Enfermedad de Von Willebrand


56.Una mujer de 65 años acude a la consulta con gingivorragia. La exploración física no aporta datos de interés, salvo la presencia de
petequias en los tobillos. La cifra de leucocitos es de 500/uL (10% de neutrófilos), el hematocrito asciende al 25% y se detectan 10
000 plaquetas/uL. En la biopsia de la m édula ósea se observa una celularidad de 10%. Las esca-sas células disponibles para el
análisis no muestran ninguna anomalía morfológica. La paciente carece de ante - cedentes de hepatitis, no ha tomado recientemente
ningún fármaco y nunca ha sufrido ningún problema similar. El tratamiento más adecuado en este caso sería:

A. Prednisona

B. Dosis bajas de ara-C

C. Plasmaféresis

D. Globulina antitimocítica, Ciclosporina, metilprednisona y G-CSF

E. G-CSF, eritropoyetina EL-11

57.La docimacia pulmonar es usada para determinar:

A. Asma bronquial

B. Neumonía purulenta

C. Atelectasia del neonato

D. Cuerpo extraño en neonatos

E. Enfisema bulloso

58.Un recién nacido presenta un cefalohematoma. Señale, de las medidas planteadas, la q ue se considera INCORRECTA:

A. Radiografía de cráneo si sospecha asociación con fractura subyacente.

B. Analítica de control de bilirrubina.

C. Fototerapia, si la ictericia es importante.

D. Incisión y drenaje quirúrgico en todos los casos.

E. Analítica de control del hematocrito.

59.Durante el desarrollo infantil, la personalidad queda básicamente estructurada:

A. Entre el 1er y 2º año

B. Durante la etapa escolar

C. En la lactancia

D. Al finalizar la adolescencia

E. En el periodo pre escolar

60.La vacuna contraindicada en personas con hipersensibilidad intensa al huevo. :

A. Fiebre amarilla.

B. Encefalitis equina.

C. Rabia.

D. Poliomielitis.

E. Encefalitis de San Luís.

61.Un niño de 3 años ha tragado un imperdible de seguridad abierto, el cual es visto radiográficamente en el estómago.

El tratamiento más adecuado es:

A. Catárticos para ocasionar un pasaje rápido

B. Extraer el imperdible mediante endoscopia

C. Observación, con la esperanza que avance y se elimine

D. Extraer el imperdible mediante gastrostomía

E. Darle alimentación de algodón para atrapar el imperdible


62.Lactante de 2 años, desde hace 3 días presenta tos exigente, y hace un día fiebre y dificultad respiratoria. Se auscultan crepitantes
en base de hemitórax derecho. El diagnóstico más probable es:

A. Neumonía intersticial.

B. Neumonía bacteriana.

C. Bronquiolitis.

D. Empiema.

E. Traqueobronquitis

63.En el recién nacido la ictericia más frecuente es:

A. La fisiológica.

B. La atresia de vías biliares.

C. La debida a infecciones.

D. La secundaria a enfermedad hemolítica del recién nacido.

E. La debida a hepatitis neonatal.

64.Para evaluar distensión abdominal, la radiografía muestra un signo de “doble burbuja”. La mejor explicación de la distensi ón
abdominal de recién nacido es:

A. Enfermedad de Hirschsprung

B. Íleo meconial

C. Tapón de meconio

D. Atresia duodenal

E. Atresia pilórica

65.Niña de 2 años acude a urgencias por cuadro de disnea, estridor y tos perruna. En la exploración destaca rinorrea intensa,
amígdalas hipertróficas con exudado puntiforme, Fiebre de 38,5. A la auscultaci ón pulmonar se aprecia hipo ventilación bilateral y
estridor inspiratorio. El diagnóstico más probable será:

A. Traqueítis bacteriana.

B. Laringitis supra glótica

C. Laringitis aguda

D. Laringitis espasmódica.

E. Epiglotitis.

66.El diagnóstico de otitis media aguda en el niño se basa en:

A. Otalgia

B. Visualización del tímpano

C. Fiebre

D. Pérdida de la audición

E. Rascado del pabellón auricular

67.Preescolar de 3 años de edad, que desde hace 3 días presenta fiebre y deposiciones liquidas en número de 4 a 6 por día,
inicialmente sin moco ni sangre, de escaso volumen; posteriormente con moco y rasgos de sangre, acompañados de dolor abdomina l
y tenesmo. El agente etiológico presuntivo es:

A. Rotavirus

B. Salmonella typhi

C. Shigella

D. Criptosporidium
E. Vibrio cholerae

68.Paciente de 5 años de edad, con tos nocturna desde hace 2 meses, sin fiebre, con antecedentes de haber acudido a Emergenci a
en 3 oportunidades por dificultad respiratoria. El diagnóstico probable es:

A. Tuberculosis pulmonar

B. Bronquitis crónica

C. Bronquiolitis

D. Fibrosis quística

E. Asma bronquial

69.Un niño de 1 año de edad, es traído porque desde hace 24 horas presenta fiebre alta 39.5° C y al parecer l e molesta al deglutir, al
examen físico se encuentran lesiones vesiculares y ulcerativas en los pilares anteriores de la orofaringe y úvula, el resto d el examen
es negativo, el germen responsable de esta enfermedad es:

A. Herpes virus

B. Coxsackievirus

C. Parvovirus

D. Ebstein Barr

E. Herpes virus 6

70.La historia de pica, con ingesta de tierra orienta a descartar infección por:

A. Toxocara.

B. Leptospira.

C. Toxoplasma.

D. A y C.

E. Ninguna de las anteriores.

71.El Kwashiorkor es un síndrome clínico resultado de una grave deficiencia en:

A. Proteínas.

B. Grasas.

C. Vitamina A.

D. Niacina.

E. Tiamina.

72.El soplo que caracteriza al conducto arterioso persistente se denomina:

A. Soplo Gibson •

B. Soplo en maquinaria •

C. Soplo en vapor •

D. Todas las alternativas son correctas •

E. Ninguna de las anteriores

73.La causa más frecuente de convulsiones en el recién nacido es:

A. Infecciosa

B. Metabólica

C. Parasitaria
D. Hipertermia

E. Hormonal

74.Se ve a un niño de 2 años bebiendo de un envase de kerosen e. Inmediatamente empieza a toser y presenta taquipnea, por lo que
lo llevan al hospital. El mejor método de tratamiento es:

A. Inducir el vómito.

B. Realizar un lavado con sonda nasogástrica.

C. Instilar aceite mineral.

D. Administrar esteroides.

E. Ninguna de las anteriores

75.Un recién nacido, a los 5 minutos de vida presenta: F.C. 60 latidos por minuto, cianosis acra, esfuerzo respiratorio ausente, tono
muscular inexistente y ausencia de respuesta al introducir un catéter por su nariz. La puntuación de Apgar en este niño será:

A. 5.

B. 4.

C. 3.

D. 2.

E. 1.

76.La emergencia quirúrgica extrauterina más frecuente en la mujer embarazada es:

A. Apendicitis aguda.

B. Diverticulitis aguda.

C. Quiste ovárico roto.

D. Compresión aguda de cava por útero grávido.

E. Fístula útero-vesical

77.Una joven de 16 años con historia familiar de Esferocitosis Hereditaria ingresa al hospital para esplenectomía como una te rapia
primaria de su enfermedad. ¿Cuál de los siguientes hallazgos es muy probable de encontrar en la ciru gía?:

A. Pancreatitis Crónica.

B. Ulcera Duodenal.

C. Colelitiasis.

D. Colecistitis Crónica.

E. Hepatitis Crónica.

78.NO constituye contraindicación para el abordaje laparoscópico:

A. Paciente con trastornos de la coagulación.

B. Paciente con múltiples cirugías abdominales previas.

C. Paciente obeso.

D. EPOC severo con oxígeno domiciliario.

E. Paciente con disfunción ventricular severa.

79.Paciente varón de 11 años de edad que ingresa a SOP con Dx de hernia inguinal derecha encarcelada. En el i ntraope- ratorio se
encuentra el saco herniario conteniendo el apéndice cecal perforado en su base. Ud. la trata como:

A. Hernia de Litré.

B. Hernia de Amyand

C. Hernia de Ritcher.

D. Hernia de Spiegel.
E. Hernia de Garengeot.

80.Paciente de 60 años, natural y procedente de Huancayo, que presenta dolor abdominal de tipo cólico, difuso, poco localizado;
vómitos fecaloideos, ausencia de expulsión de gases y heces por el ano. Al examen físico: distensión ab - dominal, timpanismo y
ruidos intestinales aumentado de frecuencia y de tono, no fiebre, deshidratación moderada, abdomen depresible, poco sensible sin
signos de irritación peritoneal. El diagnóstico probable es:

A. Peritonitis

B. Hipertrofia de píloro

C. Obstrucción intestinal

D. Ileitis regional

E. Sepsis intestinal

81.Sobre las quemaduras, señalar el enunciado correcto:

A. La povidona yodada es el agente más efectivo en la limpieza y descontaminación de las quemaduras.

B. Las flictenas no deben ser desbridadas por el riesgo de infección.

C. Las quemaduras, en pacientes hospitalizados, deben ser examinadas cada 48 horas.

D. No es necesario el tratamiento antibiótico endovenoso profilácticamente

E. En las quemaduras eléctricas, la gravedad de la lesión se correlaciona con el tamaño de la lesión cutánea.

82.Durante la intervención quirúrgica en un paciente de 26 años de edad por apendicitis aguda congestiva, accidental -

men-te se produce una perforación mínima de colon. ¿cómo se clasifica la herida quirúrgica?:

A. Sucia contaminada

B. Contaminada

C. Limpia

D. Sucia

E. Limpia contaminada

83.Paciente en el sexto día postoperatorio por peritonitis debida a perforación traumática de colon. La herida operatoria pre senta
vesículas hemorrágicas en la superficie de la piel. Al explorar la herida se encuentra aponeurosis edematosa de color gris pizarra y
tejido celular subcutáneo esfacelado. El probable diagnóstico es:

A. Gangrena gaseosa

B. Absceso de herida por gramnegativos

C. Fasceitis necrotizante

D. Celulitis por estreptococo

E. Flemón

84.Un niño de 18 m eses, quien gozaba de buena salud, fue cogido por los brazos para subirlo dos escalones; lanzo un grito y después
se limitó el empleo de su brazo izquierdo, el diagnóstico más probable es:

A. Parálisis de Erb

B. Lesión del plexo braquial

C. Subluxación de la cabeza del radio

D. Luxación del humero

E. Fractura de la clavícula.

85.Señale de las siguientes fracturas, cuál con mayor probabilidad NO precisará un tratamiento quirúrgico:

A. Fractura abierta grado II de la diáfisis tibial.

B. Ausencia de consolidación de una fractura de la diáfisis radial.


C. Fractura pertrocantérea en un paciente anciano.

D. Fractura patológica de húmero por una metástasis de un adenocarcinoma mamario.

E. Fractura desplazada de la clavícula.

86.Los factores de riesgo para el desarrollo de parálisis neonatal del plexo braquial pueden ser maternos, fetales o rela - cionados con
el parto. ¿Cuál de los siguientes es el factor de riesgo más frecuente para la aparición de esta entidad?

A. Parto asistido con fórceps.

B. Macrosomía fetal.

C. Diabetes mellitus materna.

D. Distocia de hombro.

E. Anomalía uterina.

87.Un hombre de 45 años está en tratamiento de quimioterapia por un tumor testicular. ¿Cuál de los siguientes fármacos es más
probable que se utilice como antiemético?

A. Bromocriptina.

B. Ergotamina.

C. Ibuprofeno.

D. Ondansetrón.

E. Sumatriptán.

88.Varón de 70 años que acude por hipoacusia. En la exploración tiene un Rinne negativo en OD y positivo en OI y un Weber
lateralizado hacia el OI. ¿Qué tipo de hipoacusia tiene el paciente?:

A. Cofosis de OD.

B. Hipoacusia de transmisión de OD.

C. Hipoacusia de percepción de OI.

D. Hipoacusia mixta de OD.

E. Hipoacusia de transmisión de OI.

89.Mujer de 77 años que presenta disminución bilateral y progresiva de agudización visual de vario s años de evolución. Sin embargo,
ahora es capaz de leer sin sus gafas de presbicia. ¿Cuál es el diagnóstico más probable?

A. Presbicia

B. Catarata senil

C. Degeneración macular asociada a la edad

D. Desprendimiento de retina

E. Neuropatía óptica isquémica

90.En el recién nacido, la salivación excesiva está asociada a:

A. Hernia diafragmática congénita

B. Atresia esofágica

C. Estenosis hipertrófica

D. Estenosis duodenal

E. Acalasia

91.Se tiene información acerca del porcentaje del cumplimiento y no cumplimiento del calendario de vacunaciones en un grupo de
niños del cono norte de Lima. Cuál es el gráfico que elegiría:

A. Histograma
B. Lineal

C. Tendencias

D. Circular

E. Polígono de frecuencias

92.En un estudio donde se buscaba establecer la relación entre la exposición ambiental a PM10 (partículas menores de 10 micras) y
la ocurrencia de asma, se encontró un OR de 4.5 (IC95% 3.5-5.5) al comparar el grupo expuesto frente al grupo no expuesto. Se
puede afirmar:

A. La exposición se comporta como un factor protector para el asma.

B. La relación no es estadísticamente significativa.

C. El grupo expuesto presenta 450% más riesgo de asma que el grupo no expuesto.

D. Los expuestos presentan 4.5 veces el riesgo de los no expuestos de presentar asma .

E. La hipótesis nula no se rechaza para un nivel de significación del 5%.

93.¿Cuál de las siguientes enfermedades pertenece al grupo de transmisibles metaxémicas de notificación inmediata?:

A. Ántrax

B. Dengue

C. Brucelosis

D. Fiebre amarilla.

E. Tétanos

94.¿Cuál de las siguientes corresponden a la prevención secundaria EXCEPTO?:

A. Atención integral de pacientes hospitalizados

B. Atención médica integral de la comunidad

C. Encuestas masivas

D. Medidas de rehabilitación

E. Todas las anteriores

95.El promedio de hijos vivos por mujer en el poblado de Tungasuca, zona rural andina del Perú, de acuerdo al censo de poblaci ón
2007, es de 2.9, cifra m ás alta que el promedio nacional para la zona rural (2.5). Ud. es el jefe del Centro de Salud de Tungasuca, p or
lo cual la primera medida para mejorar esta situación será:

A. Establecer un programa de capacitación sobre métodos de planificación familiar en los colegios de la zona.

B. Coordinar con la organización religiosa local para establecer programas de capac itación sobre métodos de planifi- cación
familiar

C. Establecer un programa de capacitación sobre métodos modernos de planificación familiar

D. Solicitar a la dirección regional de salud una partida presupuestaria adicional para capacitación

E. Potenciar la farmacia comunal con anticonceptivos orales, y métodos de barrera a precios subsidiados

96.En la curva endémica se denomina “zona de éxito” a la que se encuentra:

A. Por debajo de Q1.

B. Entre Q1 y Q2.

C. Entre Q2 y Q3.

D. Por encima de Q3.

E. Por encima de Q1.

97.Para la búsqueda de casos de tuberculosis, ¿qué acciones deben de ejecutarse?:

A. Radiografía de consultantes al hospital


B. Radiografía de tórax en sospechosos a la foto radiografías

C. Baciloscopia en sintomáticos respiratorios

D. Encuestas tuberculino-radiológicas en la comunidad

E. Ninguna de las anteriores

98.¿Qué bienes producir y en qué cantidad?, Se refiere al estudio de:

A. La Oferta

B. La Demanda

C. El Mercado

D. La Distribución

E. El Marketing

99.El órgano asesor del Hospital responsable de mantener la calidad técnica de la atención medica que se brinda a los pacientes es:

A. El Comité Técnico – Médico

B. El Comité Técnico – Administrativo

C. El cuerpo médico

D. La auditoría del hospital

E. Todos ellos

100. Como parte de las técnicas de recolección de información, tenemos a la “Lluvia de Ideas”. Marque la opción incorrec - ta con
relación a esta técnica:

A. Se enfatiza en la cantidad y se evita el juzgamiento

B. Ayuda a tener una perspectiva amplia sobre un problema

C. Todas las ideas son útiles, y si no son muy prácticas hay que descartarlas rápidamente para dar paso a nuevas ideas

D. Hay que disponer de tiempo al final, para organizar y dejar una idea concreta en común como conclusión.

E. Se puede crear nuevas ideas, en base a ideas de los integrantes

3 º EXAMEN ENAN
1¿Cuál es el medicamento inicial de elección en un cuadro convulsivo de emergencia?
a) Fenobarbital
b) Fenitoina
c) Diazepam
d) Ácido valproico
e) Tiopental
2.Una mujer de 25 años acude a consulta por poliuria y polidipsia. Hasta el momento las investigaciones excluyen las causas p sicógenas y diabetes. Se realiza prueba de
restricción de líquidos. Al final de la misma la osmolalidad urinaria es 240 mosm/kg y el nivel sérico de hormona antidiurética(ADH) se encuentra elevado.
¿Cuál de los siguientes diagnósticos es el más probable?
a) Defecto de la corteza suprarrenal
b) Diabetes insípida nefrógena
c) Diabetes insípida central
d) Necrosis tubular aguda
e) Enfermedad de Addison
3.Mujer 54 años, G: 8 P: 8008 con antecedente de partos domiciliarios. Acude por presentar pérdida de orina cuando ríe, tose o al subir gradas. ¿Cuál es el diagnóstico
más probable?
a) Incontinencia de urgencia
b) Incontinencia de esfuerzo
c) Incontinencia oculta
d) Prolapso genital
e) Infección del tracto urinario
4.¿Qué hernias tienen mayor riesgo de estrangulamiento?
a) Femorales
b) Inguinales indirectas
c) Inguinales directas
d) Spiegelianas
e) Umbilicales
5. Mujer de 25 años sufre accidente de tránsito. En emergencia se constata: Funciones vitales estables y fractura de rama isquiopúbica derecha que compromete vejiga
en su porción intra peritoneal. ¿Cuál es el tratamiento más adecuado?a. Cirugía. b. Colocación de sonda Foley. c. Talla vesical. d. Citostomía suprapúbica. e. Reparación
endoscópica.

6.Gestante a término no controlada, VIH+. Al examen: altura uterina compatible con embarazo de 38 semanas, sin enfermedad opo rtunista. Se inicia tratamiento
antirretroviral. ¿Cuál sería la forma del parto y recomendación para la lactancia?
a) Parto vaginal y lactancia materna
b) Cesárea y fórmula maternizada
c) Parto vaginal y fórmula maternizada
d) Cesárea y lactancia materna
e) Cesárea y lactancia mixta
7.Los nódulos de Heberden característicos de la osteoartritis se ubican en las articulaciones...
a) Interfalángicas distales.
b) Radiocarpianas.,
c) Interfalángicas proximales.
d) Metacarpofalangicas.
E) Metatarsofalángicas
8.El antibiótico de elección en otitis media aguda es:
a) Gentamicina
b) Eritromicina
c) Clindamicina
d) Amoxicilina
e) Penicilina V
9.Varón de 72 años con fibrilación auricular y alcoholismo crónico. Es traído porque desde hace 1 año presenta deterioro cogn itivo y algunos déficit focales. Al examen
PA: 130/70 mm Hg, FC: 87 F R: 17 x', despierto, orientado parcialmente, Babinski y Hoffrnan izquierdo positivos. ¿Cuál es la causa de demencia más probable?
a) Encefalopatia de korsakoff
b) Enfermedad de Alzheimer
c) Encefalopatia de wernicke
d) Multiinfarto
e) Alcoholismo crónico
10.Varón de 18 años, con herida cortante en muñeca derecha por asalto con arma blanca. Presenta hipoestesia y postura en hiperextensión metacarpofalángica del
4to. y 5to. dedo. ¿Cuál es el nervio lesionado?
a) Mediano
b) Radial
c) Cubital
d) Musculocutáneo
e) Circunflejo
11.Varón de 53 años, con tuberculosis pulmonar BK positivo y en tratamiento con esquema 1, que presenta alteraciones de la visión de colores. ¿Cuál de los siguientes
fármacos produce con mayor frecuencia este efecto adverso?
a) Pirazinamida
b) Isoniacida
c) Rifampicina
d) Estreptomicina
e) Etambutol
12.¿Cuál de las siguientes leucemias es más frecuentes en niños?
a) Linfoblástica
b) Mieloblásticas
c) Monoblástica
d) Mieloide crónica
e) Todos por igual
13. La catarata en roseta es típica de: a. Espondilitis anquilosante. b. Traumatismo ocular c. Adulto mayor (senil). d. Diabético. e. Degeneración macular

14.Paciente con infección por VIH que presenta signos meníngeos ¿Cuál es la etiología más frecuente?
a) Toxoplasmosis
b) Histoplasmosis
c) Criptococosis
d) Estreptococo pneumoniae
e) Listeria monocitógenes
15.¿Cuál de los siguientes es considerado un efecto colateral de los anticonceptivos orales?
a) Mialgia
b) Condritis
c) Hipermenorrea
d) Dismerionea
e) Mastalgia
16. Una cirugía por apendicitis en la que se encontró perforación de colon, se realiza jareta invaginante y se deja dren tubular corresponde a una herida: a. Limpia. b.
Limpia contaminada. c. Sucia. d. Infectada. e. Contaminada.

17.Varón de 71 años, hipertenso, acude por precordalgia intensa, EKG: supradesnivel del 17.segmento ST en D II, D III, aVF e infradesnivel ST en V1, ¿Cuál es el
diagnóstico más probable?
a) IMA STE diafragmático
b) IMA STNE posterior
c) IMA STE posterior
d) IMA STE inferoposterior
e) IMA cara anterior
18. La hernia indirecta que presenta anillo inguinal dilatado, sin desplazamiento de vasos epigástricos, corresponde a la clasificación de Nyhus: a. II b. I c. IIIb d. IV e. IIIa

19.Cuando existe una correlación entre dos variables cuantitativas continuas. ¿Cuál es el modelo predictivo que debe aplicarse?
a) Regresión múltiple
b) Regresión de Cox
c) De efectos fijos
d) Dispersión
e) Regresión lineal
20.¿Cuál es la técnica que se utiliza para el control de sesgo de selección?
a) Pareamiento
b) Aleatorización
c) Estratificación
d) Enmascaramiento
e) Medidas repetidas
. 21. La fractura del recluta ocurre a nivel de: a. Primer metacarpiano. b. Segundo metatarsiano. c. Calcáneo. d. Quinta falange del pie. e. Astrágalo.

22.El puerperio inmediato comprende: Las primeras dos


a) horas post parto Las primeras 12
b) horas post parto
c) Las primeras 24 horas post parto
d) Las primeras 48 horas post parto
e) La primera semana post parto
23.Mujer de 43 años, acude por convulsiones tónico-clónicas generalizadas en 4 oportunidades desde hace 35 minutos, sin recuperación de la conciencia. Durante la
evaluación nuevamente se repite el cuadro descrito ¿Cuál es el tratamiento?
a) Diazepam: 10 mg EV, carga de Fenitoina a 20 mg/k g peso a una infusión máxima de 50 mg/min.
b) Midazoiam 5 mg, fenobarbital EV diluido lento, infusión más de 50 mg/
d) Propofol EV, Fenitoina bolo de 25 mg/kg peso, infusión máxima de 75 mg/minuto
c) Diazepam 10 mg EV, seguido de Midazolam en infusión
e) Diazepam 5 mg EV, carga de 800 mg de Fenitoina diluida en dextrosa al 5%
24.Varón de 40 años acude por dolor anal, progresivo e invalidante, de dos días de evolución que se incrementa al sentarse y caminar. Al examen: tumoración
renitente con signos de flogosis en el margen anal, ¿Cuál es el tratamiento inicial?
a) AINEs
b) Antibloticoterapia
c) Baños de asiento
d) Corticoterapia
e) Drenaje quirúrgico
25.Varón de 22 años que en reyerta sufre agresión con cuchillo. Al examen: PA: 110/70 mm Hg, FC: 80 x', FR. 18 x', herida de 4cm en flanco izquierdo, con dolor local y
rebote negativo. ¿Cuál es la conducta inicial?
a) Laparotornia exploradora
b) Exploración de herida
c) Laparoscopia exploradora
d) Transfusión urgente
e) TAC abdominal
26.Varón de 60 años, con disnea progresiva desde hace 2 horas. Al examen: FR: 34 x', presenta cianosis perioral y distal, sibilantes diseminados en ambos campos
pulmonares, escasos roncantes. Laboratorio: Leucocitos 12,368, pH: 7,42, PCO2: 28, P02: 68, HCO3: 21. Mejora después de admin istrar p2 adrenérgicos. ¿Cuál es el
diagnóstico más probable?
a) Tromboembolia pulmonar
b) EPOC descompensado
c) Insuficiencia respiratoria tipo II
d) Crisis asmática
e) Alcalosis metabólica e hipoxemia
27.Mujer de 45 años, con esquizofrenia paranoide y HIV positivo, acude al hospital por convulsiones y trastorno del sensorio. Al examen: Glasgow 9, signos meníngeos
(+). LCR: levaduras, positivo a tinta china. ¿Cuál es el tratamiento a seguir?
a) Fluconazol
b) Itraconazol
c) Anfotericina B
d) Ketoconazol
e) Caspofungina
28.Varón de 66 años, diabético, acude por fiebre, malestar general, tos con expectoración verdosa. Al examen: PA: 80/50 mm Hg, FC : 110 x', FR: 35 x', MEG, pulsos
débiles y oliguria. Laboratorio: leucocitosis 16000 x' lactato 1,4 U/L. Se realiza reto de fluidos y se observa estabilización de la PA, mejora el rango diurético, ¿cuál es el
diagnóstico?
a) Sepsis grave, foco respiratorio
b) SIRS e injuria renal aguda
c) Shock séptico foco respiratorio
d) Sepsis foco urinario
e) Shock séptico foco urinari o
29.En la fractura de Monteggia hay:
a) Fractura del 1/3 distal + luxación de cabeza de radio
b) Fractura del 1/3 proximal del cubito + luxación anterior de cúpula radial
c) Fractura de cubito y radio + luxación de codo
d) Fractura de 1/3 medio del radio + luxación del codo
e) Ninguna de las alternativas.
30.A una paciente de 70 años de edad se le detecta irregularidad de la frecuencia cardiaca en una evaluación de rutina. No ha presentado nuevos síntomas en reposo o
asociados a los esfuerzos. En el ECG, no se documentan ondas P y se encuentra un intervalo RR irregular con una frecuencia de 70/ min. En su ECG de hace cuatro años
tenía ritmo sinusal ¿Cuál de las siguientes conductas es la más adecuada en el manejo de la paciente?
a) Cardioversión
b) Antiarrítmicos
c) Bloqueador Beta
d) Anticoagulación
e) Ácido acetilsalicílico
31.A los antioxidantes se les conoce como tales al Prevenir o retardan el envejecimiento:
a) Porque protege el S.N.C.
b) Porque protege al sistema hormonal
c) Porque bloquea los electrones impares de los radicales libres
d) Ninguna de las alternativas.
e) Todas las alternativas son correctas.
32.¿Cuál es el proceso inicial de la hemostasia?
a) Tapón hemostático temporal
b) Vasoconstricción de la arteriola hemostático definitivo
c)Tromboplastina hística
d) Activación de la coagulación
e)Tapón
33.Lactante de 6 meses inicia hace 7 días cuadro de resfrió común, hace dos días se agrega fiebre de 39°C. Hace un día se agr ega dificultad respiratoria. Al examen:
polipnea, crepitantes y roncantes en ambos campos pulmonares. ¿Cuál es el diagnóstico más probable?
A) Neumonítis viral
b) Neumonía bacteriana
c) Bronquiolitis
d) Neumonía complicada
e) Neumonía atípica
34.Varón de 28 años con cuadro febril de 3 semanas de evolución, sudor nocturno, artralgias y dolor en cadera bilateral. Refiere comer queso fresco hecho
tradicionalmente. Si se sospecha de Brucellosis, ¿Cuál es la prueba de screeníng recomendada?
a) 2-Mercaptoetanol
b) Aglutinación en lamina
c) Aglutinación en tubo
d) Hemocultivo
e) Rosa de Bengala
35.Multípara de 40 años que una hora después de alumbramiento de su último parto, presenta pérdida sanguínea vaginal no muy abundante, pero la PA cae a 80/50 y
el pulso se acelera a 110/m, mostrando sudoración aunque sin perder la conciencia. No refiere dolor al hacer la palpación del abdomen, no se encuentra útero, el
examen rápido de la región vulvoperineal no evidencia desgarros. Cuál de las siguientes presunciones diagnósticas es la más p robable:
a) Desgarro perineo vaginal de 4to grado
b) Coagulación intravascular diseminada
c) Inversión uterina probablemente parcial
d) Desgarro cervical
e) Retención total de placenta
36. Una mujer de 64 años notó lesiones del tipo ampollosas en sus muslos y axilas. Eran pruriginosas, pero no dolían. No tenían otros síntomas. En la exploración las
lesiones son grandes, a tensión, en forma de ampollas de contenido seroso. La biopsia confirmó el diagnóstico de Penfigoide ampolloso. ¿Cuál de las siguientes
características histológicas es típica de esta condición?
a) Cambios inespecíficos
b) Depósitos de inmunoglobulina A (Ig
c) Lesiones dentro de la epidermis (acantólisis)
d) Lesiones de la membrana basal
e) Depósitos de inmunoglobulina M (IgM)
37.La ulcera péptica es una enfermedad multifactorial de curso crónico y periódico. De los siguientes factores diga ¿cuál es el más importante para su desarrollo?
a) AINE
b) Infección por helicobacter pylori
c)Alcohol
d)Tabaco
e)Hiperclorhidria
38.Una mujer de 68 años con hipertensión arterial y dislipidemia presenta dolor estroesternal de 30 minutos de duración que se irradia al cuello. Esta su dorosa y con
moderada angustia. El ECG muestra una elevación del segmento ST en las derivaciones de la cara inferior. ¿Cuál de los siguientes mecanismos es la causa más probable
de esta entidad?
a) Rotura de una placa coronaria
b) Aortitis
c) Inflamación pericárdica
d) Vasculitis
e) Miocarditis
39.Un varón asintomático de 64 años, ejecutivo de profesión, que asiste a su chequeo regular anual. Tuvo un infarto de miocardio anterior hace cuatro años y presenta
actualmente en el EKG, supradesnivel del ST en V3-V6 ¿Cuál es su diagnóstico?
a) Pericarditis calcificada
b) Aneurisma de la pared ventricular
c) Quiste hidatídico
d) Pleuropericarditis
e) Normal
40¿Cuál de los siguientes mecanismos describe mejor el efecto de los iones del calcio sobre el miocardio?
a) Inotropismo positivo
b) Inotropismo negativo
c) Cronotropismo positivo
d) Cronotropismo negativo
e) Desacoplamiento de la excitación – contracción
41¿Cuáles son los componentes de la calidad de los servicios de salud?
a) Organizacional, técnico y humano
b) Científico y técnico
c) Administrativo y financiero
d) Humano, financiero y gerencia!
e) Técnico, científico y financiero
42.¿Que significa CREST ?
a) Calcinosis, Raynaud, endocrinopatía, telagiectasia
b) Calcinosis, Raynaud, endocrinopatía y telagiectasia
c) Calcinosis, Raynaud, alteración del esófago, esclerodactilia y telagiectasia
d) Calcinosis, Raynaud, alteración del esófago, esclerodermia y telagiectasia
e) Calcinosis, Raynaud, alteración del esófago, esclerodermia y Tremor
43. Durante una laparoscopía, accidentalmente el cirujano perfora el colon transverso, perforándolo en un 20% de su circunferencia. Inmediatamente convierte la
cirugía a una laparotomía. ¿Cuál es la conducta a seguir con el segmento lesionado?: a. Debridación de bordes y rafia. b. Resección más anastomosis. c. Drenaje. d.
Colostomía. e. Ileostomía.

44.Lactante de 7 meses, con fiebre de 39 °C de 3 días de evolución, un


vómito por día y deposiciones normales. Rechaza alimentos semisólidos pero lacta bien. Al examen: T: 39 °C. Hemodinámicamente estable, mucosa orofaringea
normal. MV presentes en ambos campos pulmonares y piel sin lesiones, ¿Cuál es el diagnóstico más probable?
a) Faringitis aguda
b) Fiebre tifoidea
c) Infección del tracto urinario
d) Mononucleosis infecciosa
e) Diarrea aguda
45. En relación a las inmunizaciones. ¿Cuál es la actitud médica frente a un niño con asplenia funcional o anatómica?
a) Sólo puede recibir vacunas inactivadas
b) Al vacunar con agentes atenuados pueden multiplicarse
c) Debe evitarse la vacuna antineumocócica
d) Están indicadas las inmunizaciones
e) No debe recibir la vacuna antimeningocócica
46.Varón de 69 años en UCI, con neumonía, que presenta al 7mo día de evolución Murphy (+) ¿Cuál' es el diagnóstico más probab le?
a) Coliecistitis aguda iitiasica
b) Coliecistitis aguda reagudizada
c) Colecistitis crónica
d) Colecístitis aguda alitiasica
e) Colecistitis crónica reagudizada
47¿Cuál es la reacción adversa más frecuente en el tratamiento de la onicomicosis?
a) Síndrome de Stevens – Johnson
b) Hepatotoxicidad
c) Nefritis intersticial
d) Hemorragia digestiva
e) Dermatitis exfoliativa
48.En qué tipo de episiotomía es más frecuente el desgarro perineo vaginal de 4to grado:
a) El medio lateral izquierdo
b) En la medio lateral derecha
c) En la mediana con peritoneo corto
d) En la medina con perineo grande y amplio
e) En la cesareada anterior
49. Los siguientes son factores de riesgo para la pre eclampsia–eclampsia, con una excepción. Márquela:
a) Historia personal de hipertensión arterial Primigravidez
b) Diabetes mellitus
c) Enfermedad trofoblástica gestacional
d) Embarazo prolongado
50.Se considera EHG (Enfermedad hipertensiva de la gestión) cuando se encuentra:
a) Hipertensión, más proteinuria después de las 20 semana gestacional
b) Hipertensión, edema y aumento exagerado de peso después de las 20 semana gestacional
c) Hipertensión, anasarca y poliuria después de las 20 semana gestacional
d) Hipertensión, cefalea, tinitus después de la 20 semana gestacional
e) Hipertensión, hipoproteinemia y trombocitopenia después de la 20 semana gestacional
51.Neonato de 15 días con lactancia materna exclusiva y diagnóstico de galactosemia
¿Cuál es la conducta a seguir?
a) Puede recibir lactancia mixta con fórmula maternizada
b) Puede recibir lactancia materna de manera relativa
c) Debe recibir fórmula maternizada e iniciar ablactancia
d) Suspender de manera absoluta la lactancia materna
e) Puede recibir una fórmula extensamente hidrolizada.
52.Mujer de 30 años acude por dolor anal intenso al defecar. Antecedente: constipación crónica. Al examen: tacto rectal doloroso, esfínter hipertónico y hemorroide
centinela. Anoscopia: úlcera lineal media posterior. ¿Cuál es el tratamiento?
a) Esfinterotomía interna lateral
b) Esfinterotomia externa lateral
c) Ligadura de paquete hemorroidal
d) Curetaje de la úlcera
e) Cauterización de la úlcera
53.El LCR en la meningoencefalitis bacteriana se caracteriza:
a) Hipoglucorraquia, hipoproteinor raquia y lactato mayor de 3.9 mEq/L
b) Hipoglucorraquia, hiperproteinor raquia y lactato menor de 1.6 mEq/L
c) Hipoglucorraquia, hiperproteinorraquia y lactato mayor de 3.9 mEq/L
d) Ninguna de las alternativas es correcta .
e) Todas las alternativas son correctas.
54¿En el caso supuesto que su respuesta hubiese sido cáncer de próstata, cuál de los siguientes análisis de laboratorio hubiese sido de mayor utilidad?
a) Fosfatasa alcalina
b) Fosfatasa acida
c) Antígeno específico prostático
d) Antígeno carcinoembrionario
e) Gonadotropinas coriónicas
55.La transición demográfica en el Perú se relaciona con la disminución de la...
a) Razón de dependencia.
b) Densidad poblacional
c) Tasa global de fecundidad
d) Tasa de migración interna.
e) Esperanza de vida al nacer.
56. En una comunidad con servicios de agua a domicilio y alcantarillado, pero no en el colegio, se ha detectado incremento de casos de diarrea aguda en los
estudiantes, en este contexto, el personal del Centro de Salud ha planificado realizar las siguientes acciones: capacitación a docentes del colegio, coordinación con la
Municipalidad y el sector educación para construir servicios sanitarios adecuados (mientras tanto, colocar letrinas). Estas actividades corresponden a:
a) Promoción
b) Prevención
c) Gestión
d) Organización
e) Recuperación
57¿Cuál de los siguientes tratamientos se considera el primer paso para el manejo inicial de las fracturas abiertas o expuest as?
a) Fijación con tutores externos
b) Desbridamiento urgente
c) Inmovilización endomedular
d) Inmovilización con placas y tornillos
e) Inmovilización con aparato de yeso
58.Varón de 70 años con dolor de espalda y extremidades al deambular, astenia de cinco meses de evolución. Al examen: PA 130/80 mm Hg, FC: 78 x', FR: 20 x', T°:
37.5 °C. Palidez de piel, hepatoesplenomegalia, Laboratorio: anemia, hipercalcemia, azoemia,
¿Cuál es el diagnóstico más probable?
a) Leucemia mieloide crónica
b) Leucemia linfática crónica
c) Linterna de células
d) Mieloma múltiple
e) Leucemia de células plasmáticas
59.¿Cuál de las siguientes enzimas plasmáticas diagnóstica enfermedad obstructiva biliar?
a) Gamma-glutamil transferasa
b) Transaminasa glutámico-oxalacetica
c) Deshidrogenasa isocítrica
d) Acetilcolinesterasa
e) Leucina aminopeptidasa
60.La tuberculosis de primoinfección es más frecuente de
a) Piel
b) Estómago
c) Pulmón
d) Ganglio linfático
e) Intestino delgado
61.Varón de 32 años que ha presentado matarla por Plasmodium vivax, para prevenir la recaída, ¿cuál es su indicación terapéutica?
a) Cloroquina 500 mg. X 4 semanas
b) Artesunato 1200 a 1600 mg. Vía oral x 3 días
c) Primaquina 30 mg x día x 14 dlaa
d) Quinidina 10 mg/kg, hasta un máximo de 600 mg. En solución salina x 2 semanas
e) Usar mosquiteros
62.La presencia de secreción vaginal amarillenta, espumosa y lesiones eritematosas en cérvix nos da la sospecha de infección por:
a) Trichomonas vaginalis
b) Candida albicans
c) Gardnerella
d) Chlamidya trachomatis
e) Neisseria gonorrheae
63.Mujer de 30 años con G: 3 P: 3003 y FUR hace 1 semana, acude a emergencia por dolor de gran intensidad en bajo vientre y fiebre de 39 °C a más. Al tacto vaginal:
dolor a la movilización de cérvix, masa palpable en anexo izquierdo y flujo vaginal de regular volumen. ¿Cuál es el diagnóstico más probable?
a) Embarazo ectópico
b) Absceso pélvico
c) Apendicitis aguda complicada
d) Pielonefritis aguda
e) Diverticulitis
64.Gestante de 41 semanas G: 2 P: 1001, El embarazo ha transcurrido normalmente. Al examen: feto en cefálico, test no estresante reactivo y Bishop favorable. ¿Cuál
es la conducta a seguir?
a) Esperar hasta que inicie el trab ajo de parto espontaneo
b) Maduración cervical
c) Inducir el parto
d) Administrar corticoides e inducir el parto pasadas las 48 horas
e) Hacer cesárea electiva.
65.Mujer de 25 años, acude por dolor abdominal en FID, hace 7dias, se auto medica con ciprofloxacino y paracetamol VO. Al examen: PA: 80/50 mm Hg, FC: 120, FR:
24, T°: 38°C. Piel pálida, sudorosa, escleras ictéricas Abdomen: distendido, RHA disminuidos, Mc Burney positivo, rebote positivo. El diagnóstico es apendicitis aguda
complicada con peritonitis generalizada con indicación quirúrgica. En el post operatorio continua ictérica, ¿Cuál de las sigu ientes patologías explica ia ictericia?
a) Hepatitis
b) Pancreatitis
c) Pi leflebitis
d) Colangitis
e) Iatrogenia
66¿Cuál es un principal criterio para endometritis?
a) Fiebre
b) Dolor pélvico
c) Secreción vaginal maloliente
d) Sangrado vaginal
e) Estreñimiento
67.¿Cómo se denomina la alteración de la fusión de los pliegues uretrales ?
a) Hipospadia
b) Agenesia del genital externo
c)Pene bífido
d) Micropene
e) Hidrocele
68.Paciente que presenta dolor precordial de 5 minutos que cede al reposo .¿Cuál es el diagnóstico más probable?
a) Angina inestable
b) Angina estable
c) Infarto de miocardio
d) Angina de Prinzmetal
e) Osteocondritis
69.Después de la penetración de un espermatozoide en el óvulo, se produce la intensificación del metabolismo del huevo. ¿Cuál es la causa del incremento del
metabolismo oxidativo?
a) Aumento de la necesidad de oxígeno y su consumo
b) Disminución del metabolismo de carbohidratos
c) Disminución del contenido de aminoácidos libres
d) Disminución de la actividad de fermentos proteolíticos
e) Aumento de la síntesis de ácidos grasos
70. Lactante que rueda de prono a supino y se sienta sin soporte. ¿Cuál es su edad en meses?
a) 4
b) 5
c) 6
d) 2
e) 3
71.Recién nacido de 14 días de edad, P: 3700 gr, con lactancia materna exclusiva, ginecoobstetra, refiere que la madre tiene BK (++). El neonato ha ganado 120 gr de
peso, luce activo, al examen físico es normal y no ha recibido BCG, ¿Cuál es su Dx y conducta con el neonato?
a) Contacto, aplicar BCG
b) Infección, RX Tórax, Hemograma.
c) Infección, aplicar PPD
d) Contacto, INH 18.5 mg/día vo
e) Contacto, suspender lactancia materna, dar sucedáneo de la LM 13% 90cc cada 3 horas.
72. La incidencia más alta en que se producen los accidentes por cuerpo extraño en niños es:
a) A los 4 años de edad.
b) A los 36 meses de edad.
c) A los 14 meses de edad.
d) Cuando el niño diferencia el día y la noche
e) Cuando el niño deja de orinarse en la cama.
73. Lesión punzocortante a la altura de L1 paravertebral derecha ¿Qué arteria puede lesionarse?
a) Arteria hepática
b) Arteria aorta
c) Arteria renal
d) Arteria colónica
e) Arteria iliaca
74. En la enfermedad de Parkinson ¿qué neurotransmisor se altera?
a) Aspartato
b) Dopamina
c) Glutamato
d) Ácido g-aminobutírico (GABA)
e) Serotonina
75.¿Cuál es la mordedura más contaminada?
a) Gato
b) Conejo
c) Hamster
d) Humana
e) Canina
76.Recién nacido de 32 semanas y 1800 gramos de peso, a las 48 horas de vida presenta ictericia, hepatoesplenomegalia y lesio nes cutáneas de aspecto petequial en el
hemograma se aprecia anemia con eritroblastosis y trombocitopenia, en la radiografía de cráneo se observan calcificaciones cerebrales periventriculares. Que
diagnostico le parece más probable:
a) Rubeola congénita
b) Citomegalovirus congénita
c) Sepsis neonatal
d) Enfermedad hemolítica del recién nacido
e) Sífilis congénita
77.Varón de 60 años, con masa dolorosa en fosa iliaca derecha y fiebre de 7 días de evolución, ¿Cuál es el diagnóstico más pr obable?
a) NM de ciego
b) Absceso de pared abdominal
c) Hernia de Spiguel Hernia Spiguel
d) Tumor carcinoide
e) Plastrón apendicular
78.Mujer de 56 años asintomática acude a su control mensual de diabetes mellítus tipo
2. Al examen: no anormalidades. Laboratorio: glucosa: 129mg/d1., urocultivo: E. coll
>100,000 UFC/ml, ante estos resultados. ¿Cuál es la conducta a seguir?
a) Jugo de Cramberry diario
b) Observacion
c) Norfloxacino 400mg / 2 veces al día por 5 días
d) Nitrofurantoína 100mg / d ía como profilaxis por 3 meses
e) Nitrofurantoina 100 mg cada 6 horas como tratamiento por 3 días
79.El fármaco que produce fibrosis pulmonar es:
a) Bleomicina Busulfan
b) Methotrexate
c) Ciclofosfamida
d) Todas las alternativas son correctas.
80.¿Cuál de las siguientes drogas en el tratamiento específico de actúa inhibiendo a la ARN polimerasa?
a) Rifampicina.
b) Etionamida
c) Estreptomicina
d) Isoniazida
e) Etambutol
81.Con respecto al tiempo de sangría marque lo correcto:
a) Evalúa los factores de coagulación
b) En pacientes hemofílicos se encuentra prolongado
c) En trombocitopenias severas está prolongado
d) En los defectos de agregación plaquetaria, se encuentra en valores normales
e) En pacientes anticoagulados el tiempo de sangría se encuentra prolongado
82.Con respecto al estudio del L.C.R:
a) La celularidad normal es menor a 05 leucocitos por ml
b) Normalmente encontramos predominancia mononuclear
c) En las meningitis asépticas puede existir hipoglucorraquia
d) Un valor de más de 200 mg/dl de proteínas me sugieren una meningitis séptica
e) En enfermedad hemorrágica meníngea no se encuentra aumento de proteínas en el L.C.R
83. Adolescente de 15 años con diagnóstico de depresión severa e intento de suicidio.
¿Cuál es el tratamiento inicial?
a) Lamotrigina
b) Haloperidol
c) Clorpromazina
d) Sertralina
e) Amitriptilina
84.Hijo pretérmino de madre RPM con signos de obstrucción y sangrado al defecar ¿Cuál es el diagnóstico más probable?
a) Neumoperitoneo
b) Neumoretroperitoneo
c) Enterocolitis necrotizante
d) Trombosis mesentérica
e) Aire en la vena porta
85. Niño de 3 años que vive en una institución para menores abandonados y al que se le ha diagnosticado MEC por gérmenes Gram negativos. Usted le in dicaría
tratamiento ATB durante:
a) Dos semanas como máximo
b) Una semana es lo ideal.
c) 10 días
d) Seis semanas
e) Tres semanas mínimo
86. El tratamiento de mantenimiento de las convulsiones febriles es:
a) Fenobarbital por 2 años.
b) Ácido valproico por 6 meses
c) No se debe dar tratamiento generalmente
d) Fenobarbital + diazepan de acuerdo a crisis convulsiva
e) Carbamazepina + ácido valpróico por dos años
87.¿Cuál es la causa más frecuente de hipertiroidismo?
a) Adenoma tóxico
b) Bocio multinodular tóxico
c) Tiroiditis de Hashimoto
d) Tiroiditis sugaguda de De Quervain
e) Enfermedad de Graves Basedow
88.¿Por dónde pasa el impulso al ventrículo?
a) Has de Bachman
b) Has de Wenckebach
c) Has de Thorel
d) Has de Hiss
e) Fibras de Purkinje
89.Sangrado rojo vinoso en tercer trimestre, escaso, con contractilidad uterina frecuente
¿Cuál es el diagnóstico probable?
a)Rotura uterina
b)Placenta previa
c)Rotura de vasa previa
d)Desprendimiento prematuro de placenta
e)Placenta de inserción baja
90.En un hijo de madre con rubeola ¿Cuál es la cardiopatía congénita más frecuente?
a) CIV
b) CIA
c) PCA
d) Transposición de grandes vasos
e) Tetralogía de Fallot
91.Gestante multípara, con embarazo doble de 36 semanas quien llega en periodo expulsivo. Luego de atender el parto en cefálica del primer producto se evidencia
que el segundo se halla en situación transversa con el dorso superior. ¿Cuál es la actitud más apropiada?
a) Versión externa
b) Cesárea de emergencia
c) Conducta expectante
d) Tracción cefálica
e) Versión interna
92.Una mujer de 63 años presenta angina de esfuerzo y dos episodios de síncope en el pasado. En la ex ploración se detecta un soplo sistólico con irradiación a las
carótidas y un S2 suave, ¿Cuál de los siguientes diagnósticos es el más probable?

a) Estenosis mitral
b)Insuficiencia mitral
c)Estenosis aórtica
d)Insuficiencia aórtica
e)Estenosis tricúspidea
93.Una niña de 4 años es llevada al médico por un cuadro de inquietud, intenso prurito anal nocturno y discreto prurito vulvar. ¿Cuál es el diagnóstico?
A. Teniasis.
B. Oxiuriasis.
C. Ascariasis.
D. Trichiuriasis.
E. Eccema del pañal.
94.Una niña de 6 meses de edad es traída donde su pediatra por presentar incremento en la frecuencia de las deposiciones durante las últimas d os semanas. Sus heces
son olorosas, “sueltas”, y flotan en el agua. Su madre refiere que después de darle de comer elimina flatos en exceso. Tiene como antecedente un episodio de
obstrucción meconial a las 24 horas de nacida. Su alimentación actual consiste en leche en fórmula y ocasionalmente arroz. El examen físico revela un peso actual por
debajo del percentil 5 para la edad. El examen de Thevennon en heces y cultivos resultan negativos. ¿Cuál de los siguientes exámenes confirmaría el diagnóstico?
A. Biopsia guiada por endoscopía del intestino delgado
B. Análisis cualitativo de grasas en heces
C. Test de cloruro en sudor
D. Ecografía de páncreas
E. Radiografía de abdomen y pelvis con contraste
95.En un niño de 8 años previamente sano, la causa más frecuente de meningitis bacteriana es:
A. H. influenzae.
B. S. pneumoniae.
C. N. meningitidis.
D. S. aureus.
E. Estreptococos del grupo B.
96.Un paciente de 32 años infectado por VIH consulta por astenia y fiebre ocasional, de unos 2 meses de evolución. En la exploración destaca: palidez cutánea,
ausencia de adenopatías periféricas, esplenomegalia 5 cm por debajo del re- borde costal. En la analítica destaca: hemoglobina 6,5 g/dl, volumen corpuscular medio
normal, leucocitos 1.300/ml, plaquetas 40.000/ml, CD4 300/ml. En la TAC de abdomen presenta esplenomegalia homogénea, sin adenopatías. De los siguientes
procesos, ¿cuál le parece más probable?
A. Infección por M. avium-intracellulare.
B. Infección por Citomegalovirus.
C. Infección por Parvovirus B19.
D. Infección por Leishmania.
E. Toxoplasmosis diseminada.
97.Un hombre de 30 años sano se realiza una radiografía de tórax para formalizar un contrato laboral en una empresa. En la radiografía se observa un patrón
intersticial bilateral de tipo reticular, adenopatías hiliares bilaterales y medias- tínicas. Se indica una broncofibroscopia con lavado broncoalveolar que en el recuento
celular muestra los siguientes resultados: linfocitos 50%, histiocitos 40%, eosinófilos 2%, polimorfonucleares 8% y el cociente CD4/CD8 5 ¿Cuál es el diagnóstico más
probable?
A. Alveolitis alérgica extrínseca.
B. Neumoconiosis.
C. Sarcoidosis.
D. Linfangitis carcinomatosa.
E. Hemosiderosis pulmonar idiopática.
98.¿Cuál es una característica del Haemophilus influenzae?
A. La inmunización no ha reducido la incidencia de infecciones por ese agente.
B. La máxima incidencia se observa en niños mayores de 5 años.
C. Los gérmenes más virulentos no pertenecen al serotipo B.
D. Forma parte de la flora respiratoria normal en el 60-90% de niños sanos.
E. Es un coco bacilo grampositivo.
99.Un hombre de 40 años, turista extranjero, es remitido a un centro de Urgencias por bajo nivel de conciencia, tempe- ratura de 37,7 °C y disnea. No tiene
traumatismos. Se realizan varias pruebas. La más inmediata es una gasometría arterial sin oxigenoterapia que muestra pH 7,33, PCO2 50 mmHg, PO2 65 mmHg, HCO3-
27mEq/l. Señale el diag- nóstico más probable entre los siguientes:
A. Tromboembolismo pulmonar.
B. Neumonía lobar.
C. Edema agudo de pulmón.
D. Hipoventilación alveolar.
E. Bronconeumonía.
100¿Cuál es el antibiótico de elección para el tratamiento de neumonía por Chlamydia pneumoniae en menores de 4 años?
A. Amoxicilina + ácido clavulánico.
B. Ceftriaxona.
C. Eritromicina.
D. Rifampicina.
E. Tetraciclina.
101. Todos los siguientes fenómenos se asocian a taponamiento cardíaco, EXCEPTO:
A. Pulso paradójico.
B. Presión venosa disminuida.
C. Presión diferencial reducida.
D. Gasto cardíaco disminuido.
E. Taquicardia sinusal.
102.Paciente de 48 años, con hipertensión pulmonar severa y cianosis, a quien en su infancia se le diagnosticó comunica- ción interauricular. La posibilidad diagnóstica
es:
A. Enfermedad de Lutembacher.
B. Hipertensión pulmonar primaria.
C. Síndrome de Eisenmeger.
D. Tetralogía de Fallot.
E. Malformación de Ebstein.
103.Una paciente de 35 años refiere presentar disnea en los últimos 10 días. A su llegada a urgencias se observa crepitantes basales y ECG con taquicardia irregular de
QRS estrecho, sin que se observen ondas P. ¿Cuál de las siguientes es la actitud más correcta?
A. Cardioversión eléctrica sincronizada.
B. Cardioversión eléctrica no sincronizada.
C. Digoxina y diuréticos, anticoagulación durante 4 semanas y cardioversión posterior.
D. Digoxina y diuréticos, anticoagulación durante 4 días y cardioversión posterior.
E. Digoxina y diuréticos, antiagregación durante 4 semanas y cardioversión posterior.
104.¿Cuál de los siguientes antiulcerosos produce quelación de otros medicamentos, impidiendo su absorción de forma significativa?
A. Cimetidina.
B. Ratinidina.
C. Misoprostol.
D. Tetraciclina.
E. Sucralfato.
105.Un paciente de 78 años de edad, previamente sano, que vive en una residencia de ancianos bastante masificada y con insuficientes recursos higiénicos, padece un
cuadro diarreico desde hace 6 semanas. Refiere molestias abdominales tipo retortijón, febrícula ocasional y 4 -6 deposiciones diarias, alguna de ellas nocturna, con
mucosidad y, en ocasiones, con hebras de sangre. Entre los diagnósticos que se enumeran a continuación seleccione el que le p arece MENOS probable:
A. Cáncer de colon.
B. Infección por Clostridium difficile.
C. Enfermedad de Crohn.
D. Colitis isquémica.
E. Salmonelosis.

106.¿Cuál de los siguientes procedimientos, es el menos indicado para el tratamiento de la úlcera péptica?
A. Régimen lácteo.
B. Anticolinérgicos.
C. Antiácidos.
D. Bloqueadores H2.
E. Tranquilizantes.
107.La tomografía cerebral en los infartos cerebrales isquémicos:
A. Muestra típicamente área hipodensa rodeada de edema.
B. Muestra área hiperdensa que capta contraste.
C. Puede ser normal en la fase inicial.
D. Distingue bien la zona de infarto de la de edema.
E. Sólo distingue bien los infartos cerebelosos.
108.Gestante en la décimo cuarta semana, desde hace semanas presenta náuseas y vómitos persistentes sin respuesta al dimenhid rinato, ha recibido en los últimos 15
días sueros glucosados por vía IV. Actualmente presenta edema de miembros inferiores, taquicardia, diplopía, dificultad para la marcha, nistagmo y alteraciones
mentales. PA: 100/70; pulso: 115 lpm. Análisis: HB: 9 g%, Na: 130 mEq/L, bilirrubina total: 2 mg%, bilirrubina directa: 1,4 m g%. ¿Cuál es la posibilidad diagnóstica?
A. Encefalopatía hepática.
B. Pre – eclampsia severa.
C. Síndrome de HELLP.
D. Hipertiroidismo de la gestante.
E. Encefalopatía de Wernicke.
109.La cefalea en racimos se caracteriza por lo siguiente:
A. El alcohol puede desencadenar los ataques ya sea en el período de cefalea o intercrítica.
B. Los ataques duran típicamente 8 – 10 horas y luego remiten rápidamente.
C. Puede haber síndrome de Horner ipsilateral.
D. Es más frecuente en el sexo femenino.
E. Todo lo anterior.
110.Un paciente diabético de 60 años consulta por primera vez respecto al tratamiento de su enfermedad, ¿qué objetivo a
alcanzar de los siguientes le recomendaría en primer lugar?
A. Mantener la tensión arterial por debajo de 110/70 mmHg.
B. Abandono del hábito tabáquico.
C. Mantener un índice de masa corporal (IMC) menor de 21.
D. Realizarse glucemia capilar basal a diario.
E. Evitar las grasas animales en la dieta.
111.En una mujer de 55 años intervenida de cáncer de mama tres años antes, con buen estado general, se comprueba una hipercalcemia de 11,1 mg/dl. ¿Cuál es la
primera prueba a realizar?
A. Determinación de PTH.
B. Determinación de 1,25(OH)2D.
C. Determinación de 25OHD.
D. Determinación de péptido relacionado con la PTH (PRPTH).
E. Gammagrafía ósea.
112.Una niña de 11 años está hospitalizada debido al aumento de nerviosismo y palpitaciones; los tranquilizantes no la han aliviado. El apetito ha sido bueno pero
registró pérdida de peso. El aprovechamiento escolar ha ido deteriorándose gradualmente. La niña fue enviada por el profesor a un tutor guía que sugirió ayuda
psiquiátrica. Al examen se en- contró un pulso amplio, la temperatura de la piel estaba aumentada, transpiración excesiva y ROT rápidos. Los ojos presentaban una
apariencia vidriosa. El diagnóstico más probable es:
A. Tirotoxicosis juvenil.
B. Psicosis juvenil.
C. Diabetes mellitus.
D. Disautonomía familiar.
E. Síndrome de Cushing.
113.¿Cuál de los siguientes trastornos no produce vasculitis de pequeños vasos?
A. Poliangeítis microscópica
B. Púrpura de Henoch-Schonlein
C. Síndrome de Churg Strauss
D. Enfermedad de Kawasaki
E. Crioglobulinemia mixta esencial
114.En el lupus cutáneo subagudo es característica de la presencia de anticuerpos:
A. Anti-Sm.
B. Anticentrómero.
C. Anti-Ro.
D. Anti-Jo1.
E. Antihistona.
115.La segunda generación de antidepresivos incluye a todos los siguientes, EXCEPTO:
A. Amoxapina.
B. Traxodona.
C. Maprotilina.
D. Amitriptilina.
E. Fluoxetina.
116.Señalar en cuál de los siguientes trastornos de la personalidad es más IMPROBABLE la aparición de sintomatología alucinatoria:
A. Trastorno límite.
B. Trastorno esquizotípico.
C. Trastorno obsesivo-compulsivo.
D. Trastorno esquizoide.
E. Trastorno paranoide.
117.¿Qué es cierto acerca de la oliguria?
A. Se define como un volumen urinario inferior a 1.000 ml/día.
B. Es un hallazgo constante en la insuficiencia renal aguda.
C. Se asocia a dolor lumbar bilateral.
D. Hace más difícil el manejo conservador del fracaso renal agudo.
E. Mejora el pronóstico del fracaso renal agudo.
118.Mujer de 28 años de edad, consulta por hemoptisis recurrente, disnea, anemia, presión arterial: 120/85 mmHg, res- piraciones 28 x’. Examen de orina: hematuria
microscópica con cilindros hemáticos; creatinina: 3,2 mg%, urea: 65 mg%; Hb: 6 g%; anticuerpos antinucleares: negativo; anticuerpos anticitoplasmáticos ANCA-C + 52
U. Radiografía de pulmones: lesión homogénea 1/3 medio del hemitórax izquierdo. ¿Cuál es el diagnóstico más probable?
A. Granulomatosis de Wegener.
B. Lupus eritematoso sistémico.
C. Tuberculosis pulmonar y renal.
D. Síndrome de Goodpasture.
E. Aspergilosis pulmonar.
119.La aparición de un pliegue extra de piel por debajo del párpado inferior es una característica de:
A. Dermatitis seborreica
B. Rosácea.
C. Pitiriasis rosada.
D. Dermatitis atópica.
E. Eccema de contacto.
F. Enfermedad injerto contra huésped.
120.En Querion el tratamiento es:
A. Antihistamínicos
B. Penicilina.
C. Clotrimazol
D. Griseofulvina.
E. Anfotericina B.
121.Durante qué mes de gestación comienza la hemopoyesis en la médula ósea:
A. Tercero.
B. Quinto.
C. Sexto.
D. Séptimo.
E. Octavo.

122.El linfoma de Burkitt está asociado a la infección de los linfocitos B con el:
A. Citomegalovirus.
B. Virus delta.
C. Virus de Epstein Barr.
D. Papilomavirus.
E. Virus del polioma.
123. La histamina se encuentra en los: a. Mastocitos. b. Eosinófilos. c. Leucocitos. d. Neutrófilos. e. Linfocitos

124.Actualmente, el tratamiento de elección del carcinoma epidermoide del canal anal es:
A. Ablación local.
B. Radioterapia sola.
C. Radioterapia y quimioterapia combinadas.
D. Resección abdómino perineal.
E. Resección anterior baja.
125.En el tratamiento con inyección para esclerosar las hemorroides, es importante inyectar:
A. En el lumen de la vena varicosa.
B. En la unión mucocutánea.
C. En la muscularis del intestino.
D. En todas estas.
E. En ninguna de las anteriores.
126.¿Cuál de los siguiente GLUT'S participa en el transporte de fructuosa ?: a. GLUT 1. b. GLUT 5. c. GLUT 2. d. GLUT 4. e. GLUT 3

127. Cuál es el electrolito encargado de la fase de repolarización ventricular en el corazón:


a. Sodio. b. Potasio. c. Cloro. d. Calcio. e. Magnesio. 39.

128.El tratamiento paliativo más adecuado en caso de cáncer de cabeza de páncreas es:
A. Quimioterapia.
B. Operación de Whipple.
C. Derivación biliodigestiva sola.
D. Derivación biliodigestiva más gastroyeyunoanastomosis.
E. Radioterapia.
129.En las próximas 30 horas después de una laparotomía puede haber fiebre de causas infecciosas importantes, EXCEP - TO:
A. Lesión intestinal con filtración de líquidO intestinal.
B. Infección de tejidos blandos por Clostridium perfringes.
C. Infección de tejidos blandos por Gram negativos.
D. Infección de tejidos blandos por Streptococo B-hemolítico.
E. Infección de tejidos blandos por Klebsiella.
130. Una herida traumática ocurrida hace más de 4-6 horas se considera que está:
A. Contaminada.
B. Limpia.
C. Sucia.
D. Supurada.
E. Infectada.
131.¿Cuál es el electrolito que más se altera en las primeras 48 horas post-quemadura?
A. Sodio.
B. Cloro.
C. Zinc.
D. Magnesio.
E. Todos.
132.La hernioplastía de tipo Liechtenstein se caracteriza por el uso de:
A. Malla de polipropileno.
B. Incisiones relajantes.
C. Tapón de polipropileno en anillo inguinal.
D. Doble sutura a surget en la fascia transversalis.
E. Técnica laparoscópica.
133.En la Fase I del ATLS (Advanced Trauma Life Support), debe realizarse todo, excepto:
A. Control de la columna cervical.
B. Respiración y ventilación.
C. Inserción de sonda nasogástrica.
D. Exponer desnudo al paciente.
E. Control de la hemorragia externa.
134.¿Cuál de las siguientes suturas es de material absorbible?.
A. Seda.
B. Poliéster.
C. Nylon.
D. Polipropileno.
E. Ácido poliglicolico
135.En la fractura hepática con hemoperitoneo ¿qué estructura se tiene como referencia, para clampar y cohibir el san- grado?
A. El ligamento de Treitz.
B. La vía biliar principal.
C. La vena cava inferior.
D. El ligamento colecistoduodenal.
E. Epiplón gastrohepático
136.El dolor abdominal postprandial intenso a repetición en un paciente anciano con varios factores de riesgo cardiovas- cular, nos debe hacer sospechar:
A. Embolia mesentérica.
B. Isquemia mesentérica crónica.
C. Colitis isquémica.
D. Rotura de aneurisma abdominal..
E. Aneurisma esplácnico.
137.¿Cuáles son las dos causas MÁS comunes de hemorragia grave del colon?
A. Diverticulosis y angiodisplasia.
B. Enfermedad hemorroidaria y poliposis.
C. Adenocarcinoma de colon y colitis ulcerativa.
D. Vólvulo del colon y amebiasis.
E. Colitis granulomatosa y prolapso rectal.
138.Mejor operación (con menor índice de recurrencia) en paciente joven con enfermedad acidopéptica:
A. Gastrectomía total.
B. Gastrectomía subtotal Billroth-II.
C. Vagotomía supraselectiva.
D. Vagotomía troncular y piloroplastia.
E. Vagotomía troncular y antrectomía.
139.Mujer de 20 años de edad, G2 P2002. Último parto por cesárea, hace 3 meses lactando. ¿Cuál es el anticonceptivo
hormonal MÁS adecuado?
A. Estrógeno oral.
B. Estrógeno más progesterona por vía oral.
C. Progestágenos orales.
D. Parche de etinilestradiol y progesterona.
E. Inyectable mensual de estrógeno y progesterona.
140.Paciente con prurito vulvar, que al examen ginecológico se encuentra secreción blanquecina maloliente. Cuello uteri- no con punteado rojizo como fresas. La
infección vaginal sería causada por:

A. Cándida albicans.
B. Trichomonas vaginalis.
C. Gardnerella vaginalis.
D. Flora mixta.
E. Chlamydia trachomatis.
141.En una gestante con condiloma genital de gran volumen, el mejor tratamiento es:
A. Podofilina local.
B. Láser de dióxido de carbono.
C. Ácido tricloroacético local.
D. 6-fluorouracilo local.
E. Interferon sistémico.
142.Paciente a quien se realizó un cono frío, cuya anatomía patológica reporta carcinoma epidermoide invasor con menos de 5 mm de profundidad en el estroma.
Según la clasificación de la FIGO, corresponde al estudio:
A. I A.
B. I B.
C. II A.
D. II B.
E. N.A.
143.Mujer de 60 años de edad. Presenta pérdida de orina al esfuerzo físico sin poder controlarlo. Al examen se corrobora Con la maniobra de VaIsaIva. ¿Cuál es el
diagnóstico MÁS probable?
A. Incontinencia urinaria de esfuerzo.
B. Incontinencia urinaria de urgencia.
C. Desgarro perineal antiguo.
D. Prolapso uretral.
E. Incontinencia urinaria por rebosamiento.

144.El tratamiento MÁS apropiado en caso de carcinoma endometrial estadío I comprende:


A. Histerectomía total abdominal.
B. Salpingooforectomía bilateral.
C. Estadiaje.
D. Sólo A y B son correctas.
E. Todas las respuestas son correctas.
145.¿Cuál de los siguientes signos NO suele aparecer en la clínica del cáncer de mama?
A. Mastodinia.
B. Telorragia.
C. Tumoración.
D. Retracción del pezón.
E. Adenopatía axilar.
146.La modificación endocrina que aparece de modo MÁS PRECOZ en la menopausia es:
A. Aumento de FSH.
B. Disminución de FSH.
C. Aumento de LH.
D. Disminución de LH.
E. Aumento de estrógenos.
147.La cianosis de la mucosa vaginal y del cérvix es un signo presuntivo de embarazo y se le denomina signo de:
A. Godell
B. Hegar
C. Noble Budin

D. Chadwick
E. Mac Donald
148.¿Qué tiempo después de la enfermedad trofoblástica molar (mola hidatiforme) generalmente hace su aparición la enfermedad trofoblástica maligna?
A. 4 a 6 años.
B. 5 a 10 años.
C. 36 meses.
D. 2 a 28 meses.
E. 6 a 24 meses.
149.En la placenta acreta hay un defecto en la decidua que consiste en la AUSENCIA de:
A. Las Estrías de Rohr.
B. La zona esponjosa.
C. La zona compacta.
D. La zona basal.
E. El estrato de Nitabuch.
150.El embarazo gemelar que se produce en el primer día de la división celular del huevo, tiene las siguientes caracterís- ticas:
A. Monoplacentario y biamniótico.
B. Monoplacentario y monocoriónico.
C. Monocoriónico y monoamniótico.
D. Monocoriónico y biamniótico.
E. Bicoriónico y biamniótico
151.En el trabajo de parto normal, cuando la flexión se completa, el diámetro de la cabeza fetal que ingresa en el estrecho superiorde la pelvis es:
A. Occipitofrontal.
B. Suboccipitobregmático.
C. Occipitomentoniano.
D. Biparietal.
E. Bitemporal.
152.Con respecto al uso del sulfato de magnesio, señale lo INCORRECTO:
A. La dosis de carga es de 4g /EV.
B. La dosis de mantenimiento es de 1 - 2 g /EV por hora.
C. No es necesario la colocación de sonda vesical, salvo cuando se maneja convulsiones.
D. Solo debe ser usado hasta 24 horas post parto con el fin de prevenir las convulsiones.
E. La arreflexia patelar y la depresión respiratoria son signos de intoxicación por sulfato de magnesio.
153.La complicación MÁS SEVERA de la hiperemesis gravídica es:
A. Deshidratación.
B. Hipotensión.
C. Encefalopatía de Wemicke.
D. Hiperkalemia.
E. Hipematremia.
154.Gestante de 22 años, con 18 semanas por FUR, grupo sanguíneo A, factor Rh negativo. Antecedente de un p arto eu- tócico, donde recibió inmunoglobulina Anti D.
el embarazo actual es de segundo compromiso. ¿Cuál es la conducta adecuada?
A. Esperar porque no hay problema.
B. Inmunoglobulina anti D a las 26 y 32 semanas.
C. Inmunoglobulina Anti D dosis única.
D. Test de Liley.
E. Tipificación sanguínea de la pareja.
155. En el perfil biofísico fetal se estudia lo siguiente, EXCEPTO:
A. Situación y posición fetal.
B. Movimientos fetales y respiratorios.
C. Frecuencia y reactividad cardíaca fetal.
D. Volumen del líquido amniótico.
E. Tono fetal.
156. En relación a la valoración Apgar, señale lo CORRECTO:
A. Se toma desde el momento que sale completamente el RN del canal del parto
B. No necesariamente sirve como guía para la reanimación del RN.
C. Un puntaje de 7 nos da un buen pronóstico neurológico a largo plazo.
D. En un recién nacido grave debe evaluarse básicamente el tono muscular.
E. Existe limitación de aplicación en el RN prematuro.
157.El hijo de madre diabética puede presentar los siguientes problemas, EXCEPTO:
A. Prematuridad.
B. Macrosomía.
C. SDR I.
D. Trombosis de la vena renal.
E. Mesomelia.
158. La forma neurotóxica de la bilirrubina en el recién nacido es la:
A. Bilirrubina no conjugada o indirecta.
B. Bilirrubina conjugada o directa.
C. Urobilinógeno.
D. Biliverdina.
E. Urobilina.
159.¿Cuál de los siguientes NO es parte del síndrome de rubéola congénita?
A. Coriorretinitis.
B. Microftalmia.
C. Sordera.
D. Conducto arterioso persistente.
E. Hepatoesplenomegalia.
160.El feto postmaduro típicamente presenta todo lo siguiente, EXCEPTO:
A. Piel suelta.

B. Uñas largas.
C. Caída del pelo.
D. Disminución del vérmix.
E. Ninguna de las anteriores.
161.Una madre se queja de que su niño de 3 meses vomita frecuentemente, el control del peso del niño durante el período de 1 1/2 meses, revela que éste se
mantiene dentro del percentil 70, ¿qué actitud tomaría usted?
A. Solicita Rx del tracto digestivo alto.
B. Solicita estudio de acidez gástrica.
C. Cambia la fórmula láctea.
D. Agrega cereales a su alimentación.
E. Ninguna de las anteriores.
162.¿Cuál de los siguientes NO ha sido implicado como agente causal en caso de una enfermedad parecida al coqueluche?
A. Bordetella pertusis.
B. Echovirus tipo 21.
C. Bordetella parapertussis.
D. Bordetella bronchiseptica.
E. Especies de Adenovirus.
163.En el primer trimestre de vida extrauterina, el crecimiento en los niños se caracteriza por, EXCEPTO:
A. Ganancia de peso diario de 30g.
B. Crecimiento en talla de 3.5 cm/mes.
C. Crecimiento de perímetro cefálico: 2 cm/mes.
D. Requerimiento calórico de 80 kcal/g/d.
E. B y D.
164.Los senos paranasales que se encuentran desarrollados al nacer son:
A. Maxilares y etmoidales.
B. esfenoidales y maxilares.
C. Frontales y etmoidales.
D. Esfenoidales y etmoidales.
E. Frontales y maxilares.
165.¿Cuál de las siguientes expresiones no es correcta respecto a la laringotraqueobronquitis?
A. Es la forma más común de obstrucción de las vías aéreas superiores.
B. El 75% de los casos es producida por los virus parainfluenza.
C. La budesonida inhalada es útil en el tratamiento.
D. El uso de antibióticos previene la infección bacteriana.
E. La dexametasona disminuye el edema inflamatorio.
166. Niño de 15 años con pligue epicanto, nariz en silla de montar, boca pequeña y lengua grande. ¿Cuál es el posible diagnostico que presenta?: a. Trisomía 21. b.
Síndrome de Turner. c. Síndrome de Klinefelter. d. Trisomía 16. e. Trisomía 18.
167. ¿Qué exceso de vitamina produce hipertensión endocraneal benigna?: a. A. b. D. c. B. d. C. e. K
. La glucosa durante la glucólisis en condiciones de alta tensión de oxígeno produce: a. Lactato b. Malato c. Acetato d . Piruvato e. Succinilcolina

168.¿Cuál de las siguientes manifestaciones se presenta en la hipoglicemia neonatal?


A. Hiperreflexia.
B. Distensión abdominal.
C. Fontanela tensa.
D. Hipertonía.
E. Temblores.
169. El petit mal del niño se caracteriza por lo siguiente, EXCEPTO:
A. Puede observarse aura.
B. Por lo general las crisis duran menos de 15 segundos.
C. Las crisis no alteran la postura del niño.
D. Puede observarse contracciones de los miembros superiores.
E. La conciencia se recupera rápidamente.
170.Niño de 2 meses de edad, es llevado a Emergencia por presentar convulsiones tónico-clónicas generalizadas desde hace 45 minutos, sin recuperar la conciencia.
Antecedente de asfixia al nacer. Presenta fiebre de 39ºC. ¿Cuál es el diagnós- tico más probable?
A. Crisis convulsiva
B. Convulsión febril compleja
C. Estatus convulsivo
D. Convulsión febril simple
E. Epilepsia
171. Un niño de 10 años presenta anemia, hemorragias pulmonares y hepatoesplenomegalia, En la placa de tórax se obser - va una masa mediastínica anterior,
¿Qué proceso descartaría en primer lugar?
A. Mononucleosis infecciosa
B. Leucemia linfoide crónica
C. Leucemia linfoblástica aguda tipo T
D. Leucemia linfoblástica aguda tipo B
E. Leucemia M2
172. Un niño de 6 años acude a consulta por un cuadro de febrícula de 3 días de evolución, con dolor a la deglución. Los datos más relavantes de la exploración
física son lesiones erosivas en el paladar y vesículas intraepidérmicas no agru- padas en palmas y plantas. Entre los siguientes diagnósticos, ¿cuál es el más probable?
A. Eritema multiforme.
B. Rickettsiosis
C. Síndrome de Steven-Jonhson.
D. Enfermedad de pie, mano, boca
E. Deshidrosis.
173.El enteropatógeno parasitario que con más frecuencia causa diarrea con malabsorción intestinal en niños menores de 5 años y en desnutridos es:
A. Isospora belli
B. Esfamoeba histolytica
C. Strongyloides stercolaris
D. Ascaris lumbricoides
E. Giardia lamblia
174.Si el valor de una canasta básica familiar en Tarma es de S/. 1500 y el valor de la canasta solo con alimentos es de S/75 0, señale usted en que grupo se encontraría
la familia “X” cuyo ingreso promedio mensual es de S/. 550 mensual:
A. Quintil 4
B. Pobre no extremo
C. Extrema pobreza
D. Pobreza
E. Clase media
175.Son factores condicionantes de la salud pública:
A. Biología
B. Medio ambiente
C. Estilo de vida
D. Sistema sanitario
E. Todas
176.¿Cuál de los siguientes documentos sobre promoción de la salud puso mayor énfasis en la perspectiva sociopolítica?
A. Informe de Lalonde (Canadá, 1974)
B. Declaración de Santa Fe De Bogotá (Colombia, 1992)
C. Declaración de Alma Ata (URSS, 1978)
D. Carta de Ottawa (Canadá, 1986)
E. Declaración de Sundsvall (Suecia, 1991)
177.¿Cuál es el programa social encargado de realizar incentivos monetarios, que promuevan y apoyen el acceso a los
1servicios en educación, salud y nutrición, para las familias muy pobres?
A. Pronaa
B. Cuna más
C. Pensión 65
D. Foncodes
E. Juntos
178.De acuerdo con las normas sectoriales, el establecimiento de Salud que cuenta con servicio de emergencia, sala de operaciones, las cuatro especialidades básicas y
mediana capacidad resolutiva, es categorizado como:
A. I-3
B. I-4
C. II-1
D. II-2
E. III-1
179. La existencia de un servicio de atención al cliente o al usuario es un indicador de:
A. De eficacia
B. De acceso
C. De estructura
D. De proceso
E. De resultado
180.Cual no es documento de gestión de un establecimiento de primer nivel:
A. Manual de organización y funciones
B. Manual de procedimientos
C. Texto único de procedimientos administrativos
D. Guía de procedimientos y trámites
E. Plan operativo anual
181.Marque la respuesta CORRECTA, de acuerdo al tema de SUNASA:
A. Es un órgano consultor adscrito al MINSA con autonomía funcional, administrativa y financiera.
B. Ejerce sus competencias y funciones desde Lima pero a un ámbito nacional, regional y local.
C. Ejerce función conciliatoria y arbitral para tramitar las controversias entre el asegurado y el seguro.
D. Uno de sus objetivos es construir un sistema de aseguramiento público sostenible.
E. Es un órgano ejecutor del Seguro Integral de Salud
182.El coeficiente de correlación de Pearson (r):
A. Varía de -1 a 0.
B. Tiene el mismo signo que la varianza de la cual procede.
C. No sirve para estudiar la relación lineal entre dos variables cuantitativas.
D. Si se eleva al cuadrado, es el coeficiente de determinación.
E. Todas son ciertas.
183. La expectativa de vida al nacer en el Perú actualmente es de:
A. 60,2 años
B. 74,1 años
C. 65,0 años
D. 63,4 años
E. 68,8 años
184.Al tratar una artritis psoriásica con azatioprina, mejora el 60% de los pacientes; la 6 -mercaptopurina mejora a un 55%. La diferencia entre ambos tratamientos es
significativa (p<0,05), lo cual se debe interpretar como una de las siguientes opciones:
A. Con azatioprina mejorarán el 60% de tus pacientes.
B. La azatioprina es en un 95% mejor que la 6-mercaptopurina.
C. El nivel de significación es del 1%.
D. Hay diferencias entre los dos tratamientos, con una probabilidad de que esta afirmación sea equivocada menor al 5%.
E. No se puede concluir nada.
185.Se llama probabilidad condicionada a:
A. La probabilidad de un suceso conocido previamente.
B. La condición que debe cumplir un suceso.
C. La probabilidad que ocurra un suceso habiendo ocurrido otro.
D. La probabilidad de que suceda de que ocurra un suceso u otro.
E. El teorema de Bayes restringido.
186. Varón de 20 años de edad, que presenta convulsiones en el servicio de Emergencia. ¿Cuál de los siguientes fármacos
administraría?
A. Carbamazepina.
B. Diazepam.
C. Metilfenitoína.
D. Valproato.
E. Difenilhidantoína.
187. Entre los efectos colaterales del litio NO se incluye:
A. Aumento de peso.
B. Gastritis.
C. Polidipsia.
D. Leucocitosis.
E. Convulsiones.
188. ¿Cuál es el efecto adverso más frecuente en la administración de Clindamicina?
A. Colitis pseudomembranosa.
B. Nefrotoxicidad.
C. Ototoxicidad.
D. Anemia aplásica.
E. Artropatia.
189. Varón de 7 años, 15 días después de cumplir tratamiento por faringoamigdalitis estreptocócica presenta náuseas, vó - mitos, cólicos abdominales e ictericia. ¿Cuál
de los siguientes fármacos se asocia a estos efectos adversos?
A. Clindamicina fosfato
B. Cefradina
C. Vancomicina
D. Ampicilina
E. Eritromicina estolato
190. ¿Cuál es el hospedero definitivo de Hymenolepis diminuta?
A. Hombre.
B. Cerdo.
C. Vaca.
D. Rata.
E. Venado.
191. ¿Cuál es el tipo de estreptococo que produce estreptolisina O?
A. Faecalis.
B. Bovis.
C. Pyogenes.
D. Agalactie.
E. Pneumoniae.
192.¿Cuál de las siguientes enfermedades es causada por un parásito intracelular?
A. Asepergilosis.
B. Histoplasmosis.
C. Lepra.
D. Malaria.
E. Tuberculosis.
193.En un ensayo clínico se comparan 3 tratamientos (p.e. placebo, tratamiento establecido y un tratamiento nuevo). La variab le respuesta es continua (p.e. nivel de
glucosa en sangre). ¿Si la variable no tiene una distribución normal, el test correcto para comparar la respuesta es?:
A.La t de Student.
B.El test de Wilcoxon.
C.Análisis de la varianza.
D.El test de Krusal-Walls.
E.El test Chi-cuadrado.
194.Si como resultado de un estudio prospectivo en el que se han comparado dos grupos de indiv iduos, unos expuestos y otros no expuestos a un factor de riesgo
causante de una enfermedad. Después de comprobar la comparabilidad de los dos grupos, obtenemos que de los 500 individuos exp uestos al factor de riesgo 50
padecen la enfermedad de estu- dio, y que de los 500 no expuestos sólo la padecen 10. ¿Cuál sería el riesgo de padecer la enfermedad que podríamos atribuir al factor
de riesgo?:
A. 0,08.
B. 0,1.
C. 0,2.
D. 0,5.
E. 10.
195.En el estudio de Framingan (Framingan Heart Study) sobre cardiopatía coronaria, se encontró que en el examen ini- cial 17 personas por cada 1.000 tenían
síntomas evidentes de cardiopatía coronaria. Elija la tasa o razón que mejor describe este hecho:
A.Tasa de incidencia.
B.Tasa de prevalencia.
C.Razón de morbilidad estandarizada.
D.Tasa específica por edad.
E.Tasa de mortalidad ajustada.
196.En un estudio de cohortes diseñado para comprobar la relación tabaco-cáncer de vejiga, se siguieron durante un año a 1.000 personas, 500 fuman y 500 no. De
los 50 cánceres producidos en ese tiempo, 45 aparecieron en fumadores.
¿Cuál es la incidencia en expuestos?:

A. 50/1.000 = 5%.
B. 50/500 = 10%.
C. 45/500 = 9%.
D. 5/500 = 1%.
E. 45/1.000 = 4,5%.
197. Niño de dos meses, sin antecedentes de importancia, presenta rinorrea hialina y fiebre de 38° C en los tres últimos días. Desde hace 24 horas, tiene tos y dificultad
respiratoria progresiva. En las últimas 12 horas rechaza todas las tomas. Ha recibido la primera dosis de DTP acelular, Hemophilus influenzae tipo B, Meningococo C,
Polio oral y dos dosis de Hepatitis B. Examen físico: frec. cardíaca 135 l.p.m., frec.respiratoria 55r.p.m. y Sat O2: 90%. Tiraje intercostal y subcostal, subcrepitantes y
sibilancias bilaterales. ¿Cuál es el diagnóstico más probable?
A.Crisis asmática moderada - severa.
B.Bronquiolitis.
C.Neumonía bacteriana.
D.Tos ferina.
E.Neumonía por Clamidia trachomatis.
198.Paciente de 9 meses que presenta discreta fiebre, adenopatías retroauriculares grandes y dolorosas y además exante- ma maculopapular de color rosáceo en
cuerpo. ¿Cuál es el diagnóstico más probable?
a). Roséola infantil
b).Rubéola
c) Sarampión
d) Mononucleosis infecciosa
e) Varicela
199.Niño de 13 años llevado a la consulta por presentar estornudos, rinorrea acuosa y prurito nasal persistentes. Al exa- men: intenso edema y palidez de la mucosa
nasal, con secreción clara. El diagnóstico probable es:
a).Cuerpo extraño
b) Mastocitosis nasal
c) Rinitis alérgica
d) Rinitis neutrofílica
e) Rinitis vasomotora
200.A los 5 minutos de vida un neonato presenta estos signos: 130 pulsaciones por minuto, manos y pies cianóticos, buen tono muscular, llanto poderoso, reactividad
normal, frecuencia respiratoria 60 por minuto. El índice de Apgar en este niño es:
a) 6
b) 7
c) 8
d) 9
e) 10

EXAMEN N°08

SIMULACRO 11A
1. Cuadro clínico con foco neumónico en radiografía se ve lesión en base de pulmón derecho que borra aurícula derecha ¿qué ló bulo
está afectado?

A. Inferior

B. Apical

C. Medio

D. Lingula

E. Superior

2. Gestante de 40 semanas que acude en periodo expulsivo del trabajo de parto. ¿Cu ál de los movimientos principales del trabajo es
el más precoz?

A. Rotación interna

B. Extensión

C. Flexion

D. Rotación externa

E. Expulsión.

3. ¿Cuál es el volumen de rehidratación que debería recibir un lactante en quien se clasifica como deshidratación severa sin s hock?

A. 200 ml/Kg/día

B. 100 ml/Kg/día

C. 100 ml/SC/día

D. 3500 ml/Kg/día

E. 150 ml/Kg/día

4. Respecto a la enfermedad diverticular, señale lo correcto:

A. La diverticulosis siempre requiere tratamiento

B. La hemorragia diverticular suele ceder espontáneamente

C. Los divertículos son más frecuentes en el color ascendente

D. En el manejo de la diverticulitis siempre se opta por la col ostomía

E. Es más frecuente entre los 30 y 50 años

5. Sobre tratamiento de HTA señale lo falso

A. Paciente con HTA con microalbuminuria es de elección captopril

B. Paciente con HTA e IRC terminal con K < 5.5 indicamos calcioantagonistas

C. Paciente con HTA y antecedente de osteoporosis indicamos tiazidas

D. Paciente con HTA de raza afroamericana indicamos beta block

E. En gestantes con HTA severa con labetalol

6. El agente Gram negativo m ás frecuente de la neumonía asociada a Ventilación es:

A. Escherichia coli

B. Enterococco faecalis

C. Pseudomona aeruginosa

D. Staphylococcus aureus

E. Haemophilus influenzae

7.¿Cuál es la tasa de filtración glomerular compatible con paciente diabético en estadio temprano?

A. 90 ml/min
B. 60 ml/min

C. 150 ml/min

D. 30 ml/min

E. 15 ml/min

8. Mujer de 20 años de edad, peladora de espárragos desde hace 4 meses, presenta lesiones eritematosas pruriginosas
micropapulares localizada en dorso de antebrazos, cara y cuello. Las lesiones también se han identificado en algunas com pañeras de
trabajo. No presenta antecedentes de atopía. Su primera impresión diagnóstica es y el tratamiento es:

A. Acarosis / permetrina

B. Dermatitis por contacto / corticoides

C. Acarosis / griseofulvina

D. Dermatofitosis / clotrimazol

E. Dermatofitosis / itraconazol

9. ¿ Cuál es el antibiótico de elección para la diarrea producida por campylobacter jejuni

A. Azitromicina

B. Dicloxacilina

C. Clindamicina

D. Cotrimoxazol

E. Ciprofloxacino

10. Son criterios mínimos para el diagnóstico de la enfermedad inflamatoria pélvica:

A. Dolor abdominal inferior, VSG elevada examen del flujo vaginal positivo para N. Gonorrhoeae

B. Dolor a la movilización cervical, absceso tuboovarico en ecograf ía, examen del flujo vaginal positivo para C.

Trachomatis

C. Dolor abdominal inferior, dolor a la movilización cervical, dolor anexial bilateral.

D. Dolor anexial bilateral, temperatura oral > 38 0C, flujo vaginal o cervical anormal.

E. Dolor hipogástrico persistente, proteína C reactiva elevada, flujo vaginal anormal.

11. Primigesta en trabajo de parto, 6 cm de dilatación con membranas íntegras prominentes, C –4. Se tacta cordón; con- jugado
diagonal 10,5 cm; ponderado fetal 3.500 g. ¿Cuál es su diagnóstico?:

A. Desproporción céfalo-pélvica por macrosomía fetal.

B. Desproporción céfalo-pélvica y prolapso de cordón.

C. Estrechez pelviana y deflexión II.

D. Estrechez pelviana y procúbito de cordón.

E. Estrechez pelviana y prolapso de cordon

12. Acude a consulta un paciente de 3 años de edad. Mama refiere que presento un cuadro de rinorrea y estornudos leves. Luego
empezó a presentar lesiones tipo maculopapulares que se extendieron en todo su cuerpo al primer dia de exan - tema y que luego
desaparecieron con descamación furfurácea al tercer día de enfermedad. ¿Cuál es el diagnostico?

A. Exantema súbito

B. Roseola

C. Sarampion

D. Rubeola

E. Escarlatina.
13. Paciente joven leptosomico con colelitiasis, adem ás se encuentra el examen físico esplenomegalia, adem ás presente astenia y en
su perfil hepático destaca la elevación de bilirrubina a predominio indirecto., ante la sospecha diagnostica solicitamos

A. Test de HAM

B. Electroforesis de hemoglobina

C. PCR enzimático

D. Test de fragilidad osmótica

E. Gota gruesa

14. En el fenómeno de Raynaud, el paciente puede recibir como terapia farmacológica:

A. Prednisona

B. Isorbide

C. Minoxidil

D. Propanolol

E. Amlodipino

15. Según la clasificación de Johnson de ulcera gástrica señale lo verdadero

A. La tipo II es la más frecuente

B. La tipo III cursa con secreción de ácido disminuido

C. La tipo I se ubica en curvatura mayor

D. La tipo IV cursa con hipersecreción de acido

E. La tipo V se asocia a lesión por AINES

16. En relación al diagnóstico de cardiopatía isquémica señale lo falso

A. En caso de duda diagnostica en la angina estable solicitamos ergometria

B. El estudio de mayor dato pronóstico es la angiografía coronario

C. La forma aguda produce un trombo que causa oclusión coronaria

D. El cuadro crónico se produce una estenosis fija que va progresando en años

E. la oclusión total se manifiesta eléctricamente sin elevación del segmento ST

17. Respecto al cáncer de colon, es incorrecto:

A. La presentación clínica del cáncer de hemicolon derecho es fundamentalmente obstructiva

B. Los adenomas colorrectales se consideran lesiones premalignas

C. El tipo más frecuente es el adenocarcinoma

D. Es más frecuente en mayores de 50 años

E. El consumo de grasas saturadas aumenta el riesgo

18. Paciente adulto con diagnóstico de neumonía por estafiloco co. Recibe tratamiento durante varios días con Vancomici- na, pero no
mejora. ¿Qué antibiótico recomendaría en este paciente?

A. Ceftriaxona

B. Piperacilina tazobactam

C. Linezolid

D. Meropenem

E. Clindamicina

19. ¿Cuál de las siguientes es un tipo de exantema conocido como la quinta enfermedad?

A. Exantema súbito
B. Eritema infeccioso

C. Varicela

D. Sarampion

E. Rubeola.

20. Mujer de 25 años, puérpera de 1 horas de parto vaginal, RN de 4 500 gr, presenta abundante sangrado rojo rutilan - te, se
encuentra pálida, taquicárdica, sudorosa, útero a 3 cm debajo de la cicatriz umbilical. ¿Cu ál es el diagnóstico probable?

A. Atonía uterina

B. Retención de restos placentarios

C. Hipotonía uterina

D. Embolia de líquido amniótico

E. Laceración del tracto genital

21. Es incorrecto respecto a las quemaduras de tercer grado:

A. La superficie es indolora

B. Destruye toda la dermis

C. Tiene un color oscuro

D. La superficie es generalmente seca

E. Presenta ampollas en la superficie

22.¿Cuál de los siguientes enteroparásitos NO transmite por fecalismo?:

A. Lumbricoides

B. Cryptosporidium parvum

C. Entamoeba histolytica

D. Strongyloides stercolaris

E. Trichuris trichiuria

23. Mujer de 64 años, G:1, P:1001, menarquia: 10 años. FUR: 55 años, obesa, diabética e hip ertensa desde hace 10 años. Acude por
ginecorragia. Ecografía: endometrio 20 mm y volumen uterino 110 cc. PAP: AGUS ¿cu ál es el diagnóstico m ás probable?

A. Adenocarcinoma endometroide

B. Carcinoma de células claras

C. Carcinoma indiferenciado

D. Carcinoma mucinoso

E. Carcinoma seroso papilar

24. No es un criterio de causalidad de Hill:

A. Fuerza de asociación

B. Número necesario a tratar

C. Experimentación

D. Consistencia

E. Gradiente biológica

25. ¿Cuál de las siguientes es INCORRECTA respecto a la leche y lactancia materna?

A. La galactosemia es una contraindicación

B. El calostro tiene más contenido de grasas en comparación a la leche madura

C. La leche madura se obtiene a partir del dia 14


D. El contenido de Calcio es menor en la leche humana en comparación a la leche de vaca

E. La hierro de la leche materna es mejor absorbido que el de la leche de vaca.

26. Es característica de la anestesia epidural:

A. Atraviesa la duramadre

B. Siempre ocasiona cefalea post punción

C. Es necesario siempre atravesar el ligamento amarillo

D. Es un tipo de anestesia general

E. Puede causar hipertensión arterial

27. Paciente varón joven con datos de trombosis venosa profunda, adem ás se encuentra ictericia a predominio indirecto con
coluria, la sospecha diagnostica es:

A. Hemoglobinuria paroxística nocturna

B. Esferocitosis

C. Talasemias

D. Hemoglobinopatías

E. Eliptocitosis

28.Mujer de 32 años de edad, con diagnóstico de artritis reumatoide. ¿Cu ál de los siguientes hallazgos indica un buen pronóstico?:

A. Elevación de la velocidad de sedimentación

B. Títulos bajos de factor reumatoide

C. Títulos elevados de haptoglobina

D. Presencia de nódulos subcutáneos

E. Presencia erosiones en radiografía de manos

29.Paciente de 80años de edad, que presenta en ambas extremidades ampollas grandes, tensas, algunas con contenido sanguíneo,
sobre una base urticariana y con moderado prurito. No hay compromiso del estado general y la histopato - logía muestra lesiones
subdermicas con acumulo de eosinofilos en la dermis papil ar. La inmunofluorescencia muestra depósito lineal de IgG y de C3. ¿Cu ál
es la dermatopatia m ás probable?

A. Dermatitis herpetiforme

B. Penfigo cicatricial

C. Penfigo seborreico

D. Penfigo vulgar

E. Penfigoide ampollar

30.Paciente varón de 3 años de edad, quien desde hace aproximadamente 4 días presenta deposiciones liquidas en fre - cuencia de 5
por día sin sangre sin moco. Al examen físico presenta deshidratación de grado moderado. ¿Cu ál es el rango de volumen que debería
recibir dicho paciente?

A. 20-80 ml/Kg/día

B. 30-80 ml/Kg/día

C. 30-80 ml/Kg/4horas

D. 80-100 ml/Kg/3horas

E. 30-50 ml/Kg/12horas.

31.Mencione el dato de mal pronóstico en pancreatitis aguda

A. El índice de severidad tomografía más de 5 puntos

B. Secuestro de líquidos al ingreso

C. Hipocalcemia a las 48 horas


D. Apache más de 7 puntos

E. Disminución de PCR

32. El precursor metabólico inmediato de la noradrenalina es:

A. Isoproterenol.

B. Ibopamina.

C. Dobutamina.

D. Dopamina.

E. Efedrina.

33. Paciente de 68 años con disnea progresiva hasta en reposo. Examen físico: tórax amplio vibraciones vocales dismi- nuidas,
hipersonoridad, murmullo vesicular disminuido, ingurgitación yugular y disnea severa. Cu ál es la actitud inmediata?

A. Tomar radiografía de tórax

B. Evaluación por cirujano de tórax

C. Drenaje torácico inmediato

D. Tomografía de pulmones

E. Antibióticos y corticoides

34.Marque lo falso respecto a los niveles de prevención:

A. La prevención primordial evita la causación primaria de la enfermedad.

B. La prevención primaria busca reducir la incidencia de enfermedad a través de pruebas de screening.

C. La prevención secundaria se sustenta en la utilidad de las pruebas de screening.

D. La prevención terciaria busca recuperar parte de la funcionabilidad previa.

E. La prevención cuaternaria evita el sobrediagnostico y sobretratamiento.

35.Con respecto a la convulsión febril, marque la alternativa incorrecta:

A. La convulsión febril más frecuente es la simple

B. Es el tipo de convulsión febril más frecuente en niños con una frecuen cia de 3-5% en menores de 5 años

C. El que los padres hayan tenido en la niñez no apoya el riesgo de que el niño lo tenga

D. Generalmente se resuelven solas

E. Existe manejo preventivo de convulsión febril

36.¿Cuál es un principal criterio para endometritis?

A. Fiebre en el primer día pureperio

B. Dolor pélvico

C. Secreción vaginal maloliente

D. Sangrado vaginal

E. Fiebre a partir del segundo día puerperio

37. Paciente de 40 años de edad, que acude a la consulta por sangrado postcoital de 2 semanas de evol ución al examen
pélvico:cérvix ulcerado hipertróficos, sangrante, poco móvil. Cuerpo uterino 6cm, anexos no palpables. Al tacto rec - tal: útero duro con
parametrios tomados en sus 2/3 internos. El diagnóstico clínico es:

A. Cáncer de cérvix IB

B. Cáncer de cérvix IIIA

C. Cáncer in situ
D. Cáncer de cérvix IIB

E. Ectropión sangrante

38. Varón de 17 años, con procesos respiratorio viral, posteriormente presenta hematuria con hematíes dismorficos ade - más presenta
disminución de complemento (C3 Y c4) ¿Cuál es su posibilidad diagnostica?

A. Glomerulonefritis por vasculitis

B. Glomerulonefritis postinfecciosa

C. Glomerulonefritis membranoproliferativa

D. Glomerulonefritis rapidament progresiva

E. Nefropatía por IgA .

39.Señora que sufre caída y acude a consulta con miembro inferior en extensión y aducción, a la radiografía no se eviden - cia fractura
¿Cuál es el diagnóstico m ás probable?

A. Luxación pubiana de la cadera

B. Luxación posterior de la cadera

C. Luxación congénita de cadera

D. Luxación obturatriz de cadera

E. Luxofractura de cadera

40.En este estudio de evalúan los efectos adversos a largo plazo y las interacciones medicamentosas en un gran número de
pacientes / es un estudio previa a la comercialización del f ármaco pero que ya recibió la aprobación de la FDA para realizar ensayos
clínicos controlados:

A. Fase III / II

B. Fase V / II

C. Fase IV / III

D. Fase IV / II

E. Fase I / IV

41.Llega a consulta una madre con su hijo de 18 meses de edad quien refiere que solo recibió la primera dosis de la Pen- tavalente.
Con respecto a esta vacuna, ¿Qué le diría usted?

A. Ya no puede vacunarse por el riesgo de ESAVI

B. Debería completar con solo una segunda dosis

C. Completar con dos dosis más separadas por 1 mes

D. Debería reiniciar las tres dosis y darlos cada 2 meses

E. Completar con dos dosis más separadas por 2 meses.

42. En un paciente con glucosa en ayunas de 130 mg/dL y dos exámenes adicionales dudosos, con un test de tolerancia a la glucosa
en limite, debemos solicitar:

A. Curva de insulina

B. Glucosa post prandial

C. Repetir el test de tolerancia a la glucosa 75

D. Hemoglobina A1c

E. Electroforesis de hemoglobina.

43. Respecto a los aneurismas cerebrales, es INCORRECTO:

A. La mayoría corresponden al tipo sacular

B. Es la principal causa de hemorragia subaracnoidea atraumática

C. La principal arteria donde se les puede encontrar es la arteria meníngea anterior


D. La mayoría son de tipo congénito

E. La diplopía y la cefalea son síntomas característicos

44. Una madre con tipo de sangre AB tiene un hijo con tipo AB. A ella le gustaría establecer la paternidad a través del tipo
sanguíneo. ¿Cuál tipo de sangre excluye a un varón de ser el padre biológico?

A. AA.

B. BB.

C. BO.

D. AO.

E. OO.

45.El tipo de medicamento indicado para el tratamiento de la Angina de Prizmetal es:

A. Calcioantagonistas

B. IECA

C. B- Bloqueantes

D. ARA II

E. Digoxina

46. Paciente mujer quien se realiza una colecistectomía laparoscópica electiva por antecedente de litiasis vesicular, acci - dentalmente
se lacera el duodeno, sin perforarlo. El tipo de herida es:

A. Sucia

B. Limpia contaminada

C. Limpia

D. Contaminada

E. Infectada

47.Mujer de 28 años, hace 6 meses presenta malestar general, Poliartritis de mano y disnea. Al examen físico: PA: 125/80 mmHg , FC:
100 x minuto, FR: 28 x minuto. Alopecia, palidez y adenopatías cervicales múltiples. Hemoglobina 7G/ dl, urea 65 mg/dl, creatinina 2.2
mg/dl. Examen de orina: hematuria, cilindros hem áticos y proteínas (+), ANA y anti- Smith (+). Rx de pulmones: lesión homogénea en
el tercio inferior del hemitórax izquierda: ¿Con respecto al diagnós- tico señale lo correcto?:

A. Mediado por una reacción de hipersensibilidad tipo II

B. El rash malar es una manifestación aguda específica

C. Asocia frecuentemente con síndrome de Sjogren

D. Mejora con la gestación

E. Cuando presenta anticuerpo anticardiolipina es buen pronóstico

48. Mujer de 25 años de edad, con gestación de 37 semanas. Acude a Emergencia por presentar dolor abdominal soste - nido, de
inico brusco y de intensidad fuerte, con leve sangrado vaginal. Al examen físico: PA 130/90 mmHg, LCF 160 por minuto. ¿Cu ál es el
diagnóstico más probable?

A. Placenta marginal

B. Ruptura uterina espontánea

C. Ruptura del seno marginal

D. Desprendimiento prematuro de placenta

E. Parto pretérmino

49.El tipo de sangrado m ás característico que producen los leimiomas uterinos:

A. Hipermenorrea
B. Metrorragia

C. Menorragia

D. Menometrorragia

E. Amenorrea

50. ¿Cuál de las siguientes vacunas es de tipo inmunización activa tipo toxoide?

A. Disteferia y Tetanos

B. Sarampion, Paperas y Rubeola

C. Pentavalente

D. Antineumococica

E. BCG.

51.Respecto a las escalas de medicina marque lo falso:

A. Las variables en la escala nominal son excluyentes.

B. La escala ordinal presenta una diferencia de grado entre variables.

C. La escala de intervalo muestra diferencias cuantificables de los valores pero no establece ausencia de fen ómeno.

D. La escala de razón muestra diferencias cuantificables de los valores y establece ausencia de fen ómeno.

E. La escala de intervalo muestra diferencias cuantificables de los valores y establece ausencia de fen ómeno.

52. En un puesto de salud se han producido 2.800 consultas médicas al año y se dispuso para el mismo 400 horas -médico.
¿Cuál es el rendimiento?

A. 9

B. 5

C. 6

D. 8

E. 7

53.¿Cuál es la característica que corresponde al sarcoma de Kaposi?

A. Es una neoplasia maligna de endotelio causada por HVS8

B. Las lesiones son dolorosas y pruriginosas causadas por HVS8

C. Es un cáncer de endotelio por HVS1

D. Es típico de VIH causado por HVS5

E. La etología es el VIH que causa cáncer de endotelio

54. ¿Cuál es la alternativa que NO corresponde a la anemia ferropenia?.

A. La respuesta al tratamiento se mide con reticulocitos al 10 día

B. La causa más frecuente es carencial en jóvenes

C. Para diferenciarla de la anemia por enfermedad crónica se mide transferrina

D. El test bioquímico mas especifico es la ferritina

E. El hierro se absorbe solo el 10% de lo ingerido a nivel intestinal.

55. En el tratamiento de la hiperkalemia ¿Qué medicamento actúa produciendo disminución del potasio corporal total?

A. Gluconato de calcio
B. bicarbonato

C. Dextrosa hipertónica

D. Beta bloqueadores

E. Resina Kayexalate

56. Gestante de 38 años, con diagnóstico de placenta previa. ¿Cu ál es el factor de riesgo m ás asociado y a partir de qué edad
gestacional se diagnostica dicha patología?

A. Cesárea – 28 semanas

B. Parto distocico – 26 semanas

C. Cesárea – 30 semanas

D. Parto distocico – 28 semanas

E. Cesárea – 22 semanas

57. Un neonato presenta ictericia, dificultad respiratoria, Tº inestable, distensión abdominal y hepatomegalia. Diagnósti - co de
presunción:

A. Kernicterus.

B. Meningitis neonatal.

C. Enfermedad granulomatosa crónica.

D. Lupus neonatorum.

E. Sepsis neonatal.

58. ¿Cómo se define talla baja en pediatría?

A. P/T<3p

B. T/E<5p

C. P/T<5p

D. T/E<3p

E. P/E<10p

59. Respecto a los tumores de esófago, marque lo INCORRECTO

A. Los tumores benignos más frecuentes son los leiomiomas

B. La acalasia crónica aumenta el riesgo

C. El consumo de alcohol y tabaco aumenta el riesgo

D. El tumor maligno más frecuente es el adenocarcinoma

E. El cáncer más frecuente es el carcinoma epidermoide.

60. Paciente con ECV con hemiparesia derecha a predominio braquial y afasia de expresión ¿qué arte ria está afectada?

A. Cerebral media rama superior izquierda

B. Cerebral media rama superior derecha

C. Cerebral anterior rama superior izquierda

D. Cerebral anterior rama superior derecha

E. Cerebral media rama inferior izquierda

61. Mide la desviación estándar de una media de todas las muestras posibles de una población infinita y se usa para la inferencia
por:

A. Error estándar de la media – intervalos de confianza

B. Desviación medial de la media – intervalos de confianza


C. Error estándar de la media – prueba de hipotesis

D. Amplitud intercalar - prueba de hipotesis

E. Correlacion lineal – intervalos de confianza

62. El impétigo ampolloso es causado por y debe ser tratado con:

A. Staphylococcus aureus coagulasa positivo / mupirocina

B. Streptococcus pneumoniae / dicloxacilina VO

C. St. Pyogenes / clindamicina VO

D. Staphylococcus aureus coagulasa positivo / dicloxacilina VO

E. Pseudomonas aeruginosa / aztreonam VO

63. Paciente de 30 años de edad, con hemorragia uterina normal y biopsia e ndometrial informada como hiperplasia sim - ple
típica. ¿Cuál es la conducta m ás apropiada a seguir?:

A. Tratamiento cíclico con progesterona

B. Histeroscopia

C. Histerectomía abdominal total

D. Esperar un nuevo control

E. Ablación endometrial

64. En un escolar de 7 años con claudicación de la marcha y dolor en la cadera, usted sospecharía en la enfermedad de:

A. Sever

B. Osgood-Schlatter

C. Khole

D. Legg-Calve-Perther

E. Panner

65. La elaboración del Plan Operativo de un establecimiento de Salud tiene co mo una de sus características que:

A. La misión expresa lo que seremos en el futuro

B. Promueve la participación del equipo de gestión solamente

C. Se programan actividades a lo largo de un año

D. Las fortalezas parten del entorno

E. Busca plantear problem as de la oferta para resolverlos operativamente

66. El antidepresivo con menor efecto sedante y el que tiene mayor efecto sedante son respectivamente:

A. Amitriptilina / sertralina

B. Clomipramina / imipramina

C. Doxepina / bupropion

D. Fluoxetina / paroxetina

E. Trazodona / sertralina

67. ¿Cuál es la frecuencia cardiaca fetal normal?

A. 60 B. 120

C. 120 a 160

D. 80 a 120

E. >180
68. Varón de 40 años que presenta en el brazo derecho lesión m áculo-papulosa de 3 cm, pruriginosa, indolora que progre- sa a
vesícula y luego a costra negruzca rodeada de edema. Ocupación crianza de ganado lanar y vacuno. Niega fiebre. Examen: Pulso:
115 x minuto, FR: 24 x minuto, PA: 100/60 mmHg. ¿Cuál es la presunción diagnóstica y el manejo adecuado?

A. Carbunco / doxiciclina

B. Leishmaniasis / nifurtimox

C. Lactrodactismo / suero especifico

D. Leishmaniasis / antimonio pentavalente

E. Carbunco / ciprofloxacino

69. La insuficiencia adrenal primaria se denomina y es causada:

A. Síndrome de Cushing / falla de regulación central

B. Enfermedad de Addison / falla de la glándula

C. Síndrome de Addison / falla de glándula

D. Enfermedad de Addison / falla de regulación central

E. Síndrome de Addison / falla de regulación central.

70. El test de Apgar valora el estado vital del neonato mediante la comprobación de los siguientes signos, EXCEPTO:

A. Frecuencia cardiaca.

B. Movimientos respiratorios.

C. Respuesta a estímulos.

D. Reactividad osteotendinosa.

E. Coloración de la piel.

71. Respecto a las hernias inguinales marque lo incorrecto

A. Las hernias inguinales directas salen mediales a los vasos epigástricos inferiores

B. Para el tratamiento de las hernias inguinales directas se prefieren las técnicas sin tensi ón

C. Las hernias indirectas ocurren por persistencia del conducto periton eo vaginal

D. Las hernias directas son generalmente congénitas

E. Las hernias indirectas recorren el canal inguinal

72. Qué diámetro es el de mayor medida en el feto:

A. Suboccipitobregmatico

B. Suboccipitomentoniano

C. Occipitofrontal

D. Bitrocantereo

E. Bi acromial

73. Agente etiológico del Chancroide y de la cervicitis respectivamente:

A. Haemophilus vaginalis / clamidia trachomatis

B. Calymmabacterium Granulomatosis / gonococo

C. Treponema pallidum / H. ducreyi

D. Chlamydia trachomatis / H. ducreyi

E. Hemophilus ducrey / C. trachomatis

74. El síndrome de Cushing se refiere a:

A. Excesiva producción adrenal de glucocorticoides


B. Al exceso de glucocorticoides producidos

C. Excesiva exposición multiorganica a glucocorticoides

D. Excesiva estimulación de las adrenales por la ACTH

E. Excesiva actividad glucocorticoide

75. El motivo por el cual la sulfonilureas deben ser usadas con cuidado en ancianos y enfermos renales es:

A. Pueden causar acidosis láctica

B. Hipoglicemia

C. Hiperkalemia

D. Agitación

E. Daño hepático

76. Se recibe en Emergencia a un niño de 5 años de edad, febril, con 15 minutos continuos de crisis convulsiva, no ha recu -
perado la conciencia aun y continua convulsionando, la madre del niño describe cuadro compatible con conv ulsiones tónico-clónicas
generalizadas. ¿Cuál es el diagnóstico m ás probable y el manejo inicial m ás adecuado?

A. Estado convulsivo / Diazepam o lorazepam

B. Epilepsia / Diazepam o lorazepam

C. Convulsión febril / fenitoina

D. Estado convulsivo / fenitoina

E. Convulsión tónico –clónica / fenobarbital

77. ¿Cuál es el parámetro más exacto para determinar la edad gestacional?

A. Ecografía previa en el primer trimestre : LF

B. Ecografía previa en el primer trimestre: DBP

C. Fecha de inicio de náuseas y vómitos

D. Ecografía previa en el primer trimestre: LCC

E. FUR confiable

78. Paciente con pancreatitis aguda severa se colicita TAC con contraste donde se informa edema de p áncreas con una
colección de líquido por lo cual según el índice de Baltazar corresponde a:

A. A

B. B

C. C

D. D

E. E

79. ¿Clínica más frecuente de esclerodermia?

A. Fenómeno de Raynaud

B. Atrofia de piel

C. Cianosis

D. Diarrea

E. Microstomia

80. Paciente varón de 75 años, con dolor abdominal intenso desde hace 8 horas, distensión abdominal marcada y vómitos
biliosos. En la radiografía se evidencia distensión de asa intestinal. El diagnóstico m ás probable es:

A. Hernia inguinal incarcerada


B. Vólvulo de ciego

C. Diverticulitis aguda

D. Vólvulo de sigmoides

E. Malrotación intestina

81. El modelo de atención de salud tiene como principio fundamental y la estrategia principal es la :

A. Calidad / APS

B. Integridad / APS

C. Eficacia / MAIS

D. Universalidad / MAIS

E. Solidaridad / APS

82. De las siguientes alternativas, la conducta de tratamiento m ás adecuado para el trauma de uretra posterior es:

A. Colocación de sonda Foley uretral

B. Talla vesical

C. Reparación primaria de la uretra

D. Ureterostomía

E. Nefrostomía

83. Se desea estudiar un factor de riesgo asociado a la prostatitis, se nota que la población presenta varios factores con - fusores
pero no pueden ser considerados dentro de los criterios de exclusión y deben muestrear a todos. Para obtener una muestra
representativa se debe optar por el muestreo:

A. Estratificado

B. Aleatorio simple con restitución

C. Aleatorio simple sin restitución

D. Sistemático

E. Por conglomerado

84. Paciente usuaria de DIU, asintom ática, con ecografía que reporta gestación intrautero de 10 semanas, Cola de DIU visible a l
examen con especulo. La conducta a seguir es:

A. Control del embarazo más antibiótico

B. Control del embarazo más progesterona

C. Retiro de DIU más antibióticos

D. Retiro de DIU más progesterona

E. Retiro de DIU y observación

85. ¿A qué edad aproximadamente empieza a salir la primera dentición?

A. 4 meses

B. 5 meses

C. 6 meses

D. 7 meses

E. 8 meses

86. Un prematuro de 32 semanas de edad; a la primera hora de vida presenta una disnea progresiva con cianosis y tiraje. En la
radiografía hay un patrón de vidrio esmerilado y broncograma aéreo. A pesar de la ventilación, el oxígeno y los antibióticos, continúa
mal. ¿Qué terapéutica añadiría en primer término?

A. Surfactante endotraqueal.
B. Indometacina oral.

C. Prednisona intravenosa.

D. Bicarbonato intravenoso.

E. Glucosa intravenosa.

87. Durante la noche en una guardia de pediatría llega a consulta un paciente varón de 20 meses quien presenta aguda - mente
un cuadro de dolor de oído leve a moderado, mama refiere que días antes presento un cuadro de r esfrió y que ahora presenta esta
molestia. Al examen presenta una T 38.5°C, orofaringe congestiva sin exudado, oído derecho con tímpano eritematomoso, congest ivo,
abombado y nivel hidroaereo, pulmones sin alteraciones. Según su diagnóstico, ¿Qué le indica ría?

A. Amoxicilina

B. Cefuroxima

C. Ibuprofeno

D. Observación

E. Azitromicina.

88. Gestante de 38 semanas con PA: 160/110 y Albuminuria ( ++++) ¿Cu ál es la alternativa terapéutica m ás apropiada?

A. Sulfato de Magnesio 6gr Ev en bolo y 2gr cada hora

B. Betametasona 12 mg EV c/12 horas

C. Misoprostol 200 ug via vaginal cada 6 horas

D. Sulfato de Magnesio 2gr IM en cada glúteo

E. Sulfato de Magnesio 1gr Ev en bolo y 1gr cada hora

89. Anciana llega a la consulta con dolor postraum ático de hombro derecho, con limitación de movimiento. Examen clíni- co:
hematoma en cara interna del tercio superior del brazo. ¿Cuál es el diagnóstico m ás probable?

A. Luxación escapulohumeral

B. Fractura de clavícula

C. Luxación acromioclavicular

D. Fractura de escápula

E. Fractura del cuello del húmero

90. ¿Qué se desarrolla en la administración de un establecimiento nivel I, al relacionar las actividades de sus diversas unidades
para alcanzar las metas propuestas?

A. Negociación

B. Organización

C. Coordinación

D. Motivación

E. Planificación

91. Infante con hipoacusia cursa con prueba de Rinne negativo, adem ás en la otoscopia se menciona membrana timpánica con
niveles hidroaéreos.

A. Otitis media aguda

B. Otitis media crónica

C. Otitis externa difusa

D. Otitis secretora

E. Otitis externa maligna.

92. Las macula densa se encuentran en:

A. La arteria aferente
B. La arteria eferente

C. Túbulo contorneado distal

D. Asa de Henle

E. Capsula de Bowman

93. ¿Cuál es la contraindicación absoluta para el tratamiento hormonal en post menopausia?

A. Ginecorragia de origen desconocido.

B. Varices superficiales

C. Bochornos

D. Dislipidemia controlada

E. HTA controlada.

94. Cuál de las siguientes es un material de sutura absorbible:

A. Nylon

B. Polidioxanona

C. Polipropileno

D. Seda

E. Acero quirúrgico

95. ¿Cuál es la profundidad a la cual se debe comprimir el torax en un lactante en el RCP?

A. 3cm

B. 4cm

C. 5cm

D. 6cm

E. 7cm.

96. Bebé de 30 horas de edad, nació con 4300 g, hijo de madre con diabetes gestacional, ha tomado bien su alimento, pero se
ve muy inquieto a los 30 minutos de vida. 15 minutos m ás tarde sufre una convulsión tónico-clónica. Diagnóstico m ás probable:

A. Hipoglucemia

B. Hipocalcemia

C. Hipomagnesemia

D. Hiponatremia

E. Hiperviscosidad

97. El grado de afectación fetal en la isoinmunización Rh se aprecia con mayor exactitud por medio de:

A. Test de Coombs indirecto.

B. Test de Coombs directo.

C. Niveles de b-HCG en líquido amniótico.

D. Medición de los niveles de bilirrubina en el líquido amniótico-curvo de Liley.

E. Test de Coombs y b-HCG.

98. La porción mayor del estómago está constituida por:


A. La unión gastroesofágica.

B. El cardias.

C. El fondo.

D. El cuerpo.

E. El antro.

99. Una niña de 9 años de edad tiene una historia de sibilancias intermitentes de varios años; no ha recibido medicamen - tos
hace algún tiempo. Se halla febril, agitada y con cianosis peribucal. Tirajes intercostales y supraesternales; los ruidos res piratorios
están disminuidos y hay sibilancias bilaterales. La intervención inicial m ás apropiada es:

A. Administración de aminofilina intravenosa.

B. Solicitar radiografía torácica.

C. Prescribir la nebulización de cromoglicato sódico.

D. Pedir biometría hemática completa y hemocultivo.

E. Prescribir salbutam ol nebulizado.

100. La trompa de Eustaquio y la mucosa del oído medio se originan en:

A. La cuarta bolsa faríngea

B. El neuroectodermo notocordal

C. El primer arco branquial

D. La cuerda del tímpano

E. La primera bolsa faríngea

SIMULACRO 12A

1. Son agentes patógenos de neumonía adquirida en la comunidad los siguientes, excepto:

A. Streptococcus pneumoniae

B. Acinetobacter baumanii

C. Mycoplasma pneumoniae

D. Hemophilus influenzae

E. Legionella pneumophila

2.¿Cuál de las siguientes no es característica de convulsion febril simple?

A. Duración menor de 15 minutos

B. 2 o más en 24 horas

C. No dejar déficit motor

D. Convulsion Tonicoclonico generalizado

E. Convulsion asociado a fiebre

3. Mujer de 30 años, acude por presentar Poliartrálgias y pápulas erite matosas aplanadas sobre los nudillos de ambas manos (signo
de Gottron) y telangiectasias periungüeales. ¿Cuál es lo correcto con respecto al diagnóstico?:

A. Existe dolor muscular

B. Asocia con esclerodermia

C. Para el diagnostico se usa biopsia muscular

D. Presencia del anti jo 1 indica buen pronostico

E. Asocia con panarteritis nodosa

4. Es la relación que existe entre los resultados con respecto a los recursos
A. Eficiencia

B. Efectividad

C. Eficacia

D. Costo – Beneficio

E. Aceptación.

5. Mujer de 20 años, con cesárea por inducción fallida. Al tercer día de puerperio presenta malestar general, fiebre y loquios
fétidos. Al Examen clínico: T: 39°C, útero subinvolucionado y doloroso a la palpación, secreción f étida. ¿Cuál es el diagnóstico más
probable?

A. Dehiscencia de la histerorrafia

B. Endometritis puerperal

C. Absceso de pared abdominal

D. Retención de restos placentarios

E. Necrosis isquémica de mioma uterino

6. ¿Cuáles son los tumores primarios que más frecuentemente producen metástasis cerebrales y cuáles son los tumores
primarios más frecuentes en adultos?

A. Cáncer de cuello uterino y de próstata // astrocitoma grado II

B. Linfoma y carcinoma gástrico / glioblastoma multiforme

C. Cáncer de pulmón y de mama // astrocitoma grado IV

D. Cáncer gástrico y de páncreas // meduloblastoma

E. Cáncer de pulmón y de mama // metastasis

7. Primigesta añosa de 34 semanas de gestación no controlada, acude por presentar cefalea, tinnitus, dolor epigástrico.
Examen físico: PA: 150/90 mm Hg, FC: 90 x ́, FR: 20 x ́, T: 36.5°C, edema en miembros inferiores. Altura uterina: 30 cm, Feto LCI,
FCF:140 x ́. Exámenes de Laboratorio: Hb:10 g/dL, plaquetas 110,000/mm3, Creatinina: 1 mg/dL, TGO: 70 UI/L, LDH:1200 UI/L,
Bilirrubina:1.4 mg/dL a predominio indirecto. ¿Cuál es el diagnóstico más probable?

A. Síndrome HELLP

B. Preeclampsia severa

C. Hipertensión gestacional

D. Hipertensión crónica

E. Eclampsia

8. ¿Cuáles son los agentes terapéuticos de uso frecuente en la epilepsia y convulsiones paciales y en las generalizadas
respectivamente?

A. Lamotrigina, fenobarbital // carbamazepina

B. Etosuximida, ácido valproico, lamotrigina // lamotrigina

C. Carbamazepina, fenilhidantoína, gabapentina // acido valproico, lamotrigina

D. Primidona, topiromato // Carbamazepina, Lamotrigina

E. Zonisanida, ácido valproico // Carbamazepina, Lamotrigina

9. RN con hepatoesplenomegalia, rinitis purulenta, anemia, rash maculopapular y descamación de Palmas y plantas. Cuál es el
diagnóstico más probable.

A. CMV

B. Sífilis congénita

C. Toxoplasmosis

D. Rubeola
E. Herpes

10. Con respecto a la HTA señale lo correcto

A. Principal factor de riesgo para hemorragia subaracnoidea

B. La hipertensión maligna genera necrosis fibrinoide

C. La hipertensión produce lesión de arterias de gran calibre

D. La estenosis arteriolar corresponde a la retinopatía grado II

E. Produce dilatación ventricular izquierda

11. Gestante de 38 semanas, acude a Emergencia por presentar dolor abdominal intenso hace 4 horas acompañado de
sangrado vaginal rojo vinoso. Examen físico: PA: 140/90 mmHg, LCF: 128 x’, AU: 37 cm, hipertonía uterina y se con - firma sangrado
de cavidad uterina ¿Cuál es el diagnóstico más probable?

A. Desprendimiento prematuro de placenta

B. Placenta previa

C. Ruptura uterina

D. Vasa previa

E. Inversión uterina

12. Si el cuadro clínico de asma bronquial cursa con sibilancias, tos, disnea menos de 2 veces por semana, exacerbaciones
breves (desde horas a dias), síntomas nocturnos mayores de una vez por semana, asintomática entre exacerbacione s.

¿Qué clasificación de gravedad le corresponde?

A. Intermitente

B. Persistente leve

C. Persistente moderado

D. Persistente grave

E. Intermitente moderado.

13. Acude a consulta un paciente de 6 meses de edad que en sus 3 primeros días de enfermedad pre senta cuadro catarral,
luego aumenta su frecuencia respiratoria por lo que acude a emergencia donde se evidencia taquipnea, tirajes, sibi - lantes. ¿Cuál es
el agente etiológico más probable?

A. Adenovirus

B. Micoplasma

C. Penumococo

D. Virus sincitial respiratorio

E. Rinovirus

14. ¿Cuál es la alternativa que NO corresponde a la anemia ferropenia?

A. La respuesta al tratamiento se mide con reticulocitos al 10 dia

B. La causa más frecuente es carencial en jovenes

C. Para diferenciarla de la anemia por enfermedad crónica se mide transferrina

D. El test bioquimico mas especifico es la ferritina

E. El hierro se absorbe solo el 10% de lo ingerido a nivel intestinal.

15. Adolescente de 15 años, con 32 semanas de gestación , sin control prenatal, acude a emer gencia por dolor epigástrico y
cefalea. Al examen impresiona como embarazo de 6 meses por altura uterina. PA: 160/110mmHg, Proteinuria (+++). El diagnostico
más probable es :

A. Eclampsia

B. Hipertension transitoria
C. Pre-eclampsia severa

D. Pre-eclampsia superpuesta

E. Sindrome de HELLP

16. Llega a emergencia un lactante de 3 meses quien 4 días antes de su ingreso desarrolla rinorrea y estornudos, ahora tiene
dificultad para respirar, taquipena y a la auscultación subcrepitos. Según su sospecha diagnostica, ¿Cuál es el manejo más indicado?

A. Iniciar amoxicilina

B. Brindar terapia de rescate con beta-agonistas

C. Dar broncodilatadores más corticoide

D. Iniciar macrolidos

E. Nebulización con suero hipertónico.

17. Varón de 56 años, que presenta pérdida de peso de 6 kg en el último año, cambios de la voz, incremento del volumen de la
glándula tiroides a predominio derecho. No adenomegalias y pruebas de función tiroidea normales. En el preope- ratorio, ¿qué
examen complementario de tiroides solicitaría?

A. Aspiración con trocar

B. Gammagrafía

C. Ultrasonografía con biopsia

D. Resonancia magnética

E. Tomografía

18. Paciente proviene de Piura, con fiebre de menos de 5 días de duración, sin afección de las vías aéreas superiores con
cefalea y mioartalgias. ¿Cuál es el diagnóstico más probable?

A. A.Dengue sospechoso.

B. B.Probable dengue grave

C. C.Probable dengue hemorrágico

D. D.Dengue confirmado

E. E.Dengue descartado

19. Inmunoglobulina aumentada en purpura de Henoch Schonlein

A. Ig G

B. Ig E

C. Ig M

D. Ig D

E. Ig A

20. Paciente mujer de 20 años con palpitaciones de inicio y fin súbito, con PA normal, la primera medida terapuetica es:

A. Digoxina

B. Atenolol

C. Verapamilo

D. Adenosina

E. Maniobras vagales

21. Gestante de 8 semanas acude por sangrado vaginal profuso desde hace 3 horas, acompañado de dolor abdominal intenso.
Examen físico: PA: 80/60 mmHg, FC: 98 x’, FR: 18 x’. Especuloscopía: cérvix con sangrado y coágulos abun- dantes. Tacto vaginal:
orificio externo e interno abiertos. ¿Cuál es el diagnóstico más probable?

A. Aborto inevitable

B. Aborto frustro
C. Aborto inminente

D. Amenaza de aborto

E. Aborto diferido

22. Mencione lo que no corresponde al score de forrest para hemorragia digestiva alta

A. El tipo Ia tiene alta tasa de mortalidad

B. El tipo IIb es coagulo adherido

C. El tipo III es sangrado inactivo

D. El tipo Ib requiere tratamiento endoscópico más antiacidos

E. El tipo IIc es ulcera con fibrina

23. ¿En qué mecanismo del trabajo de parto se produce cambio de la variedad de posicion?

A. Expulsión

B. Rotación interna

C. Descenso

D. Extensión

E. Flexion

24. Paciente programado para cirugía de apendicitis congestiva, en sala se produce mínima lesión del colon. Como lo clasificar ía.

A. Limpia

B. Sucia

C. Limpia contaminada

D. Sucia contaminada

E. Contaminada

25. El diagnóstico bioquímico del empiema pleural se realiza sobre la base de los siguientes criterios, excepto:

A. Triglicéridos mayor de 100

B. Glicemia menor de 40 mg%

C. pH menor de 7.2

D. proteínas mayor de 3g%

E. Gran celularidad a predominio de polimorfonucleares

26. Señale lo correcto respecto a la menopausia:

A. Los niveles de FSH se elevan antes que los trastornos del ciclo menstrual.

B. El estradiol es la hormona predominante.

C. La progesterona se eleva tras la menopausia.

D. El climeterio es más intenso en los primero 10 años anteriores al diagnóstico.

E. La terapia de reemplazo hormonal es lo más importante en todas las menopausicas.

27. Lactante de 8 días de nacido con distensión abdominal y vómitos biliosos abundantes, signo de la doble burbuja.

A. Páncreas tubular

B. Estenosis hipertrófica de piloro

C. Vólvulo de ciego

D. Atresia duodenal

E. Bridas
28. ¿Con que prueba se realiza el diagnostico de asma bronquial en un niño mayor de 5 años?

A. Espirometria simple

B. Flujometro

C. Espirometria forzada

D. Pletismografia

E. Radiografia de torax

29. Paciente con IMA que presenta crepitantes en ambos campos pulmonares signos de ICC severa se clasifica como

A. Killip I

B. Killip II

C. Killip III

D. Killip IV

E. Forrester III

30. Paciente con disfagia motora m ás pérdida de peso, ante la sospecha diagnostica quien confirma la etiologia

A. Ph metria de 14 hrs

B. Endoscopia alta

C. Impedanciometria

D. Manometría esofagica

E. Test de secretina

31. En relación con la etiología del RCIU II, señale factor más frecuente

A. Enfermedad hipertensiva

B. Rubéola

C. Alcoholismo

D. Drogadicción

E. Tabaquismo

32. ¿Cuál es el vector del ciclo de transmisión selvático de la fiebre amarilla en la Am érica del sur?

A. Mosquitos de genero Haemagogus

B. Mosquitos de genero Aedes

C. Mosquitos de genero Crotalus

D. Mosquitos de genero Phlebotomus

E. Mosquitos de genero Vivax

33. ¿Cuál de las siguientes entidades causa derrame pleural tipo trasudado?

A. Neumonía bacteriana

B. TBC pleural

C. Cirrosis hepática

D. Metástasis pleural

E. Mesotelioma .
34. En el Perú actual, el cuidado de la salud está en los primeros lugares de la agenda política del Acuerdo Nacional, al mismo
tiempo el Ministerio de Salud sólo ha podido desarrollar precariamente su rol rector y conductor sensorial. La combinaci ón de estos
dos factores en una matriz FODA permite identificar la siguiente estrategia:

A. Desafío

B. Limitación

C. Potencialidad

D. Restricción

E. Riesgo.

35. Paciente varon joven con datos de trombosis venosa profunda, además se encuentra ictericia a predominio indirecto con
coluria, la sospecha diagnostica es:

A. Hemoglobinuria paroxística nocturna

B. Esferocitosis

C. Talasemias

D. Hemoglobinopatías

E. Eliptocitosis

36. ¿Cuál es el diagnostico nutricional de un lactante de 11 meses que tiene P/T 94%, T/E 94%?

A. Desnutrición aguda

B. Desnutrición crónica

C. Desnutrición crónica reagudizada

D. Desnutrición global

E. Eutrófico

37. El Caso índice o caso cero Corresponde a:

A. Primer caso ocurrido en una epidemia.

B. Primer caso reportado al sistema de salud.

C. Primer caso muerto.

D. Primer caso tratado.

E. Primer caso publicado.

38. Paciente de 15 años, con náuseas, vómitos, boca seca, y somnolencia, PA 60/40 IMC 17KG/M2 , glicemia 450mg/dl; pH

7,15; HCO3 12mEq/kg , la terapia indicada comprende fluidos m ás:

A. Insulina cristalina en bolos

B. Insulina cristalina por vía SC

C. Insulina cristalina por infusión

D. Insulina cristalina por vía IM

E. Insulina intermedia por vía SC

39. Respecto a los ACO señale lo falso:

A. Si se dan gestàgenos solos el efecto anticonceptivo disminuye.

B. Si se dan progestágenos solos puede ocurrir atrofia endometrial.

C. El principal efecto anticonceptivo se obtiene a nivel central hipotalámico.

D. Los progestágenos de tercera generación tienen buen perfil lipídico.

E. Los gestagenos en depósito son de elección en adolescentes y nul íparas.


40. Con respecto a la angina inestable señale lo verdadero

A. La etiología más frecuente es la placa de ateroma con capa fibrosa gruesa

B. El factor de riesgo más importante es la hipertensión arterial

C. Las mujeres tienen más riesgo en comparación con varones

D. Una variante clínica es la angina por vasoespasmo coronario

E. Presenta elevación de CPK MB

41. Niño con alopecia, lesión en piel (acrodermatitis), diarrea, deficiencia de que vitamina:

A. Zinc

B. Niacina

C. Tiamina

D. Magnesio

E. Hierro

42. Paciente varón de 25 años, politraumatizado en accidente de tránsito. Al examen neurológic o: apertura ocular al do- lor, emite
sonidos incomprensibles y extiende el miembro superior al dolor. De acuerdo a la escala de Glasgow, cuál sería la conducta in mediata
a seguir:

A. Observación

B. Intubación orotraqueal

C. Craneotomía

D. Traqueotomía

E. TAC

43. Mujer de 40 años de edad, presenta predominantemente en la región nasogeniana legiones papulosas eritomatosas y
descamaticas, las cuales se exacerban con alguna comida y tensión emocional. el diagnóstico más probable es y el manejo adecuado
debería ser:

A. Rosácea / minociclina

B. Acné / retinoides

C. Rosácea / metronidazol topico

D. Acné / metronidazol oral

E. Psoriasis / retinoides o metronidazol

44. La nueva estructura de etapas del ciclo de vida según el modelo de atención integral de salud incluye a la etapa joven. ¿Cuál
es el rango de edad que le corresponde?

A. 18a29años

B. 19a29años

C. 20 a 29 años

D. 19a30años

E. 20a30años

45. Adulto joven en cuyo perfil hepático solo se encuetra alterado la bilirrubina que esta aumentado ligeramente a predo- minio
indirecto, la sospecha diagnostica es

A. Síndrome de rotor

B. Síndrome de dubin jonson

C. Síndrome de Gilbert

D. Sindrome de clijer najar tipo I

E. Sindrome de clijer najar tipo II


46. Varón de 45años de edad, que presenta desde hace 12 meses de manera progresiva dificultad para la marcha. Al exa - men
se evidencia hipomimia facial, lentitud para iniciar la marcha, dificultad para atar los cordones de sus zapatos, temblor fin o en miembro
superior derecho y resistencia a la flexo-extensión pasiva de dicho miembro. La sospecha diagnóstica es ------ y el manejo adecuado
es --------:

A. Alzheimer / memantina

B. Huntington / levodopa y carbidopa

C. Parkinsonismo / levodopa y memantina

D. Enfermedad de Parkinson / levodopa y carbidopa

E. Wilson / EDTA.

47. Acude a emergencia un paciente con estridor que días antes presento clínica de rinorrea, estornudos y sensación de alza
térmica, ahora presenta dicho estridor y tos perruna. Al evaluarlo se le clasifica como leve. ¿Cu ál es su manejo?

A. Antipiréticos

B. Nebulización con adrenalina

C. Nebulización con suero hipertónico

D. Dexametasona endovenoso

E. Dexametasona vía oral

48. Fractura conminuta no desplazada del primer MTC:

A. Benett

B. Rolando

C. Jones

D. Murphy

E. Mallampati

49. En la pubarquia, de acuerdo con la clasificación de Tanner, si se encuentra vello pubiano que extiende solo en el tri án- gulo
pubiano, corresponde al estadio:

A. VP1

B. VP2

C. VP3

D. VP4

E. VP5

50. Paciente varón de 18 años, quien ingresa a emergencia con dolor abdominal desde hace 7 horas, localizado en fosa iliaca
derecha y náuseas, afebril. Dolor en punto de Mc Burney positivo. Leucocitosis y PCR levemente aumentado. De acuerdo al
diagnóstico más probable, señale lo correcto:

A. La etiología es básicamente la perforación de la víscera

B. El germen que se puede identificar con más probabilidad es el Bacteroides fragilis

C. El tratamiento antibiótico requiere Amikacina + Metronidazol por una semana

D. La causa más frecuente es la hipertrofia de las placas de Peyer.

E. El germen más frecuente es E. coli

51. El establecimiento de sector salud de categoría I-2 corresponde a:

A. Puesto de salud con m édico

B. Puesto de salud sin m édico

C. Puesto de salud con 10 m édicos

D. Puesto de salud con hospitalizacion


E. Puesto de salud con quirófano.

52. ¿A qué edad aproximadamente un niño duplica su talla de nacimiento?

A. 1 año

B. 2 años

C. 3 años

D. 4 años

E. 5 años

53. Paciente con epistaxis posterior mencione lo verdadero

A. Lesión del plexo de kiesellbach

B. Fácil de controlar

C. Resuelve con taponamiento anterior

D. La arteria esfenopalatina es rama de la maxilar interna

E. La causa más frecuente es infecciones.

54. Paciente de 40 años de edad que presenta necesidad imperiosa de mi ccionar y se le escapa la orina antes de llegar al baño,
ya que no puede retenerla. Al examen se constata uretrocistocele de primer grado. El diagnóstico es:

A. Incontinencia urinaria de esfuerzo

B. Incontinencia urinaria de urgencia

C. Infección de vías urinarias

D. Cistitis intersticial

E. Cistitis aguda.

55. La denominación sinclitismo y asinclitismo se refiere a la relación entre:

A. La sutura interparietal y las tuberosidades izquierdas.

B. La sutura metódica y el promontorio.

C. La sutura occipital y el promontorio.

D. La sutura sagital con el promontorio y el pubis.

E. La sutura sagital y las espinas ciáticas.

56. Multigesta de 10 sem con sangrado vaginal y dolor abdominal, altura uterina 18 cm orificio cervical entreabierto,Además
presenta hemoptisis ¿Cuál es el diagnóstico?

A. Mola parcial

B. Mola completa

C. Aborto molar

D. Degeneración hidrópica

E. Coriocarcinoma

57. En el embarazo normal, la altura del fondo uterino se vuelve extrapelvico durante la semana y se considera un
síntoma… de diagnóstico de embarazo

A. 12 - probable

B. 16 - presuntivo

C. 18 - certeza

D. 10 - presuntivo

E. 20 – certeza
58. ¿Cuál es la neoplasia más frecuente en pediatría?

A. Leucemias

B. Linfomas

C. Tumor de willms

D. Astrocitoma

E. Meduloblastoma.

59. En la prueba de hipótesis, el investigador comete un error tipo II o beta cuando:

A. No establece el nivel de significancia

B. No rechaza la hipótesis nula siendo falsa

C. Rechaza la hipótesis alterna, siendo falsa

D. Rechaza la hipótesis alterna, siendo verdadera

E. Rechaza la hipótesis nula siendo verdadera.

60. ¿Cuál es la vacuna que no debe darse en alérgicos a la proteína del huevo?

A. BCG

B. Antineumococica

C. Antiamarilica

D. VPH

E. Rotavirus

61. Varón de 50 años de edad, que ingresa al Hospital por prese ntar un cuadro de disnea, palidez de piel y mucosas. Al examen
clínico: PA 150/90 mmHg. Se detecta anasarca y signos de derrame pleural derecho. Exámenes de laboratorio: hemoglobina 9g/dL,
riñones con pérdida de la diferenciación corticomedular ¿Cuál es el diagnóstico más probable?:

A. Síndrome nefrótico

B. Síndrome nefrítico

C. Glomerulonefritis aguda

D. Nefritis aguda

E. Insuficiencia renal crónica

62. ¿Cuál de los siguientes es el mejor parámetro para evaluar el crecimiento de un niño?

A. IMC en valores absolutos

B. Una sola visita en su vida

C. El seguimiento con las curvas de crecimiento

D. Con los datos del nacimiento

E. Evaluación psicológica

63. El índice ecográfico que mejor identifica el retardo del crecimiento fetal es… , y para evaluar sus pronostico se

utiliza la ecografía doppler de la arteria

A. La longitud del fémur - uterina

B. La circunferencia abdominal - umbilical

C. La longitud del fémur – cerebral media

D. La circunferencia abdominal – ductus venoso

E. La longitud del fémur – cerebral media


64. Varón de 65 años, con T: 38°C, náuseas, vómitos y deposiciones líquidas con moco, sangre y tenesmo. Estando hospi -
talizado presenta una convulsión por primera vez. ¿Cuál es el agente causal?

A. Entamoeba histolítica

B. Salmonella

C. Escherichia colie

D. Clostridiun perfingens

E. Shiguella biflexa.

65. Es un complicación tardía de la traqueotomía

A. Estenosis laríngea

B. Hemorragia

C. Neumotórax

D. Atelectasia pulmonar

E. Enfisema subcutáneo

66. Niño de 2 años de edad, que desde los 9 meses presenta episodios de lesiones eritematosas descamativas en mejillas,
abdomen y parte extensoras de miembros, con prurito intenso. Actualmente aparecen lesiones similares despu és de comer maní. Al
examen se evidencia pliegues redundantes infraorbitarios. ¿Qué tipo de dermatitis consideraría en su presunción diagnóstica?:

A. Por ingesta de alimentos

B. Seborreica

C. Alérgica de contacto

D. Eccematoide infecciosa

E. Atópica

67. En el tratamiento de la hiponatremia en un paciente con insuficiencia renal aguda oligúrica, la medida terapéutica más
apropiada es:

A. Restricción de agua libre

B. Restricción de sodio

C. Administración de sodio

D. Uso de diuréticos de asa

E. Uso de resina de intercambio iónico

68. Señale que cambios ocurren en el embarazo normal

A. Disminución del tamaño renal

B. Dilatación uretereal bilateral

C. Disminución del índice de filtración glomerular

D. Aumento del índice de flujo plasmático renal

E. Pronunciada proteinuria

69. ¿Cuál es la secuencia lógica en el proceso administrativo?

A. Planificación, organización, dirección y control.

B. Planificación dirección, organización, y control

C. organización, dirección control Planificación

D. Planificación, control organización, dirección


E. dirección Planificación, organización, y control.

70. RN con antecedente de DIPII y líquido amniótico meconial, nace hipotónico, apnea y bradicardico. Qu é medida to- maría.

A. Aspiración de secreciones

B. Estimulación táctil y aspiración de secreciones

C. VPP

D. VPP con mascarilla

E. Ventilación mecánica

71. En varones adultos con anemia ferropénica ¿con que prueba monitorizamos la respuesta al tratamiento?

A. Hemoglobina a los 20 dias

B. HTO a los 15 dias

C. Reticulocitos a los 10 dias

D. Reticulocitos a los 5 dias

E. Ferritina a los 30 dias

72. ¿Cuál es la característica del flujo vaginal en la Vaginosis bacteriana?

A. Seco y espumoso

B. Grumoso y blanquecino

C. Fétido y grisáceo

D. Espeso y blanquecino

E. Espumosos y sanguinolento.

73. ¿Cuál es el principal mecanismo de la ovulación a nivel hormonal

A. El pico de LH aislado.

B. El pico de FSH aislado.

C. El pico de Estradiol y FSH.

D. El pico de Estradiol y LH.

E. El pico de Progesterona aislado.

74. Recién nacido con fibrosis quística, cual es la manifestación más relacionada:

A. Ileo meconial

B. Policitemia

C. Hipoglicemia

D. Dermatitis crónica

E. Neumonía

75. ¿Qué porcentaje del área bajo la curva normal estándar se localiza entre +2 desviaciones estándares de la media?

A. 58,6

B. 95,4

C. 68,3

D. 90,4

E. 99,9

76. Las fracturas que se producen por microtraumatismos, ya sea por intensidad o por repetición que alteran las propie -dades
fisicoquímicas del hueso, se denominan y la m ás frecuente es la fractura :
A. Por fatiga / del recluta

B. Patológicas / colles

C. Por fatiga / cargador

D. Osteoporoticas / colles

E. Patologicas / metatarsiano 2

77. Marque lo falso respecto a las siguientes patologías neurológicas:

A. La demencia más frecuente es la demencia de Alzheimer.

B. La demencia de Alzheimer es predominantemente subcortical.

C. El nivel de lesión mesencefalico se evalúa con la rigidez de decorticacion o descerebración.

D. El tumor más frecuente en el SNC es metastasico de pulmón.

E. El tumor pilocitico infantil es frecuente en cerebelo.

78. Tratamiento para crisis de agitación psicomotriz:

A. Haloperidol

B. Benzodiazepinas

C. Fenitoina

D. Carbamazepina

E. Quetiapina

79. Que serotipo VPH está asociado a las verrugas genitales

A. 6 y 11

B. 16 y 18

C. 24 y 28

D. 15 y 11

E. 6 y 18

80.¿Cuál de las siguientes medidas de tendencia central es la más usada para el análisis estadístico si los datos son asi - metricos?

A. La mediana

B. La media

C. La moda

D. La desviación estándar

E. La varianza

81.Según la clasificación de Bormann para cáncer gástrico, las lesiones ulceradas infiltrantes corresponden al tipo:

A. 5

B. 4

C. 3

D. 2

E. 1

82.Varón de 23 años de edad: refiere fácil sangrado de las encías al uso del cepillo dental. Al examen; palidez y mucosas, ausencia
de gingivitis; punta de bazo palpable. Hematocrito 31%, leucocitos 2.300/dL, plaquetas 100.000/dL. Frotis de sangre perif érica:
cuerpos de Auer. El diagnóstico presuntivo es:

A. Leucemia linfoide aguda

B. Leucemia mieloide aguda


C. Leucemia mieloide crónica

D. Mielofibrosis

E. Mononucleosis infecciosa

83. Paciente de 78 años con te: 2 días, presenta deposiciones líquidas 3 v/día, volumen de ½taza por vez, s/s, con pa: 100/60
mmHg, p: 100x’, fr: 22x’; presión ocular disminuida. Además tiene Na: 120 mmol/l, k: 3,5 mmol/l, cl: 100 mmol/l. respecto al estado
hidroelectrolítico de la paciente, podemos afirmar que presenta:

A. Hiponatremia Hiperosmolar.

B. SIADH

C. Hiponatremia con hipovolemia

D. Hiponatremia con normokalemia

E. Hiponatremia con hipovolemia

84. Paciente diabético cursa con pérdida progresiva de agudeza visual, la etiología más probable seria:

A. Desprendiemiento de retina

B. Edema macular

C. Hemovitreo

D. Glaucoma de Angulo estrecho

E. Miopia maligna

85. Mujer de 22 años de edad, sexualmente activa, con dolor pélvico intenso y fiebre. Al examen: anexos dolorosos y leucorrea
maloliente. Hemograma con 15.000 leucocitos x mm3. ¿Cuál es el diagnóstico y el manejo más apropiado?

A. Enfermedad pélvica inflamatoria + hospitalización + Ceftriaxona + metronidazol + doxicicli na

B. Enfermedad pélvica inflamatoria + hospitalización + cefoxitin + metronidazol + azitromicina

C. Enfermedad pélvica inflamatoria + tratamiento ambulatorio con ciprofloxacino, doxiciclina y metronidazol.

D. Quise ovárico complicado + tratamiento quirúrgico

E. Enfermedad pélvica inflamatoria + tratamiento quirúrgico.

86. Mujer de 26 años, con fiebre de 39°C, de 4 semanas de evolución que calma con antipiréticos; fatiga intensa y malestar
general. Al Examen físico: Linfoadenomopatías cervicales, submaxilares y supraclaviculares y hepatoesplenomega - lia. Hemograma
con 15% de linfocitos atípicos y leve trombocitopenia. Anticuerpos heterofilos positivos. ¿Cu ál es el diagnóstico más probable?

A. Dengue hemorragico

B. Mononucleosis infecciosa por CMV

C. Fiebre Chikungunya

D. Mononucleosis infecciosa por VHB

E. Tuberculosis pulmonar

87. Paciente varón de 45a con DM tipo I. Acude por cefalea, somnolencia, náuseas y vómitos persistentes. Al examen físi - co,
despierto, pálido y mucosa oral seca; PA 140/70, FC: 100; FR: 28; extremidades inferiores con fóvea++ Hb 10 pH: 7.24; Hto 44 K 2.9
Na 139; HCO3 15. Glucosa 280. ¿Cuál es el Diagnostico?

A. Estado Hiperosmolar

B. Cetoacidosis Diabética

C. Pielonefritis crónica

D. Cetoacidosis ayuno

E. Insuficiencia Renal Aguda

88. En la artritis bacteriana aguda de rodilla, la conducta terap éutica inmediata es:

A. Drenaje de líquido sinovial y AINES


B. Antibióticos parenteral y drenaje

C. Antibióticos orales y AINES

D. Antibióticos intra-articulares

E. Irrigación intra-articular continua

89. Mujer de 20 años de edad, peladora de espárragos desde hace 4 meses, presenta lesiones eritematosas pruriginosas
micropapulares localizada en dorso de antebrazos, cara y cuello. Las lesiones tambi én se han identificado en algunas compañeras de
trabajo. No presenta antecedentes de atopía. Su primera impresión diagnóstica es y el tratamiento es:

A. Acarosis / permetrina

B. Dermatitis por contacto / corticoides

C. Acarosis / griseofulvina

D. Dermatofitosis / clotrimazol

E. Dermatofitosis / itraconazol

90. El antidepresivo con menor efecto sedante y el que tiene mayor efecto sedante son respectivamente:

A. Amitriptilina / sertralina

B. Clomipramina / imipramina

C. Doxepina / bupropion

D. Fluoxetina / paroxetina

E. Trazodona / sertralina

91. Mujer con infección por chlamydia, que esquema de tratamiento indicaría.

A. Metronidazol 2gr dosis única

B. Fluconazol 150mg

C. Doxiciclina 100 mg

D. Ketoconazol

E. Ceftriaxona 250 mg

92. Ubicación más frecuente del tumor de klastkin

A. En conducto cístico

B. Colédoco proximal

C. Confluencia de hepáticos

D. Colédoco distal

E. Hepático derecho

93. Varón de 19 años que refiere aumento de volumen en el hemiescroto derecho transiluminacion positiva, deber íamos
descartar

A. Hernia inguinoescrotal.

B. Hidrocele

C. Quiste del epidídimo.

D. tumor testicular tipo seminoma

E. Tumor testicular tipo no seminoma

94. Beta bloqueador utilizado en glaucoma crónico

A. Metoprolol

B. Bisoprolol
C. Nadolol

D. Carvedilol

E. Timolol

95. Paciente varón de 50 años, fumador, con Hipertensión arterial controlada, programado para cura quir úrgica de her- nia
inguinal. Su riesgo ASA es:

A. V

B. II

C. I

D. IV

E. III

96. Se cuenta con 13 valores de nivel de bilirrubinas en pacientes con ictericia y se desconoce la varianza poblacional para esta
variable. En la construcción del intervalo de confianza al 95% para el valor promedio de bilirrubina en este grupo de pacientes, la
prueba estadística requerida es:

A. ANOVA

B. Binomial

C. Chi-cuadrado

D. Exacta de Fisher

E. T de Student.

97. Paciente con WPW, cual es la alternativa correcta:

A. Se debe a fenómenos de reentrada

B. Su tratamiento definitivo es con beta bloqueadores

C. Su tratamiento de urgencia es ablación con radiofrecuencia

D. Cursa con onda delta en el EKG

E. Más frecuente en personas adultas mayores

98.¿Cómo se define un RN con peso adecuado para la edad gestacional?

A. P10-90

B. P5-95

C. P15-85

D. P3-97

E. P3-90

99.La inhibición de la agregación plaquetaria por ADP (adenosina difosfato) es característica de:

A. Ácido acetilsalicílico.

B. Tirofibán.

C. Sulfinpirazona.

D. Clopidogrel.

E. Abciximab.

100.El tipo de sangrado más característico que producen los leimiomas uterinos:

A. Hipermenorrea por intramurales

B. Metrorragia por submucosos

C. Menorragia por submucosos


D. Menometrorragia por intramurales

E. Hipermenorrea por submucosos F.

EXAMEN N° 09

SIMULACRO 101

1.Una paciente de 44 años, sometida a mastectomía izquierda con vaciamiento ganglionar, consulta un año después de la
intervención por un cuadro febril con celulitis en brazo izquierdo. ¿Cuál es el agente etiológico más probable?

A. Salmonella enteritidis.

B. Streptococcus agalactiae.

C. Escherichia coli.

D. Streptococcus pyogenes.

E. Pasteurella multocida.

2.Un paciente de 28 años, aficionado a la caza, presenta una lesión roja en muslo que se aclara en el centro a medida que se
extiende. Con respecto a la enfermedad de este paciente, señale la respuesta correcta:

A. Se adquiere por contacto con herbívoros.

B. La presencia de una «mancha» negra en la piel es un dato muy específico.

C. Puede complicarse con bloqueo auriculoventricular.

D. El tratamiento de elección es Doxiciclina, independientemente de los órganos afectados.

E. Es probable que en los próximos días se asocie artritis de la cadera ipsilateral.

3.¿Cuál es la forma habitual de adquirir la infección por Coxiella burnetii?

A. Inhalación.

B. Ingesta de carne contaminada.

C. Picadura de garrapata.

D. Contacto con agua contaminada.

E. De persona a persona.

4.Paciente de 20 años, que hace 4 días comienza con dolor brusco de oído derecho, seguido de supuración maloliente, por lo qu e es
tratado por su médico de cabecera con ciprofloxacino en gotas óticas. La fiebre, que al inicio del cuadro era de 38 °C, se eleva hasta
40 °C, con debilidad y escalofríos. Al llegar a Urgencias, la TA es de 90/50, y el paciente presenta tortícolis con dolor y e nrojecimiento
cutáneo paralelo al esternocleidomastoideo derecho, y tos intensa con dolor torácico. En la Rx de tórax se observan infiltrados
redondeados bilaterales. Señale la afirmación correcta:

A. La bacteria causante forma parte de la flora orofaríngea.

B. Los infiltrados pulmonares son secundarios a endocarditis tricuspídea.

C. El dolor cervical nos indica la presencia de un adenoflemón.

D. En el hemocultivo se aislarán cocos positivos aerobios.

E. El tratamiento de elección es cefotaxima i.v.

5.La respuesta dérmica al antígeno de la tuberculina es positiva después de...... iniciada la infección:

A. 7 – 10 días.

B. 6 – 12 meses.

C. 6 – 12 semanas.

D. 12 – 18 meses.

E. 2 – 6 semanas.
6.Paciente de 40 años de edad, fumador desde los 18 años y sin antecedentes patológicos de interés. Consulta por cuadro de 24
horas de evolución de escalofríos, fiebre de 39 °C y tos con expectoración. La radiografía simple de tórax pone de manifiesto un
aumento de densidad con patrón alveolar en el lóbulo superior derecho y el paciente es diagnosticado de neumonía adquirida en la
comunidad. ¿Cuál es el agente etiológico más frecuente y por lo tanto de obligada cobertura al elegir el tratamiento antibiótico
empírico?

A. Mycoplasma pneumoniae.

B. Chlamydia pneumoniae.

C. Streptococcus pneumoniae.

D. Legionella pneumophila serogrupo 1.

E. Haemophilus influenzae.

7.Un sujeto sano tiene una ventilación minuto de 5 litros con una frecuencia de 16 respiraciones por minuto; si dismi - nuye su
frecuencia respiratoria a 8 respiraciones por minuto y mantiene el mismo volumen ventilatorio por minuto (5 litros); señale q ue es lo
que pasará con el valor del espacio muerto fisiológico (VD):

A. Aumenta.

B. Disminuye.

C. Permanece igual.

D. Variará de acuerdo a la nueva frecuencia.

E. Ninguno de los anteriores.

8.El soplo cardiaco en la miocardiopatía hipertrófica se intensifica por, excepto:

A. Posición en cuclillas.

B. Ejercicio.

C. Isoproterenol.

D. Digital.

E. Maniobra de Valsalva.

9.La causa más común de pericarditis constrictiva es:

A. Fiebre reumática.

B. Neoplasia.

C. Pericarditis aguda benigna.

D. Tuberculosis.

E. No determinada.

10.La clonidina es un fármaco antihipertensivo de tipo:

A. Agonista adrenérgico.

B. Antagonista adrenérgico.

C. Antagonista adrenérgico mixto.

D. Vasodilatador arterial.

E. Bloqueador ganglionar.

11.Un paciente de 55 años ha sido diagnosticado de miocardiopatía alcohólica e insuficiencia cardíaca congestiva. Tiene una
capacidad funcional IV/IV de la New York Heart Association (NYHA) e hipertensión esencial de 180/110 mmHg.¿Cuál de los
siguientes sería el tratamiento más apropiado para reducir l a tensión arterial del paciente?

A. Bloqueantes α-adrenérgicos.
B. Nitratos.

C. Inhibidores de la enzima convertidora de la angiotensina.

D. Bloqueantes β-adrenérgicos.

E. Antagonistas de los canales del calcio.

12.¿En cuál de las siguientes neoplasias gástricas en estadio incipiente, se puede considerar como primera opción tera -péutica la
erradicación de H. pylori con antibioterapia e inhibidores de la bomba de protones?

A. Adenocarcinorna gástrico tipo difuso.

B. Adenocarcinorna gástrico de tipo intestinal.

C. Linfoma MALT gástrico.

D. Linfoma alto grado gástrico.

E. Linfoma de Hodgkin gástrico.

13.El hallazgo más importante en la Acalasia de esófago es:

A. La estenosis del extremo distal del esófago.

B. La dilatación esofágica marcada.

C. La ausencia de los plexo nerviosos - mioesofágicos.

D. La regurgitación alimentaria nocturna.

E. La presencia de disfagia que puede llevar a ser muy intensa.

14. El carcinoma primario de hígado acompaña más a la:

A. Hemocromatosis.

B. Cirrosis.

C. Gastritis hemorrágica.

D. Colecistitis calculosa.

E. Quiste hidático del hígado.

15.Las convulsiones febriles en la infancia son:

A. Parciales mioclónicas.

B. Parciales motoras.

C. Generalizadas mioclónicas.

D. Espasmos infantiles.

E. Generalizadas tónico - clónicas.

16.Sobre los síndromes miasténicos, señale lo correcto:

A. En la miastenia gravis, las anomalías tímicas son poco frecuentes.

B. El síndrome de Lambert Eaton se asocia concáncer prostático.

C. En el botulismo se produce bloqueo postsináptico.

D. El hipertiroidismo no debe considerarse en el diagnóstico diferencial.

E. Pueden ser desencadenados por aminoglucósidos.

17.La esclerosis múltiple se asocia a:

A. Una distribución geográfica característica.

B. Presencia de anticuerpos contra el sarampión en el líquido cefalorraquídeo.

C. Ciertos antígenos de histocompatibilidad (HLA).


D. Elevación de inmunoglobulinas oligoclonales en el LCR.

E. Todo lo anterior.

18.En relación al aumento de la prevalencia de la obesidad y la inactividad física en la población, se h a incrementado la frecuencia de
un trastorno denominado síndrome metabólico. Señale cuál de las siguientes características NO forma parte del síndrome metabó lico:

A. Hipertrigliceridemia.

B. Hipertensión arterial.

C. Hiperglucemia en ayunas.

D. Aumento del perímetro de la cintura abdominal

E. Aumento de la concentración plasmática de colesterol-LDL.

19.Uno de los siguientes signos NO es característico de la enfermedad de Graves Basedow:

A. Hipertiroidismo.

B. Macroglosia.

C. Exoftalmos.

D. Bocio difuso.

E. Mixedema pretibial.

20.En la síntesis de hormonas esteroideas a partir del colesterol, el primer paso regulatorio es la formación de:

A. Aldosterona.

B. Cortisol.

C. 17-hidroxiprogesterona.

D. Pregnenolona.

E. Tertosterona.

21.En el diagnóstico de polimiositis, la enzima que se incrementa significativamente es:

A. Transaminasa oxalacética

B. Transaminasa pirúvica

C. Deshidrogenasa láctica

D. Creatinfosfoquinasa

E. Aldolasa

22.En una esclerosis sistémica progresiva (esclerodermia) el peor pronósti co se asocia con:

A. Una extensa calcinosis cutánea.

B. La presencia de disfagia.

C. El número de articulaciones inflamadas.

D. La presencia de insuficiencia renal.

E. Fenómeno de Raynaud severo con úlceras necróticas en dedos.

23.La esquizofrenia implica cambios patológicos a nivel de:

A. Hipotálamo.

B. Sistema reticular.

C. Lóbulo occipital.

D. Cerebelo.

E. Lóbulo frontal, temporal y ganglios de la base.


24.Señale cuál de los siguientes tratamientos está indicado para el trastorno por déficit de atención con hiperactividad:

A. Fluoxetina.

B. Haloperidol.

C. Sertralina.

D. Metilfenidato.

E. Clozapina.

25.¿Cuál de los siguientes parámetros resulta más útil en el diagnóstico diferencial de la necrosis tubular aguda por

fármacos?

A. Creatinina plasmática.

B. Urea plasmática.

C. Sodio urinario.

D. Eosinofilia en plasma.

E. Nivel de C3 en suero.

26.Un niño de 4 años presenta insuficiencia renal aguda y anemia marcada con abundantes esquistocitos en el frotis de sangre
periférica. El diagnóstico probable es:

A. Glomerulonefritis aguda.

B. Fracaso renal agudo isquémico.

C. Trombosis de las venas renales.

D. Síndrome hemolítico-urémico.

E. Insuficiencia renal hemoglobinúrica.

27.Cuando se observa una inmunofluorescencia directa positiva en la piel lesional y perilesional afectando a la sustancia intercelular
de la epidermis, podemos hacer el diagnóstico de:

A. Penfi goide ampollar.

B. Dermatitis herpetiforme.

C. Epidermólisis ampollar.

D. Pénfigo vulgar.

28.El diagnóstico de ectima estreptocócico se debe sospechar cuando se presentan una o más úlceras induradas princi - palmente en:

A. Región facial

B. Región cervical

C. Miembros inferiores

D. Región torácica

E. Miembros superiores

29.Tipos de anemia que se observan generalmente en los pacientes con paludismo agudo:

A. Macrocítica.

B. Hipocrómica.

C. Normocrómica.

D. Drepanocítica.

E. Microcítica.
30.Los niveles de INR para control de anticoagulación oral deben mantenerse entre 2 y 3, con la excepción de una de las sigui entes
indicaciones:

A. Trombosis venosa profunda.

B. Prótesis valvular cardíaca mecánica.

C. Fibrilación auricular en estenosis mitral.

D. Disfunción severa de ventrículo izquierdo.

E. Tromboembolismo pulmonar.

31.¿En cuál de los siguientes casos usaría ligadura elástica como tratamiento?

A. Hemorroides de primer grado.

B. Hemorroides de segundo grado no trombosada.

C. Hemorroides de tercer grado.

D. Pólipo rectal pediculado.

E. Pólipo simple de recto.

32.Las complicaciones de las hemorroides son:

A. Tromboflebitis hemorroidal.

B. Trombosis hemorroidal.

C. Hematoma perianal.

D. Ninguna de las anteriores.

E. Todas ellas.

33.¿Cuál de las siguientes derivaciones portosistémicas es selectiva?

A. Porto-cava término-lateral.

B. Porto-cava látero-lateral.

C. Espleno-rectal convencional.

D. Interposición porto-cava.

E. Espleno-rectal distal.

34.En relación con el carcinoma de vesícula biliar, señale lo CORRECTO:

A. La frecuencia es menor en pacientes con “vesícula de porcelana”.

B. El cociente varón/mujer es igual a 2:1.

C. El adenocarcinoma no es el más frecuente.

D. El 90% de casos se asocia a cálculos de vesícula.

E. El tratamiento adyuvante con quimio o radioterapia es eficaz.

35.En el carcinoma de páncreas se presentan los siguientes síntomas, EXCEPTO:

A. Dolor abdominal.

B. Ictericia.

C. Signo de Courvosier-Terrier.

D. Bloqueo A-V.

E. Pérdida de peso.

36.¿Por qué mecanismo se da el prendido de un autoinjerto de piel?

A. Tejido epitelial.
B. Incremento de bordes.

C. Crecimiento por profundidad.

D. Tejido de granulación.

E. Conexiones vasculares.

37.El estado físico (ASA) clasifica a los pacientes en función de:

A. La gravedad de las enfermedades preoperatorias con la mortalidad.

B. La morbilidad y mortalidad intra operatoria.

C. La gravedad de la enfermedad y la reserva fisiológica.

D. La mortalidad anestésica secundaria.

E. La gravedad de la enfermedad y los accidentados anestésicos.

38.¿Qué tipo de colección contiene un seroma?

A. Serosa.

B. Linfática.

C. Intestinal.

D. Purulenta.

E. Hemática.

39.La característica más importante de la hernia incarcerada es:

A. Tener contenido visceral reductible.

B. Tener contenido irreductible con conservación de su circulación.

C. Tener contenido esfacelado por bloqueo de su circulación.

D. Tener contenido sólo de epiplón.

E. Ninguna anterior.

40.¿Cuál de los siguientes factores es el más común en la producción de evisceraciones?

A. Edad del paciente.

B. Anemia.

C. Hipoproteinemia.

D. Distensión abdominal.

E. Diabetes mellitus.

41.La complicación más seria después de una resección gástrica es:

A. Tromboflebitis.

B. Escape del muñón duodenal.

C. Hemorragia.

D. Sepsis.

E. Atelectasia.

42.¿Cuál de las afirmaciones define el concepto de politraumatismo?

A. Traumatismo craneoencefálico y lesión visceral.

B. Lesiones viscerales y periféricas.

C. Dos o más lesiones periféricas.

D. Lesiones traumáticas múltiples.


E. Lesiones traumáticas múltiples que ponen en riesgo la vida del paciente

43.En una lesión de la vesícula biliar por arma de fuego, ¿Cuál de los siguientes procedimientos es de elección?

A. Colecistectomía.

B. Sutura de la vesícula.

C. Colecistectomía + coledocotomía.

D. Colecistostomía.

E. Colecistostomía y drenaje de Petzer

44.La enfermedad congénita de las vías biliares denominada “Enfermedad de Caroli”, tiene la siguiente base anatómica:.

A. Dilatación quística del colédoco,.

B. Atresia biliar extra hepática..

C. Atresia biliar intrahepática..

D. Dilatación de los conductos biliares intrahepáticas..

E. Vesícula intrahepática.

45.Señale cuál síndrome NO se asocia con un aumento en la incidencia de cáncer colorrectal:

A. Poliposis colónica familiar.

B. Síndrome de Gardner.

C. Síndrome de Turcot.

D. Síndrome de Lynch.

E. Síndrome de Peutz – Jeghers.

46.Nulípara de 20 años de edad, con antecedente de infección de transmisión sexual y enfermedad inflamatoria pélvica a repetición.
Conoce que su pareja tiene otras compañeras sexuales. Ella no desea concebir y quiere optar por algún método de planificación
familiar. ¿Cuál es el método de elección?

A. Píldoras anticonceptivas.

B. Métodos naturales.

C. Métodos de barrera.

D. Dispositivos intrauterinos.

E. Anticoncepción parenteral.

47.En el diagnóstico diferencial del dolor pélvico agudo, la enfermedad o disfunción ginecológica MENOS probable de considera r es:

A. Endometriosis.

B. Embarazo ectópico roto.

C. Enfermedad pélvica inflamatoria aguda.

D. Quiste ovárico funcional hemorrágico.

E. Degeneración de leiomioma.

48.Paciente de 22 años, con tiempo de enfermedad de 3 días. Examen ginecológico: vesículas en racimo sobre labio mayor y
adenopatías inguinales homolaterales dolorosas. ¿Cuál es el diagnóstico PROBABLE?

A. Chancro sifilítico.

B. Condiloma vulvar.

C. Herpes genital.

D. Absceso perineal.
E. Linfogranuloma venéreo.

49.El antibiótico de elección en una mujer de 32 años de edad, con cervicitis mucopurulenta y PCR positivo para Chla - mydia
trachomatis es:

A. Azitromicina.

B. Ciprofloxacina.

C. Doxiciclina.

D. Metronidazol.

E. Penicilina benzatínica.

50.¿Hacia qué órgano se disemina con MÁS frecuencia el carcinoma cervical?

A. Hueso

B. Pulmones

C. Vejiga.

D. Cuerpo uterino.

E. Hígado.

51.Mujer de 55 años con tumoración que protruye por genitales externos. Al examen tiene una longitud de vagina de 7. El punto Ba
es +6. Según la clasificación de prolapso de órganos pélvicos (POP-Q), ¿Cuál es el grado de distopia genital?

A. III

B. I

C. II

D. V

E. IV

52.Los quistes de ovario con contenido de color achocolatado se observan en:

A. Endometriosis.

B. Quiste folicular.

C. Cistoadenoma de ovario.

D. Todas las anteriores.

E. Ninguna de las anteriores.

53. Entre las alteraciones que presenta la mujer geriátrica debido al hipoestrogenismo se incluyen las siguientes, EXCEP - TO:

A. Vaginitis.

B. Dispareunia.

C. Polaquiuria.

D. Colpocitología con predominio de células superficiales.

E. Osteoporosis.

54.¿Cuál de los siguientes marcadores de cromosomopatías del primer trimestre tiene MÁS valor?

A. Gonadotropina coriónica.

B. La alfa-fetoproteína.

C. La PAPP–A.

D. La sonoluscencia nucal.

E. El acortamiento del femur.


55.Paciente de 28 años, hace 39 días fue sometida a legrado uterino; presenta hemoptisis, ginecorragia escasa. Examen clínico PA :
120/70 mmHg, FC: 102 x minuto, útero de 14 cm, cérvix normal. Dosaje de HCG-B: 32000 Ul/ml. Hemog- lobina: 10.2 g/dL. ¿Cuál es
el diagnóstico PROBABLE?

A. Reinicio del ciclo menstrual.

B. Coriocarcinoma.

C. Retención de restos placentarios.

D. Endometritis.

E. Adenocarcinoma de endometrio.

56.Primigesta de 35 años de edad, con 37 semanas de gestación, ingresa a la Emergencia con san grado vaginal rojo ruti- lante sin
dolor; no obstante, la frecuencia cardíaca fetal se mantiene estable en 150 por minuto. ¿Qué procedimiento indicaría usted?

A. Examen digital del cérvix.

B. Test no estresante.

C. Inducción del trabajo de parto.

D. Ruptura de membranas.

E. Examen de ultrasonografía.

57.Son criterios de diagnóstico clínico de corioamnionitis, EXCEPTO:

A. Taquicardia materna mayor de 100 latidos por minuto.

B. Dolor y contracciones uterinas cada 3 minutos.

C. Taquicardia fetal mayor de 160 latidos por minuto.

D. Hipersensibilidad uterina.

E. Fiebre materna mayor de 38 grados.

58.Primigesta de 39 semanas, al examen obstétrico revela abdomen elongado en sentido vertical, cabeza fetal a nivel del fondo
uterino y miembros fetales en flanco derecho materno, con latidos fetales nítidos en cuadrante superior izquier- do. Señale la
situación, presentación, posición y probable variedad de posición:

A. Longitudinal podálica izquierda y sacra izquierda anterior.

B. Longitudinal podálica izquierda y sacra izquierda posterior.

C. Longitudinal podálica derecha y sacra derecha anterior.

D. Longitudinal podálica derecha y sacra posterior.

E. Longitudinal sacro izquierda variedad anterior.

59.¿Cómo clasificaría a una gestación que antes del embarazo tenia tensiones arteriales normales; que en la primera consulta,
realizada en la semana 8 de edad gestacional, se le detecta una tensión arterial de 140/90 mmHg; y que en la semana 28 tiene una
tensión arterial de 170/110 mmHg, sin edemas, y con una proteinuria en orina de 24 horas de 300 mg?

A. Preeclampsia.

B. Preeclampsia grave.

C. Hipertensión inducida por el embarazo.

D. Hipertensión crónica.

E. Hipertensión crónica con preeclampsia sobreañadida.

60.El embarazo es afectado por la diabetes como sigue:

A. Aumenta la incidencia de hidramnios.

B. Aumenta la incidencia de toxemia.

C. Aumenta la incidencia de malformaciones congénitas.

D. Todas las alternativas son correctas.


E. Ninguna de las anteriores.

61.Paciente gestante con diagnóstico de SIDA y sin tratamiento previo, llega a la Emergencia en periodo expulsivo. El tratamiento
retroviral elegido debe ser:

A. Zidovudina en la madre y en el neonato.

B. Lamivudina en la madre.

C. Zidovudina y Lamivudina en la madre.

D. Zidovudina en la madre.

E. Lamivudina en el neonato.

62.Puérpera de 7 días, regresa al hospital presentando fiebre de 39,5 °C, cefalea, dolor de bajo vientre, útero subinvolu - cionado con
loquios purulentos y fétidos. ¿Cuál es el diagnóstico MÁS PROBABLE?

A. Infección urinaria aguda.

B. Endometritis puerperal.

C. Vulvovaginitis mixta.

D. Vaginosis bacteriana.

E. Infección pélvica aguda.

63.En los primeros instantes del parto el recién nacido pierde más calor por:

A. Evaporación.

B. Radiación.

C. Conducción.

D. Convección.

E. Por todos ellos.

64.A las 32 semanas de gestación, en una sala de partos con temperatura ambiental de 23,9ºC, nace un bebé que pesa 1400 gms.
Pocos minutos después del nacimiento, este recién nacido tiende a manifestar los siguientes signos, EXCEPTO:

A. Palidez.

B. Escalofríos.

C. Disminución de la temperatura corporal.

D. Aumento de la frecuencia respiratoria.

E. Acidosis metabólica.

65.Lo más probable, ante la presencia de un área de pigmentación obscura en zona del sacro y nalgas en un recién nacido:

A. Eritema tóxico.

B. Melanoma maligno.

C. Mancha mongólica.

D. Hemangioma plano.

E. Meningomielocele oculto.

66.El tratamiento empírico inicial de la sepsis neonatal de inicio precoz es:

A. Meticilina y cefotaxima.

B. Ampicilina y cefotaxima.

C. Vancomicina y amikacina.

D. Ampicilina y gentamicina.

E. Penicilina G y kanamicina.
67.La documentación de sepsis en el recién nacido es a veces difícil. ¿Cuál de las siguientes localizaciones tiene un bajo ín dice de
positividad?

A. Líquido cefalorraquídeo.

B. Cultivos de aspirado endotraqueal.

C. Sangre venosa.

D. Urocultivo.

E. Cultivos de la arteria umbilical tomados al momento de la inserción.

68.La fenilcetonuria es producida por falta de:

A. Hidroxilasa de la fenilalanina.

B. Transaminasa de la fenilalanina- cc - cetoglutarato.

C. Oxidasa del homogentisato.

D. Descarboxilasa de la DOPA.

E. Sintetasa de la cistationina.

69.¿Qué afección se presenta con mayor frecuencia en recién nacidos que han sido objeto de maniobras de resucitación?

A. Fractura de costillas.

B. Neumotórax.

C. Hemorragia cerebral.

D. Todas las anteriores.

E. Ninguna de las anteriores.

70.El calostro se produce durante los primeros.....después del parto:

A. 7 días.

B. 3 días.

C. 2 días.

D. 5 días.

E. 10 días.

71.Montar un triciclo, hacer torres de 10 cubos, decir su edad y sexo, y comunicarse con lenguaje gramatical, son logros que
normalmente deben adquirirse a los:

A. 2 años.

B. 3 años.

C. 4 años.

D. 6 años.

E. 5 años.

72.Un niño que acude a control al mes de edad tiene las siguientes medidas: talla 52cm, peso: 4kg y perímetro cefálico: 37cm,
correspondiéndoles dichas medidas al percentil 40. Si el niño recibe lactancia materna exclusiva y es un niño sano, Ud. esper ará en el
siguiente control mensual:

A. Que el perímetro cefálico pase el percentil 80.

B. Que sólo la talla persista en el percentil 40.

C. Que el peso suba al percentil 90.

D. Que todas sus medidas continúen en el percentil 40.

E. Ninguna de las anteriores.

73.¿Cuál de las siguientes malformaciones congénitas, es la más frecuente en nuestro medio?


A. Atresia intestinal.

B. Hipertrofia pilórica.

C. Ano imperforado.

D. Hernia hiatal.

E. Ninguna de ellas.

74.¿Cuál de las siguientes es la fórmula dental para los dientes caducos (de leche)?

A. 2-1-2-.

B. 2-1-.

C. 3-2-1-.

D. 2-1-1-.

E. 2-1-.

75.De los siguientes antecedentes en el RN sometido a cuidados intensivos, ¿cuáles son la indicación para estudio de la audic ión?

1. Peso al nacer menor de 2.500 g.

2. Asfixia perinatal.

3. Hemorragia intracerebral.

4. Hiperbilirrubinemia indirecta mayor de 20 mg/dL.

5. Taquipnea transitoria del recién nacido.

6. Uso de aminoglucósidos.

7. Trisomía 21. A. 1,2,3,4,6.

B. 1,2,4,5,6.

C. 1,3,4,6,7.

D. 2,3,4,5,6.

E. 2,3,5,6,7.

76.Un lactante de 10 meses que presenta vómitos y diarreas, taquicardia, PA normal, mucosas secas, llenado capilar me -nor de 2
segundos e irritabilidad. ¿Qué porcentaje de peso ha perdido este paciente?

A. 13-15%.

B. 3-5%.

C. 1-2%.

D. 6-9%.

E. 10-12%.

77.Señale el enunciado CORRECTO con relación a las infecciones del SNC en niños:

A. La etiología específica es la más frecuente

B. Las causadas por Rickettsias son frecuentes.

C. Las de origen viral son las más frecuentes

D. Las debidas a hongos y parásitos son más frecuentes que las bacterianas

E. Las especies de Mycoplasma no producen infección neurológica.

78.Nos traen a Urgencias a un escolar de 7 años de edad, que estando totalmente bien, presenta fiebre de 40ºC, acompaña - da de
vómitos, cefaleas, pulso rápido y algo débil, con sensación de enfermedad g rave. A la exploración destacan amíg- dalas hipertróficas
con exudado blanco-amarillento, así como enantema en paladar blando. Además presenta adenitis satélite en ángulo mandibular
izquierdo. A las 14 horas aparece exantema puntiforme a la palpación que respeta el ángulo nasogeniano. A los 3-4 días se inicia
descamación furfurácea. El tratamiento etiológico de este proceso es:
A. Macrólidos por 5 días.

B. Tratamiento sintomático.

C. Penicilina oral durante 10 días.

D. Penicilina benzatina en 3 dosis consecutivas.

E. Asociar corticoides al antibiótico elegido.

79.En un niño con glomerulonefritis difusa aguda post-estreptocócica, ¿cuál de los siguientes hallazgos puede observarse en la
biopsia renal?

A. Fibrosis periglomerular.

B. Engrosamiento de la membrana basal glomerular.

C. Esclerosis mesangial.

D. Medias lunas.

E. Proliferación endocapilar difusa

80. El divertículo de Meckel puede:

A. Invaginarse y provocar intususcepción.

B. Secretar Ácido gástrico y perforarse.

C. Provocar obstrucción del intestino delgado.

D. Ulcerarse y provocar hemorragia.

E. Todo lo anterior.

81. Analizó los determinantes de la Salud y construyó un modelo que ha pasado a ser clásico en Salud Pública:

A. Rudolf Virchow

B. Edwin Chadwick

C. Sara Baker

D. Marc Lalonde

E. Michael Marmot

82.La Ley General de Salud (ley N° 26842) establece que la salud pública es responsabilidad:

A. Primaria de las organizaciones de la sociedad civil

B. Compartida por la sociedad y el estado

C. Primaria del estado

D. Compartida por el individuo y la sociedad

E. Compartida por el individuo, sociedad y estado

83.La fluorización a través de enjuagues bucales en niños de 6 años, corresponde a una actividad de...

A. Control sanitario

B. Prevención secundaria

C. Prevención terciaria

D. Promoción sanitaria

E. Prevención primaria

84.Cuál es el proceso que proporciona información necesaria a las personas, familia y comunidad para mejorar la salud y una m ayor
control sobre los determinantes sociales?

A. Visitas domiciliarias.

B. Protocolos de salud.
C. Participación social.

D. Alianzas estratégicas.

E. Interacción comunitaria.

85.No es un objetivo de un sistema sanitario:

A. Equidad de los usuarios

B. Eficacia micro y macroeconómica

C. Atención continuada

D. Accesibilidad de los usuarios

E. Satisfacción de los usuarios

86.La secuencia lógica del proceso administrativo es:

A. Organización-dirección-control-planificación

B. Dirección-planificación-organización-control

C. Planificación-organización-dirección –control

D. Planificación-dirección-organización-control

E. Organización-planificación-dirección-control

87.¿Qué se desarrolla en la administración de un establecimiento nivel I, al relacionar las actividades de sus diversas unida des para
alcanzar las metas propuestas?

A. Organización

B. Negociación

C. Motivación

D. Planificación

E. Coordinación

88.En el análisis FODA de un hospital, “los permanentes conflictos entre la dirección y el cuerpo médico” corresponde a:

A. Debilidad

B. Amenaza

C. Fortaleza

D. Oportunidad

E. Recurrencia

89.Un contraste de hipótesis:

A. Es un tipo de estadística descriptiva

B. La hipótesis nula plantea la existencia de diferencias.

C. La hipótesis alternativa plantea la no diferencia.

D. La hipótesis nula y la alterativa pueden no ser excluyentes.

E. Se puede saber la probabilidad de equivocarse en la afirmación.

90.¿Cómo se denomina la tasa calculada a partir del número de niños de edad superior a 28 días y menores de 1 año, que mueren en
un año por cada mil recién nacidos vivos?

A. Mortalidad neonatal

B. Morbilidad neonatal

C. Morbilidad postneonatal

D. Mortalidad postneonatal
E. Mortalidad infantil

91.Uno de los siguientes estadísticos NO es de dispersión:

A. Desviación estándar.

B. Desviación media.

C. Cuartil.

D. Rango intercuartilico.

E. Coeficiente de variación.

92.Se desea estimar la prevalencia de una enfermedad que se sospecha tiene un comportamiento estacional. Para obtener una
muestra representativa se debe optar por el muestreo:

A. Estratificado

B. Aleatorio simple con restitución

C. Aleatorio simple sin restitución

D. Sistemático

E. Por conglomerado

93.En Epidemiología, ¿Cuáles son fuentes de información primaria?

A. Observación, registros HIS, grupos focales

B. Grupos focales, historias clínicas, registros HIS

C. Reportes de necropsias, grupos focales, observación

D. Encuestas, observación, grupos focales

E. Censo poblacional, grupos focales, registros HIS

94.¿Cuál de las siguientes sustancias inhibe la motilidad intestinal?

A. Noradrenalina.

B. Insulina.

C. Motilina.

D. Colecistocinina.

E. Serotonina.

95.El músculo sartorio está inervado por el nervio:

A. Glúteo mayor.

B. Glúteo menor.

C. Ciático poplíteo interno.

D. Femoral.

E. Femorocutáneo.

96.Indicar los músculos que forman el canal del pulso:

A. Supinador largo y palmar mayor.

B. Supinador largo y palmar menor.

C. Palmar mayor y palmar menor.

D. Abductor largo y extensor largo del pulgar.

E. Supinador largo y primer radial externo.

97.La arteria meníngea media atraviesa el agujero:


A. Esfenopalatino.

B. Oval.

C. Redondo mayor.

D. Redondo menor.

E. Rasgado anterior.

98.¿Cuál es el origen embriológico de la corteza suprarrenal?

A. Mesodermo.

B. Endodermo.

C. Ectodermo.

D. Neuroblasto.

E. Células pluripotenciales.

99.El epitelio de la vejiga urinaria es de tipo:

A. Transicional.

B. Poliestratificado cornificado.

C. Poliestratificado no cornificado.

D. Cilíndrico pseudoestratificado.

E. Cilíndrico monoestratificado.

100.¿Cuál es la característica de un electrocardiograma normal?

A. Onda R positiva en aVR.

B. Onda T negativa en derivación I.

C. Onda Q en derivación I.

D. Onda T negativa, en derivación II.

E. Onda P positiva en derivación II.

SIMULACRO 102

1.En la autopsia de una mujer de 24 años se encuentra pleuritis, engrosamiento de la membrana de las paredes del capilar
glomerular, anillos concéntricos de colágeno alrededor de las arteriolas esplénicas y excrecencias en la parte inferior de la válvula
mitral. Es más probable que el análisis de sangre evidencie:

A. Anticuerpos antinucleares.

B. Concentraciones elevadas de C3.

C. Linfocitosis.

D. Gammapatía monoclonal.

E. Anticuerpos antimembrana.

2.Paciente mujer de 59 años, acude a la consulta con dolor crónico en cintura escapular y rigidez de cuello. También refiere rigidez en
región lumbar que le imposibilita agacharse a recoger objetos. Refiere estos síntomas desde hace 2 a 3 años. A la exploración se
evidencia mucho dolor y rigidez a la palpación de musculatura de hombros y cervical pos - terior, así como en la musculatura
paravertebral lumbar. Resto del examen físico normal. Al ahondar en la anamnesis tiene dificultad para conciliar el sueño. VSG: 20.
¿Cuál es el diagnóstico y tratamiento de elección?

A. Fibromialgia - AINES y ansiolíticos.

B. Espondilitis anquilosante - Corticoides a dosis bajas.

C. Polimiositis - Azatioprina.

D. Polimialgia reumática - Corticoides a dosis altas.

E. Depresión atípica - Antidepresivos tricíclicos.


3.Paciente de 54 años que desde hace unos meses, y después de iniciar una dieta, comienza a presentar un cuadro caracteriz ado por
astenia, anorexia con pérdida de peso, apatía, anhedonia, abulia, sentimientos de minusvalía y desesperanza, insomnio de desp ertar
precoz y mejoría vespertina. Antecedentes personales: episodio depresivo hace 20 años, reactivo a la muerte de un fam iliar.
Antecedentes familiares de depresión. Se descarta patología somática. El diagnóstico más probable es:

A. Depresión endógena.

B. Depresión reactiva.

C. Depresión crónica.

D. Depresión caracterial.

E. Depresión orgánica.

4.¿Cuál de los siguientes es un síntoma característico de la crisis de angustia?:

A. Euforia.

B. Irritabilidad.

C. Labilidad emocional.

D. Sensación de muerte.

E. Hiperactividad.

5.El incremento de eritropoyetina se relaciona con:

A. Carcinoma gástrico.

B. Adenocarcinoma de ovario.

C. Carcinoma de riñón.

D. Carcinoma de endometrio.

E. Metástasis a médula ósea.

6.Ante un enfermo con anemia megaloblástica que va a iniciar tratamiento con vitamina B12 ¿Cuál de las siguientes
recomendaciones es la más aconsejable?

A. Procurar tomarlo en ayunas.

B. Asociar administración de Vitamina C.

C. Asociar administración de ácido fólico.

D. La dosis elemental de Fe será 150 - 200 mg al día.

E. Mantener tratamiento mayor o igual a 6 meses.

7.El mecanismo principal de la anemia de las enfermedades crónicas es:

A. Bloqueo del hierro en los macrófagos sin poderlo utilizar los precursores eritroides.

B. Disminución de la absorción intestinal de hierro.

C. Disminución de la producción de eritropoyetina.

D. Acortamiento de la vida media del hematíe.

E. Defectos adquiridos en la membrana de los hematíes.

8.Una mujer de 56 años consulta por ptosis palpebral y debilidad muscular de la hemicara derecha, de aparición brusca. Tambié n
refiere tinitus. El examen muestra: parálisis que compromete las partes superior e inferior de la hemicara derecha, desviación de la
comisura labial hacia la izquierda y signo de Bell con reflejo corneano ausente, ageusia en los dos tercios anteriores de la lengua.
¿Cuál es el diagnóstico MÁS probable?:

A. Parálisis facial central por accidente cerebrovascular isquémico.

B. Parálisis facial (a frigore).

C. Distrofia muscular oculofaríngea.

D. Masa ocupante cortical izquierda.


E. Ninguna de las anteriores.

9.Paciente con diarrea sanguinolenta, con deshidratación sin fiebre, leucocitos: 8:00, Hb: 7 gr. Plaquetas 20,000. DHL: 800 U. Subraye
el Diagnóstico probable:

A. Shigella.

B. Estafilococo.

C. Giardia lamblia.

D. Clostridium dificile.

E. Coli 0 157: H7.

10.Varón de 34 años, procede de Lima, consulta por fiebre de 39 .5º. R: 22 por min, Hepato y esplenomegalia. Ictericia de piel y
mucosas, laboratorio: Hb: 8 gr%. Leucocitos: 3,800, Neutrófilos 60%. Abastonados: 14%. Linfocitos: 18%. Bilirrubinas: 6mg%. D irecta:
3.5, TGP: 300, TGP: 350. F. Alcalina: 800 u. Aglutinaciones: = 1/80 H 1/160, Brucella (-). El diagnostico probable es:

A. Faciolasis Hepática.

B. Leptospirosis.

C. Absceso Hepático-Amebiano.

D. Colangitis Ascendente.

E. Fiebre Tifoidea.

11.Varón de 9 años, natural y procedente de zona endémica de dengue en Piura, sin antecedentes de dengue, hace 7 días febril, con
cefalea, artromialgias, náuseas y astenia. ¿Qué examen debe solicitar para confirmar el diagnóstico?

A. PCR para serotipificación.

B. IFI.

C. ELISA para dengue.

D. IFI viral por hisopado nasofaringeo

E. Western Blot para dengue

12.Varón de 47 años, alcohólico, consulta por la presentación brusca desde hace 72 horas de dolor intenso en el hemitórax der echo,
refiere además fiebre, escalofríos y sudoración desde hace 10 días, así como tos persistente con ab undante expectoración maloliente.
Rx de pulmones: consolidación redondeada con nivel hidroaéreo en el 1/3 medio del hemi - tórax derecho. Se envía muestras de
esputo para examen directo, cultivo, ¿qué tratamiento instauraría Ud.?:

A. Claritromicina + Amikacina.

B. Ceftriaxona + Clindamicina.

C. Doxiciclina + Amoxicilina con Ácido clavulánico.

D. INH + RFP + PZA + Etambutol.

E. Cotrimoxazol + Ciprofloxacino.

13.Paciente con derrame pleural hemático y masa en pared torácica es altamente sospechoso de:

A. Tuberculosis.

B. Carcinoma broncogénico.

C. Histoplasmosis.

D. Mesotelioma.

E. Hidatidosis torácica.

14.El diagnostico de tromboembolismo pulmonar se confirma mediante:

A. Radiografía de pulmones

B. Gammagrafía pulmonar

C. Electrocardiograma
D. Estudio de gases arteriales

E. Angio TC

15.La causa más importante de bronquioectasias es:

A. Inflamación necrosante de origen infeccioso.

B. Anormalidades hereditarias.

C. Anomalíascongénitas.

D. Fibrosis quística.

E. Asma bronquial.

16.Varón de 49 años concurre a emergencia por disnea severa, cianosis Presión arterial: 220/140. A. Respiratoria: Cre - pitantes en
2/3 inferiores de ambos hemitórax, Rx. de Pulmones: Sombras densas bilaterales en alas de mariposa. La terapia indicada sería :

A. Ceftriaxona + Claritromicina.

B. Prednisona.

C. Sufametozazole.

D. Ciclofosfamida + Prednisona.

E. Furosemida + Cedilanid.

17.Anciano de 76 años, es conducido a Emergencia por Lipotimia. Al examen: Paciente recuperado, pulso: 41 por minu - to,
regular.PA: 110/70. Esta sudoroso y pálido; Ud. dispone se realice: ECG y análisis mientras tanto; Ud. recomienda la siguiente terapia:

A. Glucosa al 10%, 1 litro 50 gotas por 1.

B. Cedilanid 0.4 mg 1 ampolla VEV.

C. Sulfato de Atropina ½ mg. VEV.

D. Atenelol 50 mg. VO.

E. Amiodarona V.O.

18. Una paciente de 32 años de edad presenta una auscultación cardíaca con un click mesosistólico y un soplo mesotelesis -
tólico que se incrementa de intensidad con la maniobra de Valsalva. El diagnóstico de presunción será:

A. Miocardiopatía hipertrófica.

B. Comunicación interauricular.

C. Estenosis mitral.

D. Estenosis aórtica.

E. Prolapso mitral

19.En un paciente diagnosticado de pancreatitis aguda, a los dos días de su evolución, ¿cuál de los siguientes datos carece d e valor
para pronosticar su severidad?:

A. Score tomográfico.

B. Valores de amilasa y/o lipasa sérica elevados más de 10 veces el límite normal.

C. Tener unos valores de proteína C reactiva sérica superiores a 120 mg/dl.

D. Presentar una puntuación APACHE II (Acute Physiology and Chronic Health Evaluation) superior a 8.

E. Presentar positivos 3 ó más criterios de Ransom.

20.Paciente de 69 años, bebedor y fumador con anorexia y disminución de peso, malestar abdominal, sensación de plenitud gástr ica y
cambios en el hábito intestinal. A exploración: tinción ictérica de piel, ganglio supraclavicular izquierdo y axilar anterior izquierdo
palpable, hiperpigmentación axilar y queratosis pruriginosa aguda. Diagnóstico más probable:

A. Adenocarcinoma gástrico precoz.

B. Adenocarcinoma gástrico avanzado.

C. Carcinoma epidermoide gástrico avanzado.


D. Leiomiosarcoma gástrico.

E. Linfoma gástrico.

21.Paciente de 35 años de edad con episodios de diarrea de presentación diurna y dolor hipogástrico que calma con la defecaci ón. No
fiebre, heces sin moco ni sangre. El diagnóstico más probable es:

A. Enfermedad inflamatoria del intestino.

B. Neoplasia de colon.

C. Colon irritable.

D. Giardiosis.

E. Salmonelosis.

22.De las siguientes pruebas, ¿Cuál se debe realizar para el diagnóstico de neurosífilis?:

A. Examen de campo oscuro en LCR.

B. FTA absorbido en LCR.

C. Inmovilización del Treponmeapallidum en LCR.

D. Inmunofluorescencia directa en LCR.

E. VDRL en LCR.

23.El síndrome anticolinérgico puede ser producido por:

A. Carbamatos.

B. Gases nerviosos: Sarín.

C. Neostigmina.

D. Flumazenilo.

E. Antiparkinsonianos.

24.Varón de 42 años conducido a Emergencia por dolor abdominal, despeños diarreicos y mareos. Desde hace 4 meses presenta
debilidad, laxitus, a veces desvanecimiento y episodios de oliguria. Ha perdido 10 kg de peso. Al examen:frialdad de piel, vi tíligo en
labios y manos. PA: 80/60 mmHg. Pulso: 110 x’. T°: 38 °C. Hb: 7 g% leucocitos: 14.000, eosinófilos 10%, linfocitos 40%. Na: 1 18
mEq/L. Cl: 110 mEq/L. K: 6 mEq/L. HCO3 -: 14 mEq/L. CA: 107 mg%. Glicemia: 45 mg/dL. Creatinina: 3 mg/dL. Na urinario: 60
mEql/L. ¿Cuál es su posibilidad diagnóstica?:

A. Déficit de la enzima 11 –hidroxilasa.

B. Gastroenteritis aguda.

C. Insuficiencia suprarrenal.

D. Necrosis tubular aguda.

E. Secreción inapropiada de hormona antidiurética.

25.Un varón con las siguientes características somáticas: hipogonadismo hipogonadotrófico, anosmía, falo pequeño, fe - notipo
masculino, hábito eunucoide y sin ginecomastia. ¿A qué cuadro corresponde?:

A. Síndrome de Klinefelter.

B. Síndrome de Noonan.

C. Síndrome de Del Castillo.

D. Síndrome de Kallman-Morsier.

E. Síndrome de Reifeinstein.

26.¿Cuál es el examen auxiliar más útil para el diagnóstico de monoartritis tuberculosa de rodilla?

A. Biopsia de membrana sinovial.

B. Historia clínica.

C. Rx de rodilla en dos posiciones.


D. Rx de tórax.

E. Ninguna de las anteriores.

27.Mujer de 40 años que cursa con debilidad muscular proximal. Además presenta en los párpados y alrededor de los mismos,
lesiones eritematosas y liláceas, asociadas a pápulas eritematosas en dorso de ambas manos. Estas lesiones s on típicas de:

A. Eccema de contacto.

B. Eccema atópico.

C. Dermatomiositis.

D. Lupus eritematoso sistémico.

E. Síndrome de Sjogren.

28.Paciente con cuadro obstructivo pilórico por úlcera péptica, con cuadro antiguo de vómitos persistentes, indique qué al teraciones
metabólicas se producen en el paciente:

A. Alcalosis hipoclorémica

B. Hiperpotasemia y acidosis

C. Acidosis hipoclorémica

D. Hipercloremia y alcalosis

E. Hipernatremia sin variación de pH

29.Una reacción de hipersensibilidad que produce necrosis de los desmosomas epidérmicos se ve en:

A. Verruga Peruana.

B. Verruga vulgar.

C. Pénfigo vulgar.

D. Tuberculosis cutánea.

E. Reacción a la tuberculina.

30.Persona del sexo femenino de 38 años, murió el 10 de enero del 2006. Con antecedente de hospitalizaciones por hipere- mesis
gravídica y aborto recurrente en los años previos. Estando gestando, el 09 de enero del mismo año fue atropella -da por un automóvil
que le ocasionó fracturas múltiples en cráneo y extremidades inferiores. El día de su fallecimiento presentó infarto agudo de miocardio.
¿Cuál es la causa básica de muerte?:

A. Preeclampsia.

B. Fracturas múltiples en cráneo.

C. Accidente de tránsito.

D. Aborto.

E. Infarto del miocardio.

31. Los glucósidos digitálicos aumentan la contractilidad miocárdica principalmente por uno de estos mecanismos:

A. Abertura de los canales de calcio.

B. Liberación de calcio desde el retículo sarcoplásmico.

C. Estimulación de la ATPasa de la miosina.

D. Estimulación de la fosfolipasa C de la mem brana.

E. Inhibición de la ATPasa de Na+ - k + de la membrana.

32. En la formación del polígono arterial de Willis, contribuyen las arterias:

A. Cerebrales anteriores y cerebelosas superiores.

B. Cerebrales medias y cerebelosas posteriores.


C. Cerebrales anteriores y cerebrales posteriores.

D. Cerebrales medias y cerebrales posteriores.

E. Comunicante media y cerebrales posteriores.

33.Es un derivado de los conductos mesonéfricos:

A. Tubos seminiferos.

B. Conducto uterino.

C. Vesículas seminales.

D. Glándulas de Cowper.

E. Testículos.

34.En la formación de la “barrera hemato-encefálica” la célula que participa es:

A. La de Schwann.

B. La astroglia.

C. La neuroglía.

D. La ependimaria.

E. El astrocito.

35.Un paciente de 15 años de edad presentó una hepa titis viral aguda. La evolución clínica fue favorable y la biopsia he - pática
posterior mostró una regeneración completa de los hepatocitos dentro de la estructura lobulillar normal. ¿Cuál es el tejido q ue permitió
mantener la arquitectura normal del órgano?:

A. Fibras elásticas.

B. Fibras colágenas.

C. Colágeno tipo I.

D. Glucoproteinas.

E. Fibras reticulares.

36.La alteración en la flexión del 4º y 5º dedos por las articulaciones interfalángicas distales al tratar de cerrar el puño, y al tratar de
flexionar la articulación de la muñeca la mano se desvía lateralmente hacia la cara radial. ¿Cuál será la lesión probable?:

A. Nervio cubital.

B. Nervio radial.

C. Nervio mediano.

D. Nervio radial y nervio cubital.

E. Nervio mediano y nervio radial.

37.Un paciente de 40 años presenta la siguiente gasometría arterial respirando aire ambiente: pH 7,30; pO2 56 mmHg; pCO2 46
mmHg ¿Cómo puede ser definida esta situación?:

A. Alcalosis metabólica.

B. lnsuficiencia respiratoria hipoxémica con alcalosis.

C. Acidosis con insuficiencia respiratoria hipoxémica e hipercápnica.

D. Acidosis metabólica con hiperventilación secundaria.

E. Alcalosis respiratoria crónica.

38.Entre el grupo de fármacos antagonistas de los receptores de angiotensina II (ARA -II), señale el que tiene mayor afinidad por el
receptor AT1

A. Telmisartan.

B. Valsartan.
C. Losartan.

D. Irbesartan.

E. Olmesartan.

39.Una de las siguientes no es una cefalosporina de tercera generación. ¿De cuál se trata?:

A. Ceftriaxona.

B. Cefotaxima.

C. Ceftazidima.

D. Cefixima.

E. Cefazolina.

40.El sistema reticular activador ascendente del tronco cerebral se relaciona con:

A. El aprendizaje.

B. El contenido de la conciencia.

C. El mantenimiento del estado de alerta.

D. La coordinación de movimientos aprendidos.

E. Los comportamientos alimentarios.

41.¿Qué tipo de gemelos da origen la partición del disco embrionario más allá de los 13 días?:

A. Monoamniótico - bicoriónico.

B. Monoamniótico - monocoriónico.

C. Biamniótico – bicoriónico.

D. Biamniótico - monocoriónico.

E. Monstruos dobles.

42.La hemorragia vaginal, masa endocervical o vaginal y dolor, son una triada característica de:

A. Aborto en curso.

B. Placenta acreta.

C. Placenta previa de inserción baja.

D. Inversión uterina.

E. Alumbramiento incompleto.

43.Paciente de 29 años, con antecedente de un parto eutócico de 36 sem. hace 6 años, embarazo molar hace 4 años y embarazo
ectópico hace 11 meses. Actualmente evidencia una gestación de 24 semanas, con un estudio ecográfico que revela una gest ación
gemelar compatible con el tiempo de amenorrea. Señale su fórmula de paridad.

A. G: 4; P: 1-0-2-1.

B. G: 5; P: 0-1-1-1.

C. G: 5; P: 0-1-2-0.

D. G: 4; P: 0-1-2-1.

E. G: 4; P: 1-1-1-1.

44.Paciente de 28 años que ha tenido un parto por cesárea hace 48 horas, presenta temperatura de 39ºC luego de un tra - bajo de
parto prolongado y múltiples tactos vaginales, el diagnóstico más probable será:

A. Infección de herida operatoria.

B. Mastitis.

C. Tromboflebitis pélvica séptica.


D. Endometritis.

E. Fiebre medicamentosa.

45.Paciente gestante con diagnóstico de SIDA y sin tratamiento previo, llega a la emergencia en periodo expulsivo. El tratami ento
retroviral elegido debe ser:

A. Zidovudina en la madre y en el neonato.

B. Lamivudina en la madre.

C. Zidovudina y lamivudina en la madre.

D. Zidovudina en la madre.

E. Lamivudina en el neonato.

46.Mujer de 38 años, que presenta nódulo mamario indoloro, de bordes imprecisos. Mamografía: Imagen nodular con espículas en
todos sus márgenes y finas microcalcificaciones agrupadas en el interior. ¿Cuál es el diagnóstico más probable?:

A. Fibroadenoma.

B. Displasia mamaria.

C. Quiste.

D. Carcinoma.

E. Papiloma intraductal.

47.Una muchacha de 15 años de edad, sexualmente activa, acude a la emergencia quejándose de dolor agudo en la parte inferior del
abdomen, que incluso le dificulta la deambulación. Refiere que el dolor empezó dos días después de la terminación de su últim a
menstruación, y que padece de flujo vaginal cuyas características no precisa. En su criterio, ¿cuál sería el microorganismo que con
mayor probabilidad está causando este cuadro de EIP?:

A. Neisseria gonorrhoeae.

B. Bacteroides fragilis.

C. Actinomyces israelii.

D. Staphylococcus aureus.

E. Trichomona vaginalis.

48.Una joven de 24 años está usando correctamente anticonceptivos orales combinados de baja dosis (20 ug de etinilestra - diol)
desde hace 8 meses. Hace 2 meses que no menstrua y desea cambiar de método para colocarse DIU ¿Cuál sería la conducta
correcta?

A. Esperar que menstrúe.

B. Dosar b-HCG.

C. Colocar DIU.

D. Dar suplente de estrógenos.

E. Realizar ecografía pélvica.

49.Mujer de 56 años que refiere perdida de orina a grandes esfuerzos. ¿Cuál es el tipo de incontinencia urinaria presenta la paciente?:

A. Urgencia.

B. Esfuerzo.

C. Rebosamiento.

D. Mixta.

E. Vejiga hiperactiva.

50.Mujer de 35 años de casada, que luego de legrado uterino hace 6 meses, no presenta menstruación y sin actividad sexual has ta el
momento de la consulta. Al examen: Útero y anexos normales. El médico soli cita una ecografía ¿Cuál es el síndrome a tener en
cuenta como causa de amenorrea secundaria?:
A. Asherman.

B. Ovario poliquístico.

C. Sheehan.

D. Kalman.

E. Turner.

51.Una mujer de 28 años con anticuerpos anticardiolipina y antecedente de tres abortos en el primer trimestre del em - barazo es
evaluada en la sexta semana de un cuarto embarazo, el embarazo actual transcurre con normalidad, nunca ha sido tratada por la
positividad de los anticuerpos anticardiolipina, ¿Cuál de las siguientes es la co nducta más apropiada?

A. Observación estrecha.

B. Prednisona.

C. Aspirina.

D. Infusión intravenosa de inmunoglobulinas.

E. Heparina.

52.Paciente con 35 semanas de gestación, cesareada anterior 2 veces y diagnóstico de PPT es cesareada electivamente y se
encuentra que tiene placenta percreta. ¿Cuál sería el mejor tratamiento?:

A. Cesárea segmentaria más ligadura de trompas.

B. Cesárea corporal más ligadura de trompas.

C. Cesárea-histerectomia.

D. Método de B Lynch.

E. Taponamiento uterino.

53.Los desgarros perineales de tercer grado comprometen:

A. Horquilla, piel perineal y mucosa vaginal.

B. Piel, mucosa vaginal, cuerpo perineal y esfínter anal.

C. Piel, mucosa vaginal, aponeurosis y músculos del cuerpo perineal.

D. Mucosa rectal exponiendo la luz rectal.

E. Piel perineal y mucosa vaginal.

54.Gestante nulípara de 38 semanas de embarazo por amenorrea, con historia de pérdida de líquido, al examen se apre - cia salida de
líquido amniótico a través del cuello uterino, y prueba de papel de ni trazina es positivo. Al momento tiene 5 horas de ruptura de
membranas y no hay inicio de trabajo de parto. Cultivo de EGB negativo. ¿Cuál es el manejo más adecuado?

A. Programar cesárea.

B. Indicar cesárea de emergencia para evitar la infección materna y fetal.

C. Indicar antibiótico terapia y mantener expectante.

D. Iniciar inducción de parto.

E. Realizar ecografía y NST para tomar una decisión.

55.El parámetro ecográfico más útil para identificar el retardo del crecimiento intrauterino es:

A. El cociente circunferencia cefálica y abdominal

B. El diámetro biparietal

C. La circunferencia cefálica

D. La longitud del fémur

E. La longitud del húmero


56.Segundigesta de 36 años, en sala de partos RN un producto de 40 semanas de gestación y 2.960 gramos; el perio do de dilatación
fue de 1 hora y el expulsivo de 5 minutos, donde se utilizó oxitocina. La paciente en periodo de alumbra - miento después de pujar en 2
ocasiones y no desprenderse la placenta, siente sensación de desmayo y dolor abdominal. Sangrado vaginal. Constantes vitales
normales. Se realiza palpación abdominal con pérdida de fondo uterino. ¿Cuál sería su sospecha diagnostica?:

A. Rotura uterina.

B. Obstrucción intestinal.

C. Inversión uterina

D. Placenta percreta.

E. Miomatosis uterina.

57.Anomalía congénita asociada con oligohidramnios:

A. Atresia esofágica.

B. Anencefalia.

C. Displasia renal.

D. Espina bífida.

E. Hidrops fetalis.

58.Un paciente de 38 años de edad acude trasladado a nuestro servicio de Urgencias tras sufrir herida por ar ma blanca a nivel de
hemitórax derecho octavo espacio intercostal; a su ingreso presenta: tensión arterial de 120/60 mmHg; frecuencia cardíaca de 92
l/minuto y frecuencia respiratoria de 22 por minuto; la auscultación del hemitórax derecho demuestra disminución de los ruidos
respiratorios y la percusión del hemitórax demuestra timpanismo. ¿Cuál es la mejor conducta a seguir?

A. Tomografía computarizada de tórax.

B. Toracotomía urgente.

C. Observación clínica.

D. Radiología simple de tórax y según hallazgos colocación de drenaje pleural.

E. Ingreso en unidad de cuidados intensivos.

59.Un paciente de 42 años de edad, de sexo masculino, con antecedentes de asma bronquial, que es atropellado por auto -móvil, ha
sufrido traumatismo abdominal cerrado, presenta con más frecuencia:

A. Hematoma visible en pared abdominal.

B. Lumbalgia severa.

C. Signos de peritonismo.

D. Abdomen con poca reacción peritoneal.

E. Signos de shock hipovolémico.

60.Paciente varón de 40 años es ingresado a Emergencia debido a haber sufrido accidente automovilístico. Politraumati- zado, cursa
hipotenso, taquicárdico y con transtorno del nivel de conciencia, con pérdidas importantes de sangre. En un primer momento, l a
reposición de la volemia se logra mejor a través de:

A. Catéter de Swan-Ganz vía subclavia.

B. Disección de una vena de la flexura del codo.

C. Disección de la vena safena.

D. Catéter central vía yugular interna.

E. Dos cánulas venosas cortas percutáneas.

61.Uno de los siguientes datos es más característico de la hernia inguinal d irecta que de la indirecta:

A. Aparición en la infancia.

B. Forma alargada.

C. En la palpación digital del conducto inguinal, no se suele apreciar debilidad de la pared posterior.
D. Las maniobras de reducción son difíciles.

E. Ausencia de progresión hacia el escroto.

62.Paciente con el diagnóstico de hemorroides trombosadas, refiere mucho dolor en zona anal. El tratamiento quirúrgico prefer ido es:

A. Reblandecedores de heces y baños de asiento.

B. Incisión de las hemorroides y evacuación del coágulo sanguíneo.

C. Incisión de las hemorroides más esfinterectomía.

D. Ligadura de las hemorroides con bandas de caucho.

E. Inyección de una solución esclerosante

63.En el cáncer de colon izquierdo, la sintomatología del paciente está fundamentalmente en relación con:

A. La función absorbente del colon derecho

B. La naturaleza líquida de las heces

C. La existencia de un tumor úlcero-vegetante

D. El menor calibre luminal

E. La presencia del síndrome anémico

64.Paciente con insuficiencia respiratoria moderada o severa, se considera con riesgo quirúrgico (ASA) clase:

A. II

B. I.

C. III.

D. IV.

E. V.

65.En la fractura de la cadera, el tratamiento inicial debe ser:

A. Bota de yeso antirrotatoria.

B. Tracción esquelética supracondílea o de la tuberosidad tibial.

C. Tracción cutánea.

D. Pelvipedio.

E. Tracción esquelética supramaleolar.

66.En caso de luxación de cadera de un hombre joven, con 6 horas de evolución ¿Qué haría Ud.?

A. Dar analgésicos y transferirlo.

B. Reducir en forma incruenta y hacerle tracción continua.

C. Inmovilizarlo con un pelvipedio y transferirlo.

D. Reducir en forma cruenta y colocarle un pelvipedio.

E. Reducir e inmovilizarlo.

67.Las principales complicaciones del tratamiento de las fracturas son: infección, pseudoartrosis, retardo de la consoli- dación,
consolidación viciosa, rigidez articular, artrosis secundaria y:

A. Síndrome Compartimental.

B. Artritis.

C. Síndrome de aplastamiento.

D. Daño tendinosos muscular.

E. Atrofia ósea de Sudek.


68.El cálculo renal más frecuente es de tipo:

A. Mixto.

B. Oxalato calcio.

C. Ácido úrico.

D. Estruvita.

E. Cistina.

69.Niño de 12 años quien va regularmente a la piscina, cursa con otalgia intensa, signo del trago positivo y ocasional otorre a escasa
muy líquida. ¿Cuál es el diagnóstico más probable?

A. Otitis media aguda.

B. Otitis media crónica.

C. Otitis externa difusa.

D. Otocerumen.

E. Otitis externa circunscrita.

70.La adenopatía preauricular es más frecuente en las conjuntivitis causadas por:

A. Tóxicos

B. Irritantes

C. Atopia

D. Chlamydia y Virus

E. Bacterias

71.Neonato de 20 días de edad sin factores de riesgo, nace a termino con peso de 3000 gr Recibe lactancia materna exclu - siva. Es
traído a Emergencia por vómitos frecuentes que aumentan progresivamente desde hace 3 días. No diarrea, no fiebre. Peso actual:
3600 gr. El diagnóstico más probable es:

A. Estenosis hipertrófica congénita de píloro

B. Sepsis neonatal

C. Meningoencefalitis

D. Intolerancia a la lactosa

E. Reflujo gastroesofágico

72.Mujer de 50 años con cuadro de varios años de evolución de síndrome miccional irritativo y dolor hipogástrico que cede con la
micción. La ecografía renal y vesical, urografía, citología en orina, cultivos repetidos de orina, baciloscopía y exploración genital son
negativas. ¿Cuál es la actitud más correcta?:

A. No realizar más exploraciones.

B. Practicar cistomanometría.

C. Repetir la urografía intravenosa en 3 meses.

D. Realizar cistoscopia.

E. Realizar hidrodistensión vesical simple bajo anestesia.

73.La dermatitis atópica usualmente aparece a la edad de:

A. 2 semanas.

B. 3 meses.

C. 2 años.

D. 6 años.
E. Ninguna de las anteriores.

74.Niño de 10 meses con cuadro febril de 3 días de duración, sin otra sintomatología acompañante salvo irritabilidad con los periodos
de hipertemia. El cuarto día presenta aparición de exantema en tronco y desaparición de la fiebre. Respecto al cuadro clínico ci tado,
¿Cuál de las siguientes afirmaciones es verdadera?

A. El diagnóstico más probable es una infección por virus del sarampión.

B. El signo físico diagnóstico es la presencia de una amigdalitis exudativa.

C. El tratamiento indicado es amoxicilina oral.

D. Se asocia a una infección por virus herpético humano tipo 6.

E. La duración del exantema suele ser de 4 semanas.

75.Se admite en urgencia a un preescolar previamente sano de 15 meses con 3 horas de enfermedad caracterizada por rinorrea,
fiebre alta, convulsión tónicoclónica generalizada y un examen neurológico en el post - Ictal sin mayores al- teraciones. No tiene
antecedentes de convulsiones previas y no recibe medicación alguna. El diagnóstico más probable es:

A. Meningoencefalitis bacteriana.

B. Epilepsia secundaria.

C. Trastorno metabólico.

D. Convulsión febril.

E. Encefalomielitis.

76.Un infante de 10 meses que gozaba de buena salud, repentinamente llora por “cólico abdominal”, con severos episo - dios que
ocurren en un período de 3 horas, algunos acompañados de vómitos. El niño parece muy enfermo. Al examen hay signos de íleo,
distensión, vómitos, taquicardia. El abdomen parece no ser doloroso. La mejor descripción de las heces en esta patología es:

A. Agua de arroz.

B. En jalea de grosella.

C. En agua negra

D. Pastosas.

E. Alquitranadas.

77.Paciente de 12 años de edad, soporoso, quien presenta vómito, diarrea, sialorrea, m iosis, broncoespasmo, fasciculacio-nes
musculares y convulsiones. ¿Cuál es el diagnóstico probable?:

A. Intoxicación por organofosforados.

B. Meningitis bacteriana.

C. Gastroenteritis con deshidratación severa.

D. Intoxicación por salicilatos.

E. Epilepsia.

78.La principal causa de paro cardíaco en niños es:

A. Asistolia ventricular.

B. Insuficiencia respiratoria.

C. Fibrilación ventricular.

D. Alteraciones hidroelectrolíticas.

E. Endocrinas.

79.El lactante habitualmente comienza a mantenerse sentado a la edad de:

A. 4 a 5 1/2 meses.

B. 6 a 6 1/2 meses.

C. 8 a 8 1/2 meses.
D. 9 a 10 1/2 meses.

E. 11 a 12 meses.

80.Recién nacido de 12 horas de vida, de parto eutócico, peso 3000 gramos, asintomático, con madre que presenta sero - logía
positiva para hepatitis B. ¿Cuál es la conducta más apropiada en este recién nacido?

A. Administrar solo vacuna anti hepatitis B.

B. Administrar inmunoglobulina más vacuna contra hepatitis B.

C. Serología para hepatitis B.

D. Anticuerpos para hepatitis B.

E. Sólo vacuna contra hepatitis B.

81.Mujer de 11 años de edad, acude por presentar primer episodio de sangrado aparentemente vaginal de dos días de duración, s in
dolor y sin causa aparente. No refiere otras molestias. Refiere hace seis meses presenta algunos días una secreción genital
transparente sin ardor ni molestias asociadas. Al examen se encuentra un desarrollo de caracteres sexuales en estadio de Tann er IV.
El diagnóstico probable es:

A. Menarquia.

B. Metrorragia de causa no determinada.

C. Vulvovaginitis.

D. Hematuria de causa no determinada.

E. Abuso sexual.

82.El Síndrome de dificultad respiratoria es la causa más frecuente de ingreso de RN a la UCIN, siendo la etiología más frecu ente:

A. Enfermedad de membrana hialina.

B. Síndrome de aspiración de meconio.

C. Taquipnea transitoria del RN.

D. Persistencia de la circulación fetal.

E. Patología pulmonar congénita.

83.Un lactante de 10 meses que presenta vómitos y diarreas, taquicardia, presión arterial normal, mucosas secas, llenado capi lar
menor de 2 segundos e irritabilidad. ¿Qué porcentaje de peso ha perdido este paciente?:

A. 13 – 15%.

B. 3 – 5 %.

C. 1 – 2 %.

D. 10 – 12 %.

E. 6 – 9%.

84.Escolar de 6 años de edad de 20 k con diagnóstico de crisis asmática acude a emergencia, donde Usted decide nebuli - zarlo con
salbutamol. ¿Cuántas gotas de salbutamol prescribiría?:

A. 12.

B. 16.

C. 8.

D. 20.

E. 4.

85.Un niño de 2 años acude por presentar 5 días fiebre elevada, exantema maculo papular en tronco, inyección conjunti - val bilateral,
enrojecimiento bucal, lengua aframbuesada, faringe hiperémica sin secreción y una adenopatía cervical izquierda de 4cm. ¿Cuál es el
diagnóstico más probable?

A. Síndrome de Kawasaki.
B. Exantema súbito.

C. Escarlatina.

D. Rubéola.

E. Mononucleosis infecciosa.

86.En un niño mayor de 2 años de edad, asintomático, con examen de heces positivo para quistes de Entamoeba hysto - litica, el
tratamiento de elección es:

A. Cloroquina

B. Metronidazol

C. Dihidroemetina

D. Primaquina

E. Ninguna de las anteriores

87.Una adolescente de 14 años tiene un exagerado miedo a engordar y de hecho se ve más gorda de lo que los demás aprecian. Ha
mantenido su peso a pesar de que frecuentemente se da atracones que no puede evitar. Analíticamente encontramos hip okalemia.
Probablemente, estamos frente a un caso de:

A. Anorexia nerviosa.

B. Depresión juvenil.

C. Trastorno obsesivo-compulsivo.

D. Trastorno narcisista de la personalidad.

E. Bulimia nerviosa.

88.Niña de 2 años presenta movimientos mioclónicos de extremidades y sacudidas desordenadas de los ojos. En la explo- ración se
palpa masa abdominal en línea media y flanco derecho. Se le realiza una TAC abdominal, evidenciándose una masa de consistenci a
mixta y calcificaciones en su interior. Señale la afirmación correcta:

A. Es el tumor neurológico más frecuente en la infancia.

B. La edad más frecuente al diagnóstico son los 3 años.

C. Cursan con síndrome WARG.

D. Las catecolaminas en orina estarán elevadas.

E. Se asocia con delección del cromosoma 11.

89.La causa más frecuente de obstrucción intestinal entre los 3 meses y 3 años de edad es:

A. Enfermedad de Hirschsprung.

B. Apendicitis.

C. Bridas congénitas.

D. Invaginación intestinal.

E. Malrotación intestinal.

90.Paciente de 4 años con diagnóstico de meningoencefalitis meningococica que asiste a una guardería infantil, tiene un hermano
menor. ¿Cuál es la medida más recomendable?

A. Vacunar a todos los niños de la guardería.

B. Administrar ampicilina al hermano menor.

C. Dar sulfas a todos los niños en contacto.

D. Administrar gammaglobulina intramuscular.

E. Emplear rifampicina en el hermano y compañeros de la guardería.


91.En un colegio primario se desea detectar casos de enteroparasitosis. Se solicita a cada alumno una muestra de heces. En - tre los
negativos a esta primera muestra se solicita una segunda muestra. Al hacer este procedimiento se está logrando:

A. Aumentar la especificidad y sensibilidad.

B. Disminuir la sensibilidad y aumentar la especificidad.

C. Aumentar la sensibilidad y disminuir la especificidad.

D. Disminuir la especificidad y sensibilidad.

E. La sensibilidad y especificidad se mantienen.

92.En el análisis estratégico de una organización se considera en el contexto interno:

A. El Plan Operativo Institucional (POI).

B. Mi cliente.

C. La competencia.

D. Otros productos en competencia.

E. El marco político económico social.

93.Dos grupos de investigadores plantean determinar la prevalencia de gastritis por AINES en una misma población de pacientes que
sufren de osteoartrosis. Ambos grupos trabajarán con el mismo nivel de confianza, sin embargo el pri - mer grupo plantea trabajar con
un error relativo de muestreo del 5% y el segundo con un error relativo de muestreo del 10%. Se puede afirmar entonces:

A. Los tamaños de las muestras serán similares, ya que el nivel de confianza es el mismo.

B. El primer grupo trabajará con una muestra más pequeña.

C. Los tamaños de las muestras serán similares, ya que la prevalencia de gastritis es alta.

D. No se puede saber qué grupo tendrá una m uestra más grande.

E. El segundo grupo trabajará con una muestra más pequeña.

94.De 75 niños entre 6 y 10 años que tomaron su desayuno escolar en un centro educativo de un centro poblado rural, 40 presen taron
un cuadro de náuseas, vómitos, dolor abdominal y diarreas dentro de las 12 horas posteriores a la ingesta del alimento servido. De los
siguientes indicadores epidemiológicos, el más apropiado calcular en este contexto es:

A. Prevalencia.

B. Letalidad.

C. Tasa de ataque.

D. Densidad de incidencia.

E. Incidencia acumulada.

95.El código de Nuremberg estableció normas para:

A. Los procedimientos quirúrgicos en época de guerra.

B. Los procedimientos médicos en general.

C. Las publicaciones médicas.

D. El consentimiento informado previo a toda investigación.

E. Los procedimientos médicos en épocas de guerra.

96.En una comunidad hay abundante rabia animal y se notifican algunos casos humanos todos los años. Con el fin de fijar las b ases
para iniciar un programa de control, entre las cuales se señalan las siguientes medidas: ¿Cuál a su juicio, puede dar resultados más
permanentes?:

A. Campaña para exterminar perros vagabundos.

B. Campaña masiva de educación sanitaria en la población.

C. Campaña masiva para hacer tratamiento antirrábico a todas las personas mordidas.

D. Establecer un programa de vacunación canina (80% de los perros estimados) en un año.


E. Sistema de inspección de carne de los mataderos.

97.Los elementos básicos en la Programación en salud son:

A. La población, las atenciones y los instrumentos.

B. La política de salud, la población y la infraestructura física.

C. Los diferentes niveles organizacionales y los niveles de atención.

D. La doctrina sanitaria y los establecimientos de salud.

E. Todos los anteriores.

98.En una institución de salud, el concurso público para cubrir vacantes requiere:

A. Instrumentos de evaluación y equipo calificador.

B. Análisis de puestos e inventario de recursos.

C. Inventario de recursos y equipo calificador.

D. Cumplir con los requerimientos de las características de los puestos.

E. Instrumentos de evaluación y una conversación cordial.

99.Un exitoso convenio vigente entre un establecimiento de salud pública y una entidad privada debe ser considerado:

A. Oportunidad.

B. Debilidad.

C. Fortaleza.

D. Amenaza.

E. Desafío.

100.En una distribución de valores de una variable cuantitativa, el percentil 50 corresponde a:

A. Mediana.

B. Moda.

C. Media aritmética.

D. Media geométrica.

E. Los percentiles no se corresponden con las medidas de tendencia central.

4TO EXAMEN ENAM

1. GO Primigesta con 32 semanas de gestación por última regla, acude a emergencia por dolor en epigastrio e hipocondrio derecho . PA:
180/110 mmHg, proteinuria ++, tacto vaginal: cérvix posterior, orificio externo cerrado, pelvis ginecoide. El diagnóstico más probable es:
A. Preeclampsia severa y trabajo de parto
B. Preeclampsia superpuesta e hipertensión crónica
C. Preeclampsia severa
D. Preeclampsia severa y colecistitis
E. Preeclampsia severa y DPP
2. GO Puérpera de parto gemelar que presenta sangrado vaginal profuso luego del alumbramiento. El diagnóstico más pro- bable es:
A. Laceración de cuello uterino
B. Ruptura uterina
C. Coagulación
D. Atonía uterina
E. Trombocitopenia
3. GO Gestante a término de 35 años acude a emergencia por contracciones uterinas cada 3 minutos y sangrado vaginal de 10cc. PA: 120/70,
P: 84x’, FR: 16x’, Ecografía: feto único en LCI, placenta previa marginal. LCF: 148x’. Se realiza TV en condiciones de operab ilidad: incorporación: 90%,
dilatación: 8cm, membranas íntegras, altura de presentación: 0, variedad de posición: OIIA, pelvis ginecoide. ¿Cuál es la conducta a seguir?
A. Acentuación del trabajo de parto
B. Esperar a que continúe el trabajo de parto espontáneo
C. Parto instrumentado
D. Preparar para cesárea inmediata
E. Transfusión de sangre
4. GO Paciente de 49 años, con mioma uterino de tamaño equivalente a una gestación de 12 semanas, que presenta hiperme - norreas y
hemoglobinemia de 9 g%. No existe patología asociada. La paciente está en lista de espera para la práctica de una histerectomía programada para
dentro de 3-4 meses. En esta paciente está indicado el tratamiento preopera- torio con:
A. Estrógenos.
B. Ergóticos dopaminérgicos.
C. Inhibidores de la fibrinolisis.
D. Análogos de la GnRH.
E. Derivados del cornezuelo del centeno.
5. GO Mujer con tumoración que protruye por genitales externos. Al examen tiene una longitud de vagina de 11. El punto Aa
es +3. Según la clasificación de prolapso de órganos pélvicos (POP-Q), ¿cuál es el grado de distopia genital?
A. III
B. I
C. II
D. V
E. IV
6. GO Paciente de 26 años con 21 semanas de gestación, presenta dolor abdominal y sangrado vaginal escaso, de 2 días de evolució n. Al
examen: altura uterina 20 cm, movimientos fetales presentes. Especuloscopía: se observa membranas ovulares prominentes e integras, que
protruyen por el orificio externo abierto. ¿Cuál es el diagnóstico?
A. Óbito fetal
B. Aborto incompleto
C. Aborto inevitable
D. Aborto inminente
E. Aborto frustro
7. GO Primigesta en la 39 semana de gestación y con contracciones de parto. Todo ha transcurrido con normalidad hasta que ha roto la bolsa
espontáneamente con 3 cm de dilatación. Nada más romper la bolsa ha comenzado a sangrar (sangre roja en mediana cantidad) y h an surgido
signos de sufrimiento fetal muy grave. La causa más probable es:
A. Placenta previa.
B. Abruptio placentae.
C. Lesión de cérvix.
D. Síndrome de Hellp.
E. Rotura de vasa previa
8. GO Gestante en la décimo cuarta semana, desde hace semanas presenta náuseas y vómitos persistentes sin respuesta al dimenhidrinato,
ha recibido en los últimos 15 días sueros glucosados por vía IV. Actualmente presenta edema de miembros inferiores, taquicard ia, diplopía, dificultad
para la marcha, nistagmo y alteraciones mentales. PA: 100/70; pulso: 115 lpm. Análisis: HB: 9 g%, Na: 130 mEq/L, bilirrubina total: 2 mg%, bilirrubina
directa: 1,4 mg%. ¿Cuál es la posibilidad diagnóstica?:
A. Encefalopatía hepática.
B. Pre – eclampsia severa.
C. Síndrome de HELLP.
D. Hipertiroidismo de la gestante.
E. Encefalopatía de Wernicke.
9. GO Después de un expulsivo normal y tras 60 minutos de periodo de alumbramiento no se aprecian signos de despren - dimiento placentario a
pesar de haberse aplicado masaje uterino y de haber incrementado moderadamente la dosis de oxitocina. Se indica una extracción m anual de
placenta y la pared uterina ¿Cuál es el diagnóstico más probable?
A. Placenta incarcerada
B. Engatillamiento placentario
C. Placenta adherente por acretismo placentario
D. Placenta succenturiada con cotiledón aberrante
E. Placenta circunvalata
10. CIR Paciente de 55 años, general en su activo, con antecedentes de hipertensión, diabético y angina inestable, traído a la urgencia tras
caída montando en bicicleta. Radiográficamente presenta una fractura intracapsular desplazada del fémur proximal. ¿Cuál será el tratamiento más
adecuado?
A. Reducción y osteosíntesis con tornillo
B. Artroplastía parcial de cadera
C. Reducción y osteosíntesis con tornillos canulados
D. Reducción incruenta
E. Artroplastía total de cadera
11. CIR Un hombre fumador de 50 años consulta por ronquera, afonía e hipo. En la exploración ORL se evidencia parálisis de la cue rda vocal
izquierda como única anomalía. ¿Dónde localizaría la lesión?
A. Esófago distal
B. Mediastino posterior
C. Pleura
D. Cavum
E. Língula
12. CIR Un paciente consulta por pérdida de peso, dolor abdominal y trombosis venosa. En la exploración tiene esplenomega -
lia e ictericia. En las pruebas de imagen se confirma la sospecha clínica de tumor abdominal de:
A. Hígado
B. Vesícula
C. Páncreas
D. Estómago
E. Colon
13. CIR Si un paciente queda disfónico después de haber sido sometido a una hemitiroidectomía izquierda, es porque se ha lesionado:
A. Nervio laríngeo superior izquierdo
B. Nervio frénico izquierdo
C. Cadena simpática izquierda
D. Nervio laríngeo inferior izquierdo
E. Nervio frénico
14. CIR Tras hacer la historia clínica y ordenar las pruebas complementarias se llega al diagnóstico de que un paciente de 45 a ños tiene una
apendicitis aguda sin peritonitis que requiere una apendicectomía urgente, el paciente le pregunta sobre la laparoscopía. Su contestación es:
A. La laparoscopía solo sirve para el diagnóstico en casos de apendicitis aguda
B. La cirugía laparoscópica solo está indicado para la colecistectomía
C. La apendicitis aguda es una contraindicación absoluta para el abordaje laparoscópico
D. La apendicitis aguda sin peritonitis puede tratarse por laparoscopía y puede ofrecer algunas ventajas sobre la laparotomía
E. La única indicación de cirugía laparoscópica es el plastrón inflamatorio palpa ble en fosa ilíaca derecha
15. CIR Paciente de 52 años de edad que acude a consulta manifestando que desde hace tres días nota visión borrosa con mo - lestias en ojo
derecho que se acentúan cuando está en lugares muy iluminados o en la calle si hace sol. A l ser explorado encontramos una discreta hiperemia
conjuntival y tras la tinción con colirio de Fluoresceína sódica observamos sobre la córnea central una figura lineal ramific ada de unos 5mm de
longitud que nos sugiere que estamos ante:
A. Queratoconjuntivitis epidémica
B. Queratitis bacteriana
C. Queratitis vírica
D. Queratitis por acanthamoeba
E. Queratitis medicamentosa
16. CIR Paciente de 37 años que desde hace 4 días presenta dolor en región anal y desde hace 2 días fiebre. En urgencia se le ob serva una
zona indurada tumefacta y enrojecida en la zona perianal derecha. A la presión es muy dolorosa. Se le prescribe antibióticos y se le recomienda
consulta en Cirugía a las 48 horas, ¿Cuál es el origen más probable de su patogenia actual?
A. Fisura anal
B. Prolapso rectal
C. Fístula anorrectal
D. Hemorroides internos
E. Rectocele
17. MED Una paciente diabética de 65 años acude a urgencias por fiebre de 38.5°C y malestar general junto con síndrome mic - cional. PA:
90/60 mmHg, FC: 105x’ y FR: 22x’ con SO2: 89%. En la exploración física destacaba PPL (+). En el labo- ratorio sobresalía leucocitosis de 15700/mm3
(87%: neutrófilos) y creatinina de 1.4 mg/dL. Plaquetas: 90000; señale aquella medida inicial cuyo retraso tenga una mayor in fluencia negativa en la
evolución del cuadro:
A. Ingreso en UCI para ventilación mecánica asistida
B. Administración precozmente de bicarbonato
C. Administración de tratamiento inotrópico con dobutamina
D. Administración de antibióticos y estabilización hemodinámica
E. Administración precozmente de corticoides y proteína C activada
18. MED Mujer de 51 años de edad que consulta por debilidad generalizada y visión doble de 3 meses de evolución. Se le realiza u n test de
edrofonio y una electromiografía que permiten un diagnóstico de miastenia gravis. En Rx. y TAC torácico de localización anteroposterior de 5x4x4cm,
y densidad homogénea. ¿Qué patología mediastínica debemos sospechar como más probable?
A. Bocio endotorácico
B. Linfoma
C. Timoma
D. Teratoma
E. Tumor neurogénico
19. MED Un paciente de 65 años diagnosticado de EPOC, es trasladado al hospital por disnea progresiva y expectoración purulenta. Durante el
traslado en la ambulancia se le administra oxígeno y fluidoterapia. A su llegada el paciente está obnubilado y te mbloroso. Se auscultan crepitantes
en la base derecha. La gasometría arterial muestra un pH: 7.08, PO2: 106. El tratamiento inicial más adecuado sería:
A. Reducir flujo de oxígeno a 1L/min. y administrar bicarbonato sódico
B. Mantener flujo de oxígeno y administrar acetazolamida
C. Intubación y ventilación mecánica
D. Retirar oxígeno y administrar bicarbonato sódico
E. Administrar epinefrina i.v. y bicarbonato sódico
20. MED Mujer de 35 años sin factores de riesgo cardiovascular, con historia de una muerte fetal de 20 semanas, que presenta un infarto de
la cerebral media derecha. En el hemograma presenta plaquetopenia y en el estudio de coagulación se observa un alargamient o del TTPa que no
corrige con plasma fresco congelado. ¿Cuál es el diagnóstico más probable?
A. Poliarteritis nodosa
B. Micropoliangeítis
C. Síndrome antifosfolípido
D. Hemofilia
E. Esclerosis sistémica
21. MED Mujer de 62 años que acude a la consulta por presentar dolor en el borde interno del antebrazo y a nivel del dedo anular y meñique
de la mano derecha, también explica hormigueos, frialdad y pérdida de sensibilidad. ¿Qué tipo de neuropatía por compresión ti ene el paciente?
A. Síndrome canal carpiano
B. Atrapamiento de nervio mediano y cubital a nivel de la muñeca
C. Compresión proximal del nervio cubital
D. Sección nerviosa cubital
E. Compresión del nervio radial
22. MED Mujer de 72 años, con debilidad marcada que le impide caminar, diarreas de 3 meses de duración, pérdida de peso 15 kg e n un año.
Antecedentes no contributorios. Al examen: palidez severa de piel y mucosas, glositis, edema de miem -bros inferiores. Hb: 4g%; VCM: 130, hcm: 37;
leucocitos: 2,500; plaquetas: 25.000; creatininemía: 1mg%; glucosa: 160 mg%; lámina periférica: macrocitosis y polisegmentaci ón de neutrófilos.
¿Cuál es su posibilidad diagnóstica?:
A. Leucemia aguda aleucémica.
B. Anemia megaloblástica por déficit de folatos.
C. Aplasia medular.
D. Anemia megaloblástica.
E. Púrpura trombótica trombocitopénica.
23. PED Una niña de 12 años presenta como molestiaprincipal ptosis palpebral y visión doble. El examen revela debilidad de los músculos
extraoculares, de los músculos del cuello y de los músculos faciales. Se observa debilidad progresiva ante las contracciones musculares sostenidas o
repetidas. Cuando se solicita a la paciente que mantenga la mirada dirigida hacia arriba, se observa ptosis progresiva. La debilida d muscular mejora
con el reposo. Los estudios de tiroides son normales. El diagnóstico más probable es:
A. Parálisis de Bell.
B. Miastenia gravis juvenil.
C. Ptosis congénita.
D. Parálisis del nervio facial congénita.
E. Miositis osificante progresiva.
24. MED Un paciente con cerebro esclerosis que presenta disturbios mnésicos, hipomimia, bradicinesia, hipertonía, Hiperreflexia p rofunda,
Babinski bilateral, temblor de actitud, voz “monótona”, corresponde a la forma clínica siguiente:
A. Pseudobulbar.
B. Parkinson arteriopático.
C. Demencia arteriopática.
D. Forma lacunar.
E. Neurasténica.
25. MED Mujer de 19 años que en los últimos 15 días presenta febrícula, sudoración nocturna, astenia y dolor torácico derecho. En la Rx de
tórax se aprecia derrame pleural derecho importante, sin otras alteraciones. Se realiza toracocentesis diagnóstica, obteniénd ose un líquido
serohemático con: pH 7,36, proteínas 4,5 g/dl (sangre 7,5), LDH 850 U/l (sangre 150), glucosa 40 mg/dl y amilasa en rango de normalidad. El Gram
no muestra gérmenes. La citología muestra pre- dominio de linfocitos. Ante la sospecha diagnóstica, ¿cuál sería el siguiente paso a da r?
A. Repetir de nuevo la toracocentesis, para descartar que sea un empiema.
B. Realizar biopsia pleural.
C. Realizar toracoscopía.
D. Iniciar tratamiento antibiótico y esperar respuesta.
E. Esperar el resultado del cultivo de Lowenstein.
26. MED Paciente de 25 años de edad, hace dos semanas presentó cuadro de influenza. Desde ayer parestesias en miembros infe - riores y
debilidad progresiva que imposibilita deambular. Hoy pérdida de fuerza en miembros superiores y dificultad respiratoria. Al e xamen: cuadriparesia
hipotónica e hiporreflexia. El diagnóstico probable es:
A. Polimiositis aguda.
B. Síndrome de Guillain- Barré.
C. Polineuropatía carencial.
D. Mielitis transversa.
E. Botulismo.
27. MED Paciente de 40 años con disnea progresiva de esfuerzo hasta disnea de decúbito, pulso con arritmia completa, frecuen- cia cardiaca
156pm, soplo diastólico de tonalidad grave en area mitral PA 115/80 mmHg, crepitantes en ambas bases pulmonares ¿cuál es el d iagnóstico más
probable?
A. Insuficiencia aortica
B. Coartación de aorta
C. Estenosis mitral
D. Insuficiencia mitral
E. Estenosis aortica
28. MED Un varón de 35 años lleva varios años trabajando de pintor y ahora presenta cefalea de varios meses de evolución, di fi- cultad de
concentracion y artralgias. En la exploración física neuropatía periférica. Laboratorio: anemia normocitica
¿cuál de estas pruebas de laboratorio confirmaría el diagnostico?
A. Concentración de plomo en el suero
B. Concentración de plomo en la sangre
C. Concentración de arsénico en la sangre
D. Concentración de arsénico en el suero
E. Concentración de cadmio en el suero
29. MED Paciente mujer de 45 años con enfermedad de Crohn y una pequeña fistula intestinal desarrolla tetania durante la 2da semana de
nutrición parenteral. Los resultados de laboratorio incluyen del Ca 8.2 meq/L; Na 135 meq/L ; K 3.2 meq/L; Cl 103 meq/L ; PO4 2.4 meq/L ; albumina
2.4 ; pH 7.48 ; bicarbonato 25 meq/L. La causa más probable de la tetania del paciente es:
A. Hipomagnesemia
B. Hiperventilación
C. Hipocalcemia
D. Deficiencia esencial del ácido graso
E. Convulsiones focales
30. MED Un varón de 30 años ha experimentado dificultades para la deglución tanto de sólidos como de liquidos en los últimos
6 meses. ¿Cuál es el diagnóstico más probable?
A. Carcinoma esofágico
B. Anillos de Schatzki
C. Acalasia
D. Estenosis esofágica benigna
E. Esófago de Barret
31. PED Un recién nacido postmaduro con el antecedente de asfixia intraparto presenta en el momento del nacimiento una atelectasia
pulmonar asociada a neumotórax y neumomediastino, presentando en la radiología condensaciones algo- donosas diseminadas. ¿Cuál sería su
diagnóstico?
A. Persistencia de circulación fetal
B. TTRN
C. SALAM
D. Neumonía estafilocócica
E. EMH
32. PED Un recién nacido varón presenta una malformación congénita que consiste en agenesia de timo y paratiroides, arcoaórtico derecho,
atresia esofágica y anomalías faciales. ¿Qué síndrome polimalformativo sospecharía?
A. Martin-Bell
B. Potter
C. Pierre-Robin
D. Prune-Belly
E. Di George
33. PED Un varón de 10 meses presenta desde hace unos días signos de coriza y febrícula, hace unas horas comienza con un episodio agudo de
dificultad respiratoria con tiraje intercostal y sibilancias en la auscultación. Los padres nos comen- tan que nunca había sufrido un episodio similar.
¿Cuál es el agente etiológico que con más frecuencia relaciona este cuadro clínico?
A. Virus sincitial respiratorio
B. Parainfluenzae
C. Adenovirus
D. Mycoplasma
E. Idiopático
34. PED Varón de 3 años de edad que, encontrándose previamente bien, comienza de forma brusca con fiebre, dolor de gargan - ta, estridor,
disnea, acúmulo de secreciones orales y emisión de saliva por la boca. Un hermano de 2 años lleva 72 horas ingresado por una meningitis
bacteriana. El agente etiológico de ambos cuadros clínicos será con más probabilidad:
A. Providencia rettgeri
B. Haemophilus influenzae
C. Klebsiella pneumoniae
D. Streptococcus pneumoniae
E. Neisseria meningitidis
35. PED En relación a las etapas de desarrollo mamario, según Tanner, la presencia de montículos mamarios, incremento de la areo la y
desarrolla de pezón en grado variable, corresponde a:
A. Etapa 5
B. Etapa 3
C. Etapa 2
D. Etapa 0
E. Etapa 1
36. PED Un recién nacido presenta un área edematosa de contenido seroso en la zona de presentación. Presenta alguna pete - quia aislada,
pero no existe colección de sangre. Se observa también cabalgamiento de parietales. Este cuadro desapa- rece en 24 horas. ¿Cómo se denomina
dicho cuadro?
A. Cefalohematoma
B. Caput venoso
C. Necrosis adiposa del tejido subcutáneo
D. Hemorragia subaponeurótica difusa
E. Caput succedaneum
37. PED Un bebé a término tiene una clínica de vómitos tardíos, retraso en la eliminación de meconio, episodios alternantes de diarrea -
estreñimiento y enterocolitis. Se le practica un tacto rectal observándose hipertonía con recto vacío. Usted decide hacer una biopsia de colon con el
hallazgo de ausencia de células ganglionares de los plexos mioentéricos en un segmento colónico. ¿Cuál es el segmento que con más frecuencia se
ve afectado en esta enfermedad?
A. Colon transverso
B. Recto-sigma
C. Colon descendente
D. Recto
E. Colon ascendente
38. PED Una madre acude con su hijo de 2 semanas al pediatra ya que le encuentra irritable, rechaza el alimento, tiene diarrea
mucosanguinolenta y tinte ictérico. A la exploración presenta distensión abdominal con palpación dolorosa y aspecto séptico. En una radiografía
descubre una perforación intestinal. ¿Cuál será la causa más frecuente de este hallazgo teniendo en cuenta que el niño todaví a pertenece al período
neonatal?
A. Invaginación intestinal
B. Estenosis pilórica
C. Enterocolitis necrotizante
D. Celíaca precoz
E. Apendicitis aguda
39. PED Un niño de 7 meses deshidratado, con vómitos, diarrea y fiebre; nos llega la siguiente analítica: Hb 11,6 g %; Hto: 39%; Leucocitos:
14.900 mm3 (Neutrófilos: 63%; Linfocitos: 30%; Monocitos: 7%); Plaquetas: 322.000 mm3; Os- molaridad: 295 mOs/l; Na: 137 mEq/l; K: 5.6 mEq/l;
Ca iónico: 1,2 mmol/l; pH: 7,20; pCO2: 25 mmHg; HCO3: 11 mEq/l; E.B.: -19 mEq/l; Lactato: 5,3 mmol/l, Creatinina: 4,2 mg %. ¿Cuál es la valoración
metabólica de esta deshi- dratación?
A. Deshidratación isotónica con acidosis mixta.
B. Deshidratación hipotónica con acidosis metabólica.
C. Deshidratación isotónica con acidosis metabólica.
D. Deshidratación hipertónica con acidosis metabólica.
E. Deshidratación isotónica con acidosis respiratoria.
40. PED Un niño de 2 años, previamente saludable, desarrolla, fiebre de 40,6º C y presenta convulsión generalizada que dura 60 se g. El
examen físico revela una infección respiratoria alta y los estudios de laboratorio son normales, mostrándose asintomático al día siguiente. Ud.
Explica a los padres que la convulsión es febril y que tiene buen pronóstico, no necesitando quimioprofilaxis con fenobarbita l. Su opinión está basada
en:
1) Estuvo previamente sano hasta el inicio de la convulsión.
2) La convulsión ocurrió luego del año de edad.
3) El episodio agudo duró menos de 15 minutos.
4) Al día siguiente no presentó manifestación neurológica residual
5) La convulsión estuvo asociada a una infección respiratoria alta benigna.
A. La respuesta correcta es: B. 1,2 y 3.
C. 1,3 y 5.
D. 2,4 y 5.
E. 2,3 y 4.
F. 1,4 y 5.
41. PED ¿A los cuántos meses de edad se recomienda colocar la vacuna contra la hepatitis A?
A. 4
B. 9
C. 12
D. 2
E. 6
42. PED La asociación apropiada de antibióticos para tratar la sepsis en un recién nacido es:
A. Vancomicina + Amoxicilina
B. Amikacina + Penicilina
C. Amikacina + Ampicilina
D. Gentamicina + Cefalotina
E. Cloranfenicol + Ampicilina
43. CCBB ¿Cuál de los siguientes cambios es un fenómeno de adaptación?
A. Autolisis
B. Metaplasia
C. Apoptosis
D. Cariolisis
E. Cariorrexis
44. CCBB¿Cuál de las siguientes vitaminas se comporta como hormona esteroidea?
A. A
B. B5
C. E
D. K
E. D
45. CCBB¿Cuál de las siguientes alteraciones caracteriza a las células malignas?
A. Atrofia
B. Hipertrofia
C. Anaplasia
D. Metaplasia
E. Hiperplasia
46. SSPP Un paciente ha tenido un resultado positivo en un Mantoux que se la ha realizado por sospecha de tuberculosis.
¿Cómo se denomina la probabilidad de que este individuo tenga realmente la enfermedad cuando el test realizado ha resultado positivo?
A. Especificidad
B. Riesgo atribuible
C. Valor predictivo positivo
D. Sensibilidad
E. Valor predictivo negativo

47. SSPP Entre los factores de riesgo modificables de la aterosclerosis se encuentran el tabaquismo, la obesidad, la vida sedenta- ria, los
trastornos de los lípidos, la hipertensión, la resistencia a la insulina, entre otros; como factores no modificables están la edad, el sexo masculino y
factores genéticos. Un varón de 35 años sufre un infarto agudo de miocardio. Es fumador desde hace 20 años por lo que se le aconseja dejar de
fumar. Con esto usted realiza una:
A. Promoción de la salud
B. Atención curativa
C. Prevención terciaria
D. Prevención primaria
E. Prevención secundaria
48. SSPP Se determina la sensibilidad de la prueba a partir de la siguiente fórmula
Resultado de la prueba Enfermos
SI NO
Positiva a b
Negativa c d
A. a/a+c
B. c/a+c
C. d/c+d
D. b/b+d
E. a/a+b
49. SSPP ¿Qué tipo de estudio epidemiológico emplearía en el estudio de una enfermedad rara, crónica, de larga evolución, y de larga
inducción?
A. Transversal
B. Cohortes
C. Test de Welch
D. Casos y controles
E. Incidencia
50. SSPP Un médico se encuentra realizando SERUMS en Cieneguilla y desea realizar una actividad de promoción en salud. Su equipo de salud
le envía el siguiente listado de actividades. Considerando el concepto de promoción. ¿Cuál es la actividad que debe elegir?
A. Consejería grupal de adolescentes sobre la transmisión del VIH.
B. Charla a la comunidad en nutrición saludable.
C. Campaña de vacunación en menores de 5 años.
D. Búsqueda en la comunidad de sintomáticos respiratorios.
E. Control vectorial para disminuir casos de enfermedades metaxénicas.
51. SSPP Si el valor de una canasta básica familiar en Tarma es de S/. 1500 y el valor de la canasta sólo con alimentos es de S/750, señale uste d
en que grupo se encontraría la familia “X” cuyo ingreso promedio mensual es de S/. 550 mensual:
A. Quintil 4.
B. Pobre no extremo.
C. Extrema pobreza.
D. Pobreza.
E. Clase media.
52. SSPP En el análisis FODA de un hospital, “los permanentes conflictos entre la dirección y el cuerpo médico” corresponde a:
A. Debilidad.
B. Amenaza.
C. Fortaleza.
D. Oportunidad.
E. Recurrencia.
53. MED Varón de 18 años de edad, presenta un forúnculo en la cara y a los pocos días aparece edema con signos de inflama - ción en la
rodilla izquierda, asociado a fiebre. A los 3 días aparece tos, disnea, dolor torácico, fiebre elevada y sig nos de toxicidad sistémica. Una radiografía de
tórax muestra múltiples infiltrados nodulares y neumatocele. El agente etiológico más probable es:
A. Staphylococcus aureus
B. Peptostreptococcus magnus
C. Streptococcus pyogenes
D. Pseudomonas aeruginosa
E. Mycobacterium tuberculosis
54. MED Paciente varón de 18 años que presenta limitación funcional y dolor en región de columna dorsal, tiene como ante - cedente haber
sufrido de “pleuresia” y enfermedad venérea .Es consumidor de queso fresco. Radiográficamente se evidencia lesión en columna D12 con
destrucción de las vértebras contiguas preservación de pedículos. El diagnóstico más probable seria
A. Brucelosis
B. Neoplasia
C. Espondilitis anquilosante
D. Mal de pott
E. Sd de Reiter
55. MED Paciente con antecedente de haber tenido impétigo luego hipertensión, edema de párpados, oliguria, ictericia, hígado aumentado de
tamaño ¿Cuál es el diagnóstico?
A. Enfermedad de Berger
B. Glomerulonefritis post estreptocócica
C. GMN rápidamente progresiva
D. Glomerulopatía membranosa
E. Cambios mínimos
56. MED Paciente de 54 años de edad, procedente de Pucallpa. Acude a hospital de la capital porque tiene fiebre intermitente y p alidez. ¿Cuál
es el diagnóstico más probable?
A. Hepatitis C
B. Fiebre tifoidea
C. Malaria
D. Brucelosis
E. Bartonelosis
57. MED Mujer de 32 años llega a consulta por cefalea intensa, vómitos y rigidez de nuca. El LCR presenta pleocitosis linfoci - taria. ¿Cuál es el
diagnóstico más probable?
A. Meningitis por hongos
B. Meningitis viral
C. Meningitis bacteriana
D. Hemorragia subaracnoidea
E. Meningitis por VIH
58. MED Paciente portador de prótesis cardiaca metálica, alérgico a la penicilina, al que se le va a realizar broncoscopia diag -nóstica. ¿Qué
profilaxis antibiótica utilizaría?
A. Cefalexina
B. Eritromicina
C. Clindamicina
D. Vancomicina
E. Gentamicina
59. MED Anciana de 70 años sin antecedente de DM es llevada a emergencia por presentar estado mental alterado, cuadro de deshidr atación
severa, el familiar solo informa que días antes presentó polidipsia, poliuria. El diagnóstico pla nteado seria.
A. DM tipo 1
B. DM tipo 2
C. Cetoacidosis diabética
D. Coma hiperosmolar no cetósico
E. Hipoglicemia
60. MED Un hombre asintomático presenta en dos análisis de sangre rutinarios glucemias basales de 132 y 130mg/dl. ¿Cuál será la actitud
correcta?
A. Se le pide una curva de glucemia para diagnostico
B. Se inicia tratamiento con antidiabéticos orales
C. Se inicia tratamiento con dosis de insulina NPH
D. Se le pide nuevos análisis dentro de un año
E. Se le diagnostica diabetes mellitus tipo 2 y se le inicia tratamiento con dieta
61. MED Paciente varón de 65 años con antecedente de diabetes mellitus en control regular que acude a su cita con los siguientes resultados:
glicemia, creatinina sérica y electrolitos en límites normales, en el examen de orina se aprecia una cruz de proteínas. En el mes previo sus presiones
arteriales han fluctuado entre 150-160 / 70-80. ¿Cuál sería la mejor terapia farmacológica para su hipertensión arterial?
A. Calcio antagonistas
B. Beta bloqueadores
C. Inhibidores de la enzima convertidora de angiotensina
D. Diuréticos tiazídicos
E. Diuréticos ahorradores de potasio
62. MED Paciente varón fumador de 68 años, que presenta tos con expectoración matutina esporádica desde hace 15 años y que desde los
últimos 3 la tos y la expectoración se han incrementado añadiéndose disnea al realizar grandes esfuerzos, las molestias ceden parcialmente con el
uso de broncodilatadores. ¿Cuál es la prueba que Ud. le solicitaría primero para completar el diagnóstico?
A. Tomografía axial computarizada de tórax
B. Espirometría
C. Prueba de difusión de PCO2
D. Búsqueda de bacilos de Koch en el esputo
E. Test de hiperreactividad bronquial con metacolina
63. MED Paciente mujer de 65 años que acude a la consulta con dolor en rodilla derecha de 15 días de evolución, que se incre - menta al
caminar y cede al reposo, sin historia de trauma previo; tiene un índice de m asa corporal en 32, aumento de volumen de la rodilla derecha pero sin
enrojecimiento ni incremento de temperatura; a la movilización pasiva se percibe crujido. ¿Cuál es el diagnóstico más probabl e de la paciente?
A. Monoartritis infecciosa
B. Artropatía por cristales
C. Osteoartritis de rodilla
D. Artritis reumatoide de rodilla
E. Meniscopatía de rodilla derecha
64. MED Ante un varón con antecedentes de tabaquismo y alcoholismo que consulta porque desde hace meses nota una sen- sación de
cuerpo extraño al tragar, parestesias faríngeas, pinchazos en los oídos y una adenopatía cervical. ¿Qué debemos sospechar?
A. Cáncer de las cuerdas vocales
B. Cáncer subglótico
C. Laringitis aguda
D. Cáncer de cavum
E. Cáncer de supraglotis
65. PED Ante un niño de 5 años con un cuadro de hipertensión endocraneal, alteraciones visuales e hipotalámicas, que pre- senta una
radiografía lateral del cráneo con calcificaciones en forma de paréntesis a nivel supraselar. ¿Cuál será su diagnóstico presu ntivo?
A. Meduloblastoma
B. Craneofaringioma
C. Adenoma hipofisario
D. Glioma del nervio óptico
E. Pinealoma productor de hidrocefalia
66. MED Paciente de 65 años, con cuadro de ictericia progresiva, coluria y dolor abdominal. En las últimas 58 horas se agregan fi ebre y
escalofríos. ¿Cuál es el diagnóstico más probable?
A. Absceso hepático.
B. Colangitis aguda.
C. Hepatitis aguda.
D. Neoplasia de la vesícula biliar.
E. Ninguna de las anteriores.
67. MED Varón de 56 años de edad, desde hace 3 horas presenta dolor en la fosa ilíaca izquierda, fiebre, vómitos. Niega moles- tias urinarias.
¿Cuál es la primera impresión diagnóstica?:
A. Intoxicación aguda por plomo
B. Apendicitis aguda
C. Cólico ureteral izquierdo
D. Crisis hemolítica
E. Diverticulitis colónica aguda
68. MED Paciente de 24 años de edad, que sufre un politraumatismo por caída del segundo piso y a las 6 horas fallece. ¿Cuál cree usted qu e es
la causa más probable de muerte?:
A. Pancreatitis aguda
B. Edema cerebral
C. Lesión de colon
D. Perforación ileal
E. Hemorragia
69. MED Paciente de 50 años de edad que consulta por dolor en la fosa renal, poliaquiuria, disuria y hematuria. En el análisis de orina se
observa piuria y pH ácido con cultivos repetidamente negativos. ¿Cuál sería la primera posibilidad diagnóstica, de entre las siguientes?
A. Pielonefritis aguda.
B. Síndrome nefrítico.
C. Tuberculosis genitourinaria.
D. Prostatitis aguda.
E. Carcinoma renal de células claras.
70. GO A una mujer de 49 años de edad se le realiza un Papanicolaou y el resultado se informa como neoplasia intracervical III o lesión
intraepitelial escamosa de alto grado, usted indicaría el siguiente procedimiento :
A. Colposcopia y biopsia dirigida
B. Cono frio diagnostico
C. Histerectomía total
D. Inspección visual con ácido acético
E. Aplicación del test de Schiller
71. GO Ingresa una primigesta de 18 años de edad con 32 semanas de gestación refiriendo cefalea intensa y epigastralgia tipo opre sivo. Al
examen tiene presión arterial 180/120 mm Hg. No hay dinámica uterina, presenta movimientos fetales y los latidos del feto fluctúan entre 130 y
150, la altura uterina es de 25 cms. con feto en LCI. Los reflejos patelares están en 3+/4+. ¿Cuál es el manejo farmacológico inmediato?
A. Sulfato de magnesio 4 gms EV en 20’, nifedipino 10 mg VO
B. Diazepam 10 mg EV, labetalol 200 mg EV
C. Sulfato de magnesio 10 gms IM, alfa metil dopa 1 gm VO
D. Fenitoina 900 mg VO, hidralazina 10 mg EV
E. Diazepam 5 mg EV, nifedipino 10 mg Sublingual
72. GO Gestante a término en fase activa de trabajo de parto, presenta ruptura de membranas hace 30 minutos, y usted de- tecta frecuencia
cardiaca fetal basal de 132 latidos por minuto. En 3 oportunidades ha presentado latidos fetales en 110 latidos por minuto du rante 30 segundos
fuera de las contracciones y asociado a movimiento del bebe. ¿Cuál es la causa más probable de estas desaceleraciones en la frecuencia cardiaca
fetal?
A. Compresión de cabeza fetal
B. Acidemia fetal
C. Compresión de cordón umbilical
D. Hipoxia fetal
E. Desprendimiento de placenta
73. GO Paciente en su tercer día de puerperio de cesárea por expulsivo prolongado, la intervención quirúrgica discurrió sin complica ciones. El
día de hoy presenta temperatura 38.50 C, pulso 120 por minuto, frecuencia respiratoria 20 por mi - nuto, presión arterial 120/70 mmHg, llenado
capilar < 2 segundos. Luce en regular estado, al examen muestra mamas induradas y dolorosas, útero doloroso a la palpación co n altura 3 cms. por
encima del ombligo, la herida no muestra flogosis, loquios escasos sin mal olor, el resto del examen es normal. ¿Cuál es el diagnóstico más probable?
A. Atelectasia pulmonar
B. Flebitis
C. Congestión mamaria
D. Endometritis
E. Infección urinaria
74. GO Primigesta de 34 semanas diagnosticada de infección por VIH a las 18 semanas iniciándose tratamiento con zidovudi- na y lamivudina.
Acude a control prenatal. ¿Cuál de las siguientes alternativas sería la más apropiada para el manejo del parto?
A. Amniocentesis a las 38 semanas para madurez pulmonar
B. Cesárea segmentaria a las 40 semanas
C. Inducción de parto a las 39 semanas
D. Medir carga viral a las 36 semanas
E. Medir leucocitos CD4 a las 36 semanas
75. GO Primigesta de 35 semanas acude a emergencia refiriendo hace 30 minutos dolor abdominal y sangrado vaginal oscuro de 300 cc . Al
examen detecta Presión arterial 90/60 mm de Hg, pulso 120 por minuto, respiraciones 24 por minuto, afebril. Abdomen con altura uterina d e 40 cm,
contracciones uterinas cada 2 minutos, 50 segundos de duración, intensidad 4 cruces, latidos fetales 100 a 120 por minuto; en el examen con
especulo sangrado a través del cervix, el cual se encuentra cerrado y largo. Se indica vía EV, hemograma grupo y Rh, prueba c ruzada y perfil de
coagulación.
¿Cuál debe ser la siguiente conducta?
A. Ecografia transvaginal
B. Perfil biofísico
C. Inducción de parto con oxitocina
D. Amniotomia y estimulación con oxitocina
E. Cesárea
76. GO Paciente mujer de 27 años de edad, con IMC de 30, presenta oligomenorrea, acné, hirsutismo y desde hace 3 años busca salir
gestando. ¿Cuál sería el diagnóstico de la paciente?
A. Hipotiroidismo
B. Hiperplasia Suprarrenal
C. Síndrome de Ovario Poliquistico
D. Deficiencia de la Fase Lútea
E. Hiperprolactinemia
77. GO Mujer de 53 años de edad, quien cursa con sequedad vaginal, disminución del deseo sexual, disuria y poliaquiuria con urocultivos
negativos. Estos síntomas están asociados a disminución de:
A. Progesterona
B. FSH
C. Estrógenos
D. LH
E. Prolactina
78. GO Paciente mujer de 21 años de edad, con pareja estable, quien tuvo su segundo parto vagina l hace 3 días con recién nacido sano. Planea
amamantar exclusivamente. ¿Qué anticonceptivo seria el apropiado en este caso?
A. Anticonceptivo Oral Combinado
B. Ligadura Tubárica Bilateral
C. Preservativos y espermicida
D. Progestágenos Orales
E. Anticonceptivos Inyectables mensuales
79. PED Si un niño de 8 años hace shock anafiláctico por penicilina, el medicamento de elección es:
A. Clorfenamina
B. Suero fisiológico
C. Epinefrina
D. Dexametasona
E. Dopamina
80. PED La medicación que permite mejoría rápida de un lactante con crup infeccioso grave es:
A. Adrenalina en nebulización
B. Antibióticos
C. Dexametasona vía endovenosa
D. Ambiente húmedo
E. Salbutamol en inhalación
81. PED El diagnóstico de desnutrición aguda se hace en base a la relación:
A. Peso/talla2
B. Peso/edad
C. Peso/talla
D. Talla/edad
E. Talla/peso
82. PED La panencefalitis esclerosante subaguda de Dawson tiene como etiología al virus de:
A. Rubéola
B. Sarampión
C. Influenza
D. Varicela
E. Hepatitis B
83. PED En las niñas, el agente causal más frecuente de infección urinaria es:
A. Escherichia coli.
B. Ureoplasma.
C. Parvovirus.
D. Campilobacter.
E. Chlamydia trachomatis.
84. PED Un niño de 4 años de edad con un cuadro de fiebre, conjuntivitis, adenopatía cervical, lengua de fresa y edema con descamación
membranosa de los dedos de manos y pies, orientará nuestro diagnóstico hacia:
A. Enfermedad de Behcet
B. PAN
C. Forma juvenil del lupus discoide
D. Enfermedad de Still
E. Enfermedad de Kawasaki
85. MED La saturación de oxígeno media en la arteria pulmonar es aproximadamente de:
A. 0.33
B. 0.75
C. 0.5
D. En personas sanas es superior al 95%
E. Depende de varios factores, pero se mueve en el intervalo 80-95%
86. CCBB Su alteración es muy sensible en la fase inicial del trastorno tiroideo a veces el único cambio:
A. TSH
B. T3 libre
C. T3
D. T4 libre
E. T4
87. CCBB ¿Cuál de las siguientes acciones corresponde a la vitamina E?
A. Cicatrización de las heridas
B. Anitoxidante
C. Síntesis de algunos factores de la coagulación
D. Síntesis de colágeno
E. Regularización de los niveles de calcio y fósforo en sangre
88. CCBB Al nacer, el volumen sanguíneo es:
A. 65 cc. x Kg.
B. 80 cc. x Kg.
C. 110 cc. x Kg.
D. 125 cc. x Kg.
E. 150 cc. x Kg.
89. CCBB El contenido de hierro corporal total en el adulto normal está entre:
A. 5 y 10 mg.
B. 50 y 100 mg.
C. 25 y 50 mg. D. 0.5 y 1.5 mg.
E. 2 y 5 g.
90. CCBB El hierro molecular, Fe, es:
A. Almacenado primariamente en el bazo.
B. Excretado en la orina como Fe++.
C. Almacenado en el cuerpo en combinación con ferritina.
D. Absorbido en el intestino por la transferrina.
E. Absorbido en la forma férrica, Fe+3.
91. CCBB La vitamina B12, se absorbe en el:
A. Íleon distal.
B. Yeyuno proximal.
C. Duodeno.
D. Colon.
E. Yeyuno medio.
92. CCBB El sistema nervioso simpático que sale de la médula espinal:
A. Cesa de funcionar después de sección del bulbo.
B. Contiene solamente fibras adrenérgicas.
C. Contiene solamente fibras colinérgicas.
D. Incluye una sinapsis gangliónica.
E. Ninguna de las anteriores.
93. CCBB Se dice que el sistema extrapiramidal está conformado por estratos o niveles superpuestos. Estos niveles son:
1. Cuerpo estriado.
2. Núcleo óptico.
3. Hipotálamo.
4. Núcleos de la región subopto-estriada.
5. Sustancia reticular. Es cierto solamente:
A. 1, 2.
B. 2, 3.
C. 3, 4.
D. 1, 4.
E. 1, 3.
94. SSPP En un centro de salud la enfermera da una charla sobre control prenatal a un grupo de madres, la semana siguiente la obstetriz da la
misma charla al mismo grupo de madres. La enfermera ordena a una técnica de enfermería que realice una visita domiciliaria, m ientras que el
médico manda a la misma técnica que lo ayude en consultorio externo. ¿Estos problemas a que componentes de la administración corresponden?:
A. Planificación.
B. Organización.
C. Control.
D. Dirección.
E. Evaluación.
95. SSPP Un centro de salud programó para el año 2009 realizar 2 000 controles de crecimiento y desarrollo, para 1 500 niños menores de 5
años, utilizando 800 horas enfermera. AI finalizar el año atendió 200 consultantes nuevos, 300 reingre - santes y 500 continuadores, utilizando 600
horas enfermera. El porcentaje de avance de meta en atenciones fue:
A. 50.
B. 85.
C. 25.
D. 75.
E. 100.
96. SSPP Se quiere graficar un conjunto de datos cuantitativos continuos, trabajados en una escala de razón. ¿Qué tipo de grá - fico sería más
adecuado?:
A. Gráfico de barras.
B. Gráfico circular.
C. Histograma.
D. Pictograma.
E. Gráfico semi-logarítmico.
97. SSPP En una muestra de 50 estudiantes la estatura promedio es de 1,65 m. Luego, considerando un intervalo de confianza
del 95%, el límite inferior es de 1,56 m. y el superior:
A. 91,74 m.
B. 1,68 m.
C. 1,70 m.
D. 1,77 m.
E. No se puede calcular con la información disponible.
98. SSPP Una encuesta determina que las mujeres consumen bebidas alcohólicas unas 0,65 veces menos que los varones. En este caso, la
medición usada es un(a):
A. Proporción.
B. Fracción aritmética.
C. Porcentaje.
D. Mediana.
E. Razón.
99. SSPP En un país “X”, durante los últimos cinco años, el número total de casos de malaria mantiene una tendencia ascen - dente, superior a
la del crecimiento de la población. Este cambio anual en la frecuencia total de la malaria, tiene su origen en la modificación de su:
A. Incidencia.
B. Letalidad.
C. Mortalidad.
D. Riesgo atribuible.
E. Riesgo relativo.
100. SSPP La tasa de incidencia de leucemia, en población expuesta a radiación ionizante es de 4 en mi l y la población no expues- ta de 0,5 en
mil. ¿Cuál es el riesgo atribuible al factor de exposición?:
A. 8,0.
B. 4,5.
C. 3,5.
D. 4,0.
E. 0,5.
101. CIR Son las células encargadas de la contracción de la herida, durante el proceso de cicatrización:
A. Neutrófilos.
B. Polimorfonucleares.
C. Fibroblastos.
D. Macrófagos.
E. Fibrocitos.
102. CIR La cirugía moderna del cáncer gástrico, tiene fundamento en la disección de las diferentes estaciones ganglionares. La ex tirpación del
grupo N° 9, corresponde a:
A. Arteria coronaria estomáquica.
B. Arteria esplénica.
C. Arteria hepática.
D. Pedículo hepático.
E. Tronco celíaco
103. CCBB La pepsina es inactivada por:
A. La secreción acuosa del páncreas.
B. La concentración gástrica de hidrogeniones.
C. Las prostaglandinas gástricas.
D. El ácido desoxicólico.
E. La concentración de gastrina.
104. CCBB La mayor absorción de sodio se realiza en:
A. La porción ascendente del asa de Henle
B. La porción descendente del asa de Henle
C. El túbulo proximal
D. El túbulo distal
E. El túbulo colector
105. CCBB ¿Cuál es el antibiótico que inhibe la DNA girasa?
A. Ciprofloxacino
B. Penicilina
C. Amikacina
D. Clindamicina
E. Ceftriaxona
106. CCBB En relación a las fases del potencial de acción cardíaco:
A. La fase 4 en las células marcapaso es estable.
B. El nodo sinusal es el marcapaso cardíaco en condiciones normales porque la pendiente de la fase 4 es la menor.
C. La pendiente de la fase 0 de una célula marcapaso es mayor que en una célula del sistema de conducción.
D. En el nodo sinusal, el ion responsable de la fase 0 es el calcio.
E. El período refractario relativo engloba la fase 1 del potencial de acción.
107. CIR Normalmente la válvula ileocecal se cierra en qué circunstancias:
A. Ocurre una onda peristáltica
B. Incrementa la presión ileal
C. Incrementa la presión colonica
D. Existe estimulación vagal
E. Se contrae el ciego
108. CCBB ¿Cuál de los siguientes cambios es un fenómeno de adaptación?
A. Autolisis
B. Metaplasia
C. Apoptosis
D. Cariolisis
E. Cariorrexis
109. CCBB ¿Cuál de las siguientes alteraciones caracteriza a las células malignas?
A. Atrofia
B. Hipertrofia
C. Anaplasia
D. Metaplasia
E. Hiperplasia
110. CCBB ¿Cuál de las siguientes acciones corresponde a la vitamina E?
A. Cicatrización de las heridas
B. Anitoxidante
C. Síntesis de algunos factores de la coagulación
D. Síntesis de colágeno
E. Regularización de los niveles de calcio y fósforo en sangre
111. SSPP Un centro de salud realizó la evaluación de sus servicios correspondientes al año 2009. Poblaci ón total 12,000 habitan- tes. Atendidos
3000 personas, consultas médicas realizadas 6000 ¿cuál es la intensidad de uso o concentración (%)?
A. 1.3
B. 4
C. 25
D. 2
E. 1
112. SSPP Se realizó un estudio para relacionar la ingesta rica/pobre en grasas y la existencia / no existencia posterior de cáncer de próstata en
adultos mayores de 40 años. La observación duró 10 años. ¿Cuál es el tipo de estudio de la investiga - ción?
A. Casos y controles
B. Ecológica
C. Cohortes
D. Experimental
E. Transversa
113. SSPP En una localidad determinada, durante los recientes 3 años, el número total de casos de fiebre amarilla mantiene una tendenci a
ascendente, superior a la del crecimiento de la población. Este cambio anual en la frecuencia total de fiebre amarilla, tiene su origen en la
modificación de su:
A. Incidencia
B. Letalidad
C. Mortalidad
D. Riesgo atribuible
E. Riesgo relativo
114. SSPP Un estudio observacional en el que se comparan 110 individuos con cáncer de senos paranasales con 220 individuos sin la
enfermedad, corresponde a un estudio:
A. Longitudinal
B. Caso control
C. De cohortes concurrente
D. Transversal
E. Cohortes no concurrente
115. SSPP En un distrito rural del país se notificaron durante la primera semana de Octubre del 2005, dos casos locales o autóc - tonos de rabia
humana, enfermedad que no se observó́ en las últimas tres décadas. Esta situación se define como una:
A. Endemia
B. Epidemia
C. Epizootia
D. Hiperendemia
E. Holoendemi
116. SSPP Se desarrolla un estudio con la finalidad de buscar la asociación entre el polvo de una mina y la presencia de proble- mas
respiratorios en la población. Se estudian dos poblaciones: Una expuesta al polvo de la mina y la otra de agri - cultores; se hace seguimiento por un
año y se registra la aparición de los casos en ambas poblaciones. ¿A qué tipo de diseño epidemiológico corresponde el estudio?
A. Cohortes
B. Casos y controles
C. Cuasi experimental
D. Longitudinal
E. Transveral
117. SSPP Durante su ejercicio profesional, en el centro de salud se presenta un elevado número de casos de diarrea acuosa, neumonía,
desnutrición infantil y parasitosis. De acuerdo a su observación, ¿qué acciones preventivas recomendaría?
A. Incrementar la compra de medicamentos para el centro de salud
B. Solicitar más personal profesional y técnicos de salud
C. Mejorar los servicios de emergencia del establecimiento
D. Ampliar el número de camas del establecimiento
E. Acceso a agua potable, mejoramiento de la vivienda y nutrición
118. SSPP En el análisis situacional de salud de un establecimiento se encuentra que su comunidad de referencia carece de agua potable, el año
anterior se presentaron 40 casos de cólera con 8 fallecidos, y el establecimiento no tiene protocolos de atención. Según el a nálisis FODA
institucional, la situación mencionada corresponde a:
A. Oportunidades y amenazas
B. Oportunidades y fortalezas
C. Debilidades y fortalezas
D. Amenazas y debilidades
E. Debilidades y oportunidades
119. SSPP En un centro de salud la enfermera da una charla sobre control prenatal a un g rupo de madres, la semana siguiente la obstetriz da la
misma charla al mismo grupo de madres. La enfermera ordena a una técnica de enfermería que realice una visita domiciliaria, m ientras que el
médico manda a la misma técnica que lo ayude en consultorio externo. ¿Estos problemas a que componentes de la administración corresponden?
A. Planificación
B. Organización
C. Control
D. Dirección
E. Evaluación
120. SSPP En un estudio para comprobar la relación entre el consumo de tabaco y el cáncer de vejiga, se sig uió durante un año a 1.000
personas, de las cuales 500 fumaban, y otras 500 no; de las 50 personas que padecieron cáncer de vejiga durante este tiempo, 45 fumaban. ¿Cuál es
el riesgo individual de enfermar entre los expuestos al tabaquismo?:
A. 0,09.
B. 0,10.
C. 0,01.
D. 1,00.
E. 1,11.
121. PED Niño preescolar presenta bruscamente tos intensa. Al examen físico pulmonar: timpanismo y sibilantes unilaterales. Debe
sospecharse:
A. Epiglotitis
B. Síndrome de escape de aire
C. Cuerpo extraño
D. Neumonitis intersticial
E. Crisis asmática.
122. PED Un niño de 3 años de edad presenta hace 3 días un cuadro catarral leve, con fiebre leve, adenopatías retroauriculares y e xantema
maculopaular rosado de inicio en cara. El diagnóstico más probable es:
A. Eritema infeccioso.
B. Escarlatina.
C. Exantema súbito.
D. Rubéola.
E. Sarampión
123. PED Un lactante de seis meses, amamantado con leche materna, presenta desde hace 5 horas episodios repetitivos de llanto inte nso y
vómitos. Presenta también desde hace 2 horas dos deposiciones semilíquidas y sanguinolentas con moco. No se refiere fiebre. El diagnóstico más
probable es:
A. Diarrea por Shigella.
B. Diarrea por Campylobacter.
C. Divertículo de Meckel.
D. lnvaginación intestinal.
E. Alergia a la proteína de leche.
124. PED Un recién nacido, de seis horas de vida, presenta párpados edematizados y conjuntivas enrojecidas con secreción ocu- lar clara. El
diagnóstico más probable es:
A. Conjuntivitis química
B. Dacriocistitis
C. Oftalmia neumococica
D. Oftalmia gonocócica
E. Conjuntivitis por clamydia
125. PED Un lactante de 1700 gr sufrió asfixia al nacimiento y después de maniobras de reanimación presentó numerosos episo - dios apneicos.
Al tercer día de vida empezó a vomitar y presentó distensión abdominal con evacuaciones sanguinolen- tas. El diagnóstico más probable es:
A. Intususcepción
B. Enterocolitis necrotizante
C. Vólvulo
D. Aganglionosis
E. Gastroenteritis.
126. PED RN pretérmino que, a las 6 h de vida, presenta quejido espiratorio a la auscultación, ausencia de retracción esternal, tiraje intercostal
leve con movimientos toraco-abdominales sincrónicos y sin aleteo nasal. Su puntuación en el Score de Silverman será:
A. 2
B. 3
C. 4
D. 5
E. 6
127. PED Un recién nacido de parto vaginal y peso al nacer de 4 kilos, presenta al examen: deformación de la cabeza que sobre- pasa las suturas
craneales y que deja fóvea a la presión. ¿Cuál es el diagnóstico más probable?
A. Caput succedaneum
B. Céfalohematoma
C. Fractura de cráneo
D. Rodete
E. Hidrocefalia
128. PED Un lactante de seis meses y sin antecedentes de importancia, presenta coriza y fiebre y tos desde hace tres días. Desde h ace 12 horas
se agrega respiración rápida y al examen se auscultan roncantes y subcrepitantes. ¿Cuál es el diagnós - tico más probable?
A. Neumonía
B. Bronquiolitis
C. Laringotraqueitís
D. Rinofaringitis
E. Crisis asmática
129. PED Un niño de 3 años traído a emergencia por haber presentado convulsiones tónico-clónicas generalizadas que duraron 5’. Al examen: T:
40°C, se auscultan roncantes en ambos campos pulmonares y presenta déficit motor en miembro in- ferior derecho y somnolencia post convulsión.
No tiene antecedentes relevantes. ¿Cuál es el diagnóstico más probable?
A. Convulsión febril compleja
B. Episodio de escalofríos
C. Meningoencefalitis bacteriana aguda
D. Convulsión febril simple
E. Convulsión asociada a fiebre
130. PED Niño de 4 años que presenta tos y dificultad respiratoria todas las noches y durante la actividad física. ¿Cuál es el
tratamiento a seguir?
A. Corticoide inhalado a altas dosis y beta 2 de acción prolongada
B. Corticoide inhalado a baja dosis y adrenalina
C. Corticoide inhalado a baja dosis y cromonas
D. Bromuro de ipratropio y fluticasona
E. Corticoide inhalado de acción corta y teofilina
131. PED Después de haber sufrido un traumatismo craneal, un niño de 10 años tiene 4 puntos en la escala de Glasgow y presenta res piración
irregular, en este momento está indicado:
A. Practicarle intubación endotraqueal.
B. Administrarle naloxona.
C. Administrarle Ringer lactato.
D. Administrarle manitol.
E. Administrarle dexametasona.
132. PED La persistencia del sexto arco aórtico izquierdo y que se manifiesta en el periodo neonatal por dificultad respiratoria,
precordio hiperdinámico y pulsos periféricos amplios es la:
A. Atresia pulmonar con septum interventricular intacto.
B. Persistencia del conducto arterioso.
C. Comunicación interventricular.
D. Conexión anómala de las venas pulmonares.
E. Tetralogía de Fallot.
133. PED En un neonato que en las primeras horas de vida presenta vómitos de contenido biliar, y cuya radiografía de tórax y abdomen muestra
el signo de la “doble burbuja”, el diagnóstico más probable es:
A. Estenosis congénita del píloro.
B. Atresia duodenal.
C. Invaginación intestinal.
D. Hernia diafragmática.
E. Malrotación intestinal.
134. PED Un escolar de seis años que presenta temperatura de 39 grados C., movimientos tónicos del miembro superior dere - cho, midriasis y
pérdida del estado de alerta, tiene una crisis convulsiva de tipo:
A. Tónico.
B. Mioclónico.
C. Parcial simple.
D. Parcial compleja.
E. Febril compleja.
135. PED Una lactante de 6 meses que tiene potasio sérico de 7.5 mEq/L y que además presenta arritmia cardiaca, debe ser tratada c on:
A. Bicarbonato de sodio, glucosa e insulina.
B. Gluconato de calcio, salbutamol inhalado y glucosa con Insulina.
C. Glucosa y furosemida.
D. Bicarbonato de sodio, sulfato de magnesio y furosemida.
E. Resinas de intercambio iónico, glucosa y furosemida.
136. PED Un niño de 10 años ingresa al hospital por presentar un cuadro clínico de tres semanas de evolución caracterizado por edema
generalizado. Las pruebas de laboratorio informan: hemoglobina 10.5 g/dl, leucocitos: 3 500/mm3, Urea 15 mg/dl, creatinina 2 mg/dl, albúmina
sérica 2.2 g/dl, globulinas séricas 4 g/dl, y colesterol total 280 mg/dl. El diagnóstico más probable es:
A. Síndrome nefrótico.
B. Glomerulonefritis aguda.
C. Lupus eritematoso.
D. Hipotiroidismo.
E. Insuficiencia renal.
137. PED Un paciente de 7 años de edad presenta gingivorragia y lesiones petequiales en tronco y extremidades, sin otras alte- raciones al
examen físico. Hma: 8 940/mm3, Hb: 11.5 g/dl, Plaquetas: 60 000/mm3. El diagnóstico más probable es:
A. Púrpura Trombocitopénica idiopática
B. Linfoma de Hodking
C. Leucemia linfoblástica aguda
D. Síndrome de Wiskott Aldrich
E. Púrpura de schonlein Henoch
138. PED ¿Cuántos gramos de proteína le corresponden a un escolar de 30 Kg?
A. 30 gr/día
B. 15 gr/día
C. 20 gr/día
D. 1.5 gr/día
E. 2 gr/día
139. GO En una paciente que se le realiza una amniografia y se aprecia imagen en panal de abeja. A que patología nos estamos refiriendo.
A. Aborto espontáneo
B. Embarazo ectópico
C. Pseudociesis
D. Down
E. Mola
140. GO Femenina de 22 años de edad, casada hace 6 meses, la cual refiere amenorrea de 16 semanas de evolución. A la explo- ración se
observa, signo de Chadwick, Godell y útero entre la sinfisis del pubis y cicatriz umbilical. Con estos datos usted puede hace r el diagnóstico de
A. Posible embarazo
B. Embarazo confirmado
C. Embarazo positivo
D. Posibilidad de embarazo
E. Probable embarazo
141. GO Femenina de 23 años la cual cursa con su 38 semana de gestación. Refiere que hace 2 horas arrojo una masa muco- sanguinolenta. A la
exploración se observa cuello con 3 cm de dilatación, presentación cefálica en OIA, FCF 138. Con estos datos usted haría el diagnostico de:
A. Placenta previa
B. DPPNI
C. Embarazo a término
D. Trabajo de parto en fase pasiva
E. Trabajo de parto en fase activa
142. GO En una paciente de 38 semanas de gestación la cual usted le pide un eco y este le reporta todo normal con líquido am - niótico de 800
ml. Con estos datos usted hace el diagnostico de:
A. Polihidramnios
B. ECO normal
C. Embarazo normal
D. Insuficiencia útero placentaria
E. Oligohidramnios
143. GO Gestante a término de 35 años acude a emergencia por contracciones uterinas cada 3 minutos y sangrado vaginal de 10cc. PA: 120/70,
P: 84x’, FR: 16x’, ecografía del día informa: feto único en LCI, placenta previa marginal. LCF: 148x’, s e incida vía, análisis y se realiza TV en condiciones
de operabilidad: incorporación: 90%, dilatación: 8cm, membranas íntegras, altura de presentación: 0, variedad de posición: OI IA, pelvis ginecoide.
¿Cuál es la conducta a seguir?
A. Acentuación del trabajo de parto
B. Esperar a que continúe el trabajo de parto espontáneo
C. Parto instrumentado
D. Preparar para cesárea inmediata
E. Transfusión de sangre
144. GO Puérpera de parto gemelar que presenta sangrado vaginal profuso luego del alumbramiento. El diag nóstico más pro- bable es:
A. Laceración de cuello uterino
B. Ruptura uterina
C. Coagulación
D. Atonía uterina
E. Retención de restos placentarios
145. GO Primigesta con 32 semanas de gestación por última regla, acude a emergencia por dolor en epigastrio e hipocondrio derecho. PA:
180/110 mmHg, proteinuria ++, tacto vaginal: cérvix posterior, orificio externo cerrado, pelvis ginecoide. El diagnóstico más probable es:
A. Preeclampsia severa y trabajo de parto
B. Preeclampsia superpuesta e hipertensión crónica
C. Preeclampsia severa
D. Preeclampsia severa y colecistitis
E. Preeclampsia severa y DPP
146. GO Una gestante a término de 18 años, sin antecedentes de importancia, con 180/100 mm Hg de presión arterial, en 2 tomas, y c on una
proteinuria de 350 mg en orina de 24 horas su diagnóstico más correcto sería:
A. Preeclampsia leve.
B. Hipertensión transitoria leve.
C. Preeclampsia severa.
D. Hipertensión transitoria severa
E. HTA crónica con PE severa
147. GO En una paciente que cursa su 32 semana de gestación usted le pide un ECO ya que la altura del fondo uterino es mayor a la esperada y
el ECO le reporta que el líquido amniótico es de 2100 ml. Con esto usted puede decir que la paciente tiene
A. Diabetes gestacional
B. Producto con anencefalia
C. Oligohidramnios
D. Producto con atresia esofágica
E. Polihidramnios
148. GO Mujer obesa en edad reproductiva, con oligomenorrea, infertilidad e hirsutismo. El diagnóstico más probable es:
A. Síndrome de ovario poliquístico
B. Tumor funcionante de ovario
C. Hiperprolactinoma
D. Hiperplasia suprarrenal
E. Síndrome de Kallman
149. GO Mujer de 35 años, acude a consulta por metrorragia de dos meses, sin otro antecedente de importancia. Examen clínico: páli da, peso
61 kg, PA 110/76, pulso 68 X’, útero se palpa dos cm por encima del pubis. Al examen pélvico: útero anterior con nódulo de 4 cm a nivel fúndico,
anexos normales. Hemoglobina 9 g/dL. ¿Cuál es el diagnóstico más probable?
A. Endometrioma
B. Leiomioma uterino
C. Quiste ovárico
D. Sarcoma uterino
E. Teratoma
150. GO Una mujer de 53 años de edad que no ha menstruado durante un año ha comenzado con tratamiento de restitución hormonal cícl ico.
Tiene ligero sangrado vaginal que dura dos días, mientras comienza su segundo ciclo de restitución. Está sana, su índice de masa corporal (BMI) es
de 21, su presión arterial es normal, y usó anticonceptivos orales hasta los 42 años. Se rehúsa a una muestra endometrial. El siguiente paso más
apropiado para tratar su sangrado es:
A. Comenzar un calendario menstrual.
B. Hacer un Papanicolaou, lo que incluye una muestra endocervical.
C. Insistir en una muestra endometrial.
D. Realizar una colposcopia.
E. Realizar una ecografía transvaginal para medir el grosor endometrial.
151. GO Una mujer de 33 años no puede sentir el hilo de su dispositivo intrauterino. Su última menstruación fue hace una semana. Una prueba
sérica de embarazo es negativa. La mejor acción a seguir es:
A. Insertar otro DIU para reemplazar el que se perdió
B. Hacer una radiografía abdominal
C. Hacer una ecografía pélvica
D. Realizar una histerosalpingografía
E. Palpar el cuello uterino cuidadosamente para jalar el hilo
152. GO Una mujer de 29 años de edad G 6 P 1041 se presenta con retraso menstrual de 2 semanas; su régimen catamenial es 3/28 regu lar,
tiene un examen de gonadotrofina en orina positivo. Ha tenido su primer embarazo a término normal, luego 2 abortos espontáneos, un aborto
provocado y el último fue un embarazo ectópico. No refiere enfermedades previas, tuvo salpingectomía derecha. ¿Cuál de la s siguientes alternativas
sería la primera medida a adoptar?
A. Dosaje sérico cuantitativo de HCG Beta.
B. Test triple (HCG, Alfa feto proteína, estriol)
C. Anticuerpos anticardiolipinas
D. VDRL
E. Ecografía transvaginal
153. GO Ingresa una primigesta de 18 años de edad con 32 semanas de gestación refiriendo cefalea intensa y epigastralgia tipo opresivo. Al
examen tiene presión arterial 180/120 mm Hg. No hay dinámica uterina, presenta movimientos fetales y los latidos del feto flu ctúan entre 130 y
150, la altura uterina es de 25 cms. con feto en LCI. Los reflejos patelares están en 3+/4+. ¿Cuál es el manejo farmacológico inmediat o?
A. Sulfato de magnesio 4 g EV en 20’, nifedipino 10 mg VO
B. Diazepam 10 mg EV, labetalol 200 mg EV
C. Sulfato de magnesio 10 g IM, alfa metil dopa 1 g VO
D. Fenitoína 900 mg VO, hidralazina 10 mg EV
E. Diazepam 5 mg EV, nifedipino 10 mg Sublingual
154. GO Una paciente de 47 años acude refiriendo sensación de bulto en vagina, no hay incontinencia urinaria, su última regla fue hace dos
años. Al examen de acuerdo a la clasificación POPQ presenta las siguientes medidas: Aa +2; Ba +3; C
-5; Hiato Genital (gH) 5; Cuerpo perineal (pB) 3; longitud total de vagina (Tvl) 10; Ap -1; Bp -2; D -6.
+2 +3 -5
5 3 10
-1 2 -6
¿Cuál es el grado de distopia de la paciente?
A. 0
B. 1
C. 2
D. 3
E. 4
155. GO Paciente mujer de 30 años de edad, que acude por dolor pélvico de una semana de evolución que se inicia dos días después d e haber
sido sometida a una histerosalpingografía por infertilidad. Hace tres días refiere fiebre de 38 0C. Al examen presenta dolor en abdomen bajo y se
insinúa rebote, el examen bimanual del útero y anexos es difícil de evaluar por el dolor, el resto del examen es normal. ¿Cuá l es el diagnóstico más
probable en este caso?

A. Apendicitis aguda
B. Endometriosis
C. Enfermedad pélvica inflamatoria
D. Ruptura folicular
E. Infección urinaria
156. CIR Paciente varón de 19 años de edad con fractura de fosa orbitaria derecha, al examen presenta: Ptosis palpebral y m irada hacia adentro
y abajo, que nervios podrían encontrarse afectados:
A. IV nervio craneal y rama interna del oftálmico
B. VI nervio craneal y rama interna del oftálmico
C. III nervio craneal y rama interna del oftálmico
D. III y IV nervios craneales
E. III y VI nervios craneales
157. CIR Paciente que viene a consultar por sufrir de tumoración en la parte lateral superior e inferior derecha del hígado, evide nciada por
tomografía hepática, se pregunta: ¿Cuáles son los segmentos hepáticos afectados?
A. I Y II
B. II Y IV
C. V Y VIII
D. V Y VI
E. VI Y VII
158. CIR Paciente que sufre un accidente de tránsito por impacto o choque de vehículo con otro, por detrás. El conductor del vehíc ulo
impactado resulto muerto asido al timón, se pregunta: ¿Qué articulación(es ) vertebral resultaron compro- metidos?
A. Las 3 primeras articulaciones interapofisarias cervicales
B. Articulación atloidoaxoidea media
C. Articulación interapofisaria de la prominente
D. Sexta articulación interapofisaria cervical
E. Articulación odontoatloidea
159. CIR Ante un niño de 5 años con un cuadro de hipertensión endocraneal, alteraciones visuales e hipotalámicas, que pre - senta una
radiografía lateral del cráneo con calcificaciones en forma de paréntesis a nivel supraselar. ¿Cuál será su diagnóstico presuntivo?
A. Meduloblastoma
B. Craneofaringioma
C. Adenoma hipofisario
D. Glioma del nervio óptico
E. Pinealoma productor de hidrocefalia
160. CIR Para un paciente con antecedentes de nefropatía crónica y apendicitis aguda perforada con perit onitis localizada.
¿Cuál de las siguientes alternativas de manejo sería la más efectiva y segura?
A. Apendicectomía, drenaje y una asociación de Amikacina y Metronidazol por 48 horas.
B. Apendicectomía, drenaje y una asociación de Amikacina y Metronidazol por siete dÍas.
C. Apendicectomía, lavado de cavidad y una asociación de Cefuroxima y Clindamicina por siete dias.
D. Apendicectomía, drenaje y una asociación de Cefuroxima y Metronidazol por siete días.
E. Apendicectomía, drenaje y antibiótico-profilaxis con cualquiera de los esquemas mencionados.
161. CIR Un paciente de 60 años acude a emergencia con quemaduras de 2do grado en la región anterior del tronco, los genita -les y el dorso de
la mano derecha. ¿Qué porcentaje de superficie corporal quemada presenta?
A. 20%.
B. 18%.
C. 11%.
D. 36%.
E. 38%.
162. CIR Chofer que sufre accidente de tránsito (choque) es llevado a emergencia porque presenta dolor abdominal. Al examen clínic o presenta
PA 80/40 mmHg. Pulso 105 por minuto, despierto, dolor abdominal difus o a la palpación, reacción peritoneal dudosa, ruidos hidroaereos escasos. Su
conducta a seguir será:
A. Hospitalizar al paciente y programar a sala de operaciones por tratarse de un traumatismo abdominal cerrado.
B. Enviar al paciente a su domicilio e indicarle que regrese si empeora.
C. Mantener al paciente en observación con hidratación parenteral, sonda Foley, control de hematocrito seriado.
D. Colocar vía parenteral, aplicar antiespasmódico y solicitar TAC abdominal.
E. Realizar paracentesis diagnóstica para determinar si es traumatismo.
163. CIR Al ser atropellado, un hombre de 52 años recibe una contusión directa en la pelvis. En la exploración física se encuen - tra sangre en la
uretra; al efectuar el tacto rectal se percibe que la próstata está despla zada. El diagnóstico clínico más probable es:
A. Hematoma prostático
B. Lesión vesical.
C. Laceración de la uretra.
D. Laceración del trígono.
E. Desgarro perineal.
164. CIR Para un paciente de 16 años, con diagnóstico de torsión testicular de ocho horas de evolución, el tratamiento más adecuado consist e
en practicar la:

A. Orquiectomía.
B. Destorsión manual.
C. Orquidopexia.
D. Destorsión quirúrgica.
E. Resección del área afectada.
165. CIR Una mujer de 20 años recibió un golpe contuso en la región lumbar izquierda. Acude al hospital 8 días después con icteri cia
generalizada, dolor en el hombro izquierdo, temperatura de 38.5 grados C., frecuencia cardiaca de 90 por minuto y pres ión arterial de 90/60 mmHg.
El procedimiento quirúrgico más adecuado en este momento es:
A. Esplenectomía.
B. Nefrectomía.
C. Drenaje del hematoma.
D. Laparotomía exploradora.
E. Lavado peritoneal.
166. CIR Varón de 78 años, con antecedente de hiperplasia prostática, que presenta elevación del PSA (100 ng/ml) y lumbago. En el tacto rectal
se encuentra una próstata grande, de consistencia dura en ambos lóbulos, de superficie nodular y de bordes no bien definidos. AL realizarle los
procedimientos que corresponden se realiza el diagnostico de adeno- carcinoma de próstata pobremente diferenciado que afecta ambos lóbulos y
que infiltra las vesículas seminales, con metástasis ósea. ¿Cuál es el tratamiento más adecuado?
A. Extirpación total de la próstata
B. Quimioterapia enérgica
C. Radioterapia pelviana externa
D. Braquiterapia prostática.
E. Hormonoterapia
167. CIR Una mujer de 24 años que acusa visión borrosa y enrojecimiento del ojo derecho de aproximadamente 15 días de evo- lución. Hace 5
días se agrega dolor en el mismo ojo acompañado de lagrimeo. El examen revela AV de 20/400 en ese ojo, inyección conjuntival periquerática y una
pupila irregular aparentemente arreactiva. En un caso como éste, ¿cuál sería la causa más probable del síndrome?
A. Glaucoma agudo congestivo
B. Escleritis necrotizante
C. Neuritis óptica
D. Iridociclitis aguda
E. Queratoconjuntivitis por cuerpo extraño
168. CIR Paciente post operada de apendicitis + peritonitis. Presenta cuadro respiratorio con tos y bulto que protruye a través de herida
operatoria con asas intestinales exteriorizadas. Se trata de:
A. Eventración
B. Evisceración
C. Fuga intestinal
D. Estrangulación
E. Hernia inscional
169. CIR Paciente mujer de 52 años que acude a la emergencia por presentar dolor abdominal y tumora ción en región umbilical de 12 horas de
evolución, se trata de reducir y con mucho esfuerzo se consigue pero protruye nuevamente de inmedia - to, se trata de una hernia:
A. Indirecta
B. Inguino-escrotal
C. Incoercible
D. Encarcelada
E. Por deslizamiento
170. CIR Paciente mujer de 64 años con cuatro días de enfermedad, caracterizado por dolor abdominal tipo cólico, náuseas, vómitos
fecaloideos, dificultad para defecar y no eliminación de flatos; al examen: distensión abdominal, ausencia de ruidos hidroaé reos, poco depresible.
Ecografía: Vesícula pequeña, de paredes engrosadas, con cálculos, colédoco 6 mm; Radiografía de abdomen: niveles hidroaéreos, edema de pared
intestinal, neumobilia. ¿Cuál es la causa más probable de obstrucción intestinal?
A. Hernia crural complicada.
B. Íleo biliar.
C. Cáncer de ciego.
D. Bridas y adherencias.
E. Hernia interna.
1 71. CIR Mujer de 62 años que acude a la consulta por presentar dolor en el borde interno del antebrazo y a nivel del dedo
anular y meñique de la mano derecha, también explica hormigueos, frialdad y pérdida de sensibilidad. ¿Qué tipo de
neuropatía por compresión tiene el paciente?
A. Síndrome canal carpiano
B. Atrapamiento de nervio mediano y cubital a nivel de la muñeca
C. Compresión proximal del nervio cubital
D. Sección nerviosa cubital
E. Compresión del nervio radial
172. MED Un hombre asintomático presenta en dos análisis de sangre rutinarios glucemias basales de 132 y 130mg/dl ¿Cuál será la ac titud
correcta?
A. Se le pide una curva de glucemia para diagnóstico
B. Se inicia tratamiento con glibenclamida
C. Se inicia tratamiento con dosis de insulina NPH
D. Se le pide nuevos análisis dentro de un año
E. Se le diagnostica diabetes mellitus tipo 2 y se le inicia tratamiento con dieta
173. MED Paciente adulto traído a la Emergencia por los Bomberos, quienes lo encontraron inconsciente en la calle. Al examen: pálido
sudoroso, sin signos de agresión. PA: 120/60 mmHg, FC: 100 por minuto, FR: 12 por minuto, saturación de oxígeno 95%, pupilas centrales de 3,5
mm, fotorreactivas lentas. Su diagnóstico más probable es coma…:
A. Hiperglicémico
B. Hipoglicémico
C. Barbitúrico
D. Hipoxémico
E. Vigil
174. MED Mujer de 38 años de edad, obesa, con trastorno del sensorio, sequedad de piel, caída del cabello, bradipsiquia e hipo- tensión
arterial. Dosaje de T4 disminuida y TSH elevada. ¿Cuál es el tratamiento de elección?:
A. Hormona tiroidea de inicio sin corticoides
B. Hormona tiroidea a dosis altas y corticoides a dosis bajas
C. Corticoides a dosis altas, luego hormona a tiroidea a dosis bajas
D. Corticoides y luego adicionar hormona tiroidea
E. Hormonas tiroideas, corticoides y diuréticas
175. MED Varón de 56 años, presenta dolor retroesternal súbito e intenso de 6 horas de evolución. El EKG revela onda Q y seg - mento ST
elevado en DII, DIII, AVF ¿Cuál es el diagnóstico y que fármaco usaría?
A. Tromboembolismo pulmonar / heparina.
B. Infarto agudo de miocardio / heparina.
C. Pleurodinea / antiinflamatorios no esteroideos.
D. Trombosis pulmonar / trombolítico.
E. Infarto agudo de miocardio / trombolítico.
176. PED Paciente de 13 años, 15 días antes viajó a Piura. Presenta fiebre desde hace 5 días, artralgia, cefalea, dolor retro ocul ar y erupción
cutánea. ¿Cómo se clasifica el caso?
A. Probable dengue con signos de alarma
B. Probable dengue grave
C. Probable dengue sin signos de alarma
D. Dengue confirmado
E. Dengue descartado
177. MED Mujer de 24 años con varios días de fiebre, de mayor intensidad por las tardes. Recibe tratamiento sintomático sin mejorí a; hace 3
días se agrega dolor a nivel sacro iliaco, persistiendo el cuadro febril. ¿Cuál es el diagnóstico más probable?
A. Salmonelosis
B. Malaria
C. Pielonefritis aguda
D. Leptospirosis
E. Brucelosis
178. MED Una mujer de 64 años notó lesiones del tipo ampollosas en sus muslos y axilas. Eran pruriginosas, pero no dolían. No tenían otros
síntomas. En la exploración las lesiones son grandes, a tensión, en forma de ampollas de contenido seroso. La biopsia confirm o el diagnóstico de
penfigoide ampolloso. Cuál de las siguientes características histológicas es típica de esta condición:
A. Cambios inespecíficos
B. Depósitos de inmunoglobulina A (igA)
C. Lesiones dentro de la epidermis (acantolisis)
D. Depósitos de inmunoglobulina M (igM)
E. Lesiones de la membrana basal
179. MED Es una causa de acidosis con hiato aniónico elevado:
A. Cetoacidosis diabética
B. Diarrea
C. Acidosis tubular renal
D. Inhibidores de la enzima convertidora de angiotensina
E. Hipopotasemia
180. MED Una mujer de 85 años tiene lesiones ampollosas extensas en el abdomen que aparecen y desaparecen de manera espon- tánea, sin
ningún tratamiento, El signo de Nikolsky es negativo. Cuál de los siguientes diagnósticos es el más probable
A. Pénfigo vulgar
B. Dermatitis herpetiforme
C. Herpes gestacional
D. Eritema multiforme
E. Penfigoide ampolloso
181. MED Masculino de 40 años con antecedente de ingesta crónica de esteroides y salicilatos, que ingresa a urgencias por dolor ab dominal
que inicio 6 horas antes, el cual se inició de manera súbita en epigastrio. Se difundió a todo el vientre y que describe como muy intenso. Está en
posición de gatillo y no quiere movilizarse; aumenta su dolor a la palpación. El abdomen se palpa en madera. A la percusión h ay ausencia de la
matidez hepática y se ausculta silencio abdominal.
T.A. 140/90, pulso 100, respiración 20 y temperatura 37.5, su diagnóstico más probable es:
A. Oclusión intestinal
B. Apendicitis aguda
C. Pancreatitis
D. Cólico renal
E. Ulcera péptica perforada
182. MED Cuando al explorar la presión venosa yugular, se encuentra que el paciente tiene una onda “v” gigante, ¿cuál es el diagnóstico más
probable?
A. Estenosis aórtica
B. Insuficiencia mitral
C. Insuficiencia tricuspídea
D. Estenosis mitral
E. Insuficiencia aórtica
183. MED Un hombre de 72 años, oxigenodependiente, con antecedente de tabaquismo intenso desde su juventud, inicia su pade- cimiento
actual hace 4 días con disnea, tos en accesos, expectoración purulenta, fiebre y dolor torácico. La radiografía de tórax mues tra infiltrado basal y
broncograma aéreo. El agente etiológico más probable en este caso es:
A. Klebsiella pneumoniae.
B. Haemophilus influenzae.
C. Mycoplasma pneumoniae.
D. Estreptococcus pneumoniae.
E. Moraxella catarrhalis.
184. MED Varón de 58 años quien tiene diagnóstico desde hace 6 años de colitis ulcerativa, presenta desde hace 5 días fiebre, dia- rrea
inflamatoria, y dolor abdominal. Al realizarle rectosigmoidoscopía el resultado fue úlceras y exudado purulento y hemorrágico . .Luego de 72 horas de
tratamiento con antibióticos y corticoides se asocia hipotensión, taquicardia y dolor abdominal intenso. En la radiografía de abdomen se encuentra
dilatación de 9 cm del colon ¿Cuál es el diagnós- tico más probable y el tratamiento más adecuado?
A. Colitis pseudomembranosa asociada / Imipenem
B. Perforación de colon / Colectomía de urgencia
C. Brote severo de colitis ulcerativa / Corticoides y antibióticos
D. Sobreinfección / Moxifloxacino
E. Megacolon tóxico / Colectomía si no hay mejoría en 24 h
185. MED Paciente varón de 59 años, fumador, con diagnóstico de carcinoma broncogénico en pulmón izquierdo. Se le realizan los estudios
correspondientes encontrándose un nódulo ganglionar en la región paratraqueal baja derecha .Si aplica - mos la clasificación TNM ¿Cuál sería el
resultado?
A. T4.
B. N1.
C. N3.
D. N2.
E. M1.

186. MED Ante la sospecha de quilotórax en un paciente que tiene un dosaje de triglicéridos en el líquido pleural de 100mg/dl
¿Cuál es el estudio que se de realizar en el líquido pleural?
A. LDL en el líquido pleural
B. VLDL en el líquido pleural
C. Quilomicrones en el líquido pleural
D. Gradiente de albúmina sérico .líquido pleural
E. IDL en el líquido pleural
187. MED Paciente masculino de 19 años de edad, quien acude a consultar porque presenta episodios intermitentes de disnea, la cual llega a
ser de pequeños esfuerzos, el cuadro se acompaña de tos irritativa y de ¨silbidos¨; es predominantemente nocturno, y este últ imo episodio ha
durado más de 24 horas. A la exploración física se encuentra con frecuencia res - piratoria de 28 por minuto, taquicárdico 120 por minuto, con aleteo
nasal y tiros intercostales, y en la auscultación de tórax se encuentran sibilancias difusas bilaterales. Con estos datos su sospecha clínica es:
A. Neumotórax espontáneo primario
B. Fibrosis pulmonar idiopática
C. Enfisema pulmonar
D. Asma bronquial
E Neumonía
188. MED Mujer de 42 años acude por consulta por prurito generalizado. Ella niega otros síntomas. En los exámenes de labora - torio se
encuentra una fosfatasa alcalina aumentada en 10 VNS. Para confirmar una etiología hepatobiliar se solicita gammaglutamiltransferasa la cual esta
aumentada 5 VNS ¿cuál es el marcador serológico que se debería solicitar?
A. Anticuerpos antinucleares
B. Antimusculo liso
C. Microsomico hepatorrenal
D. Anticuerpos antimitocondriales
E. Alfafetoproteina
189. MED Hombre de 53 años acude a Emergencia por dolor abdominal severo, náuseas y vómitos alimentarios. Exámenes au - xiliares:
Leucocitos: 18 000/ mm3 y amilasa sérica marcadamente elevada. ¿Qué otra anormalidad en los exámenes de laboratorio podría encontrarse?
A. Hipoglicemia
B. Hipercolesterolemia
C. Hiperglicemia
D. Hipercalcemia
E. Hipercalemia
190. MED Varón de 70 años presenta lesiones eritematosas y ampollares pruriginosas que predominan en las axilas, en l a ingle y muslos. Hay
algunas lesiones en los antebrazos y en los miembros inferiores, además lesiones orales moderadamente dolorosas. El signo de Nikolsky es negativo.
No hay compromiso de los ojos. El diagnóstico más probable es:
A. Dermatitis herpetiforme
B. Pénfigo vulgar
C. Penfigoide buloso
D. Penfigoide cicatricial
E. Epidermolisis bulosa
91. MED Paciente varón de 40 años presenta desde hace 3 meses lesiones papulosas poligonales, localizadas en la cara anterior de las
muñecas y tobillos, acompañadas de prurito .El estudio histopatológico de una de las lesiones muestra una der- matitis de interfase con
degeneración vacuolar de las células basales. ¿Cuál es diagnóstico?
A. Pitiriasis rosada
B. Papulosis linfomatoide
C. Psoriasis eruptiva
D. Liquen plano
E. Acantosis nigricans
192. MED Paciente de 75 años que consulta por una mácula pigmentada heterocroma , de contorno irregular, de 2 x 3 cm de diámetro,
localizada en la mejilla derecha, que ha tenido un crecimiento muy lento en los últimos años ¿Cuál sería el diagnóstico más p robable?
A. Melanoma lentiginoso acral
B. Lentigo maligno
C. Carcinoma basocelular pigmentado
D. Nevo de Ota
E. Eritema fijo pigmentario
193. MED Paciente mujer de 40 años de edad, que acude a Emergencia por presentar hace dos semanas dolor intenso en la cara anterio r del
cuello, que aumenta con la deglución, y malestar general. Al examen físico: dolor a la palpación en la re- gión anterior del cuello, con leve
crecimiento asimétrico del lado izquierdo de la tiroides. El diagnóstico más probable es tiroiditis:
A. Aguda
B. Subaguda
C. De Hashimoto
D. Tuberculosa
E. De Riedel
194. MED Mujer de 20 años, presenta confusión, somnolencia, convulsiones tónico-clónicas generalizadas, al examen rigidez de nuca,
brudzinski positivo, vibraciones vocales abolidas y egofonía en base de hemitórax derecho. Punción lumbar: pleocitosis, glucosa: 15, proteínas 150,
ADA: 7. Toracocentesis: exudado. Western blot para cisticercosis en sangre: negativo. HIV: negativo. TAC cerebral: múltiples imágenes heterogéneas
hipo e hiperdensas redondeadas en el parén- quima cerebral. El diagnóstico más probable es:
A. Neurocisticercosis.
B. Tuberculosis cerebral.
C. Toxoplasmosis cerebral.
D. Criptococosis cerebral.
E. Melanoma metastásico.
195. MED Una paciente de 25 años, sin antecedentes de interés acude a su consulta por presentar molestias precor diales inter- mitentes, a
veces prolongadas, que no son claramente opresivas y que en ocasiones se modifican con la respiración y los movimientos. La e xploración física es
anodina salvo que en la auscultación cardíaca se pone de manifiesto un click mesosis tólico con un soplo suave telesistólico. Teniendo en cuenta la
sospecha diagnóstica, cuál sería el tratamiento más indicado para esta paciente:
A. AAS, nitratos y calcioantagonistas.
B. AAS a dosis antinflamatorias y reposo en cama al menos 2 semanas.
C. Corticoides a dosis altas y, si no hay respuesta, añadir inmunosupresores.
D. Hielo, AINEs y vendaje compresivo.
E. Propranolol.
196. MED Mujer de 35 años, que desde hace 6 meses refiere dolor y edema en ambas manos. Examen físico: Dolor con a umento de volumen
en articulaciones metacarpofalángicas proximales y desviación cubital. ¿Cuál es el diagnóstico más pro- bable?
A. Artritis reumatoide
B. Fiebre reumática
C. Lupus eritematoso sistémico
D. Artritis gotosa
E. Artritis infecciosa
197. MED Paciente mujer de 20 años que presenta debilidad muscular en miembros inferiores; hiporeflexia, disminución del
tono muscular y se queja de parestesias, disestesias y allodinea. Cuál es el diagnóstico más probable:
A. Polineuropatía.
B. Síndrome piramidal.
C. Síndrome de motoneurona del asta anterior.
D. Síndrome de placa mioneural.
E. Mielitis transversa.
198. MED Mujer diabética de 42 años es llevada a emergencia por presentar un cuadro de sopor alteración del sensorio. Glucosa en s angre:
350mg/dl. ¿Cuál de los siguientes hallazgos confirma el diagnóstico de cetoacidosis?
A. Polidipsia, polifagia y poliuria.
B. Sudoración, enfriamiento generalizado y trastorno de conciencia.
C. Deshidratación severa hiperosmolaridad y convulsiones.
D. Dolor abdominal, respiratorio de Kussmaul y cetonuria.
E. Fiebre, decaimiento y malestar general.
199. MED ¿Qué medicamento elegiría en primer lugar para prevenir nuevos episodios maníacos en un paciente diagnosticado de trastor no
maníaco-depresivo y sin otros trastornos somáticos acompañantes?:
A. Carbamazepina.
B. Propanolol.
C. Carbonato de litio.
D. Diazepán.
E. Clonidina.
200. MED ¿Cuál de las siguientes actuaciones es la más importante en la prevención del fracaso renal agudo?:
A. Administración de manitol
B. Administración de furosemida
C. Evitar los aminoglucósidos
D. Valorar y corregir la hipovolemia
E. Vigilar la diuresis

DECIMO EXAMEN ENAM 1A. PARTE

SIMULACRO 10A

1. Cuadro clínico con foco neumónico en radiografía se ve lesión en base de pulmón derecho que borra aurícula derecha
¿qué lóbulo está afectado?
A. Inferior
B. Apical
C. Medio
D. Lingula
E. Superior

2. Gestante de 40 semanas que acude en periodo expulsivo del trabajo de parto. ¿Cuál de los movimientos principales del
trabajo es el más precoz?
A. Rotación interna
B. Extensión
C. Flexion
D. Rotación externa
E. Expulsión.

3. ¿Cuál es el volumen de rehidratación que debería recibir un lactante en quien se clasifica como deshidratación severa
sin shock?
A. 200 ml/Kg/día
B. 100 ml/Kg/día
C. 100 ml/SC/día
D. 3500 ml/Kg/día
E. 150 ml/Kg/día

4. Respecto a la enfermedad diverticular, señale lo correcto:


A. La diverticulosis siempre requiere tratamiento
B. La hemorragia diverticular suele ceder espontáneamente
C. Los divertículos son más frecuentes en el color ascendente
D. En el manejo de la diverticulitis siempre se opta por la colostomía
E. Es más frecuente entre los 30 y 50 años

5. Sobre tratamiento de HTA señale lo falso


A. Paciente con HTA con microalbuminuria es de elección captopril
B. Paciente con HTA e IRC terminal con K < 5.5 indicamos calcioantagonistas
C. Paciente con HTA y antecedente de osteoporosis indicamos tiazidas
D. Paciente con HTA de raza afroamericana indicamos beta block
E. En gestantes con HTA severa con labetalol

6. El agente Gram negativo más frecuente de la neumonía asociada a Ventilación es:


A. Escherichia coli
B. Enterococco faecalis
C. Pseudomona aeruginosa
D. Staphylococcus aureus
E. Haemophilus influenzae

7. ¿Cuál es la tasa de filtración glomerular compatible con paciente diabético en estadio temprano?
A. 90 ml/min
B. 60 ml/min
C. 150 ml/min
D. 30 ml/min
E. 15 ml/min

8. Mujer de 20 años de edad, peladora de espárragos desde hace 4 meses, presenta lesiones eritematosas pruriginosas
micropapulares localizada en dorso de antebrazos, cara y cuello. Las lesiones también se han identificado en algunas
compañeras de trabajo. No presenta antecedentes de atopía. Su primera impresión diagnóstica es y el tratamiento es:
A. Acarosis / permetrina
B. Dermatitis por contacto / corticoides
C. Acarosis / griseofulvina
D. Dermatofitosis / clotrimazol
E. Dermatofitosis / itraconazol

9. ¿ Cuál es el antibiótico de elección para la diarrea producida por campylobacter jejuni


A. Azitromicina
B. Dicloxacilina
C. Clindamicina
D. Cotrimoxazol
E. Ciprofloxacino

10. Son criterios mínimos para el diagnóstico de la enfermedad inflamatoria pélvica:


A. Dolor abdominal inferior, VSG elevada examen del flujo vaginal positivo para N. Gonorrhoeae
B. Dolor a la movilización cervical, absceso tuboovarico en ecografía, examen del flujo vaginal positivo para C.
Trachomatis

C. Dolor abdominal inferior, dolor a la movilización cervical, dolor anexial bilateral.


D. Dolor anexial bilateral, temperatura oral > 38 0C, flujo vaginal o cervical anormal.
E. Dolor hipogástrico persistente, proteína C reactiva elevada, flujo vaginal anormal.

11. Primigesta en trabajo de parto, 6 cm de dilatación con membranas íntegras prominentes, C–4. Se tacta cordón; con-
jugado diagonal 10,5 cm; ponderado fetal 3.500 g. ¿Cuál es su diagnóstico?:
A. Desproporción céfalo-pélvica por macrosomía fetal.
B. Desproporción céfalo-pélvica y prolapso de cordón.
C. Estrechez pelviana y deflexión II.
D. Estrechez pelviana y procúbito de cordón.
E. Estrechez pelviana y prolapso de cordon

12. Acude a consulta un paciente de 3 años de edad. Mama refiere que presento un cuadro de rinorrea y estornudos leves.
Luego empezó a presentar lesiones tipo maculopapulares que se extendieron en todo su cuerpo al primer dia de exan-
tema y que luego desaparecieron con descamación furfurácea al tercer día de enfermedad. ¿Cuál es el diagnostico?
A. Exantema súbito
B. Roseola
C. Sarampion
D. Rubeola
E. Escarlatina.

13. Paciente joven leptosomico con colelitiasis, además se encuentra el examen físico esplenomegalia, además presente
astenia y en su perfil hepático destaca la elevación de bilirrubina a predominio indirecto., ante la sospecha diagnostica
solicitamos
A. Test de HAM
B. Electroforesis de hemoglobina
C. PCR enzimático
D. Test de fragilidad osmótica
E. Gota gruesa

14. En el fenómeno de Raynaud, el paciente puede recibir como terapia farmacológica:


A. Prednisona
B. Isorbide
C. Minoxidil
D. Propanolol
E. Amlodipino

15. Según la clasificación de Johnson de ulcera gástrica señale lo verdadero


A. La tipo II es la más frecuente
B. La tipo III cursa con secreción de ácido disminuido
C. La tipo I se ubica en curvatura mayor
D. La tipo IV cursa con hipersecreción de acido
E. La tipo V se asocia a lesión por AINES
16. En relación al diagnóstico de cardiopatía isquémica señale lo falso
A. En caso de duda diagnostica en la angina estable solicitamos ergometria
B. El estudio de mayor dato pronóstico es la angiografía coronario
C. La forma aguda produce un trombo que causa oclusión coronaria
D. El cuadro crónico se produce una estenosis fija que va progresando en años
E. la oclusión total se manifiesta eléctricamente sin elevación del segmento ST

17. Respecto al cáncer de colon, es incorrecto:


A. La presentación clínica del cáncer de hemicolon derecho es fundamentalmente obstructiva
B. Los adenomas colorrectales se consideran lesiones premalignas
C. El tipo más frecuente es el adenocarcinoma
D. Es más frecuente en mayores de 50 años
E. El consumo de grasas saturadas aumenta el riesgo

18. Paciente adulto con diagnóstico de neumonía por estafilococo. Recibe tratamiento durante varios días con Vancomici-
na, pero no mejora. ¿Qué antibiótico recomendaría en este paciente?
A. Ceftriaxona
B. Piperacilina tazobactam
C. Linezolid
D. Meropenem
E. Clindamicina

19. ¿Cuál de las siguientes es un tipo de exantema conocido como la quinta enfermedad?
A. Exantema súbito
B. Eritema infeccioso
C. Varicela
D. Sarampion
E. Rubeola.

20. Mujer de 25 años, puérpera de 1 horas de parto vaginal, RN de 4 500 gr, presenta abundante sangrado rojo rutilan- te, se
encuentra pálida, taquicárdica, sudorosa, útero a 3 cm debajo de la cicatriz umbilical. ¿Cuál es el diagnóstico probable?
A. Atonía uterina
B. Retención de restos placentarios
C. Hipotonía uterina
D. Embolia de líquido amniótico
E. Laceración del tracto genital

21. Es incorrecto respecto a las quemaduras de tercer grado:


A. La superficie es indolora
B. Destruye toda la dermis
C. Tiene un color oscuro
D. La superficie es generalmente seca
E. Presenta ampollas en la superficie

22. ¿Cuál de los siguientes enteroparásitos NO transmite por fecalismo?:


A. Lumbricoides
B. Cryptosporidium parvum
C. Entamoeba histolytica
D. Strongyloides stercolaris
E. Trichuris trichiuria

23. Mujer de 64 años, G:1, P:1001, menarquia: 10 años. FUR: 55 años, obesa, diabética e hipertensa desde hace 10 años.
Acude por ginecorragia. Ecografía: endometrio 20 mm y volumen uterino 110 cc. PAP: AGUS ¿cuál es el diagnóstico más
probable?
A. Adenocarcinoma endometroide
B. Carcinoma de células claras
C. Carcinoma indiferenciado
D. Carcinoma mucinoso
E. Carcinoma seroso papilar

24. No es un criterio de causalidad de Hill:


A. Fuerza de asociación
B. Número necesario a tratar
C. Experimentación
D. Consistencia
E. Gradiente biológica

25. ¿Cuál de las siguientes es INCORRECTA respecto a la leche y lactancia materna?


A. La galactosemia es una contraindicación
B. El calostro tiene más contenido de grasas en comparación a la leche madura
C. La leche madura se obtiene a partir del dia 14
D. El contenido de Calcio es menor en la leche humana en comparación a la leche de vaca
E. La hierro de la leche materna es mejor absorbido que el de la leche de vaca.

26. Es característica de la anestesia epidural:


A. Atraviesa la duramadre
B. Siempre ocasiona cefalea post punción
C. Es necesario siempre atravesar el ligamento amarillo
D. Es un tipo de anestesia general
E. Puede causar hipertensión arterial

27. Paciente varón joven con datos de trombosis venosa profunda, además se encuentra ictericia a predominio indirecto con
coluria, la sospecha diagnostica es:
A. Hemoglobinuria paroxística nocturna
B. Esferocitosis
C. Talasemias
D. Hemoglobinopatías
E. Eliptocitosis

28. Mujer de 32 años de edad, con diagnóstico de artritis reumatoide. ¿Cuál de los siguientes hallazgos indica un buen
pronóstico?:
A. Elevación de la velocidad de sedimentación
B. Títulos bajos de factor reumatoide
C. Títulos elevados de haptoglobina
D. Presencia de nódulos subcutáneos
E. Presencia erosiones en radiografía de manos

29. Paciente de 80años de edad, que presenta en ambas extremidades ampollas grandes, tensas, algunas con contenido
sanguíneo, sobre una base urticariana y con moderado prurito. No hay compromiso del estado general y la histopato- logía
muestra lesiones subdermicas con acumulo de eosinofilos en la dermis papilar. La inmunofluorescencia muestra depósito
lineal de IgG y de C3. ¿Cuál es la dermatopatia más probable?
A. Dermatitis herpetiforme
B. Penfigo cicatricial
C. Penfigo seborreico
D. Penfigo vulgar
E. Penfigoide ampollar

30. Paciente varón de 3 años de edad, quien desde hace aproximadamente 4 días presenta deposiciones liquidas en fre-
cuencia de 5 por día sin sangre sin moco. Al examen físico presenta deshidratación de grado moderado. ¿Cuál es el rango
de volumen que debería recibir dicho paciente?
A. 20-80 ml/Kg/día
B. 30-80 ml/Kg/día
C. 30-80 ml/Kg/4horas
D. 80-100 ml/Kg/3horas
E. 30-50 ml/Kg/12horas.
31. Mencione el dato de mal pronóstico en pancreatitis aguda
A. El índice de severidad tomografía más de 5 puntos
B. Secuestro de líquidos al ingreso
C. Hipocalcemia a las 48 horas
D. Apache más de 7 puntos
E. Disminución de PCR

32. El precursor metabólico inmediato de la noradrenalina es:


A. Isoproterenol.
B. Ibopamina.
C. Dobutamina.
D. Dopamina.
E. Efedrina.

33. Paciente de 68 años con disnea progresiva hasta en reposo. Examen físico: tórax amplio vibraciones vocales dismi-
nuidas, hipersonoridad, murmullo vesicular disminuido, ingurgitación yugular y disnea severa. Cuál es la actitud
inmediata?
A. Tomar radiografía de tórax
B. Evaluación por cirujano de tórax
C. Drenaje torácico inmediato
D. Tomografía de pulmones
E. Antibióticos y corticoides

34. Marque lo falso respecto a los niveles de prevención:


A. La prevención primordial evita la causación primaria de la enfermedad.
B. La prevención primaria busca reducir la incidencia de enfermedad a través de pruebas de screening.
C. La prevención secundaria se sustenta en la utilidad de las pruebas de screening.
D. La prevención terciaria busca recuperar parte de la funcionabilidad previa.
E. La prevención cuaternaria evita el sobrediagnostico y sobretratamiento.

35. Con respecto a la convulsión febril, marque la alternativa incorrecta:


A. La convulsión febril más frecuente es la simple
B. Es el tipo de convulsión febril más frecuente en niños con una frecuencia de 3 -5% en menores de 5 años
C. El que los padres hayan tenido en la niñez no apoya el riesgo de que el niño lo tenga
D. Generalmente se resuelven solas
E. Existe manejo preventivo de convulsión febril

36. ¿Cuál es un principal criterio para endometritis?


A. Fiebre en el primer día pureperio
B. Dolor pélvico
C. Secreción vaginal maloliente
D. Sangrado vaginal
E. Fiebre a partir del segundo día puerperio

37. Paciente de 40 años de edad, que acude a la consulta por sangrado postcoital de 2 semanas de evolución al examen
pélvico:cérvix ulcerado hipertróficos, sangrante, poco móvil. Cuerpo uterino 6cm, anexos no palpables. Al tacto rec- tal:
útero duro con parametrios tomados en sus 2/3 internos. El diagnóstico clínico es:
A. Cáncer de cérvix IB
B. Cáncer de cérvix IIIA
C. Cáncer in situ
D. Cáncer de cérvix IIB
E. Ectropión sangrante

38. Varón de 17 años, con procesos respiratorio viral, posteriormente presenta hematuria con hematíes dismorficos ade -
más presenta disminución de complemento (C3 Y c4) ¿Cuál es su posibilidad diagnostica?
A. Glomerulonefritis por vasculitis
B. Glomerulonefritis postinfecciosa
C. Glomerulonefritis membranoproliferativa
D. Glomerulonefritis rapidament progresiva
E. Nefropatía por IgA .

39. Señora que sufre caída y acude a consulta con miembro inferior en extensión y aducción, a la radiografía no se eviden- cia
fractura ¿Cuál es el diagnóstico más probable?
A. Luxación pubiana de la cadera
B. Luxación posterior de la cadera
C. Luxación congénita de cadera
D. Luxación obturatriz de cadera
E. Luxofractura de cadera

40. En este estudio de evalúan los efectos adversos a largo plazo y las interacciones medicamentosas en un gran número de
pacientes / es un estudio previa a la comercialización del fármaco pero que ya recibió la aprobación de la FDA para realizar
ensayos clínicos controlados:
A. Fase III / II
B. Fase V / II
C. Fase IV / III
D. Fase IV / II
E. Fase I / IV

41. Llega a consulta una madre con su hijo de 18 meses de edad quien refiere que solo recibió la primera dosis de la Pen-
tavalente. Con respecto a esta vacuna, ¿Qué le diría usted?
A. Ya no puede vacunarse por el riesgo de ESAVI
B. Debería completar con solo una segunda dosis
C. Completar con dos dosis más separadas por 1 mes
D. Debería reiniciar las tres dosis y darlos cada 2 meses
E. Completar con dos dosis más separadas por 2 meses.

42. En un paciente con glucosa en ayunas de 130 mg/dL y dos exámenes adicionales dudosos, con un test de tolerancia a la
glucosa en limite, debemos solicitar:
A. Curva de insulina
B. Glucosa post prandial
C. Repetir el test de tolerancia a la glucosa 75
D. Hemoglobina A1c
E. Electroforesis de hemoglobina.

43. Respecto a los aneurismas cerebrales, es INCORRECTO:


A. La mayoría corresponden al tipo sacular
B. Es la principal causa de hemorragia subaracnoidea atraumática
C. La principal arteria donde se les puede encontrar es la arteria meníngea anterior
D. La mayoría son de tipo congénito
E. La diplopía y la cefalea son síntomas característicos

44. Una madre con tipo de sangre AB tiene un hijo con tipo AB. A ella le gustaría establecer la paternidad a través del tipo
sanguíneo. ¿Cuál tipo de sangre excluye a un varón de ser el padre biológico?

A. AA.
B. BB.
C. BO.
D. AO.
E. OO.

45. El tipo de medicamento indicado para el tratamiento de la Angina de Prizmetal es:


A. Calcioantagonistas
B. IECA
C. B- Bloqueantes
D. ARA II
E. Digoxina
46. Paciente mujer quien se realiza una colecistectomía laparoscópica electiva por antecedente de litiasis vesicular, acci-
dentalmente se lacera el duodeno, sin perforarlo. El tipo de herida es:
A. Sucia
B. Limpia contaminada
C. Limpia
D. Contaminada
E. Infectada

47. Mujer de 28 años, hace 6 meses presenta malestar general, Poliartritis de mano y disnea. Al examen físico: PA: 125/80
mmHg, FC: 100 x minuto, FR: 28 x minuto. Alopecia, palidez y adenopatías cervicales múltiples. Hemoglobina 7G/ dl, urea
65 mg/dl, creatinina 2.2 mg/dl. Examen de orina: hematuria, cilindros hemáticos y proteínas (+), ANA y anti- Smith (+). Rx
de pulmones: lesión homogénea en el tercio inferior del hemitórax izquierda: ¿Con respecto al diagnós- tico señale lo
correcto?:
A. Mediado por una reacción de hipersensibilidad tipo II
B. El rash malar es una manifestación aguda específica
C. Asocia frecuentemente con síndrome de Sjogren
D. Mejora con la gestación
E. Cuando presenta anticuerpo anticardiolipina es buen pronóstico

48. Mujer de 25 años de edad, con gestación de 37 semanas. Acude a Emergencia por presentar dolor abdominal soste - nido,
de inico brusco y de intensidad fuerte, con leve sangrado vaginal. Al examen físico: PA 130/90 mmHg, LCF 160 por minuto.
¿Cuál es el diagnóstico más probable?
A. Placenta marginal
B. Ruptura uterina espontánea
C. Ruptura del seno marginal
D. Desprendimiento prematuro de placenta
E. Parto pretérmino

49. El tipo de sangrado más característico que producen los leimiomas uterinos:
A. Hipermenorrea
B. Metrorragia
C. Menorragia
D. Menometrorragia
E. Amenorrea

50. ¿Cuál de las siguientes vacunas es de tipo inmunización activa tipo toxoide?
A. Disteferia y Tetanos
B. Sarampion, Paperas y Rubeola
C. Pentavalente
D. Antineumococica
E. BCG.

51. Respecto a las escalas de medicina marque lo falso:


A. Las variables en la escala nominal son excluyentes.
B. La escala ordinal presenta una diferencia de grado entre variables.
C. La escala de intervalo muestra diferencias cuantificables de los valores pero no establece ausencia de fenómeno.
D. La escala de razón muestra diferencias cuantificables de los valores y establece ausencia de fenómeno.
E. La escala de intervalo muestra diferencias cuantificables de los valores y establece ausencia de fenómeno.

52. En un puesto de salud se han producido 2.800 consultas médicas al año y se dispuso para el mismo 400 horas-médico.
¿Cuál es el rendimiento?
A. 9
B. 5
C. 6
D. 8
E. 7

53. ¿Cuál es la característica que corresponde al sarcoma de Kaposi?


A. Es una neoplasia maligna de endotelio causada por HVS8
B. Las lesiones son dolorosas y pruriginosas causadas por HVS8
C. Es un cáncer de endotelio por HVS1
D. Es típico de VIH causado por HVS5
E. La etología es el VIH que causa cáncer de endotelio

54. ¿Cuál es la alternativa que NO corresponde a la anemia ferropenia?


A. La respuesta al tratamiento se mide con reticulocitos al 10 día
B. La causa más frecuente es carencial en jóvenes
C. Para diferenciarla de la anemia por enfermedad crónica se mide transferrina
D. El test bioquímico mas especifico es la ferritina
E. El hierro se absorbe solo el 10% de lo ingerido a nivel intestinal.

55. En el tratamiento de la hiperkalemia ¿Qué medicamento actúa produciendo disminución del potasio corporal total?

A. Gluconato de calcio
B. bicarbonato
C. Dextrosa hipertónica
D. Beta bloqueadores
E. Resina Kayexalate

56. Gestante de 38 años, con diagnóstico de placenta previa. ¿Cuál es el factor de riesgo más asociado y a partir de qué
edad gestacional se diagnostica dicha patología?
A. Cesárea – 28 semanas
B. Parto distocico – 26 semanas
C. Cesárea – 30 semanas
D. Parto distocico – 28 semanas
E. Cesárea – 22 semanas
57. Un neonato presenta ictericia, dificultad respiratoria, Tº inestable, distensión abdominal y hepatomegalia. Diagnósti- co
de presunción:
A. Kernicterus.
B. Meningitis neonatal.
C. Enfermedad granulomatosa crónica.
D. Lupus neonatorum.
E. Sepsis neonatal.

58. ¿Cómo se define talla baja en pediatría?


A. P/T<3p
B. T/E<5p
C. P/T<5p
D. T/E<3p
E. P/E<10p

59. Respecto a los tumores de esófago, marque lo INCORRECTO


A. Los tumores benignos más frecuentes son los leiomiomas
B. La acalasia crónica aumenta el riesgo
C. El consumo de alcohol y tabaco aumenta el riesgo
D. El tumor maligno más frecuente es el adenocarcinoma
E. El cáncer más frecuente es el carcinoma epidermoide.

60. Paciente con ECV con hemiparesia derecha a predominio braquial y afasia de expresión ¿qué arteria est á afectada?
A. Cerebral media rama superior izquierda
B. Cerebral media rama superior derecha
C. Cerebral anterior rama superior izquierda
D. Cerebral anterior rama superior derecha
E. Cerebral media rama inferior izquierda

61. Mide la desviación estándar de una media de todas las muestras posibles de una población infinita y se usa para la
inferencia por:
A. Error estándar de la media – intervalos de confianza
B. Desviación medial de la media – intervalos de confianza
C. Error estándar de la media – prueba de hipotesis
D. Amplitud intercalar - prueba de hipotesis
E. Correlacion lineal – intervalos de confianza

62. El impétigo ampolloso es causado por y debe ser tratado con:


A. Staphylococcus aureus coagulasa positivo / mupirocina
B. Streptococcus pneumoniae / dicloxacilina VO
C. St. Pyogenes / clindamicina VO
D. Staphylococcus aureus coagulasa positivo / dicloxacilina VO
E. Pseudomonas aeruginosa / aztreonam VO

63. Paciente de 30 años de edad, con hemorragia uterina normal y biopsia endometrial informada como hiperplasia sim-
ple típica. ¿Cuál es la conducta más apropiada a seguir?:

A. Tratamiento cíclico con progesterona


B. Histeroscopia
C. Histerectomía abdominal total
D. Esperar un nuevo control
E. Ablación endometrial

64. En un escolar de 7 años con claudicación de la marcha y dolor en la cadera, usted sospecharía en la enfermedad de:
A. Sever
B. Osgood-Schlatter
C. Khole
D. Legg-Calve-Perther
E. Panner

65. La elaboración del Plan Operativo de un establecimiento de Salud tiene como una de sus características que:
A. La misión expresa lo que seremos en el futuro
B. Promueve la participación del equipo de gestión solamente
C. Se programan actividades a lo largo de un año
D. Las fortalezas parten del entorno
E. Busca plantear problemas de la oferta para resolverlos operativamente.

66. El antidepresivo con menor efecto sedante y el que tiene mayor efecto sedante son respectivamente:
A. Amitriptilina / sertralina
B. Clomipramina / imipramina
C. Doxepina / bupropion
D. Fluoxetina / paroxetina
E. Trazodona / sertralina

67. ¿Cuál es la frecuencia cardiaca fetal normal?


A. 60
B. 120
C. 120 a 160

D. 80 a 120

E. >180

68. Varón de 40 años que presenta en el brazo derecho lesión máculo-papulosa de 3 cm, pruriginosa, indolora que progre- sa
a vesícula y luego a costra negruzca rodeada de edema. Ocupación crianza de ganado lanar y vacuno. Niega fiebre.
Examen: Pulso: 115 x minuto, FR: 24 x minuto, PA: 100/60 mmHg. ¿Cuál es la presunción diagnóstica y el manejo
adecuado?
A. Carbunco / doxiciclina
B. Leishmaniasis / nifurtimox
C. Lactrodactismo / suero especifico
D. Leishmaniasis / antimonio pentavalente
E. Carbunco / ciprofloxacino

69. La insuficiencia adrenal primaria se denomina y es causada:


A. Síndrome de Cushing / falla de regulación central
B. Enfermedad de Addison / falla de la glándula
C. Síndrome de Addison / falla de glándula
D. Enfermedad de Addison / falla de regulación central
E. Síndrome de Addison / falla de regulación central.

70. El test de Apgar valora el estado vital del neonato mediante la comprobación de los siguientes signos, EXCEPTO:
A. Frecuencia cardiaca.
B. Movimientos respiratorios.
C. Respuesta a estímulos.
D. Reactividad osteotendinosa.
E. Coloración de la piel.

71. Respecto a las hernias inguinales marque lo incorrecto


A. Las hernias inguinales directas salen mediales a los vasos epigástricos inferiores
B. Para el tratamiento de las hernias inguinales directas se prefieren las técnicas sin tensión
C. Las hernias indirectas ocurren por persistencia del conducto peritoneo vaginal
D. Las hernias directas son generalmente congénitas
E. Las hernias indirectas recorren el canal inguinal

72. Qué diámetro es el de mayor medida en el feto:


A. Suboccipitobregmatico
B. Suboccipitomentoniano
C. Occipitofrontal
D. Bitrocantereo
E. Bi acromial

73. Agente etiológico del Chancroide y de la cervicitis respectivamente:


A. Haemophilus vaginalis / clamidia trachomatis
B. Calymmabacterium Granulomatosis / gonococo
C. Treponema pallidum / H. ducreyi
D. Chlamydia trachomatis / H. ducreyi
E. Hemophilus ducrey / C. trachomatis

74. El síndrome de Cushing se refiere a:


A. Excesiva producción adrenal de glucocorticoides
B. Al exceso de glucocorticoides producidos
C. Excesiva exposición multiorganica a glucocorticoides
D. Excesiva estimulación de las adrenales por la ACTH
E. Excesiva actividad glucocorticoide

75. El motivo por el cual la sulfonilureas deben ser usadas con cuidado en ancianos y enfermos renales es:
A. Pueden causar acidosis láctica
B. Hipoglicemia
C. Hiperkalemia
D. Agitación
E. Daño hepático

76. Se recibe en Emergencia a un niño de 5 años de edad, febril, con 15 minutos continuos de crisis convulsiva, no ha recu-
perado la conciencia aun y continua convulsionando, la madre del niño describe cuadro compatible con convulsione s
tónico-clónicas generalizadas. ¿Cuál es el diagnóstico más probable y el manejo inicial más adecuado?
A. Estado convulsivo / Diazepam o lorazepam
B. Epilepsia / Diazepam o lorazepam
C. Convulsión febril / fenitoina
D. Estado convulsivo / fenitoina
E. Convulsión tónico –clónica / fenobarbital

77. ¿Cuál es el parámetro más exacto para determinar la edad gestacional?


A. Ecografía previa en el primer trimestre : LF
B. Ecografía previa en el primer trimestre: DBP
C. Fecha de inicio de náuseas y vómitos
D. Ecografía previa en el primer trimestre: LCC
E. FUR confiable

78. Paciente con pancreatitis aguda severa se colicita TAC con contraste donde se informa edema de páncreas con una
colección de líquido por lo cual según el índice de Baltazar corresponde a:
A. A
B. B
C. C
D. D
E. E

79. ¿Clínica más frecuente de esclerodermia?


A. Fenómeno de Raynaud
B. Atrofia de piel
C. Cianosis
D. Diarrea
E. Microstomia

80. Paciente varón de 75 años, con dolor abdominal intenso desde hace 8 horas, distensión abdominal marcada y vómitos
biliosos. En la radiografía se evidencia distensión de asa intestinal. El diagnóstico más probable es:
A. Hernia inguinal incarcerada
B. Vólvulo de ciego
C. Diverticulitis aguda
D. Vólvulo de sigmoides
E. Malrotación intestina

81. El modelo de atención de salud tiene como principio fundamental y la estrategia principal es la :
A. Calidad / APS
B. Integridad / APS
C. Eficacia / MAIS
D. Universalidad / MAIS
E. Solidaridad / APS

82. De las siguientes alternativas, la conducta de tratamiento más adecuado para el trauma de uretra posterior es:
A. Colocación de sonda Foley uretral
B. Talla vesical
C. Reparación primaria de la uretra
D. Ureterostomía
E. Nefrostomía
83. Se desea estudiar un factor de riesgo asociado a la prostatitis, se nota que la población presenta varios factores con-
fusores pero no pueden ser considerados dentro de los criterios de exclusión y deben muestrear a todos. Para obtener
una muestra representativa se debe optar por el muestreo:
A. Estratificado
B. Aleatorio simple con restitución
C. Aleatorio simple sin restitución
D. Sistemático
E. Por conglomerado

84. Paciente usuaria de DIU, asintomática, con ecografía que reporta gestación intrautero de 10 semanas, Cola de DIU visible
al examen con especulo. La conducta a seguir es:
A. Control del embarazo más antibiótico
B. Control del embarazo más progesterona
C. Retiro de DIU más antibióticos
D. Retiro de DIU más progesterona
E. Retiro de DIU y observación

85. ¿A qué edad aproximadamente empieza a salir la primera dentición?

A. 4 meses
B. 5 meses
C. 6 meses
D. 7 meses
E. 8 meses

86. Un prematuro de 32 semanas de edad; a la primera hora de vida presenta una disnea progresiva con cianosis y tiraje. En
la radiografía hay un patrón de vidrio esmerilado y broncograma aéreo. A pesar de la ventilación, el oxígeno y los
antibióticos, continúa mal. ¿Qué terapéutica añadiría en primer término?

A. Surfactante endotraqueal.
B. Indometacina oral.
C. Prednisona intravenosa.
D. Bicarbonato intravenoso.
E. Glucosa intravenosa.

87. Durante la noche en una guardia de pediatría llega a consulta un paciente varón de 20 meses quien presenta aguda-
mente un cuadro de dolor de oído leve a moderado, mama refiere que días antes presento un cuadro de resfrió y que
ahora presenta esta molestia. Al examen presenta una T 38.5°C, orofaringe congestiva sin exudado, oído derecho con
tímpano eritematomoso, congestivo, abombado y nivel hidroaereo, pulmones sin alteraciones. Según su diagnóstico,
¿Qué le indicaría?
A. Amoxicilina
B. Cefuroxima
C. Ibuprofeno
D. Observación
E. Azitromicina.

88. Gestante de 38 semanas con PA: 160/110 y Albuminuria ( ++++) ¿Cuál es la alternativa terapéutica más apropiada?
A. Sulfato de Magnesio 6gr Ev en bolo y 2gr cada hora
B. Betametasona 12 mg EV c/12 horas
C. Misoprostol 200 ug via vaginal cada 6 horas
D. Sulfato de Magnesio 2gr IM en cada glúteo
E. Sulfato de Magnesio 1gr Ev en bolo y 1gr cada hora
89. Anciana llega a la consulta con dolor postraumático de hombro derecho, con limitación de movimiento. Examen clíni- co:
hematoma en cara interna del tercio superior del brazo. ¿Cuál es el diagnóstico más probable?
A. Luxación escapulohumeral
B. Fractura de clavícula
C. Luxación acromioclavicular
D. Fractura de escápula
E. Fractura del cuello del húmero

90. ¿Qué se desarrolla en la administración de un establecimiento nivel I, al relacionar las actividades de sus diversas
unidades para alcanzar las metas propuestas?
A. Negociación
B. Organización
C. Coordinación
D. Motivación
E. Planificación

91. Infante con hipoacusia cursa con prueba de Rinne negativo, además en la otoscopia se menciona membrana timpánica
con niveles hidroaéreos.
A. Otitis media aguda
B. Otitis media crónica
C. Otitis externa difusa
D. Otitis secretora
E. Otitis externa maligna.

92. Las mácula densa se encuentran en:


A. La arteria aferente
B. La arteria eferente
C. Túbulo contorneado distal
D. Asa de Henle
E. Capsula de Bowman

93. ¿Cuál es la contraindicación absoluta para el tratamiento hormonal en post menopausia?


A. Ginecorragia de origen desconocido.
B. Varices superficiales
C. Bochornos
D. Dislipidemia controlada
E. HTA controlada.

94. Cuál de las siguientes es un material de sutura absorbible:


A. Nylon
B. Polidioxanona
C. Polipropileno
D. Seda
E. Acero quirúrgico

95. ¿Cuál es la profundidad a la cual se debe comprimir el torax en un lactante en el RCP?


A. 3cm
B. 4cm
C. 5cm
D. 6cm
E. 7cm.

96. Bebé de 30 horas de edad, nació con 4300 g, hijo de madre con diabetes gestacional, ha tomado bien su alimento, pero se
ve muy inquieto a los 30 minutos de vida. 15 minutos más tarde sufre una convulsión tónico-clónica. Diagnóstico más
probable:
A. Hipoglucemia
B. Hipocalcemia
C. Hipomagnesemia
D. Hiponatremia
E. Hiperviscosidad

97. El grado de afectación fetal en la isoinmunización Rh se aprecia con mayor exactitud por medio de:
A. Test de Coombs indirecto.
B. Test de Coombs directo.
C. Niveles de b-HCG en líquido amniótico.
D. Medición de los niveles de bilirrubina en el líquido amniótico-curvo de Liley.
E. Test de Coombs y b-HCG.

98. La porción mayor del estómago está constituida por:


A. La unión gastroesofágica.
B. El cardias.
C. El fondo.
D. El cuerpo.
E. El antro.

99. Una niña de 9 años de edad tiene una historia de sibilancias intermitentes de varios años; no ha recibido medicamen- tos
hace algún tiempo. Se halla febril, agitada y con cianosis peribucal. Tirajes intercostales y supraesternales; los ruidos
respiratorios están disminuidos y hay sibilancias bilaterales. La intervención inicial más apropiada es:
A. Administración de aminofilina intravenosa.
B. Solicitar radiografía torácica.
C. Prescribir la nebulización de cromoglicato sódico.
D. Pedir biometría hemática completa y hemocultivo.
E. Prescribir salbutamol nebulizado.

100. La trompa de Eustaquio y la mucosa del oído medio se originan en:


A. La cuarta bolsa faríngea
B. El neuroectodermo notocordal
C. El primer arco branquial
D. La cuerda del tímpano
E. La primera bolsa faríngea
101. Son agentes patógenos de neumonía adquirida en la comunidad los siguientes, excepto:
A. Streptococcus pneumoniae
B. Acinetobacter baumanii
C. Mycoplasma pneumoniae
D. Hemophilus influenzae
E. Legionella pneumophila

102. ¿Cuál de las siguientes no es característica de convulsion febril simple?


A. Duración menor de 15 minutos
B. 2 o más en 24 horas
C. No dejar déficit motor
D. Convulsion Tonicoclonico generalizado
E. Convulsion asociado a fiebre
103. Mujer de 30 años, acude por presentar Poliartrálgias y pápulas eritematosas aplanadas sobre los nudillos de ambas
manos (signo de Gottron) y telangiectasias periungüeales. ¿Cuál es lo correcto con respecto al diagnóstico?:
A. Existe dolor muscular
B. Asocia con esclerodermia
C. Para el diagnostico se usa biopsia muscular
D. Presencia del anti jo 1 indica buen pronostico
E. Asocia con panarteritis nodosa

104. Es la relación que existe entre los resultados con respecto a los recursos
A. Eficiencia
B. Efectividad
C. Eficacia
D. Costo – Beneficio
E. Aceptación.

105. Mujer de 20 años, con cesárea por inducción fallida. Al tercer día de puerperio presenta malestar general, fiebre y loquios
fétidos. Al Examen clínico: T: 39°C, útero subinvolucionado y doloroso a la palpación, secreción fétida. ¿Cuál es el
diagnóstico más probable?
A. Dehiscencia de la histerorrafia
B. Endometritis puerperal
C. Absceso de pared abdominal
D. Retención de restos placentarios
E. Necrosis isquémica de mioma uterino

106. ¿Cuáles son los tumores primarios que más frecuentemente producen metástasis cerebrales y cuáles son los tumores
primarios más frecuentes en adultos?
A. Cáncer de cuello uterino y de próstata // astrocitoma grado II
B. Linfoma y carcinoma gástrico / glioblastoma multiforme
C. Cáncer de pulmón y de mama // astrocitoma grado IV
D. Cáncer gástrico y de páncreas // meduloblastoma
E. Cáncer de pulmón y de mama // metastasis

107. Primigesta añosa de 34 semanas de gestación no controlada, acude por presentar cefalea, tinnitus, dolor epigástrico.
Examen físico: PA: 150/90 mm Hg, FC: 90 x ́, FR: 20 x ́, T: 36.5°C, edema en miembros inferiores. Altura uterina: 30 cm,
Feto LCI, FCF:140 x ́. Exámenes de Laboratorio: Hb:10 g/dL, plaquetas 110,000/mm3, Creatinina: 1 mg/dL, TGO: 70 UI/L,
LDH:1200 UI/L, Bilirrubina:1.4 mg/dL a predominio indirecto. ¿Cuál es el diagnóstico más probable?
A. Síndrome HELLP
B. Preeclampsia severa
C. Hipertensión gestacional
D. Hipertensión crónica
E. Eclampsia

108. ¿Cuáles son los agentes terapéuticos de uso frecuente en la epilepsia y convulsiones parciales y en las generalizadas
respectivamente?
A. Lamotrigina, fenobarbital // carbamazepina
B. Etosuximida, ácido valproico, lamotrigina // lamotrigina
C. Carbamazepina, fenilhidantoína, gabapentina // Ácido valproico, lamotrigina
D. Primidona, topiromato // Carbamazepina, Lamotrigina
E. Zonisanida, ácido valproico // Carbamazepina, Lamotrigina

109. RN con hepatoesplenomegalia, rinitis purulenta, anemia, rash maculopapular y descamación de Palmas y plantas. Cuál
es el diagnóstico más probable.
A. CMV
B. Sífilis congénita
C. Toxoplasmosis
D. Rubeola
E. Herpes
110. Con respecto a la HTA señale lo correcto
A. Principal factor de riesgo para hemorragia subaracnoidea
B. La hipertensión maligna genera necrosis fibrinoide
C. La hipertensión produce lesión de arterias de gran calibre
D. La estenosis arteriolar corresponde a la retinopatía grado II
E. Produce dilatación ventricular izquierda

111. Gestante de 38 semanas, acude a Emergencia por presentar dolor abdominal intenso hace 4 horas acompañado de
sangrado vaginal rojo vinoso. Examen físico: PA: 140/90 mmHg, LCF: 128 x’, AU: 37 cm, hipertonía uterina y se con- firma
sangrado de cavidad uterina ¿Cuál es el diagnóstico más probable?
A. Desprendimiento prematuro de placenta
B. Placenta previa
C. Ruptura uterina
D. Vasa previa
E. Inversión uterina

112. Si el cuadro clínico de asma bronquial cursa con sibilancias, tos, disnea menos de 2 veces por semana, exacerbacione s
breves (desde horas a dias), síntomas nocturnos mayores de una vez por semana, asintomática entre exacerbaciones.
¿Qué clasificación de gravedad le corresponde?
A. Intermitente
B. Persistente leve
C. Persistente moderado
D. Persistente grave
E. Intermitente moderado.

113. Acude a consulta un paciente de 6 meses de edad que en sus 3 primeros días de enfermedad presenta cuadro catarral,
luego aumenta su frecuencia respiratoria por lo que acude a emergencia donde se evidencia taquipnea, tirajes, sibi-
lantes. ¿Cuál es el agente etiológico más probable?
A. Adenovirus
B. Micoplasma
C. Penumococo
D. Virus sincitial respiratorio
E. Rinovirus

114. ¿Cuál es la alternativa que NO corresponde a la anemia ferropenia?


A. La respuesta al tratamiento se mide con reticulocitos al 10 dia
B. La causa más frecuente es carencial en jovenes
C. Para diferenciarla de la anemia por enfermedad crónica se mide transferrina
D. El test bioquimico mas especifico es la ferritina
E. El hierro se absorbe solo el 10% de lo ingerido a nivel intestinal.

115. Adolescente de 15 años, con 32 semanas de gestación , sin control prenatal, acude a emergencia por dolor epigástrico y
cefalea. Al examen impresiona como embarazo de 6 meses por altura uterina. PA: 160/110mmHg, Proteinuria (+++). El
diagnostico más probable es :

A. Eclampsia
B. Hipertension transitoria
C. Pre-eclampsia severa
D. Pre-eclampsia superpuesta
E. Sindrome de HELLP

116. Llega a emergencia un lactante de 3 meses quien 4 días antes de su ingreso desarrolla rinorrea y estornudos, ahora tiene
dificultad para respirar, taquipena y a la auscultación subcrepitos. Según su sospecha diagnostica, ¿Cuál es el manejo más
indicado?
A. Iniciar amoxicilina
B. Brindar terapia de rescate con beta-agonistas
C. Dar broncodilatadores más corticoide
D. Iniciar macrolidos
E. Nebulización con suero hipertónico.

117. Varón de 56 años, que presenta pérdida de peso de 6 kg en el último año, cambios de la voz, incremento del volumen de
la glándula tiroides a predominio derecho. No adenomegalias y pruebas de función tiroidea normales. En el preope-
ratorio, ¿qué examen complementario de tiroides solicitaría?
A. Aspiración con trocar
B. Gammagrafía
C. Ultrasonografía con biopsia
D. Resonancia magnética
E. Tomografía

118. Paciente proviene de Piura, con fiebre de menos de 5 días de duración, sin afección de las vías aéreas superiores con
cefalea y mioartalgias. ¿Cuál es el diagnóstico más probable?
A. A.Dengue sospechoso.
B. B.Probable dengue grave
C. C.Probable dengue hemorrágico
D. D.Dengue confirmado
E. E.Dengue descartado

119. Inmunoglobulina aumentada en purpura de Henoch Schonlein


A. Ig G
B. Ig E
C. Ig M
D. Ig D
E. Ig A

120. Paciente mujer de 20 años con palpitaciones de inicio y fin súbito, con PA normal, la primera medida terapuetica es:
A. Digoxina
B. Atenolol
C. Verapamilo
D. Adenosina
E. Maniobras vagales

121. Gestante de 8 semanas acude por sangrado vaginal profuso desde hace 3 horas, acompañado de dolor abdominal
intenso. Examen físico: PA: 80/60 mmHg, FC: 98 x’, FR: 18 x’. Especuloscopía: cérvix con sangrado y coágulos abun- dantes.
Tacto vaginal: orificio externo e interno abiertos. ¿Cuál es el diagnóstico más probable?
A. Aborto inevitable
B. Aborto frustro
C. Aborto inminente
D. Amenaza de aborto
E. Aborto diferido

122. Mencione lo que no corresponde al score de forrest para hemorragia digestiva alta
A. El tipo Ia tiene alta tasa de mortalidad
B. El tipo IIb es coagulo adherido
C. El tipo III es sangrado inactivo
D. El tipo Ib requiere tratamiento endoscópico más antiacidos
E. El tipo IIc es ulcera con fibrina

123. ¿En qué mecanismo del trabajo de parto se produce cambio de la variedad de posicion?
A. Expulsión
B. Rotación interna
C. Descenso
D. Extensión
E. Flexion

124. Paciente programado para cirugía de apendicitis congestiva, en sala se produce mínima lesión del colon. Como lo
clasificaría.
A. Limpia
B. Sucia
C. Limpia contaminada
D. Sucia contaminada
E. Contaminada

125. El diagnóstico bioquímico del empiema pleural se realiza sobre la base de los siguientes criterios, excepto:
A. Triglicéridos mayor de 100
B. Glicemia menor de 40 mg%
C. pH menor de 7.2
D. proteínas mayor de 3g%
E. Gran celularidad a predominio de polimorfonucleares

126. Señale lo correcto respecto a la menopausia:


A. Los niveles de FSH se elevan antes que los trastornos del ciclo menstrual.
B. El estradiol es la hormona predominante.
C. La progesterona se eleva tras la menopausia.
D. El climeterio es más intenso en los primero 10 años anteriores al diagnóstico.
E. La terapia de reemplazo hormonal es lo más importante en todas las menopausicas.

127. Lactante de 8 días de nacido con distensión abdominal y vómitos biliosos abundantes, signo de la doble burbuja.
A. Páncreas tubular
B. Estenosis hipertrófica de piloro
C. Vólvulo de ciego
D. Atresia duodenal
E. Bridas

128. ¿Con que prueba se realiza el diagnostico de asma bronquial en un niño mayor de 5 años?
A. Espirometria simple
B. Flujometro
C. Espirometria forzada
D. Pletismografia
E. Radiografia de torax

129. Paciente con IMA que presenta crepitantes en ambos campos pulmonares signos de ICC severa se clasifica como
A. Killip I
B. Killip II
C. Killip III
D. Killip IV
E. Forrester III

130. Paciente con disfagia motora más pérdida de peso, ante la sospecha diagnostica quien confirma la etiología
A. Ph metria de 14 hrs
B. Endoscopia alta
C. Impedanciometria
D. Manometría esofagica
E. Test de secretina

131. En relación con la etiología del RCIU II, señale factor más frecuente
A. Enfermedad hipertensiva
B. Rubéola
C. Alcoholismo
D. Drogadicción
E. Tabaquismo

132. ¿Cuál es el vector del ciclo de transmisión selvático de la fiebre amarilla en la América del sur?
A. Mosquitos de genero Haemagogus
B. Mosquitos de genero Aedes
C. Mosquitos de genero Crotalus
D. Mosquitos de genero Phlebotomus
E. Mosquitos de genero Vivax

133. ¿Cuál de las siguientes entidades causa derrame pleural tipo trasudado?
A. Neumonía bacteriana
B. TBC pleural
C. Cirrosis hepática
D. Metástasis pleural
E. Mesotelioma .

134. En el Perú actual, el cuidado de la salud está en los primeros lugares de la agenda política del Acuerdo Nacional, al mismo
tiempo el Ministerio de Salud sólo ha podido desarrollar precariamente su rol rector y conductor sensorial. La combinación
de estos dos factores en una matriz FODA permite identificar la siguiente estrategia:
A. Desafío
B. Limitación
C. Potencialidad
D. Restricción
E. Riesgo.

135. Paciente varon joven con datos de trombosis venosa profunda, además se encuentra ictericia a predominio indirecto
con coluria, la sospecha diagnostica es:
A. Hemoglobinuria paroxística nocturna
B. Esferocitosis
C. Talasemias
D. Hemoglobinopatías
E. Eliptocitosis

136. ¿Cuál es el diagnostico nutricional de un lactante de 11 meses que tiene P/T 94%, T/E 94%?
A. Desnutrición aguda
B. Desnutrición crónica
C. Desnutrición crónica reagudizada
D. Desnutrición global
E. Eutrófico

137. El Caso índice o caso cero Corresponde a:


A. Primer caso ocurrido en una epidemia.
B. Primer caso reportado al sistema de salud.
C. Primer caso muerto.
D. Primer caso tratado.
E. Primer caso publicado.
138. Paciente de 15 años, con náuseas, vómitos, boca seca, y somnolencia, PA 60/40 IMC 17KG/M2 , glicemia 450mg/dl; pH
7,15; HCO3 12mEq/kg , la terapia indicada comprende fluidos más:
A. Insulina cristalina en bolos
B. Insulina cristalina por vía SC
C. Insulina cristalina por infusión
D. Insulina cristalina por vía IM
E. Insulina intermedia por vía SC

139. Respecto a los ACO señale lo falso:


A. Si se dan gestàgenos solos el efecto anticonceptivo disminuye.
B. Si se dan progestágenos solos puede ocurrir atrofia endometrial.
C. El principal efecto anticonceptivo se obtiene a nivel central hipotalámico.
D. Los progestágenos de tercera generación tienen buen perfil lipídico.
E. Los gestagenos en depósito son de elección en adolescentes y nulíparas.

140. Con respecto a la angina inestable señale lo verdadero


A. La etiología más frecuente es la placa de ateroma con capa fibrosa gruesa
B. El factor de riesgo más importante es la hipertensión arterial
C. Las mujeres tienen más riesgo en comparación con varones
D. Una variante clínica es la angina por vasoespasmo coronario
E. Presenta elevación de CPK MB

141. Niño con alopecia, lesión en piel (acrodermatitis), diarrea, deficiencia de que vitamina:
A. Zinc
B. Niacina
C. Tiamina
D. Magnesio
E. Hierro

142. Paciente varón de 25 años, politraumatizado en accidente de tránsito. Al examen neurológico: apertura ocular al do- lor,
emite sonidos incomprensibles y extiende el miembro superior al dolor. De acuerdo a la escala de Glasgow, cuál sería la
conducta inmediata a seguir:
A. Observación
B. Intubación orotraqueal
C. Craneotomía
D. Traqueotomía
E. TAC

143. Mujer de 40 años de edad, presenta predominantemente en la región nasogeniana legiones papulosas eritomatosas y
descamaticas, las cuales se exacerban con alguna comida y tensión emocional. el diagnóstico más probable es y el manejo
adecuado debería ser:
A. Rosácea / minociclina
B. Acné / retinoides
C. Rosácea / metronidazol topico
D. Acné / metronidazol oral
E. Psoriasis / retinoides o metronidazol

144. La nueva estructura de etapas del ciclo de vida según el modelo de atención integral de salud incluye a la etapa joven.
¿Cuál es el rango de edad que le corresponde?
A. 18a29años
B. 19a29años
C. 20 a 29 años
D. 19a30años
E. 20a30años

145. Adulto joven en cuyo perfil hepático solo se encuetra alterado la bilirrubina que esta aumentado ligeramente a predo-
minio indirecto, la sospecha diagnostica es
A. Síndrome de rotor
B. Síndrome de dubin jonson
C. Síndrome de Gilbert
D. Sindrome de clijer najar tipo I
E. Sindrome de clijer najar tipo II

146. Varón de 45años de edad, que presenta desde hace 12 meses de manera progresiva dificultad para la marcha. Al exa- men
se evidencia hipomimia facial, lentitud para iniciar la marcha, dificultad para atar los cordones de sus zapatos, temblor
fino en miembro superior derecho y resistencia a la flexo-extensión pasiva de dicho miembro. La sospecha diagnóstica
es ------ y el manejo adecuado es --------:
A. Alzheimer / memantina
B. Huntington / levodopa y carbidopa
C. Parkinsonismo / levodopa y memantina
D. Enfermedad de Parkinson / levodopa y carbidopa
E. Wilson / EDTA.

147. Acude a emergencia un paciente con estridor que días antes presento clínica de rinorrea, estornudos y sensación de
alza térmica, ahora presenta dicho estridor y tos perruna. Al evaluarlo se le clasifica como leve. ¿Cuál es su
manejo?

A. Antipiréticos
B. Nebulización con adrenalina
C. Nebulización con suero hipertónico
D. Dexametasona endovenoso
E. Dexametasona vía oral

148. Fractura conminuta no desplazada del primer MTC:


A. Benett
B. Rolando
C. Jones
D. Murphy
E. Mallampati

149. En la pubarquia, de acuerdo con la clasificación de Tanner, si se encuentra vello pubiano que extiende solo en el trián- gulo
pubiano, corresponde al estadio:
A. VP1
B. VP2
C. VP3
D. VP4
E. VP5

150. Paciente varón de 18 años, quien ingresa a emergencia con dolor abdominal desde hace 7 horas, localizado en fosa iliaca
derecha y náuseas, afebril. Dolor en punto de Mc Burney positivo. Leucocitosis y PCR levemente aumentado. De acuerdo
al diagnóstico más probable, señale lo correcto:
A. La etiología es básicamente la perforación de la víscera
B. El germen que se puede identificar con más probabilidad es el Bacteroides fragilis
C. El tratamiento antibiótico requiere Amikacina + Metronidazol por una semana
D. La causa más frecuente es la hipertrofia de las placas de Peyer.
E. El germen más frecuente es E. coli

151. El establecimiento de sector salud de categoría I-2 corresponde a:


A. Puesto de salud con médico
B. Puesto de salud sin médico
C. Puesto de salud con 10 médicos
D. Puesto de salud con hospitalizacion
E. Puesto de salud con quirófano.

152. ¿A qué edad aproximadamente un niño duplica su talla de nacimiento?


A. 1 año
B. 2 años
C. 3 años
D. 4 años
E. 5 años

153. Paciente con epistaxis posterior mencione lo verdadero


A. Lesión del plexo de kiesellbach
B. Fácil de controlar
C. Resuelve con taponamiento anterior
D. La arteria esfenopalatina es rama de la maxilar interna
E. La causa más frecuente es infecciones.

154. Paciente de 40 años de edad que presenta necesidad imperiosa de miccionar y se le escapa la orina antes de llegar al
baño, ya que no puede retenerla. Al examen se constata uretrocistocele de primer grado. El diagnóstico es:
A. Incontinencia urinaria de esfuerzo
B. Incontinencia urinaria de urgencia
C. Infección de vías urinarias
D. Cistitis intersticial
E. Cistitis aguda.

155. La denominación sinclitismo y asinclitismo se refiere a la relación entre:

A. La sutura interparietal y las tuberosidades izquierdas.


B. La sutura metódica y el promontorio.
C. La sutura occipital y el promontorio.
D. La sutura sagital con el promontorio y el pubis.
E. La sutura sagital y las espinas ciáticas.

156. Multigesta de 10 sem con sangrado vaginal y dolor abdominal, altura uterina 18 cm orificio cervical entreabierto,.
Además presenta hemoptisis ¿Cuál es el diagnóstico?
A. Mola parcial
B. Mola completa
C. Aborto molar
D. Degeneración hidrópica
E. Coriocarcinoma

157. En el embarazo normal, la altura del fondo uterino se vuelve extrapelvico durante la semana …………………y se considera
un síntoma…………..de diagnóstico de embarazo
A. 12 - probable
B. 16 - presuntivo
C. 18 - certeza
D. 10 - presuntivo
E. 20 – certeza

158. ¿Cuál es la neoplasia más frecuente en pediatría?


A. Leucemias
B. Linfomas
C. Tumor de willms
D. Astrocitoma
E. Meduloblastoma.

159. En la prueba de hipótesis, el investigador comete un error tipo II o beta cuando:


A. No establece el nivel de significancia
B. No rechaza la hipótesis nula siendo falsa
C. Rechaza la hipótesis alterna, siendo falsa
D. Rechaza la hipótesis alterna, siendo verdadera
E. Rechaza la hipótesis nula siendo verdadera.

160. ¿Cuál es la vacuna que no debe darse en alérgicos a la proteína del huevo?
A. BCG
B. Antineumococica
C. Antiamarilica
D. VPH
E. Rotavirus

161. Varón de 50 años de edad, que ingresa al Hospital por presentar un cuadro de disnea, palidez de piel y mucosas. Al
examen clínico: PA 150/90 mmHg. Se detecta anasarca y signos de derrame pleural derecho. Exámenes de laboratorio:
hemoglobina 9g/dL, riñones con pérdida de la diferenciación corticomedular ¿Cuál es el diagnóstico más probable?:
A. Síndrome nefrótico
B. Síndrome nefrítico
C. Glomerulonefritis aguda
D. Nefritis aguda
E. Insuficiencia renal crónica

162. ¿Cuál de los siguientes es el mejor parámetro para evaluar el crecimiento de un niño?
A. IMC en valores absolutos
B. Una sola visita en su vida
C. El seguimiento con las curvas de crecimiento
D. Con los datos del nacimiento
E. Evaluación psicológica

163. El índice ecográfico que mejor identifica el retardo del crecimiento fetal es…………., y para evaluar sus pronostico se
utiliza la ecografía doppler de la arteria
A. La longitud del fémur - uterina
B. La circunferencia abdominal - umbilical administrativo?
C. La longitud del fémur – cerebral media A. Planificación, organización, dirección y control.
D. La circunferencia abdominal – ductus venoso B. Planificación dirección, organización, y control
E. La longitud del fémur – cerebral media C. organización, dirección control Planificación
D. Planificación, control organización, dirección
164. Varón de 65 años, con T: 38°C, náuseas, vómitos
E. dirección Planificación, organización, y control.
y deposiciones líquidas con moco, sangre y
tenesmo. Estando hospi- talizado presenta una
170. RN con antecedente de DIPII y líquido amniótico
convulsión por primera vez. ¿Cuál es el agente meconial, nace hipotónico, apnea y
causal?
bradicardico. Qué medida to- maría.
A. Entamoeba histolítica
A. Aspiración de secreciones
B. Salmonella B. Estimulación táctil y aspiración de secreciones
C. Escherichia colie C. VPP
D. Clostridiun perfingens D. VPP con mascarilla
E. Shiguella biflexa. E. Ventilación mecánica

165. Es un complicación tardía de la traqueotomía


171. En varones adultos con anemia ferropénica ¿con
A. Estenosis laríngea que prueba monitorizamos la respuesta al
B. Hemorragia tratamiento?
C. Neumotórax
D. Atelectasia pulmonar A. Hemoglobina a los 20 dias
E. Enfisema subcutáneo B. HTO a los 15 dias
C. Reticulocitos a los 10 dias
166. Niño de 2 años de edad, que desde los 9 meses D. Reticulocitos a los 5 dias
presenta episodios de lesiones eritematosas E. Ferritina a los 30 dias
descamativas en mejillas, abdomen y parte
extensoras de miembros, con prurito intenso. 172. ¿Cuál es la característica del flujo vaginal en la
Actualmente aparecen lesiones similares Vaginosis bacteriana?
después de comer maní. Al examen se evidencia A. Seco y espumoso
pliegues redundantes infraorbitarios. ¿Qué tipo B. Grumoso y blanquecino
de dermatitis consideraría en su presunción C. Fétido y grisáceo
diagnóstica?: D. Espeso y blanquecino
A. Por ingesta de alimentos E. Espumosos y sanguinolento.
B. Seborreica
C. Alérgica de contacto 173. ¿Cuál es el principal mecanismo de la ovulación a
D. Eccematoide infecciosa nivel hormonal
E. Atópica A. El pico de LH aislado.
B. El pico de FSH aislado.
C. El pico de Estradiol y FSH.
167. En el tratamiento de la hiponatremia en un D. El pico de Estradiol y LH.
paciente con insuficiencia renal aguda oligúrica, E. El pico de Progesterona aislado.
la medida terapéutica
más apropiada es: 174. Recién nacido con fibrosis quística, cual es la
A. Restricción de agua libre manifestación más relacionada:
B. Restricción de sodio A. Ileo meconial
C. Administración de sodio B. Policitemia
D. Uso de diuréticos de asa C. Hipoglicemia
E. Uso de resina de intercambio iónico D. Dermatitis crónica
E. Neumonía
168. Señale que cambios ocurren en el embarazo
normal 175. ¿Qué porcentaje del área bajo la curva normal
A. Disminución del tamaño renal estándar se localiza entre +2 desviaciones
B. Dilatación uretereal bilateral estándares de la media?
C. Disminución del índice de filtración glomerular A. 58,6
D. Aumento del índice de flujo plasmático renal
E. Pronunciada proteinuria B. 95,4

C. 68,3
169. ¿Cuál es la secuencia lógica en el proceso
D. 90,4 y
1
E. 99,9 1
E. 6 y 18
176. Las fracturas que se producen por
microtraumatismos, ya sea por intensidad o por
repetición que alteran las propie- 180. ¿Cuál de las siguientes medidas de tendencia
dades fisicoquímicas del hueso, se denominan central es la más usada para el análisis
y la más frecuente es la fractura : estadístico si los datos son asi- metricos?
A. Por fatiga / del recluta
A. La mediana
B. Patológicas / colles B. La media
C. Por fatiga / cargador
C. La moda
D. Osteoporoticas / colles
D. La desviación estándar
E. Patologicas / metatarsiano 2
E. La varianza

177. Marque lo falso respecto a las siguientes 181. Según la clasificación de Bormann para cáncer
patologías neurológicas:
gástrico, las lesiones ulceradas infiltrantes
A. La demencia más frecuente es la demencia de
corresponden al tipo:
Alzheimer.
A. 5
B. La demencia de Alzheimer es B. 4
predominantemente subcortical. C. 3
C. El nivel de lesión mesencefalico se evalúa con
D. 2
la rigidez de decorticacion o descerebración.
E. 1
D. El tumor más frecuente en el SNC es
metastasico de pulmón.
182. Varón de 23 años de edad: refiere fácil sangrado
E. El tumor pilocitico infantil es frecuente en
de las encías al uso del cepillo dental. Al examen;
cerebelo.
palidez y mucosas, ausencia de gingivitis; punta
de bazo palpable. Hematocrito 31%, leucocitos
178. Tratamiento para crisis de agitación psicomotriz:
2.300/dL, plaquetas 100.000/dL. Frotis de sangre
A. Haloperidol
periférica: cuerpos de Auer. El diagnóstico
B. Benzodiazepinas
presuntivo es:
C. Fenitoina
A. Leucemia linfoide aguda
D. Carbamazepina
B. Leucemia mieloide aguda
E. Quetiapina
C. Leucemia mieloide crónica
D. Mielofibrosis
179. Que serotipo VPH está asociado a las verrugas
genitales E. Mononucleosis infecciosa
A. 6
y 183. Paciente de 78 años con te: 2 días, presenta
1 deposiciones líquidas 3 v/día, volumen de ½taza
1 por vez, s/s, con pa: 100/60 mmHg, p: 100x’, fr:
B 22x’; presión ocular disminuida. Además tiene
. Na: 120 mmol/l, k: 3,5 mmol/l, cl: 100 mmol/l.
1 respecto al estado hidroelectrolítico de la
6 paciente, podemos afirmar que presenta:
y A. Hiponatremia Hiperosmolar.
1 B. SIADH
8 C. Hiponatremia con hipovolemia
C D. Hiponatremia con normokalemia
. E. Hiponatremia con hipovolemia
2
4 184. Paciente diabético cursa con pérdida progresiva
y de agudeza visual, la etiología más probable seria:
2 A. Desprendiemiento de retina
8 B. Edema macular
D C. Hemovitreo
. D. Glaucoma de Angulo estrecho
1 E. Miopia maligna
5
185. Mujer de 22 años de edad, sexualmente activa, lesiones eritematosas pruriginosas
con dolor pélvico intenso y fiebre. Al examen: micropapulares localizada en dorso de
anexos dolorosos y antebrazos, cara y cuello. Las lesiones también se
leucorrea maloliente. Hemograma con 15.000 han identificado en algunas compañeras de
leucocitos x mm3. ¿Cuál es el diagnóstico y el trabajo. No presenta antecedentes de atopía. Su
manejo más apropiado? primera impresión diagnóstica es y el
A. Enfermedad pélvica inflamatoria + tratamiento es:
hospitalización + Ceftriaxona + metronidazol + A. Acarosis / permetrina
doxiciclina B. Dermatitis por contacto / corticoides
B. Enfermedad pélvica inflamatoria + C. Acarosis / griseofulvina
hospitalización + cefoxitin + metronidazol + D. Dermatofitosis / clotrimazol
azitromicina E. Dermatofitosis / itraconazol
C. Enfermedad pélvica inflamatoria + tratamiento
ambulatorio con ciprofloxacino, doxiciclina y 190. El antidepresivo con menor efecto sedante y el
metronidazol. que tiene mayor efecto sedante son
D. Quise ovárico complicado + tratamiento respectivamente:
quirúrgico A. Amitriptilina / sertralina
E. Enfermedad pélvica inflamatoria + tratamiento B. Clomipramina / imipramina
quirúrgico. C. Doxepina / bupropion
D. Fluoxetina / paroxetina
186. Mujer de 26 años, con fiebre de 39°C, de 4 E. Trazodona / sertralina
semanas de evolución que calma con
antipiréticos; fatiga intensa y malestar general. Al 191. Mujer con infección por chlamydia, que esquema
Examen físico: Linfoadenomopatías cervicales, de tratamiento indicaría.
submaxilares y supraclaviculares y A. Metronidazol 2gr dosis única
hepatoesplenomega- lia. Hemograma con 15% B. Fluconazol 150mg
de linfocitos atípicos y leve trombocitopenia. C. Doxiciclina 100 mg
Anticuerpos heterofilos positivos. ¿Cuál es el D. Ketoconazol
diagnóstico más probable? E. Ceftriaxona 250 mg
A. Dengue hemorragico
B. Mononucleosis infecciosa por CMV 192. Ubicación más frecuente del tumor de klastkin
C. Fiebre Chikungunya A. En conducto cístico
D. Mononucleosis infecciosa por VHB B. Colédoco proximal
E. Tuberculosis pulmonar C. Confluencia de hepáticos
D. Colédoco distal
E. Hepático derecho
187. Paciente varón de 45a con DM tipo I. Acude por
cefalea, somnolencia, náuseas y vómitos 193. Varón de 19 años que refiere aumento de
persistentes. Al examen físi- co, despierto, pálido volumen en el hemiescroto derecho
y mucosa oral seca; PA 140/70, FC: 100; FR: 28; transiluminacion positiva, deberíamos
extremidades inferiores con fóvea++ Hb 10 pH: descartar
7.24; Hto 44 K 2.9 Na 139; HCO3 15. Glucosa 280. A. Hernia inguinoescrotal.
¿Cuál es el Diagnostico? B. Hidrocele
A. Estado Hiperosmolar C. Quiste del epidídimo.
B. Cetoacidosis Diabética D. tumor testicular tipo seminoma
C. Pielonefritis crónica E. Tumor testicular tipo no seminoma
D. Cetoacidosis ayuno
E. Insuficiencia Renal Aguda 194. Beta bloqueador utilizado en glaucoma crónico

188. En la artritis bacteriana aguda de rodilla, la A. Metoprolol


conducta terapéutica inmediata es: B. Bisoprolol
A. Drenaje de líquido sinovial y AINES C. Nadolol
B. Antibióticos parenteral y drenaje D. Carvedilol
C. Antibióticos orales y AINES E. Timolol
D. Antibióticos intra-articulares
E. Irrigación intra-articular continua
195. ciente varón de 50 años, fumador, con
189. Mujer de 20 años de edad, peladora de Hipertensión arterial controlada, programado
espárragos desde hace 4 meses, presenta para cura quirúrgica de her- nia inguinal. Su
riesgo ASA es:
A. V
B. II
C. I
D. IV
E. III

196. Se cuenta con 13 valores de nivel de bilirrubinas


en pacientes con ictericia y se desconoce la
varianza poblacional para esta variable. En la
construcción del intervalo de confianza al 95%
para el valor promedio de bilirrubina en este
grupo de pacientes, la prueba estadística
requerida es:
A. ANOVA
B. Binomial
C. Chi-cuadrado
D. Exacta de Fisher
E. T de Student.

197. Paciente con WPW, cual es la alternativa correcta:


A. Se debe a fenómenos de reentrada
B. Su tratamiento definitivo es con beta
bloqueadores
C. Su tratamiento de urgencia es ablación con
radiofrecuencia
D. Cursa con onda delta en el EKG
E. Más frecuente en personas adultas mayores

198. ¿Cómo se define un RN con peso adecuado para


la edad gestacional?

199. La inhibición de la agregación plaquetaria por


ADP (adenosina difosfato) es característica de:
A. Ácido acetilsalicílico.
B. Tirofibán.
C. Sulfinpirazona.
D. Clopidogrel.
E. Abciximab.

200. El tipo de sangrado más característico que


producen los leimiomas uterinos:
A. Hipermenorrea por intramurales
B. Metrorragia por submucosos
C. Menorragia por submucosos
D. Menometrorragia por intramurales

También podría gustarte